Author: Манида С.Н.  

Tags: физика   задачи по физике  

ISBN: 5-288-03499-0

Year: 2004

Text
                    С. Н. Манида
СТУДЕНТАМ
УЧИТЕЛЯМ ШКОЛЬНИКАМ
физика
Решение задач повышенной
сложности
ПО МАТЕРИАЛАМ ГОРОДСКИХ ОЛИМПИАД ШКОЛЬНИКОВ
УДК 53(075.4)
ББК 22.3
М23
Рецензенты: кафедра методики преподавания физики Рос. гос. пед. ун-та им. А. И. Герцена (зав. каф. акад. РАО, д-р физ.-мат. наук, проф. А. С. Кондратьев);
д-р физ.-мат. наук А. Г. Изергин (С.-Петерб. отд-ние Математического ин-та им. В. А. Стеклова РАН)
Печатается по постановлению Редакционно-издательского совета С.-Петербургского государственного университета
Манида С.Н.
М23 Физика. Решение задач повышенной сложности: По материалам городских олимпиад школьников: Учеб, пособие. — 2-е изд. ~ СПб.: Издательство С.-Петербургского университета, 2004. 440 с. ISBN 5-288-03499-0
Основное содержание пособия составляют задачи повышенной сложности, предлагавшиеся на школьных олимпиадах по физике. В книгу включено около трехсот пятидесяти задач с решениями. Задачи распределены по тематическим разделам, которым предшествуют комментарии, поясняющие наиболее сложные понятия.
В раздел "Добавления” вынесены методические разработки по темам, выходящим за рамки школьного курса физики и редко рассматриваемым в вузах.
Пособие может служить руководством для студентов физических специальностей университетов и педагогических вузов. Оно может быть использовано учителями при подготовке школьников к олимпиадам и вступительным экзаменам в вузы, а учащимися—для самостоятельной работы.
Ил. 148.
ББК 22.3
ISBN 5-288-03499-0
© С. Н. Манида, 2004
© Издательство
С .-Петербургского университета, 2004
Предисловие
Физический факультет Ленинградского — ныне Санкт-Петербургского государственного университета уже более шестидесяти лет участвует в проведении городских и районных олимпиад школьников. Десятки ученых-физиков предложили за эти годы школьникам тысячи оригинальных задач. По традиции факультета студенты, бывшие в свое время победителями школьных олимпиад, активно участвуют в подготовке и проведении этих турниров юных физиков.
С 1974 года на протяжении пятнадцати лет издавались сборники олимпиадных задач. Большая часть решений задач, включенных в первые сборники, принадлежит выдающемуся физику-методисту физического факультета Виталию Рейнгольдовичу Саулиту. Над составлением задач много лет работал Анатолий Георгиевич Изергин, известный физик-теоретик, профессор физического факультета. В дальнейшем их сменили ученики — Юрий Петрович Малышев, один из ведущих организаторов школьного образования в Петербурге, и Антон Юрьевич Алексеев, молодой талантливый ученый, работы которого по квантовой теории поля получили международное признание.
Сборники олимпиадных задач уже давно стали библиографической редкостью, а регулярный выпуск новых сборников практически невозможен из-за многочисленных проблем нашего времени.
Данное пособие создавалось с целью сохранить, обобщить и передать следующему поколению накопленный методический опыт. Автор надеется, что, кроме исторической ценности, это пособие послужит развитию физического образования.
Студентам физических специальностей университетов и педагогических вузов это пособие пригодится при изучении курса ” Методика преподавания физики”. Преподаватели школ, гимназий, лицеев получат возможность разнообразить учебный процесс, проводить индивидуальную работу с наиболее одаренными учащимися. Школьникам пособие будет полезно при самостоятельной подготовке к олимпиадам и вступительным экзаменам.
Пособие организовано по следующей схеме. Задачи распределены по тринадцати тематическим разделам. Порядок расположения
3
этих разделов определяется не столько последовательностью изложения материала в школьном курсе физики, сколько характером восприятия этого материала учащимися. Например, в первом разделе представлены задачи, в решении которых используется закон Архимеда. Такое уважение к этому закону связано не только с его древним происхождением, но, главное, с теми трудностями, которые вызывает его применение у учащихся (и у их учителей). В каждом разделе сначала приводится небольшой теоретический обзор тех физических принципов, на которых построены методы решения задач данной темы. Этот обзор предполагает знакомство читателя с общими положениями курса физики и не претендует на роль учебника. Далее следуют тексты задач с указанием степени сложности (* — сложная, ** — очень сложная), года и уровня олимпиады (“р” — районная, “г” — городская) с подробными решениями. В представленных решениях особое внимание уделяется не вычислениям, а физической сути исследуемого явления или процесса, обоснованию методов применения тех или иных известных школьнику физических законов в различных конкретных случаях.
В заключительный раздел „Добавления” вынесены методические разработки по темам, выходящим за рамки школьного курса физики и редко рассматриваемым в вузах.
Автор благодарит студентов физического факультета Е. Всемир-нову, Ю. Голод, А. Григоряна, Т. Данилову, А. Катасонова, Н. Кра-гель, Е. Лобынцеву, А. Маляренко, И. Маниду, И. Миронову, Г. Му-зафарову, Ю. Несветаева, И. Семенову, Н. Солонец, И. Сорину и особенно Н. Меркулову за помощь в компьютерном наборе.
Автор будет признателен читателям за любые замечания по структуре и содержанию пособия.
Используемые обозначения
t —время
г —радиус-вектор
ж, у, z —координаты точки в прямоугольной (декартовой) системе координат
v —скорость; |v| =. v; vXj vyj vz—компоненты вектора скорости в прямоугольной (декартовой) системе координат
а —ускорение; |а| = а; аХу ау, az—компоненты вектора ускорения в прямоугольной (декартовой) системе координат
g —ускорение свободного падения; |g| — д ял 10 м/с2
т —масса тела
F, Т, Q,... —силы
р — импульс (количество движения)
М —момент силы
А —приращение физической величины
к —жесткость пружины при упругих деформациях
// — коэффициент трения
Q, —плоские углы
А — механическая работа
К — кинетическая энергия
U — потенциальная энергия
W —полная механическая энергия
Е —энергия свободного тела (с учетом энергии покоя); освещенность
N —мощность; число нейтронов в ядре; поток напряженности электрического поля
G —гравитационная постоянная, G = 6,672 • 10-11Н • м2/кг2
vi, vii —I и II космические скорости
R —радиус кривизны; электрическое сопротивление
/?з —радиус Земли (радиус круговой орбиты низколетящего спутника)
Мз —масса Земли
р —давление
р —объемная плотность; удельное сопротивление
а —коэффициент поверхностного натяжения
д —электрический заряд
е —элементарный электрический заряд, е = 1,6-10”19 К л
Е —напряженность электрического поля
* Векторные величины выделяются полужирным шрифтом: г, v, a, g, F, Т, Q и т. д.
5
р —потенциал; фаза колебаний
U —падение напряжения
б —диэлектрическая проницаемость
I —сила тока; сила света
j —плотность тока
8 —электродвижущая сила (ЭДС)
В — магнитная индукция
Ф —магнитный поток; световой поток
С —электрическая емкость
бо —электрическая постоянная, бо = 8,85 • 10-12 Кл2/(Н • м2)
—магнитная постоянная, //о = 1,26 • 10”6Н/А2
А —амплитуда колебаний; массовое число (для ядер)
w —угловая скорость (циклическая частота)
v — частота колебаний
Т — период колебаний
А —длина волны
L —индуктивность
Z —импеданс; число протонов в ядре
п — коэффициент преломления
а, Ь —расстояния от линзы до источника и изображения
f —фокусное расстояние
D —оптическая сила
с — скорость распространения электромагнитного излучения (скоро света),ее значение в вакууме с = 2,99792458- 108м/с
Л —постоянная Планка, Л = 6,6 • 10~34 Дж-с
h —редуцированная постоянная Планка, h = Л/(2тг)
Единицы физических величин
Величина	Размерность	Единица СИ	
		Наименование	Обозначение
Длина	L	метр	м
Масса	М	килограмм	кг
Время	Т	секунда	с
Скорость	LT-1	метр в секунду	м/с
Ускорение	LT~2	метр на секунду	м/с2
Угол	—	в квадрате радиан	рад
Угловая скорость	т-1	радиан в секунду	рад/с
Частота	т-1	герц	Гц
Плотность	ML~3	килограмм на	кг/м3
Сила	MLT~2	кубический метр ньютон	Н
Момент силы	ML2T~2	ньютон-метр	Нм
Импульс	MLT-1	ньютон-секунда	Нс
Давление	ML~1T~2	паскаль	Па
Работа, энергия	ML2T~2	джоуль	Дж
Мощность	ml2t~3	ватт	Вт
Жесткость	MT~2	ньютон на метр	Н/м
Электрический заряд	TI	кулон	Кл
Н апряженность	MLT~3I~l	вольт на метр	В/м
электрического поля Электрический	ML2T~3rl	вольт	В
потенциал Сила электрического	I	ампер	А
тока Электрическое	ML2T~3I~2	ом	Ом
сопротивление Магнитная индукция	мт~гг'	тесла	Тл
Магнитный поток	ml2t-2i-'	вебер	Вб
Электрическая емкость	М-гЬ-2ТЧ2	фарад	Ф
Индуктивность	ml2t~2i~2	генри	Гн
Сила света	J	кандела	кд
Световой поток	J	люмен	лм
Освещенность	JL~2	люкс	лк
Оптическая сила	L~l	диоптрия	дптр
7
1.	ЗАКОН АРХИМЕДА
Любое тело, погруженное в жидкость, подвергается сжимающему и выталкивающему действию со стороны жидкости.
Представим такую ситуацию: ученый, владеющий современными приборами и мощным математическим аппаратом, решил вычислить силу, выталкивающую из жидкости погруженное в нее тело.
Он экспериментально установит, что на единицу поверхности тела, погруженного в жидкость с плотностью р, действует по нормали к поверхности сила гидростатического давления р, зависящая от глубины погружения Л по определенному закону (р — pgh) и не зависящая от ориентации поверхности.
Он сложит векторы сил давления, действующих на различные элементы поверхности тела и направленные по нормали к ним; для этого потребуется вычислить так называемый поверхностный интеграл от некоторой векторной функции по поверхности тела сложной формы. С помощью современного математического аппарата и мощных компьютеров этот интеграл может быть вычислен. Но каково же будет изумление этого ученого, когда окажется, что полученный результат численно равен весу жидкости в объеме погруженной части тела! Этот результат был получен греческим ученым Архимедом 2200 лет назад, причем в общем виде — для тел любой формы!
Попробуем восстановить ход рассуждений Архимеда и вывести его закон.
На рис. 1 изображено тело, помещенное в жидкость. На это тело со стороны жидкости действует описанная выше сила гидростатического давления. Для нахождения этой силы вместо вычисления сложных интегралов проведем мысленный эксперимент: уберем тело и рассмотрим жидкость в объеме V, который занимала погруженная часть тела (рис. 2). На эту жидкость действует сила тяжести pVg и сила гидростатического давления F. Выделенный объем наг-ходится в равновесии, следовательно, сумма сил, действующих на
8
жидкость в этом объеме, равна нулю: F 4- plzg = 0. Отсюда следует выражение для силы гидростатического давления:
F - -p/g.
Мы нашли силу, действующую на поверхность жидкости, заполняющей объем V. Но поверхность тела, погруженного в жидкость, совпадает с поверхностью жидкости в нашем мысленном эксперименте, следовательно, найденное выражение и есть “выталкивающая” сила — сила Архимеда Ед:
Fa = -pVg.
Это равенство носит название закон Архимеда.
Казалось бы, решение задач с использованием этого закона не должно вызывать затруднений. Однако неверные решения отдельных задач на закон Архимеда встречаются не только у школьников, но и в ряде задачников.
Дело в том, что при использовании этого (как и любого другого) закона надо всегда помнить, как и для каких ситуаций он выводился. Так, например, мы вычисляли силу гидростатического давления, действующую на поверхность неподвижного объема жидкости, находящейся в равновесии, т. е. имеющей нулевые скорость и ускорение. Следовательно, и использовать выведенное выражение для силы Архимеда можно только в тех случаях, когда и скорость, и ускорение тела равны нулю.
Покажем, что применение этого закона в других ситуациях абсолютно неправомочно, так как приводит к неверным результатам.
Рассмотрим легкое тело, привязанное ниткой ко дну сосуда, заполненного жидкостью (рис. 3). Тело погружено в жидкость и находится в равновесии. На него действуют вниз сила тяжести mg = pT^g и сила натяжения нити Т, а вверх — сила гидростатического давления F = Fa = -pVg, где рт — плотность тела, р— плотность жидкости. Условие равновесия тела
- pi/g + Т + pTVg = 0.	(1)
Пусть в некоторый момент нить обрывается (т.е. исчезает сила натяжения Т), равенство (1) перестает выполняться, и тело начинает двигаться вверх (“всплывать”) с некоторым ускорением а, которое можно найти из уравнения движения
FBbIT + pTVg = pTIza,	(2)
9
Рис. 3
где FBbIT —выталкивающая сила. Предположив, что в этом случае можно использовать закон Архимеда, подставим — pVg в левую часть равенства (2) вместо FBbIT. Для ускорения тела получаем выражение
« Р Рт„ а =------—g-
Рт
Исследуем выражение (3). Ускорение тела направлено против ускорения свободного падения (что абсолютно верно), а его величина, равная
(3)
(4)
Р — Рт а = ------<7,
рт
неограниченно возрастает при уменьшении плотности тела. Такой результат противоречит как наблюдениям, так и здравому смыслу.
Таким образом, закон Архимеда неприменим к телам, ускорение которых относительно жидкости отлично от нуля (даже при равной нулю скорости).
Точный расчет гидростатического давления на поверхность ускоренно движущегося тела возможен только с применением аппарата математической физики, а ответ представим в аналитическом виде лишь для частных случаев. Уравнения, описывающие движение тела в жидкости, были впервые получены профессором Петербургского университета Леонардом Эйлером в середине XVIII века. Решение этой задачи для частного случая тела сферической формы, размеры которого много меньше размеров сосуда, приведено в Добавлении I (для читателей, владеющих методами постановки и решения граничных задач математической физики). Полученный там результат отличается от (3):
_ _ л Р ~~ Рт „ а — “л	££♦
р + 2рт
(5)
Из этого выражения следует, в частности, и то, что даже бесконечно легкий шарик всплывает с конечным ускорением 2д.
Несмотря на изложенные выше соображения о неправомочности использования закона Архимеда при движении тел в жидкостях, в
10
большинстве задачников такие задачи встречаются. В данном сборнике мы помещаем ряд задач —13 (случай 5), 14 (случай 2), 21, в которых рассматривается движение тел, помещенных в воду, и приводим такие методы решения этих задач, при которых не приходится использовать явные выражения для силы Архимеда.
При решении задач на закон Архимеда у школьников часто возникают затруднения, связанные с определением объема “погруженной” части тела. Обращаем ваше внимание на задачу 17, в которой объем “погруженной” части тела значительно превышает объем самого тела, и задачу 18, в которой объем “погруженной” части тела оказывается отрицательным.
В задачах 19 и 20 жидкость с погруженным в нее телом движется ускоренно, при этом меняется распределение давления внутри жидкости и, следовательно, выталкивающая сила определяется выражениями, отличными от закона Архимеда.
* * *
Задача 1 (79-р)
Герметически закрытый сосуд заполнен водой. Площадь нижнего основания сосуда Si = 100 см2, верхнего основания S2 = 200 см2, высота сосуда h = 50 см. Сила давления воды на верхнее основание F2 = 100 Н. Найдите силу давления на нижнее основание сосуда, если плотность воды р = 1000 кг/м3, ускорение свободного падения д = 10м/с2.
Решение. Давление воды на верхнее основание р% = Ръ/Зъ) на нижнее основание pi = Р2 + pgh = F2/S2 + pgh, Сила давления на нижнее основание
Fj =PlSi =	+ pghSt = 100 H.
^2
Задача 2 (81-р)
Стакан, имеющий массу т = 100 г и объем V = 200см3, подвешен к динамометру в перевернутом состоянии, причем края стакана касаются поверхности воды, а внутренний объем стакана заполнен водой. Почему вода не выливается из стакана? Что показывает динамометр?
Решение. На воду в стакане в вертикальном направлении действуют три силы: сила тяжести pgV, сила давления Fi со стороны дна стакана, направленные вниз, и сила давления F остальной
11
воды на уровне поверхности, направленная вверх. Вода в стакане находится в равновесии, если
F = F1 + pgV.	(1)
Разделив (1) почленно на площадь дна стакана S, получим
Ратм = p + pgh,	(2)
где Ратм— давление воды на уровне поверхности, равное атмосферному давлению; р — давление дна стакана на воду (по третьему закону Ньютона оно равно давлению воды у дна стакана) ; Л— высота стакана. Эта формула дает известную зависимость давления от глубины внутри несжимаемой жидкости. Вода не выливается из стакана, так как разность сил давления на поверхности и у дна стакана в точности уравновешивает вес воды в стакане.
Рассмотрим силы, действующие в вертикальном направлении на стакан: сила тяжести тд и сила атмосферного давления FaTM, направленные вниз, сила натяжения пружины динамометра Т и сила давления воды на дно стакана F\y направленные вверх. Стакан находится в равновесии, т. е.
Т + F1 = тд + ^атм-	(3)
Из соотношения (3) с учетом равенства (2) имеем
Т = -Ратм - Fi+mg = (ратм - p)S + тд = (m + pV)g. (4)
Отсюда Т = 3 Н. Из формулы (4) видно, что показания динамометра равны суммарному весу стакана и заполняющей его воды.
Задача 3 (81-р)
Цилиндрический тонкостенный стакан с площадью дна S поставили вверх дном на ровный горизонтальный стол, покрытый слоем упругой резины. Дно стакана имеет маленькое отверстие, в которое вставлена вертикально тонкостенная трубка. По этой трубке в стакан наливают воду. Вода заполняет весь стакан и часть трубки до уровня Ло от дна стакана. При дальнейшем увеличении количества воды она начинает вытекать из стакана. Определите массу т стакана с трубкой.
Решение. На стакан с трубкой в вертикальном направлении действуют четыре силы: сила тяжести тд и сила атмосферного давления FaTM = Ратм(»5 — So) (So —площадь отверстия в дне стакана) действуют вниз; сила реакции стола Q и сила давления воды на
12
дно стакана FR = pA(S — So) (Рд—давление воды у дна стакана) действуют вверх. Давление в трубке меняется с глубиной по закону
р(Л) - Ратм + pgh.
У дна стакана h = Ло и
Гл = (р атм + pgh0)(S - So).
(’такай покоится, т. е. сумма сил, действующих на него, равна нулю:
тд + Ратм(5 - So) - Q - (Ратм + pgh0)(S - So) = о, откуда получаем
mg - Q - pgh0(S - So) = 0.	(1)
Из (1) видно, что если /го увеличивать, то величина Q будет уменьшаться. При некотором значении Ло сила реакции стола Q становится равной нулю. Начиная с этого момента, стакан к столу не прижат—вода из-под него начинает вытекать. Полагая в (1) Q = 0, имеем:
т = p/i0(S - So).	(2)
По условию задачи отверстие в дне стакана мало, т. е. So S, следовательно, величиной So по сравнению с S можно пренебречь. В результате из (2) с хорошей степенью точности получим: т = phgS.
Задача 4* (76-р)
Герметически закрытый абсолютно жесткий сосуд высотой Н доверху заполнен водой. Давление воды у верхней стенки сосуда равно пулю. У дна сосуда находится маленький пузырек воздуха. Как изменится давление воды в сосуде, если пузырек всплывет? Считать, что вода несжимаема. Растворимостью воздуха в воде пренебречь.
Решение. Давление у дна сосуда (на глубине Я)
р(Я) = рдН,
где р —плотность воды.
Давление внутри пузырька, находящегося у дна,
Ро = рдН + 2а/г,
где а — коэффициент поверхностного натяжения воды; г — радиус пузырька. Так как пузырек всплывает при постоянной температуре
13
и растворимостью воздуха в воде можно пренебречь, то давление ро внутри пузырька и его объем V связаны формулой роV = const. При всплытии радиус пузырька г и его объем V остаются неизменными, так как по условию задачи вода несжимаема. Следовательно, не изменяется и ро- Таким образом, для давления р в воде у верхней стенки сосуда после всплытия пузырька можно написать:
р(0) = Ро - 2а/г = рдН\
давление на глубине Л будет равно р(Л) = р(0) + pgh = рд(Н + /г), т. е. на величину рдН больше, чем до всплытия пузырька.
Задача 5 (76-г)
Царь приказал Архимеду отмерить столько золота, сколько весит слон. Таких больших весов нигде не оказалось. В распоряжении Архимеда был плот. Как Архимед решил задачу?
Р е ш е н и е. На тело, погруженное в воду, действует выталкивающая сила, прямо пропорциональная объему погруженной части тела. Тело плавает (находится в равновесии), когда вес тела равен выталкивающей силе. Поэтому плот, нагруженный разными грузами одного и того же веса, будет погружаться на одну и ту же глубину. Следовательно, для решения задачи надо завести слона на плот, отметить уровень погружения плота, затем убрать слона и нагружать плот золотом до тех пор, пока он не погрузится до того же уровня.
Следует отметить, что этот способ не очень точный. Небольшой перекос плота может привести к большой ошибке.
Существует более точный способ. Нужно поместить плот в небольшой бассейн, заполненный до краев водой. Под весом слона плот опустится, и часть воды выльется из бассейна. Убрав слона, надо нагружать плот золотом до тех пор, пока вода не поднимется до краев бассейна. При таком способе измерения возможные перекосы плота на точность взвешивания не влияют.
Задача 6 (74-г)
Полностью погруженное в жидкость тело плавает на некоторой глубине, находясь в равновесии. От каких физических параметров зависит устойчивость этого равновесия? Запишите условие устойчивости равновесия, считая, что температура жидкости не зависит от глубины.
Р е ш е н и е. На тело, погруженное в жидкость, действуют сила тяжести тд, направленная вниз, и выталкивающая сила рхдт/р2У направленная вверх (здесь т — масса тела; р\ и /?2 — плотности жидкости и тела на некоторой глубине; очевидно, что
14
объем тела равен т/ръ)* Если тело находится в равновесии, то равнодействующая этих сил равна нулю, следовательно, р\ = р2. Сместим тело от положения равновесия, например вниз, на некоторую глубину. При этом плотности жидкости и тела изменятся (pi и р2)-Вели равновесие устойчиво, то тело будет стремиться вернуться в прежнее положение, следовательно, р\тпд/р2 > W, т.е. р\ > р2. Это говорит о том, что сжимаемость жидкости больше сжимаемости тела (очевидно, что этого достаточно для устойчивости равновесия).
Задача 7 (75-г)
Два металлических шара (свинцовый и железный) уравновешены на разноплечих весах. При опускании шаров в воду равновесие не нарушилось. Попробуйте найти объяснение этому явлению.
Решение. Пусть 1\ и /2, и т2, Ц и V%— длина плеч весов, масса и объем шаров соответственно. Условие равновесия на таких весах: mi/i = m2Z2. Так как сила Архимеда пропорциональна объему данного тела, то при погружении в воду равновесие весов не будет нарушено в случае, если Vi/i = УзЬ- Разделим почленно первое равенство на второе: m^/Vi = m2/V2, т. е. р\ = р2. Это говорит о том, что средние плотности шаров равны. Отсюда ясно, что в более плотном (свинцовом) шаре имеется полость, вследствие чего равновесие сохраняется.
Задача 8 (75-р)
Льдинка с вмороженной в нее пробкой (дробинкой) плавает в стакане с водой при температуре О °C. Как изменится уровень воды в стакане, если лед растает, а температура сохранится?
Решение. Обозначим массу льда т и массу тела то. Вытесняемый ими объем воды
_ т + т0
Р
где р —плотность воды.
Когда лед растает, он займет объем Ул = т/р.
Если плотность тела ро меньше или равна плотности воды, то тело плавает и вытесненный при этом объем воды Vo = то/р. Очевидно, что в этом случае V = Ул 4- Vo, т. е. уровень воды в стакане не изменится.
Если плотность тела ро больше плотности воды, то оно тонет, вытесняя объем, равный его собственному Vo = то/ро- В этом случае V > Ул 4- Vo и уровень воды в стакане понизится.
15
Задача 9 (77-г)
В большом сосуде с водой плавает в вертикальном положении тонкостенный стакан, в который налито некоторое количество воды. Разность уровней воды в сосуде и стакане равна х. Как изменится эта разность, если в стакан опустить пробку?
Решение. Пусть т — масса пустого стакана, S — площадь его поперечного сечения, V — объем налитой в стакан воды, Ц —объем погруженной части стакана, р — плотность воды и д — ускорение свободного падения. Стакан находится в равновесии, т. е. сила тяжести, действующая на него, уравновешивается выталкивающей силой:
тд + pgV = pgVi.
Очевидно, Vi = V 4- Sx . Тогда
т х = pS
не зависит от количества налитой в стакан воды.
Пусть в стакан брошена пробка, М — ее масса и Vz— объем погруженной в воду части пробки. По закону Архимеда имеем:
Мд - P9V2 •
Если теперь пробку заменить объемом воды V2 (он имеет массу М), то уровень воды в стакане не изменится. Глубина погружения стакана также не изменится. Следовательно, разность уровней воды в стакане и сосуде останется неизменной. Так как разность уровней не зависит от количества воды в стакане, то ясно, что величина х при опускании пробки в стакан не изменится.
Задача 10 (79-г)
Толстостенная лодка с вертикальными стенками и отверстием в дне достаточно долго свободно плавает в озере. Затем отверстие снаружи затыкают и внутрь лодки опускают бревно. Повысится или понизится после этого уровень воды в лодке относительно уровня воды в озере? Почему?
Решение. Пусть т — масса бревна, Si — площадь дна лодки снаружи и Sz — площадь дна лодки внутри. По условию задачи
Si > S2.	(1)
При помещении в лодку бревна она, в силу увеличения общего веса системы, глубже (на некоторую величину hi) погружается в озеро, вытесняя при этом дополнительно объем воды
V = hiSi,	(2)
16
масса которой равна массе бревна т. Бревно, погружаясь в заполняющую лодку воду, вытесняет тот же объем жидкости V. В результате уровень воды в лодке повышается относительно ее дна на некоторую величину /12—такую, что
V = h2S2.	(3)
Из (2) и (3) получаем:
hi _ S2
h2 S\ ’
откуда, в силу неравенства (1), следует, что h2 > hi. Таким образом, после погружения бревна уровень воды в лодке станет выше уровня воды в озере на величину, равную разности h2 — hi.
Задача 11 (82-р)
Шайба массой М} имеющая форму цилиндра с площадью основания S и высотой Л, плавает на границе раздела двух несмвшивающихся жидкостей с плотностями pi и р2 (pi < р2). Основание шайбы параллельно границе раздела жидкостей. Найдите глубину погружения шайбы в нижнюю жидкость.
Решение. Очевидно, что более легкая жидкость находится сверху (в противном случае равновесие системы неустойчиво и любые сколь угодно малые флуктуации поверхности раздела приведут систему в устойчивое равновесное состояние). Плотность вещества шайбы обозначим через р. По условию задачи р = M/(Shy
Если р < piy то шайба всплывает. Если р > р2} то шайба тонет. Следовательно, из условия задачи вытекает, что pi < р < р2-Обозначим высоту части шайбы, находящейся в нижней жидкости, через h2. Пусть pi —давление в жидкости на уровне верхнего основания шайбы, р2 — на уровне нижнего основания, р2 превышает pi па величину, численно равную весу соответствующего столба жидкости с единичной площадью поперечного сечения: р2 = Pi +<z(pi (Л— /н) + P2h2)y где д— ускорение свободного падения.
Сила гидростатического давления Fa, действующая на шайбу и выталкивающая ее вверх, может быть записана как
Fa = (рг -pi)S = gS(pi(h -h2) + p2h2).
'Гак как шайба находится в равновесии, то Fa = Мд. Следовательно, искомая величина
Р2 ~ Р1
Из полученного выражения видно, что при изменении р от р2 до pi глубина погружения в нижнюю жидкость меняется от h до 0.
17
Задача 12 (78-р)
Кубик плавает в сосуде с водой так, что его верхняя грань параллельна поверхности воды. При этом половина кубика погружена в воду. Какой слой масла надо долить, чтобы кубик плавал полностью погруженным в жидкость, если плотность масла в два раза меньше плотности воды и длина ребра кубика равна а? Масло с водой не смешивается.
Решение. Сначала определим плотность вещества кубика р. Плотность воды обозначим через рв. Сначала на кубик действовали направленная вниз сила тяжести
F = рда3	(1)
и сила давления со стороны воды —выталкивающая сила Fa, направленная вверх и равная весу вытесненной кубиком воды:
Fa = ^Рвда3.	(2)
Так как кубик находился в равновесии, то
F = Fa.	(3)
Подставляя выражения (1) и (2) в формулу (3), получаем:
1
Р = 2^>	(4)
т.	е. плотность вещества кубика равна плотности масла.
Предположим, что в сосуд налили такой слой масла, что кубик погрузился полностью и его верхняя и нижняя грани горизонтальны. Пусть fli—высота части кубика, находящейся в масле; тогда a — ai —высота его части, погруженной в воду. На кубик действуют сила тяжести F и выталкивающая сила F^, равная
Fa = Рв9(а - ai)a? + рда^а2.
Так как и теперь кубик находится в равновесии, то можно приравнять F = и получить выражение для нахождения а:
рда = рдаг + ръд(а - си).
С учетом равенства (4) получаем, что а, = а, т. е. в состоянии равно-весия кубик полностью погружен в масло. Так как плотности масла и кубика равны, то толщина h слоя масла роли не играет, лишь бы кубик мог полностью в него погрузиться. Таким образом, необходимо долить слой масла h > а.
18
Задача 13 (80-р)
Вес Р системы, состоящей из стакана с водой и пробкового шарика, измеряется в пяти случаях:
1)	шарик свободно плавает в стакане (показание весов Pi);
2)	шарик лежит на чаше весов рядом со стаканом (Рг);
3)	шарик удерживается в полностью погруженном состоянии тонкой невесомой нитью, прикрепленной ко дну стакана (Рз);
4)	шарик удерживается в полностью погруженном состоянии тонкой невесомой спицей, закрепленной над стаканом (Р4);
5)* шарик, удерживавшийся в утопленном состоянии, освобождается и начинает свободно всплывать (Ps)«
Расставьте показания весов в порядке их возрастания.
Решение.В первых трех случаях на систему, состоящую из шарика и стакана с водой, действуют две внешние силы: сила тяжести Мд (М— суммарная масса стакана, воды и шарика), направленная вниз, и сила реакции (Pi, Р2 или Рз) со стороны чаши весов, направленная вверх. Так как центр тяжести системы покоится, обе силы компенсируют друг друга. Следовательно, Рх ±= Р2 = Рз = Мд.
В четвертом случае кроме сил Р4 и Мд на шарик действует еще одна внешняя сила F со стороны спицы, мешающая всплытию шарика и направленная вниз. Так как система находится в равновесии, то ясно, что Р4 = F + Мд, т. е. Р4 > Мд.
В пятом случае на систему действуют только две внешние силы: Мд и сила реакции Р5 со стороны неподвижной чаши весов. Но теперь, в отличие от первых трех случаев, шарик всплывает. Вследствие этого центр тяжести системы опускается, хотя стакан и неподвижен. Характер движения центра тяжести определяется характером движения шарика. В начальный момент шарик не имел скорости, а в следующие моменты его скорость направлена вверх, следовательно, в начальный момент он двигается с некоторым ускорением, направленным вверх. При этом центр масс системы опускается вниз с некоторым ускорением. Пусть это ускорение равно а. По II закону Ньютона Ma = Мд — Р3. Отсюда Р3 = М[д — а) < Мд.
Окончательно получаем: Р3 < Pi = Р2 = Рз < Рл-
Задача 14 (80-р)
На чашке весов стоят гиря и стакан с водой. Сравните показание весов Р с показаниями весов в следующих случаях:
1) гиря подвешена на нити к внешнему штативу и погружена в воду (Pi);
2)* подвес оборвался, и гиря начинает тонуть (Pi)-
19
Решение. Подвес будет действовать на систему с некоторой силой, направленной вверх. Проводя рассуждения, аналогичные решению задачи 13, легко получить, что показание весов в этом случае уменьшается: Р\ < Р.
После обрыва подвеса действие внешней силы прекращается, но центр масс системы двигается ускоренно вниз, следовательно, Р2 < Р. Чтобы сравнить величины Fi и Р2, рассмотрим силы, действующие на стакан с водой в двух случаях. В первом случае на стакан с водой действует сила тяжести тд вниз, сила Pi вверх и сила давления со стороны гири вниз, равная численно силе гидростатического давления на гирю Fa. При этом центр масс стакана с водой неподвижен, следовательно,
Pi = mg + Fa.
Во втором случае действует та же сила тяжести, сила реакции опоры Р2 и некоторая сила давления со стороны гири FA. Легко показать, что FA > Fa- Действительно, гиря на подвесе давила на окружающую воду, а вода находилась в неподвижном состоянии. В момент обрыва гиря начинает ускоренно двигаться вниз и смещать близлежащие слои воды в ту же сторону, следовательно, сила давления гири на воду должна возрасти. Под действием всех сил центр масс воды в стакане движется ускоренно вверх, следовательно, Р2 > mg + F'a > mg + Fa = Pi-
Окончательный результат: Р > Р2 > Р\.
Задача 15* (79-р)
Пробка прикреплена посредством легкой пружинки к дну сосуда с водой. И пробка и пружинка полностью погружены в воду. Увеличится или уменьшится длина пружинки, если сосуд начнет двигаться вертикально вверх (вниз) с постоянным ускорением?
Решение. Уравнение движения пробки запишем в виде
та = Fa — тд — кх,
где Fa —сила гидростатического давления; а — ускорение системы; т — масса пробки; к — коэффициент жесткости пружины; х— величина растяжения пружины. Для определения зависимости х от а необходимо определить Fa- Рассмотрим сосуд, наполненный водой и движущийся вертикально вверх (вдоль оси я) с постоянным ускорением а. Мысленно выделим внутри жидкости некоторый объем и рассмотрим силы, действующие на него. Так как этот объем тоже движется вверх с ускорением а, то равнодействующая всех сил,
20
приложенных к нему, направлена вверх и равна
F = Fa - Af д = М,
। д<‘ М —масса выделенного объема. Из этого равенства находим
Fa = М(д + а),
а из уравнения движения пробки следует, что
кх = (М — т)(а + д).
Но условию задачи пробка легче воды, следовательно, х > 0 при а > 0.
Задача 16* (79-р)
Воздушный шарик, наполненный водородом, привязан резиновым жгутом к дну лифта. Увеличится или уменьшится длина жгута, если лифт начнет двигаться вниз с постоянным ускорением, меньшим ускорения свободного падения?
Решение. Решение аналогично решению задачи 15. Длина жгута уменьшится.
Задача 17 * (78-г)
В сосуд с водой опущена труба диаметром d. В трубу поместили шарик того же диаметра. Центр шарика оказался на глубине h. Найдите плотность вещества шарика. Зазор между трубой и шариком отсутствует. Сила трения между ними равна нулю.
Решение. Пусть т и р — масса и плотность шарика, V = •И (I) —его °бъем. Рассмотрим действующие на шарик силы. Это гпла тяжести тду направленная вниз, и направленная вверх сила давления воды Q, действующая на обращенную вниз поверхность шарика. Так как шарик покоится, то по II закону Ньютона имеем
тд - Q = 0.	(1)
Найдем силу Q, Для этого мысленно удалим шарик и дольем в трубу воду так, чтобы уровень воды в трубе был тот же, что и снаружи. Очевидно, что при этом система останется в равновесии. Следовательно,
Мд - Q = 0,	(2)
где М—масса долитой воды. Сравнив (1) и (2), найдем, что
т = М, piVr = pV,	(3)
21
где р\ — плотность воды и
„	(d\2 . 2тг/<Л3
Vi = ’UJ +t(v ()
— объем долитой воды (он складывается из объема цилиндра высотой Л с площадью основания тг(с?/2)2 и из объема половины шарика). Из (3) и (4) найдем:
1 + ЗЛ/d
Задача 18 * (87-р)
Раковина в ванной комнате имеет сливное отверстие радиусом г. В раковину опускают шар массой т и радиусом R > г и начинают медленно наливать воду. Опишите качественно дальнейшее поведение системы в зависимости от соотношения между тп, Я, г.
Решение. Возможны два варианта:
1) вначале шар попадает в сливное отверстие, и вода начинает наполнять раковину; в некоторый момент шар всплывает, вода вытекает, пока шар снова не попадает в отверстие; процесс циклически повторяется;
2) шар не всплывает, и вода переливается через верхний край раковины.
Поведение шара зависит от двух параметров: отношения р/р' (р = Зш/(4тгЯ3)—плотность шарика; р'— плотность воды) и отношения г/R.
На шар действуют три силы: сила тяжести год, сила гидростатического давления F и сила реакции опоры. Шар будет всплывать, если F > тд. Сила F действует только на часть поверхности шара, соприкасающуюся с водой. В зависимости от соотношения между параметрами задачи (радиусами шара и сливного отверстия, высотой налитой жидкости) эта сила может оказаться “выталкивающей” или “прижимающей” шар к отверстию. Вычисление силы гидростатического давления на поверхность тела можно свести к вычислению некоторых объемов.
22
На рис. 4, а шар выступает над поверхностью воды, а на рис. 4, б он полностью погружен в воду. Сила давления воды на поверхность шара в случае a (FJ вычисляется обычным образом. Она направлена вверх и равна весу воды в объеме части шара, заштрихованной с правым наклоном.
Если бы вся поверхность шара на рис. 4, б соприкасалась с водой, сила давления была бы направлена вверх и равна весу воды в объеме всего шара. Но вода не давит на часть поверхности шара, оказавшуюся в отверстии. Это приводит к тому, что сила давления на верхнюю часть шара (F2), равная весу воды в объеме, заштрихованном с левым наклоном, прижимает шар к отверстию. Полная сила давления равна F\ — F^- При достаточно высоком уровне воды на рис. 4, б сила давления может оказаться отрицательной, т. е. “прижимающей”, а не “выталкивающей”.
Задача 19 * (77-г)
Шарик, плотность которого меньше плотности воды, с помощью гонкой нити прикреплен к дну большого сосуда, заполненного водой. (’осуд движется вправо с ускорением а. В какую сторону и на какой угол а отклонена нить от вертикали?
Рис. 5. К задаче 19.
Решение. Предположим сначала, что в движущемся с ускорением сосуде пока никакого шарика нет, и мысленно выделим в жидкости шаровой объем, равный объему этого шарика (рис. 5, а). На выделенный мысленно объем воды, имеющий массу М, действуют две силы: сила тяжести Мд, направленная вниз, и сила давления окружающей воды F, направленная таким образом, что ее вертикальная составляющая уравновешивает силу тяжести, а горизонтальная сообщает выделенной массе воды М ускорение а. На
23
рис. 5, а показана сила F и ее составляющие —Мд и Ма, а также сила тяжести Мд (поверхность воды ориентирована так, что сила давления F направлена в каждой точке по нормали к поверхности). Сила давления воды F составляет с вертикалью такой угол а, что
tga = а/д.
Теперь заменим выделенный объем воды шариком с массой т, привязанным нитью ко дну и движущимся с тем же ускорением а (рис. 5, б). На этот шарик со стороны воды по-прежнему действует сила F с составляющими Ма и —Мду кроме того, на него действуют сила тяжести тд и сила натяжения нити Т, составляющая с вертикалью некоторый угол /?. Равнодействующая этих трех сил сообщает шарику ускорение а. Согласно II закону Ньютона:
F -h mg 4- Т = ma.	(1)
Проецируя это векторное равенство на горизонтальное и вертикальное направления, получаем систему из двух скалярных уравнений:
Ma — Tsinft = ma,	(2)
Mg — mg — Tcosp = 0.	(3)
Преобразуем систему уравнений (2), (3) к виду
Т sin/? = (М — m)a, Tcos/? = (M — m)g.
Разделив почленно первое равенство на второе, получим
tg /3 = а/д.
Таким образом, нить, соединяющая шарик с дном сосуда, направлена по нормали к поверхности жидкости.
Задача 20* (79-г)
Шарик, наполненный гелием, прикреплен нитью к полу вагона, движущегося с постоянной скоростью v = 100 км/ч по окружности радиусом R = 2,3 км. На какой угол айв какую сторону от вертикали отклоняется нить?
Решение. Шарик вместе со всем вагоном совершает равномерное движение по окружности радиусом R с центростремительным ускорением а = v2/R, направленным горизонтально к центру
24
икру ж пости, следовательно (см. решение задачи 19), нить отклонимся в ту же сторону на угол
Подставляя данные из условия задачи, получаем
tga« 3,4-10"2.
11,ля таких малых значений тангенса справедливо tga « а, так что о ~ 3,4 • 10“2 рад « 2°.
Итак, нить отклоняется в сторону центра окружности, по которой движется вагон, на угол а 2° от вертикали.
Задача 21 ** (87-г)
Две одинаковые батисферы плавают во взвешенном состоянии, первая на глубине 2d, вторая — на глубине cl (d = 1 км). В начальный момент времени первая батисфера сбрасывает балласт и начинает всплывать. Когда она всплывает до глубины d, вторая ьа гнсфера сбрасывает балласт и начинает всплывать. Первая ба-шсфера появилась на поверхности воды на Т = 10 с раньше второй. Известно, что первая батисфера вторую половину своего пути двигалась с практически постоянной скоростью vo = 1 м/с. Найдите выталкивающую силу. Масса батисферы без балласта равна т. (*илу сопротивления движению батисферы со стороны воды можно « читать прямо пропорциональной скорости батисферы.
Решение. Сила сопротивления движению батисферы F = —fcv, где v — ее скорость относительно воды; k— постоянный коэффициент (знак минус указывает на то, что сила сопротивления направлена против скорости). Уравнение движения батисферы без балласта:
та = — kv + Fa — mg.	(1)
При установившемся движении со скоростью vo уравнение (1) принимает вид
0 = -kvQ + Fa - mg.	(2)
'Теперь можно переписать уравнение (1) так:
та — —к(у — fo).	(3)
Применим это уравнение к движению второй батисферы. В скобках в правой части этого уравнения стоит относительная скорость
25
двух батисфер на втором этапе движения. Умножим равенство (3) на Д/ <С Т и учтем, что аД/ = Ди— изменение скорости второй батисферы, а (ио — и)Д/ = AS—изменение расстояния между батисферами. Их связь следует из уравнения (3):
тДи = к AS.	(4)
В момент всплытия первой батисферы вторая будет находиться на глубине S « vqT = 10 м— много меньше d. Это означает, что наше приближение достаточно точное и в этот момент вторая батисфера будет двигаться с установившейся скоростью vq (как и первая в конце первой половины пути).
Переходя в уравнении (4) от бесконечно малых к конечным изменениям скорости и расстояния (т. е. складывая уравнения (4) для всего времени подъема двух батисфер), получаем связь mvQ = kS = kvoTy или к = m/T. С учетом этого значения коэффициента к получаем из уравнения (2) ответ:
Fa - mg 4-	= mg ( 1 4-	« l,01m£.
Г \ д! J
Задача 22 * (75-г)
На горизонтальной плоскости лежит капля ртути объемом V. Сверху на каплю поставили брусок массой М так, что капля сильно сплющилась (рис. 6). Оцените расстояние Л между бруском и плоскостью, если коэффициент поверхностного натяжения ртути а и все поверхности абсолютно не смачиваются ртутью.
М
----2R ----
Рис. 6. К задаче 22.
Р е ш е н и е. На рис. 6 изображено вертикальное сечение, делящее каплю на две равные части. Найдем силу поверхностного натяжения F, с которой одна половинка капли притягивает другую:
F = а(4/? 4- 2тгг).
(1)
Сила внутреннего давления Fi, стремящаяся раздвинуть эти две половинки, равна
Fi = р(тгг2 + 2ЯЛ),	(2)
26
। ц<‘ p— давление внутри жидкости. Так как капля находится в равновесии, то F = Fi, и из (1), (2) имеем
0-(4Я + 2тгг)
№ + 2Rr
Ио условию задачи капля сильно сплющена, это значит, что R г. Пренебрегая малыми слагаемыми в числителе и знаменателе (3), получаем:
р = 2<r/h.
Но закону Паскаля давление внутри капли передается одинаково но всем направлениям. Это же давление удерживает брусок, следовательно:
p=Mg/S,	(4)
|де S — площадь соприкосновения бруска и капли. С хорошей степенью точности S = V/h> поэтому
р = Mgh/V.	(5)
Из (4) и (5) находим:
у Мд ’
В процессе решения мы пренебрегли изменением давления с высотой внутри капли. Это можно сделать, если pghS <С Мд> т. е. in Л/, где р и т —плотность и масса ртути.
Задача 23* (76-р)
Металлическая иголка диаметром d = 1 мм и длиной I d гмазана жиром так, что ее поверхность абсолютно не смачивает-<•>! водой. Найдите максимально возможную плотность р вещества иголки, при которой она не будет тонуть в воде. Коэффициент поверхностного натяжения воды <т принять равным 0,07Н/м, а ускорение свободного падения д = 10м/с2.
Рис. 7. К задаче 23.
27
Решение. Максимальное значение Fmax силы поверхностного натяжения Fa, действующей на иголку, будет достигаться в случае, показанном на рис. 7 (изображено поперечное сечение). При этом сила поверхностного натяжения будет определяться по формуле
— 2о7.	(1)
Иголка находится в равновесии, если Fa = тду следовательно, с учетом связи массы и плотности т — plnd2 /4^ получаем
Ртах — р?
и, подставляя сюда данные из условия задачи, находим
Ртах = 1,8 • Ю4КГ/М3.
В приведенном выше решении мы пренебрегли действием выталкивающей силы Архимеда. Оценим ее влияние на результат вычислений. Для этого найдем выталкивающую силу для конфигурации на рис. 7:
Fa = pgl(nd2/8 + dti),	(2)
где h — глубина погружения оси иголки. Для определения h найдем силы, действующие на заштрихованный объем воды V в горизонтальном направлении. Это сила поверхностного натяжения налево (F = crZ) и сила гидростатического давления направо. Среднее значение давления р = pgh/2, сила давления F = phi = pgh2l/2. В равновесии F — F, откуда
(3>
Подставляя выражение (3) в формулу (2), получаем
_	. (ird2 , [2а\
Сравним численно силы поверхностного натяжения (1) и выталкивающую силу (4) на единицу длины иголки:
Fmax// « 0,14 Н/м, FA/l « 0,044 Н/м.
Из этих вычислений видно, что силой Архимеда нельзя пренебречь. Окончательный результат с учетом (4):
Ртах —	лз
2,2 - 104^. Md
28
Задача 24 (78-г)
(/винцовый и алюминиевый шарики одинакового радиуса г святцы невесомой нерастяжимой нитью, длина которой намного больше размера шариков. Шарики опустили в сосуд с глицерином. После । гою они пришли в движение с нулевой начальной скоростью. Сила ^противления движению шариков пропорциональна их скорости, причем коэффициент пропорциональности одинаков для обоих шариков. Найдите силу натяжения нити при установившейся скорости цвижепия шариков. Плотности алюминия и свинца pi из-
вещение. Исследуем сначала движение одного свободного шарика, имеющего плотность р. Обозначим объем шарика V. На шарик действуют три силы: сила тяжести pVg, направленная вниз, вы-i.i л кивающая сила Fa, направленная вверх, и сила сопротивления
/сv, направленная против скорости шарика v. Применяя к движению шарика II закон Ньютона, получаем
рУа = pVg + FA - kv,
или в проекции на направление вниз:
pVа = pVд - Fa — kv,
где а — ускорение шарика в тот момент, когда его скорость равна v. Г’сли в начальный момент скорость шарика равна нулю, он начнет двигаться вниз с некоторым ускорением. По мере увеличения ско-I >< )сти сила сопротивления возрастает, ускорение будет уменьшаться и при достижении скорости
станет равным нулю. При этом равнодействующая всех сил, приложенных к шарику, обратится в ноль, и движение продолжится с постоянной скоростью v = vmax.
Нетрудно показать, что независимо от величины и направления начальной скорости установившееся движение шарика— это вертикальное падение с постоянной скоростью. Из формулы (1) следует, что чем больше плотность шарика, тем больше скорость его установившегося движения.
Рассмотрим теперь движение связанных шариков. Поскольку нить длинная, величина взаимодействия любого из шариков с глицерином не будет зависеть от присутствия второго шарика и будет такой же, как и в его отсутствие. Между собой шарики взаимодействуют только посредством натяжения нити.
29
При установившемся движении шарики будут двигаться вертикально вниз с некоторой скоростью v, нить будет расположена вертикально и натянута, внизу будет находиться более тяжелый свинцовый шарик.
Уравнения установившегося вертикального движения шариков:
piVg - kv - FA + T = О, P2Vд - kv - Fa-T — 0.
С учетом выражения для объема шарика V = |тгг3 находим т-
1 = ттгг 9--“--•
3	2
30
2. ДИНАМИКА ПРЯМОЛИНЕЙНОГО ДВИЖЕНИЯ
В этом разделе представлены задачи, в которых динамика прямолинейного движения может быть исследована на основе законов Ньютона. Так, II закон Ньютона устанавливает количественную связь внешнего воздействия на тело с изменением параметров ни движения:
II инерциальных системах отсчета сила F, действующая на те-|о п течение времени Д/, приводит к изменению количества движения (импульса) тела на величину
Ap = FAt	(1)
’ 1н,ссь введены обозначения: сила F — векторная величина, описывающая внешнее воздействие на изучаемый объект; импульс тела р — векторная величина, пропорциональная скорости тела, причем коэффициент пропорциональности имеет определенное значение для каждого тела и называется массой тела т. В классической механике масса тела является аддитивной величиной— складывается из маге отдельных его частей.
Используя определение импульса и ускорения, можно перепи-< ать закон (1) в иной форме:
F = та.	(2)
(Мфатим внимание на то, что сила в левой части равенства (2) — результат взаимодействия данного тела с окружающими телами:
F = EF> i
В приведенных далее задачах этими силами являются чаще всего гила тяжести и силы, возникающие в результате соприкосновения изучаемого объекта с опорой, подвесом и т. п. Из того экспериментального факта, что все тела в поле тяготения падают вниз с одинаковым ускорением g, следует, что сила тяжести, действующая на тело в поле тяготения, пропорциональна его массе: FT = mg.
При движении тел в некоторой среде или при взаимном смещении соприкасающихся поверхностей возникают силы, препятствующие движению. Их называют силами трения. Учет сил трения при решении задач производится в рамках какой-либо модели. Наиболее распространена модель сухого трения. Эта модель
31
наилучшим образом работает при рассмотрении процессов, в которых происходит взаимное смещение соприкасающихся сухих поверхностей— скольжение. Возникающую при этом силу сопротивления движению (силу трения скольжения) считают пропорциональной силе нормального давления между поверхностями и не зависящей от их взаимной скорости. Коэффициент пропорциональности называют коэффициентом трения скольжения. Он зависит только от материала и степени шероховатости соприкасающихся поверхностей. В ряде задач разобраны случаи, когда наличие силы трения приводит не к торможению, а к ускорению тела. В задаче 73 и в Добавлении II рассматривается влияние силы трения на очень быстрые изменения направления движения тела (например, при ударе о преграду).
Другая модель силы сопротивления движению— вязкое трение при движении в жидкости или газе. В этом случае сила сопротивления считается пропорциональной скорости (или квадрату скорости) тела относительно окружающей среды. Коэффициент пропорциональности зависит от размеров, формы и материала тела, от состава и плотности среды.
При решении задач с использованием II закона Ньютона необходимо установить наличие взаимодействия (сил) между телами, выбрать систему отсчета (систему координат), записать II закон Ньютона для каждого из взаимодействующих тел в векторной форме или в проекциях на оси системы координат. В большинстве задач число неизвестных (ускорений или сил) будет превосходить число выписанных уравнений. Наиболее сложной частью решения является установление связей между различными величинами (ускорениями и силами).
Связь между силами воздействия тел друг на друга устанавливает III закон Ньютона:
Силы взаимодействия двух тел равны по величине и противоположны по направлению.
Заметим, что эти силы приложены к разным телам. Если тело 1 действует на тело 2 с силой Fi-2, а тело 2 на тело 1 с силой F2-1, то
Fi-2 — —F2-1.	(3)
Из равенства (3) следует, что силы Fi-2 и F2-1 направлены по параллельным прямым, но не обязательно по одной прямой (см. Добавление III).
Между координатами (скоростями, ускорениями) взаимодействующих тел могут существовать механические связи, например при движении тел, связанных нитью. В большинстве задач нить
32
предполагается невесомой и нерастяжимой. Нерастяжимость нити математически выражается в неизменности ее длины в процессе и пижонил. Постоянство длины нити — это математическое соотношение (связь) между координатами точек, соединенных такой питью. Связь между координатами приводит к аналогичным соотношениям между скоростями и ускорениями.
Невесомость (безмассовость) нити приводит к обращению в ноль • уммы сил, действующих на эту нить (в противном случае ее ускорение стало бы сколь угодно большим). По III закону Ньютона это о тачает равенство сил, с которыми нить действует на связанные ею тела.
* * *
Задача 25 (80-г)
Как спустить с крыши высотой h = 16 м груз массой т = 45 кг < помощью веревки, у которой сопротивление Т на разрыв равно 100 II? Скорость тела в момент удара о землю не должна превышать шипения vmax = 7 м/с. Длина веревки немного превосходит высоту ш>ма.
Решение. Сила тяжести груза тд = 441Н превосходит силу • '<'противления нити на разрыв Т = 400 Н, а при свободном падении । руз достигает поверхности земли со скоростью v = \/2д71 = 18 м/с, превышающей vmax = 7м/с.
Исли привязанный к веревке груз спускать не с постоянной скоростью, а с некоторым ускорением а, то сила натяжения нити F ьудет меньше силы тяжести тд. Применяя к движению груза второй закон Ньютона, имеем:
тд — F — та или
F ' т(д - а).
Выполнение требования F <Т приводит к неравенству
-	т(д - а) <Т.
Зтому неравенству эквивалентно следующее:
а > д — Т/т = 0,9м/с2.
33
Итак, если груз спускать с ускорением а > 0,9 м/с* 2, то сила натяжения веревки не будет превышать Т = 400 Н. При этом скорость груза в момент удара о землю не будет превышать значения
v = V2ah = 5,4 м/с,
которое меньше г>тах = 7 м/с.
Задача 26* (74-г)
Змея, лежащая на полу, начинает подниматься с вертикальной скоростью v. Найдите силу давления на пол, если змея однородная, масса ее равна т, длина I.
Решение. Длина части змеи, поднявшейся за время /, равна х = vt. Эта часть змеи имеет скорость v, а остальная часть длиной I — х покоится. Из этого следует, что скорость центра масс
и = vx/l = v2t/l.
Видим, что эта скорость линейно зависит от времени, следовательно, ускорение центра масс направлено вверх и равно а = v2/l.
Сумма сил тяжести mg и силы реакции опоры N должна быть равна та. Равенство этих величин в проекции на вертикальное направление имеет вид
N — тд = та — mv2/I.
По III закону Ньютона сила давления змеи на пол (Р) по величине равна силе реакции опоры N:
Р = N = тд 4- mv2/I.
Задача 27 (79-г)
Мячик падает в воздухе с очень большой высоты и абсолютно упруго ударяется о землю. Найдите ускорение мячика сразу после удара, считая силу сопротивления, действующую на мячик со стороны воздуха, пропорциональной его скорости.
Р е ш е н и е. На падающий мячик массой т действуют силы тяжести mg и сопротивления воздуха F = —kv} где v — скорость мяча, к — коэффициент пропорциональности. Под действием результирующей этих сил мячик приобретает ускорение
mg + F	к
а =------= g----v.
т	т
(1)
34
Ускорение максимально в начальный момент и равно g. С ростом скорости, как следует из уравнения (1), ускорение уменьшается и, в конце концов (мячик падает с очень большой высоты!), становится практически равным нулю. При этом скорость мячика постоянна и равна т
v0 = yg,
и на него действует постоянная по величине сила сопротивления воздуха
Fo = -fcvo = -mg.
При абсолютно упругом ударе о землю мячик меняет направление движения на противоположное и приобретает скорость Vi = vq. Следовательно, на мячик со стороны воздуха действует сила сопротивления
Fi = —kvi = fcvo == mg.
К роме того, на него действует сила тяжести mg. В результате мячик получает ускорение
mg + Fi .--^—-28.
Задача 28 (81-р)
В системе, изображенной на рис. 8, а, тело массой т привязано невесомой и нерастяжимой нитью к потолку кабины. Кабина двигается с ускорением а, направленным под углом а к горизонту. Найдите силу натяжения нити Т и угол отклонения подвеса от вертикали (3.
Решение. Тело двигается под действием двух сил — искомой гплы натяжения нити Т, направленной вдоль нити, т. е. под углом // к вертикали, и силы тяжести mg. Векторная сумма этих сил по II чакону Ньютона должна быть равна та.
Выбирая направления осей координат, как показано на рис. 8, б', получаем равенства: в векторной форме
ma = Т + mg
а	б
35
и в проекциях на оси координат
Ох:	та cos а = Т sin /3,	(1)
Оу:	masina = Teas/3 — тд.	(2)
Решим систему уравнений (1), (2) относительно Т и /?. Перенеся тд в уравнении (2) в левую часть, имеем:
та sin а + тд = Т cos /3.	(3)
Разделив почленно (1) на (3), получим
Л a cos а tg 13 = —:--— •
a sin а + д
Для определения Т возведем равенства (1), (3) в квадрат и сложим почленно. После несложных преобразований находим
Т = т\/а2 + (J2 4- 2а</ sin а.
Эти же результаты можно получить непосредственно из рис. 8, б, используя теорему косинусов.
Задача 29 (74-г)
Снаряд массой т вылетает из пушки со скоростью v под углом а к горизонту. Сила сопротивления движению снаряда в воздухе пропорциональна скорости: F = —fcv. Какова должна быть ширина I очень глубокой пропасти (рис. 9), чтобы снаряд достиг дна, если пушка стоит на краю пропасти (при попадании в стенку снаряд взрывается и до дна ямы не долетает)?
v X
—Л.— х	—
I
[	[	Рис. 9. К задаче 29.
Решение. За время At импульс снаряда изменяется на mAv = FAZ. Проецируя это равенство на горизонтальное направление, получаем
mhvx = —kvx№ = —к&х.
Уменьшение горизонтальной составляющей скорости до нуля соответствует перемещению вдоль горизонтальной оси на х = mvx/ky откуда с учетом равенства vx = vcosa получаем окончательный ответ: I > mvcosa/k.
36
Задача 30 (80-г)
'Голо массой т = 10 кг тянут по горизонтальной плоскости, прикладывая силу |F| = 50 Н, направленную под углом а = 30° к горизонту. Ускорение движения а = 3,5 м/с2. Найдите коэффициент iрения р между телом и плоскостью.
Рис. 10. К задачам 30, 31.
Решение. Все силы, действующие на тело, показаны на рис. 10. ' )го силы: тяжести mg, внешняя F, нормальной реакции опоры Q и । рения скольжения FT. Величины двух последних сил связаны друг < другом соотношением FT = fiQ. По II закону Ньютона
mg + F + Q + FT = ma.
Проецируя это векторное равенство на горизонтальное и вертикальное» направления, получаем два скалярных уравнения
F cos а — (1Q = та,
(1)
Fsina + Q — тд = 0.
(2)
Исключая из системы уравнений (1) и (2) неизвестную величину Q,
находим:
F cos а — та н =---------—-------= 0,11.
тд — F sin а
(3)
Величина, стоящая в числителе выражения (3), всегда положительна, как легко увидеть из уравнения (1). Величина, стоящая в шаменателе, обращается в ноль при Fsina = тд и отрицательна при Fsina > тд. Из уравнения (2) видно, что эти случаи соответствуют обращению в ноль силы реакции опоры, что означает отрыв тела от плоскости.
Задача 31 (76-р)
Тело массой т стоит на горизонтальной плоскости. К телу приложена сила F под углом а к горизонту. При каких значениях коэффициента трения /2 между телом и плоскостью оно останется неподвижным?
37
Решение. На тело действуют силы (см. рис. 10 — задача 30): тяжести mg, реакции опоры Q, трения FT и внешняя F. По условию задачи тело находится в покое, поэтому сумма сил, действующих на него, равна нулю:
F + mg + FT + Q = 0.	(1)
При этом сила трения покоя
|FT| < nN,	(2)
где N — величина силы нормального давления. (По III закону Ньютона N = |Q|.)
Спроецируем уравнение (1) на вертикальное и горизонтальное направления:
Q 4- F sin о — тд = 0,	(3)
Fcosa - Гт = 0.	(4)
Решая уравнения (3) и (4) относительно Q и FT и подставляя результат в (2), получаем неравенство
F cos а < }i(—F sin а + тд).	(5)
Если а > 0, то правая часть этого неравенства может стать отрицательной при sin а > mg/F. Но при этом из уравнения (3) следует Q < 0, что противоречит условию задачи. Следовательно, правая часть неравенства (5) положительна, и можно найти допустимые значения /л:
Feos а
М > ------—•	(6)
” тд — F sin а
Заметим, что при а < 0 и /J > |ctg а| неравенство (6) выполняется при сколь угодно большой силе F.
Задача 32 (76-р)
Тело массой т прижимают к потолку с силой F, направленной под углом а к горизонту. При каких значениях коэффициента трения ц между телом и потолком тело останется неподвижным?
Решение. Действующие на тело силы изображены на рис. И (Q — сила реакции потолка, FT — сила трения покоя). Поскольку тело предполагается неподвижным, то
F + mg + Q + FT = 0.
38
При itom обязательно
|FT| < pN.	(1)
Ьпч ь .V—сила нормального давления.
Q
Рис. 11. К задаче 32.
('читая для определенности, что 0 < а < тг/2, находим
Fsina — тд — Q = 0, Feos a— FT = 0.	(2)
II н их равенствах величины F, Q и FT положительные. Решая уравнения (2) относительно Q и FT, получаем
Q = Fsina — mg, FT = Fcosa.	(3)
I Ьщ( тавив выражения (3) для |FT| и N = Q в неравенство (1), имеем
Feos а < p(Fsin а — тд).
1*н ।решим это неравенство относительно с учетом того, что
F sin а > тд
(и противном случае тело не прижимается к потолку):
Feos а и > ----------.
“ F sin а - тд
Задача 33 (86-г)
1 [ гобы вытащить пробку, застрявшую в горлышке термоса, в нее in 1.1 кают тонкое шило. Под каким углом 0 к оси термоса можно hi икать шило, не опасаясь, что пробка провалится внутрь термоса? Коэффициент трения пробки о стенки горлышка р = 0,5.
Решение. Пусть, втыкая шило, мы действуем на пробку с • иной F. Эту силу можно разложить на две составляющие: вдоль иен горлышка, равную Feos0, и перпендикулярную к ней, равную /•'жп 0. Если при этом пробка остается в равновесии, то Feos 0 = FT, । до FT— сила трения. Ясно, что FT < /iN, где N — сила нормального и,пиления пробки на стенки горлышка. Заметим, что N > Fsin0, I пк как пробка находится в сжатом состоянии и давит на горлышко путылки даже при F = 0. Условие, гарантирующее, что пробка не провалится, есть Fcos0 < pF sin 0, т. е. 0 > arcctg 0,5 « 63°.
39
Задача 34 (88-г)
Известно, что сила сухого трения между двумя телами практически не зависит от площади соприкосновения этих тел. Однако чем глубже вставлена пробка в горлышко бутылки, тем труднее ее вынуть. Почему?
Решение. Действительно, сила трения прямо пропорциональна силе нормального давления У:
FT = pN,
где /2 —коэффициент трения между соприкасающимися поверхностями. В рассматриваемой ситуации сила нормального давления прямо пропорциональна площади соприкосновения между поверхностями пробки и горлышка бутылки. Поэтому чем глубже вставлена пробка в горлышко бутылки, тем больше сила трения между соприкасающимися поверхностями и тем крепче держится пробка в горлышке бутылки.
Задача 35 (74-р)
Поезд, подъезжая к станции со скоростью v = 60 км/ч, начинает тормозить. За какое минимальное время можно остановить поезд, чтобы лежащие на полках чемоданы не сдвинулись с места? Считать, что при торможении поезд движется с постоянным ускорением, а коэффициент трения (л чемодана о полку равен 0,2.
Решение. Чемоданы не соскальзывают, если а < цд. Таким образом, максимально возможное ускорение атах = ЩЬ Ему отвечает минимально необходимое для остановки время
.	V	V Л А
^min —	—	— 8>4 С.
атах №9
Задача 36 (74-р)
Два сцепленных вагона массой т = Ют каждый катятся по рельсам. Коэффициент трения скольжения между колесами вагонов и рельсами ц = 0,3, трением качения можно пренебречь. Неожиданно оси заднего вагона заклинивает так, что его колеса перестают вращаться. Найдите ускорение вагонов и силу, действующую на сцепку (устройство, соединяющее вагоны) после заклинивания осей. Сколько времени пройдет до остановки вагонов, если их начальная скорость v = 10 м/с?
Решение. Ускорение вагонов после заклинивания осей заднего вагона а = цтд/2 = 1,5 м/с2. Сила, действующая на сцепку, F = та = 1,5 Н. Вагоны остановятся через t = v/a = 6,7 с.
40
Задача 37 (78-г)
Паровоз массой М тянет вагон массой т с постоянной скоро-• । ью по прямому горизонтальному пути. В некоторый момент вагон • •грывается и проходит путь I до остановки. Какой путь пройдет па-ривоз от момента отрыва до момента остановки вагона? Силу тяги и провоза и силу сопротивления движению считать постоянными.
а	б
2_| |-TTr~LnJ
1 п-м
Рис. 12. К задаче 37.
Решение. В системе паровоз — вагон до момента отрыва питона действуют следующие силы (рис. 12, а): сила тяги F, дей-• । кующая на паровоз (это не что иное как, сила трения покоя или । кольжения, возникающая между колесами паровоза и рельсами); • ила Т, с которой действует вагон на паровоз (по III закону Нью-ин Iа паровоз действует на вагон с силой — Т), и, наконец, Q — сила । рения качения, действующая на колеса вагона со стороны рель-«он. (Мы пренебрегли здесь силами сопротивления, действующими • in тороны воздуха на паровоз и вагон, которые зависят от скорости ипижепия, считая их малыми в сравнении с силами F и Q.) После • нрыва вагона действие сил Т и —Т прекращается, а силы F и Q вплоть до остановки вагона остаются прежними (рис. 12, б).
Применяя к движению сцепленных паровоза и вагона II закон Ньютона, получаем
F + Q = 0.	(1)
Уравнения движения расцепленных паровоза и вагона имеют вид
(4)
F = Мап,	(2)
Q = тав,	(3)
। дг а„ и ав— ускорения паровоза и вагона после отрыва вагона.
Из уравнений (1) — (3) находим отношение ускорений: ап _ т ав~~ М'
()бозпачим скорость паровоза и вагона до разрыва связи между ними через v. После отрыва вагон движется равнозамедленно и оста-। in вливается спустя t = v/aB. За время t он проходит путь
aBt2 v2
I — vi---— = -—.
2 2ав
(5)
41
Паровоз за то же самое время, двигаясь равноускоренно, проходит
путь
r ant2 v2 / ап А
L = vt+^- = — 1 2 + — 1.
2 2ав у (Zq у
(6)
Из (4)—(6) находим:
L — I (2 + —)
\ М /
Задача 38* (87-г)
На гладком горизонтальном столе лежат два одинаковых шарика массой т каждый, скрепленные нерастяжимой невесомой натянутой нитью. На один из шариков в течение очень малого времени At действует большая сила F, направленная горизонтально под углом 0 > тг/2 к нити. Определите скорости шариков сразу после окончания действия силы.
Решение. За время действия силы координаты шариков практически не меняются, а нить натягивается. Обозначив силу натяжения нити Т, ускорения шариков вдоль нити ощ и ац2, ускорения поперек нити an и ai2, напишем уравнения движения шариков в проекциях на эти направления:
тац2 = Т,	(1)
ma±2 = 0,	(2)
ma||i = F cos(tt — 0) — Т,	(3)
тац = Fsm(7r-0).
(4)
Так как нить нерастяжима, проекции скоростей (а следовательно, и ускорений) шариков на направление вдоль нити всегда одинаковы, т. е. ац2 = ащ = ац. Учитывая это и складывая уравнения (1) и (3) почленно, получаем
F all = 2^cos(’r“^-
Следовательно, продольные скорости шариков после удара
F
*|| = 2m C0S^ “	Л/’
а поперечная скорость первого шарика
F г>1 = — sin(7r — 0) At.
771
42
11и1|цпя скорость первого шарика получается равной
v =	+ v1? = ——\Zcos2 О 4- 4 sin2 0.
V II 1 2m
' Iплача 39* (75-г)
(  поверхности Земли стартует ракета. Двигатель ракеты рабо-нн'1’ к течение времени to- Топливо поступает в камеру сгорания с •»• •• । ин иной скоростью. Скорость истечения газов из сопла постоян-<»•! и равна v. Максимальное ускорение, которое приобретает ракета •и время работы двигателя, атах. Определите отношение веса то-н num к общему весу ракеты в момент старта; to = 50 с, v = 5 км/с, ♦ •«11•• *	20 .
Решение. Обозначим через Mq массу ракеты без топлива, че-|и» ш массу топлива в начальный момент и через M(t) —полную мт г у ракеты в момент t после старта. Пусть за время dt сгорает чи» га топлива dm. В результате ракета получает дополнительную ..рость du. Изменение количества движения системы за время dt I in нно импульсу силы Fdt:
М (t)du + dm v = Fdt.	(1)
hire в F = M(t)g —сила тяжести. Проецируя уравнение (1) на вер-। анальное направление, после деления обеих частей равенства на dt получаем
.ч du dm ,	.
-~dfv = ~Mms-	<2>
Но условию задачи величина dm/dt, постоянна и равна отношению мнгсы т загруженного в ракету топлива ко времени to его выгорании. Ускорение движения ракеты a(t) = du/dt. Разрешая равенство (7) относительно a(t), находим
mu ““’“мод-9-
Максимальное значение ускорения атах отвечает минимальному значению величины M(t). Очевидно, что = М(0) и
mv
(3)
Отсюда
М(0) _ у
т (атах+р)<о
43
Таким образом, искомая величина т/(т + М(0)) равна т _	1	_ 2
М(0) ” 14- v/[(amax 4-^)/0] ~ 3*
В процессе решения мы считали, что д = const, предполагая, что за время работы двигателя ракета не может подняться так высоко, что величина д изменится заметным образом. Оценим высоту подъема ракеты. Для этого примем, что движение осуществлялось с постоянным ускорением а = птах. При этом ракета за время to поднимется на высоту
Атах =	(4)
Подставляя в (4) данные из условия задачи, получаем hmax =250 км. Реальная высота подъема h < Лтах, т. е. наше предположение о постоянстве д в процессе движения ракеты верно с точностью 8 %.
Задача 40 (77-р)
На гладкой горизонтальной поверхности лежат два бруска с массами mi и m2, соединенные невесомой нерастяжимой нитью (рис. 13). Внешняя сила F приложена к телу mi и направлена горизонтально. Найдите установившееся ускорение системы и силу натяжения нити.
Решение. Силы, действующие на тела в вертикальном направлении (силы тяжести и силы реакции опоры), уравновешивают друг друга. Поскольку, по условию задачи, плоскость гладкая, то силы трения отсутствуют. В горизонтальном направлении на тело mi действуют только внешняя сила F и сила натяжения Т, направленная вдоль нити. Из III закона Ньютона с учетом невесомости нити следует, что нить действует на тело m2 с равной по величине, но противоположно направленной силой. По II закону Ньютона тело m2 двигается в горизонтальном направлении с некоторым ускорением а:
т2а = Т.	(1)
т2
—
F
Рис. 13. К задаче 40.
Так как нить нерастяжима, то тело mi двигается в том же направлении с тем же ускорением:
гща = F — Т.
(2)
44
r« nnui систему двух уравнений (1) и (2) относительно неизвестных и и /’, получаем ответ
mi + m2 ’	mi + тп2
' Задача 41	(92-р)
На гладком горизонтальном столе покоятся два груза с массами m и 2m, связанные легкой нерастяжимой нитью. В некоторый момент к грузу массой тп приложили силу F, а к грузу массой 2m приложили силу —2F (рис. 14). Найдите силу натяжения нити.
F
2F
2m —
Рис. 14. К задаче 41.
Решение. Задача аналогична задаче 40. Не будем приво-III । г подробного решения, а укажем, что, записав уравнения дви-•и си и я для каждого тела и исключив из них неизвестную величину ускорение системы тел, получим одно уравнение для силы на-1Н/К<‘ния нити. Решив его, найдем: Т = |F.
Задача 42* (92-р)
На горизонтальной поверхности лежат два бруска с массами mi и пи, соединенные невесомой нерастяжимой нитью. Внешняя сила F приложена горизонтально. Определите силу натяжения нити, если ho >ффициенты трения скольжения между поверхностью и брусками рпппы pi и Д2 соответственно.
Решение. Если величина F силы F не превосходит максималь-ни(| силы трения покоя первого бруска FTi = mi/zi<7, то брусок покоится даже при ненатянутой нити. Если FTi < F < FTi + FT2, то си-। |сма покоится и, следовательно, сила натяжения нити Т = F — FTi.
При выполнении условия FTi + Ft2 < F сила Т превзойдет силу । рения покоя второго бруска, система будет двигаться с постоянным ускорением
а _ -F	+ Ш2М2)
ТП1 +Ш2
II
Т = т2а + Л1 = ——[F + mig(fJ,2 -Д1)].
mi + m2
Задача 43 (87-р)
Па правый конец нерастяжимой веревки длиной /, находящейся ио идеально гладком горизонтальном столе, действует в горизон-i ильном направлении вдоль веревки сила F. Найдите натяжение пгревки на расстоянии х от ее левого конца.
45
Решение. В силу нерастяжимости веревки все ее точки движутся под действием внешней силы F с одинаковым ускорением а = F/М, где М — масса веревки. Левая часть веревки длиной х имеет массу т(х) = Мх/l. Она движется с ускорением а под действием искомой силы натяжения Т(х). Согласно II закону Ньютона Т(х) = m(x)a — Fx/l.
Задача 44 (87-р)
На концы находящейся на идеально гладком горизонтальном столе нерастяжимой веревки действуют силы Fi и F2, направленные в плоскости стола в противоположные стороны вдоль веревки, Найдите зависимость натяжения веревки от расстояния до ее конца, Длина веревки /.
Решение. Так как веревка нерастяжима, то все ее точки движутся в направлении действия силы Fi с одинаковым ускорением
a=(Fl-F2)/M)	(1)
где М —масса веревки.
Будем отсчитывать расстояние вдоль веревки от того конца, к которому приложена сила F\. Рассмотрим движение куска веревки, ограниченного точками с координатами 0 и я соответственно. Масса куска т(х) определяется выражением
т(я) = Мх/1.	(2)
Вдоль линии движения на него действуют две силы: в одну сторону сила Fi, а в другую — сила натяжения Т(х). Уравнение его движения
Fi — Т(х) = тп(х) а,	(3)
Из уравнений (1) — (3) находим
Т(х) = Fi - (Fx - F2)x/l.
Задача 45 (87-р)
Тело массой mi, расположенное на горизонтальной плоскости, тянут за привязанную к нему веревку, действуя с силой Fi, направленной параллельно плоскости. Масса веревки т2у длина I. Найдите натяжение веревки в зависимости от расстояния до тела. Веревка нерастяжима, трения нет.
Решение. Так как веревка нерастяжима, то все ее точки и тело движутся в направлении силы Fi с одинаковым ускорением,
46
мн|м‘дсляемым формулой а = Fi/(mi + m2). Со стороны веревки на Н’Лн действует сила
Fz = mia =
/л 1 Fi
mi + т2
(1)
I ^гласно III закону Ньютона такая же по величине, но направлении н противоположно сила действует со стороны тела на веревку. В । м • |ультате к концам веревки оказываются приложенными силы Fi н /*2, направленные в противоположные стороны, и решение сводит-• н к нахождению силы Т(х) по силам Fi и F2i как и в предыдущей шдаче:
ТМ = F, - (F, - F,)i = '"‘CT^F,.
I mi + 7712
Задача 46 (77-p)
Два тела с массами mi и ?7i2 связаны невесомой нерастяжимой нитью, перекинутой через невесомый блок, так, что тело m2 виги т, а тело mi лежит на горизонтальной поверхности. К телу mi (рис. 15, а) приложена в горизонтальном направлении сила F. Бай-пите ускорения тел и силу натяжения нити. Трение отсутствует.
Рис. 15. К задаче 46.
Решение. Так как нить нерастяжима, то оба тела движут-сл с одинаковыми по величине ускорениями а, первое — в горизонтальном направлении, второе — в вертикальном. Все силы, приложенные к телам, показаны на рис. 15, б). Сила тяжести Pi и сила реакции R уравновешивают друг друга. Первое тело приобретает ускорение а под действием разности сил F и натяжения нити Т. Для определенности будем считать, что направление ускорения совпадает с направлением силы F. Согласно II закону Ньютона
F - Т = mia.
(1)
47
Так как трение в блоке отсутствует и нить невесома, то на тело m2 в вертикальном направлении вверх действует та же по величине сила натяжения нити Г, что и на тело mi в горизонтальном направлении. Вниз на тело m2 действует сила тяжести Р2 = над где д — ускорение свободного падения. Под действием разности этих сил Т — Р2 второе тело и получает ускорение а. По II закону Ньютона
Т — т2д = гп2а.	(2)
Решая систему из двух уравнений (1) и (2) относительно неизвестных а и Т, находим
F - т2д
CL —	,
mi + m2
.l — tii,2-----*
mi + m2
Сделанное нами предположение о том, что направление ускорения а совпадает с направлением силы F (а > 0), справедливо, если F > rri2g. В противном случае ускорение а имеет обратное направление. Если F = гп2ду то система находится в покое или оба тела движутся равномерно с одинаковыми по величине скоростями.
Задача 47 (74-р)
Какой груз (рис. 16, а) будет опускаться, а какой подниматься? Массы грузов одинаковы, массами нитей и блоков пренебречь, нити нерастяжимы, трение в осях блоков отсутствует. Найдите ускорения грузов, если масса каждого груза равна т.
Рис. 16. К задаче 47.
Р е ш е н и е. Из условий невесомости нити и отсутствия трения в блоках следует постоянство силы натяжения по всей длине нити. Из условия нерастяжимости нити получаем связь ускорений двух тел:
2ai = -<i2.	(1)
48
Inn к минус в (1) указывает, что ускорения тел направлены в про-। нппположные стороны. Учитывая все силы, действующие на грузы (риг. 16, б), и записывая уравнения их движения вдоль вертикаль-н»di оси (за положительное выбрано направление вниз), получаем
тд — Т = таъ,
(2)
тд — 2Т = та\.
(3)
Исключив величину Т из системы уравнений (2), (3) и учтя связь Vi корений (1), находим сц = — а2 = |т#.
Задача 48* (80-г)
Три груза, массы которых mi = m, m2 = 2m, m3 = m, подвешены на невесомых нерастяжимых нитях с помощью системы неве-• пмых блоков (рис. 17, а). Начальные скорости грузов равны нулю. Найдите их ускорения после обрыва нити в точке А. Трение между нитями и блоками отсутствует.
а
Рис. 17. К задаче 48.
Решение. После обрыва нити все грузы и левый блок приходит в движение. Нить, соединяющая грузы mi, m3, будет при этом пес время натянута. Так как трение между этой нитью и блоками < псутствует и нить невесома, то сила натяжения нити Т будет в лю-<К1М ее сечении одна и та же. Все силы, действующие на грузы и левый блок во время движения, показаны на рис. 17, б).
Применим II закон Ньютона к движению левого блока. Так как, по условию задачи, блок невесом (его масса М = 0), то
2Т + Т1 = 0.
() тсюда
Ti = 2Т.
(1)
49
Обозначим ускорения грузов с массами mi, m2, m3 через ai, аз соответственно. Принимая направление вниз за положительное и выписывая уравнение движения для каждого груза, получаем еле-дующую систему уравнений:
тд + Т = mai, 2тд — Т\ — 2та2) тд — Т — таз •
(2)
(3)
(4)
Система четырех уравнений (1)—(4) содержит пять неизвестных. Чтобы найти недостающее уравнение, обратим внимание на то, что нити, по условию задачи, нерастяжимы.
Двигаясь с постоянными ускорениями, грузы за произвольное время t с начала движения проходят пути
а?! = ах/2/2, х2 = «2^/2, хз = аз^2/2.
(5)
(6) (П
Так как нить, соединяющая левый блок со вторым грузом, нерастяжима, то за время t этот блок проходит тот же путь Х2, что и второй груз. Вследствие этого за время t расстояние между первым грузом и левым блоком изменится на величину
Д/1 = Ху — •
За это же время расстояние между блоками меняется на величину
Д/2 = ®2,
а расстояние между правым блоком и третьим грузом — на величину
Д/з — #з-
В силу нерастяжимости нити, соединяющей первый и третий грузы, сумма всех Д/» (i = 1, 2, 3) должна быть равна нулю, т. е.
Д/1 + Д/2 И" Д/з —	4" 2д?2 —	— 0.
Отсюда, учитывая равенства (5)—(7), получаем уравнение
аз + 2аг — Д1 = 0,
(8)
связывающее между собой искомые величины.
50
Решая систему из пяти уравнений (1)—(4), (8) относительно не-н « гных ai, а2, оз, находим
3	1
01 = о2 = аз — -д.
’ 1пдача 49* (76-р)
Два тела с массами mi и т2 подвешены на невесомых нерастя-•пнмых нитях с помощью системы блоков (рис. 18, а). Найдите уско-|н пи» тел и направления вращения блоков. Считать, что нить двига-• и н по блокам без проскальзывания. В начальный момент скорости hoi и блоков равны нулю. Массой блоков и трением пренебречь.
Рис. 18. К задаче 49.
Р е ш е н и е. В силу отсутствия у нити массы ее натяжение во пгех точках одинаково. На рис. 18, б показаны все силы, приложенные к движущимся блокам и телам. Записывая уравнения движения /•..ня блоков 1, 2 и тел mi, m2 и принимая во внимание, что масса Uцоков равна нулю, получаем
Т-2Т=0,	(1)
2Т-Т1 = 0,	(2)
т^д — Т=	(3)
т2д - Т — т2а2.	(4)
11ч равенства (1) находим, что Т = 0, откуда с учетом равенства (2) получаем, что и = 0. В соответствии с этим равенства (3), (4) принимают вид
тп^д = miai,	(5)
т2д = т2а2.	(6)
Из (5), (6) следует, что al = а2 = д, т. е. оба тела движутся вниз с ускорением свободного падения д.
51
Найдем теперь направления вращения блоков. Обозначим длины нити от тела т\ до блока 1 через Zoi, от блока 1 до блока 2 через /12, от блока 2 до блока 3 через /23 и от блока 3 до блока 1 через /31. Пусть полная длина нити равна I. Так как нить нерастяжима, то в любой момент времени имеет место равенство
/ = /01 + ^12 + /23 + /31 •	(7)
Очевидно, что в силу нерастяжимости нити наряду с равенством (7) должно иметь место и такое равенство:
/12+/31 = Ьз-	(8)
Заменяя в (7) сумму /12 + /31 на правую часть (8), получаем
/ = /01 + 2/гз<	(9)
В более поздние моменты времени вместо (9) будем иметь
/ = ^1+2/'3.	(10)
Вычитая почленно из равенства (10) равенство (9), находим
0 = Д/о1+2Д/2з,	(П)
т. е. Д/oi и Д/23 имеют разные знаки. Поскольку расстояние /23 увеличивается (блок 2 движется вниз с ускорением д), то Д/23 > 0. В соответствии с этим и формулой (11) Д/oi < 0, т. е. расстояние между блоком 1 и телом mi сокращается (это возможно, если блок 1 движется вниз с ускорением а > д). Отсюда ясно, что блок 1 вращается по часовой стрелке.
Так как расстояние /31 растет и нить движется по блоку 3 без проскальзывания, то ясно, что блок 3 вращается против часовой стрелки.
Из того, что любой участок нити /23 перемещается вверх, а блок 2 — вниз и нет проскальзывания между нитью и блоком, следует, что блок 2 вращается в том же направлении, что и блок 3.
Задача 50 (75-р)
В системе, изображенной на рис. 19, трение в блоках и между любыми поверхностями отсутствует. Если грузику массой т позволить двигаться, то за какое время он достигнет подставки? Начальная скорость грузика равна нулю, начальное расстояние от грузика до подставки Л, нить невесомая и нерастяжимая.
52
Рис. 19. К задаче 50.
Решение. На систему подставка—груз в горизонтальном направлении действуют силы натяжения двух нитей (2Т, где Т—сила ипглжения одной нити). Эти силы сообщают системе горизонтальное ускорение аг, и по II закону Ньютона
(M + m)ar = 2T.	(1)
На груз в вертикальном направлении действуют сила тяжести и । ила натяжения нити, причем
тд — Т = таъ,	(2)
им’ нв— вертикальное ускорение груза. Нетрудно заметить, что
ов —— 2cip.	(3)
(!пязь ускорений (3) вытекает из того, что смещение подставки на расстояние Дж приводит к перемещению груза по вертикали на расстояние 2Дж.
Решая систему уравнений (1) — (3), находим ускорение груза по вертикали
_ 4m °в $5т + М
При равноускоренном движении и нулевой начальной скорости груз пройдет путь Л за время
/IE- М(5т + М)
V ав у 2тд
Задача 51* (78-г)
Через неподвижный блок перекинута длинная нерастяжимая веревка. На одном конце веревки висит груз массой т = 10 кг, за другой конец хватается обезьяна массой М = 30 кг. С каким ускорением относительно веревки должна двигаться обезьяна, чтобы оставаться на одной и той же высоте над поверхностью Земли? Массой веревки и трением в блоке пренебречь.
53
Р е ш е н и е. В силу невесомости веревки сила натяжения Т в любом ее сечении одна и та же. На обезьяну действуют: сила натяжения веревки Т — вверх, сила тяжести Мд — вниз. Под действием этих сил обезьяна должна покоиться относительно земли, т. е. ее уравнение движения по II закону Ньютона
Т-Мд = 0.	(1)
На груз действуют силы: Т — вверх и тпд—вниз, в результате чего груз движется относительно земли вверх с ускорением а:
Т — тд = та.	(2)
Решая систему уравнений (1), (2), находим ускорение:
(М \ Л а = 5-----1=20.
\ т J
Нерастяжимость веревки означает, что она движется относительно земли с таким же ускорением а = 2д. Следовательно, ускорение веревки и относительно обезьяны равно 2д, откуда заключаем, что обезьяна должна двигаться относительно веревки с ускорением 2д.
Задача 52** (77-р)
Через бревно перекинута невесомая веревка. К одному концу веревки привязано тело массой т, к другому приложена направленная вертикально вниз некоторая сила F. Из-за трения между веревкой и бревном минимальное значение силы, при котором тело т висит неподвижно, оказывается равным атд, где а— меньший единицы и не зависящий от т коэффициент. Найдите минимальное значение силы F, при котором тело т будет подниматься вверх.
Решение. При изменении силы F в пределах от Fmin = атд до -Ртах тело m неподвижно. Незначительное увеличение силы F сверх Fmax приведет к подъему тела т. Следовательно, Fmax и есть та сила, которую надо найти по условию задачи.
Итак, по условию задачи, веревка не двигается, если к одному ее концу приложена сила тд, а к другому— F = атд. Особое значение в условии задачи имеет утверждение о том, что о не зависит от массы, а значит, и от величины силы тд. Если к одному концу веревки приложить произвольную силу F*, то к другому достаточно приложить силу aFf < F', чтобы веревка осталась неподвижной. Пусть к одному концу веревки приложена сила F* = тд/а, а
54
к другому— тд < Ff. Веревка неподвижна. При незначительном увеличении силы F' тело т начинает подниматься вверх.
'Таким образом, искомое минимальное значение силы, при котором тело массой т поднимается вверх, равно тд/а.
Задача 53** (77-р)
Через бревно перекинута невесомая веревка. К одному концу веревки привязано тело массой т. К другому концу вертикально вниз приложена некоторая сила F. Из-за трения между веревкой и бревном минимальное значение силы, при котором тело поднимается вверх, оказывается равным атд, где а —больший единицы и не зависящий от т коэффициент. Найдите минимальное значение силы /*’, достаточное для удержания тела т в неподвижности.
Решение. Задача полностью аналогична задаче 52. Минимальное значение силы F, при котором тело т неподвижно, равно /'min = тд/а.
Задача 54 * (78-г)
Брусок массой М лежит на горизонтальной поверхности и привязан ниткой к стене (рис. 20, а). На нем расположен брусок массой ш. На верхний брусок действует постоянная горизонтальная сила F. ('Корость бруска постоянна и равна v. В некоторый момент нить пережигают. Как будет двигаться нижний брусок? Коэффициент трения между нижним бруском и поверхностью равен р.
Рис. 20. К задаче 54.
Решение. Силы, действующие на бруски в горизонтальном направлении до пережигания нити, показаны на рис. 20, б (Fi — сила трения скольжения, действующая на верхний брусок со стороны нижнего). Так как верхний брусок движется с постоянной скоростью, то сумма приложенных к нему сил равна нулю: F + Fi = 0.
По III закону Ньютона верхний брусок действует на нижний с силой трения F2 = — Fi = F. Кроме того, на нижний брусок со
55
стороны плоскости действует сила трения покоя FT и со стороны нити — сила натяжения Q. Так как нижний брусок покоится, то
F2 + FT + Q = О,
или, что то же самое, F + FT + Q = 0, причем FT = |FT| <	+
После пережигания нити на нижний брусок действуют только силы трения: F со стороны верхнего бруска и FT <	+ М)д со
стороны плоскости. Если F <	+ М)д, то нижний брусок двигаться не будет. Если F >	+ М)д, то нижний брусок начнет
движение с ускорением
„ т 4- М
a = F-—j^~ м-	(1)
При этом верхний брусок двигается с прежней скоростью v относительно горизонтальной плоскости. Такое движение продолжается до тех пор, пока скорость нижнего бруска относительно плоскости не сравняется со скоростью верхнего. Это произойдет за время
a F — 1л(т + М)д	v 7
с момента пережигания нити.
После того как скорости брусков сравняются, между ними будет действовать сила трения покоя. Величина этой силы Fq установится такой, что оба бруска будут двигаться как одно целое с ускорением ао- Применяя к этой системе II закон Ньютона, получаем
F — д(т + М)д = (т 4- М)ао, откуда
а0 — --------г?---•	(о)
т 4- М	v 7
Ответ:
При выполнении неравенства f < ц(т + М)д после пережигания нити нижний брусок продолжает покоиться.
При выполнении неравенства f > p(m4- М)д после пережигания нити нижний брусок в течение времени /, определяемого формулой (2), движется сначала равноускоренно с ускорением а, задаваемым формулой (1), а затем— равноускоренно с ускорением ао < а, задаваемым формулой (3).
56
’ Ьмцача 55 (86-г)
I la краю тележки массой М и длиной /, стоящей на гладком сто-!•', находится маленький кубик массой т. Кубик толкают с посто-П111К1Й силой F, направленной горизонтально к противоположному |1|тк> тележки. Через какое время кубик достигнет противоположишь края тележки? Коэффициент трения между кубиком и тележ-|"41 равен р. Кубик не переворачивается.
Решение. Рассматривая силы, действующие на кубик и тележ-ь у, и исследуя уравнения движения кубика и тележки, приходим к • цедующему результату.
Исли F < рпдт^-. то груз вообще не будет двигаться относи-irni.no тележки.
Исли F > ртд™^* , груз проскальзывает, и его ускорение отно-• ик'льно стола
F
а=------цд.
т
На тележку в горизонтальном направлении действует только сини трения, поэтому ее ускорение относительно стола
Груз движется относительно тележки с ускорением
F	/	т\
а0 = а - fli =-цд	( i + — )
т	\	М /
н достигнет края тележки за время
/2[_	/	21
v ао v £~(1 + $)’
Задача 56 (81-р)
На тележке массой М стоит кубик массой т. К кубику прикладывают горизонтально направленную силу F. Коэффициент трения между поверхностями кубика и тележки равен д. Трением между тележкой и поверхностью земли можно пренебречь. Найдите силу трения между поверхностями кубика и тележки.
Решение. На тележку в горизонтальном направлении действует лишь одна сила FT, приложенная со стороны кубика (рис. 21). Это либо сила трения скольжения, либо сила трения покоя.
FTp[w]-JL» | ] чтЛтт>т^яяятк Рис- 21. К задаче 56.
57
На кубик в горизонтальном направлении действуют две силы внешняя сила F и сила трения FT со стороны тележки, причем, согласно III закону Ньютона, FT = — FT.
Выбрав направление действия внешней силы за положительное и обозначив через а\ и а 2 ускорения кубика и тележки соответственно, напишем для них II закон Ньютона:
F — FT = mai,	(1)
FT = Ма2.	(2)
Если Я1 > а2, то кубик скользит по тележке и сила трения
FT = цтд.	(3)
Из уравнений (1) — (3) находим ускорения кубика и тележки:
F
ai	=	— -цд,	(4)
т
т	/СП
°2	=	МР9'	®
Условие скольжения ai > а2 с учетом формул (4) и (5) принимает вид
771
F> (т+ М)—цд.	(6)
Если это условие по данным задачи не выполняется, то кубик относительно тележки неподвижен, т. е. ai = а2, и сила трения является силой трения покоя, поэтому FT < цтд. Полагая в уравнениях (1) и (2) ai = а2, получаем
F----.
т + М
(7)
Итак, при выполнении условия (6) ответ дается формулой (3), в противном случае — формулой (7).
Задача 57 (77-г)
Система состоит из трех блоков и трех тел массой т каждое, связанных невесомыми нерастяжимыми нитями (рис. 22). Блоки А и В жестко закреплены на общей оси. Масса блока С пренебрежимо мала. Трение между нитью и блоками отсутствует. Найдите ускорения тел.
58
Решение. Поскольку нить невесома и трение между нитями к ।шоками отсутствует, то сила натяжения нити Т в любом ее ceil и п и одинакова. В соответствии с этим на левый и средний грузы и II»’г пуст вверх одинаковая сила Т, а вниз — одинаковая сила тд. I |иц действием этих сил оба тела получают одинаковое ускорение и. удовлетворяющее уравнению
тд — Т = та.
(1)
Гак как нить нерастяжима, то правый груз будет двигаться в нрогивоположном направлении с ускорением —а. Вместе с ним бу-। двигаться и блок С,
Рис. 22. К задаче 57.
Выпишем уравнение движения системы, образованной правым । рузом и блоком С. Поскольку, по условию задачи, масса блока пренебрежимо мала, то масса системы есть т. Со стороны двух нитей на систему вверх действует сила 2Т, вниз — сила тд. Под действием разности этих сил система приобретает ускорение —а. Согласно II ткону Ньютона
тд — 2Т = —та.	(2)
Исключая из двух уравнений (1), (2) неизвестное Т и решая получающееся уравнение относительно а, находим а = д/3.
Таким образом, левый и центральный грузы движутся вниз с ускорением д/3, а правый — вверх с таким же ускорением.
Задача 58 (77-р)
В нижней части длинной наклонной плоскости с углом наклона к основанию а находится тело массой т. Коэффициент трения между поверхностями тела и плоскости равен р. Тело получает мгновенный толчок вверх по наклонной плоскости и приобретает начальную скорость г>о- Найдите время движения тела вдоль наклонной плоскости и количество выделившегося тепла.
Решение. Рассмотрим силы, действующие на движущееся по наклонной плоскости тело. Это сила тяжести mg, сила нормальной
59
реакции наклонной плоскости N и сила трения FT, направленная вниз при движении тела вверх (рис. 23, а) и направленная вверх при движении тела вниз (рис. 23, б).
Рис. 23. К задаче 58.
Величина силы FT определяется формулой
где Q — величина силы нормального давления тела на наклонную плоскость. По III закону Ньютона Q = N и, в связи с этим,
FT = pN.	(1)
Силу тяжести удобно разложить на две составляющие: перпендикулярную к наклонной плоскости
Fi = тд cos а	(2)
и параллельную наклонной плоскости
F2 = тд sin а.
(3)
Движения в направлении, перпендикулярном наклонной плоскости, не происходит, следовательно, N = Fp С учетом этого соотношения и формулы (2) равенство (1) принимает вид
FT — цтд cos а.
Вверх тело двигалось равнозамедленно с начальной скоростью ио и ускорением
ai = — (F2 + FT) = </(sin о 4- pcosa) т
в течение времени

ti = ^0 ___________________
ai #(sina 4- /icosa) ’
пройдя до остановки путь
S = voti - а-у
vo / •	\
= -у «/(sin а + pcosoj.
60
hi’ini'iecTBO выделившегося при этом тепла W равно работе сил тре-
пни:
о w = Fts = ^ АЛ
cos а
sin а 4- cos а
Пели F2 < FT, или, иначе, > tga, то тело, достигнув верхней м«чки, остановится.
Пели же /^ < tga, то тело, достигнув наивысшей точки подъема, hvncT скользить вниз с ускорением
аг — --(^2 — FT) = gfsinot — ucosa) т
и пройдет путь S за время
vo
1__________
а — /л2 cos2 а
Полное время движения вдоль наклонной плоскости равно 4- /2 (при ц < tga) или /1 (при р > tga).
При спуске выделится такое же количество тепла, как и при подъеме, так как путь пройден такой же и величина силы трения । и же.
Задача 59 (78-р)
Кубик массой т находится на наклонной плоскости, имеющей при основании угол а. Коэффициент трения между поверхностями кубика и плоскости равен р. На кубик в горизонтальном направлении действует сила F (рис. 24, а). Найдите ускорение кубика, счи-।пи, что в начальный момент скорость кубика Vo направлена вниз пдоль наклонной плоскости.
а
Рис. 24. К задаче 59.
Р е ш е н и е. На кубик действуют силы (рис. 24, б): сила тяжести mg, сила нормальной реакции наклонной плоскости Q, внешняя сила F и сила трения FT, направленная против движения и равная i‘Q-
Уравнение движения кубика
та = mg 4- Q 4- F 4- FT.
61
В проекциях на направление вдоль наклонной плоскости (вниз): та = тд sin а — Feos а — FT,	(1)
на направление, перпендикулярное наклонной плоскости:
О = Q - тд cos а — F sin а.	(2)
Используя равенство (2), находим силу трения скольжения и ускорение кубика из уравнения (1)
FT = ц(тд cos а 4- Fsin а),	(3)
F	/	F . \
а = д sin а-cos а — и о cos а 4-sin а .	(4)
т	у	mJ
Введем угол /3 по формуле р = tg ft и преобразуем равенство (4) к виду а _ gsin(a-^) - £соз(а-/3) cos ft	’	;
Кубик движется равноускоренно при а > 0, т. е. при
< tg (а - 0).	(6)
т
Кубик движется равнозамедленно при а < 0, т. е. при
£>tg(a-/?)	(7)
т
в течение времени t = г>о/|а| и затем останавливается. После остановки кубик может начать движение вверх по наклонной плоскости. При этом сила трения изменит, свое направление и уравнение (5) примет вид
а - 9s[n(Q + ^ ~ £cos(a + ^) < 0	,8)
cos ft	’	k 7
Неравенство (8) имеет место только при
cos(a 4- ft) > 0,	(9)
£>tg(a + /?).	(10)
т
Действительно, если коэффициент трения /г и угол в основании наклонной плоскости а настолько велики, что а 4- ft > тг/2 и неравенство (9) не выполняется, то оба слагаемых в числителе выражения (8) положительны. При таких условиях движение тела вверх по наклонной плоскости невозможно, сколь бы ни была велика сила F.
62
Зидана 60** (83-р)
В приведенной на рис. 25, а механической системе коэффици-н|| г чрения между блоком и его осью равен угол а = 90°. При каких углах /3 грузы будут находиться в покое? Нить невесома, не-рагтяжима, не проскальзывает по блоку и проходит параллельно » тронам клина. Блок невесом. Трение между грузами и сторонами к пипа отсутствует. Радиус блока Я, радиус его оси г.
Решение. Пусть оба груза покоятся. При этом на них дей-। । ну ют силы: тяжести mg, натяжения нити Ti, Т2 и реакции опоры N ।, No. Из условия равенства нулю суммы сил, действующих на каждый груз, нетрудно найти силы натяжения
71 =	sin/?,	(1)
То = тдsin — /3^ = тд cos/?.	(2)
Рассмотрим силы, действующие на покоящийся блок. Со стороны нити на него действуют силы натяжения Ti и Т2, направленные по касательным к блоку, со стороны оси— сила нормальной реакции N и сила трения покоя FT (FT = /zAf), направленная по касательной и пси блока. Сумма сил, действующих на блок, равна нулю:
N 4-FT + Ti + Т2 = 0.
Принимая во внимание, что FT перпендикулярна N, a Ti перпендикулярна Т2, получаем:
№ + Ft2 = T12 + 7’22.	(3)
При покоящемся блоке должна быть равна нулю не только сумма всех приложенных к нему сил, но и сумма моментов всех этих сил пгносительно центра блока. Поэтому, учитывая, что момент силы N равен нулю, а силы Ti, Т2 и FT перпендикулярны соответствующим радиусам, получаем:
\T2-T1\R = Frr.	(4)
63
При /3 = тг/4
Ti=T2) FT = 0.
При уменьшении (или увеличении) /3 от тг/4 величина FT растет и при некотором значении угла /3 = /3mjn (или (3 = /Зтах) достигает своего максимально возможного значения
^ттах — ftN.
(5)
При дальнейшем изменении угла начнется движение грузов. Неизвестную величину силы N найдем из равенства (3), заменив в нем 71, Т2, и FT на правые части формул (1), (2) и (5):

тд у/1*?2
Подставляя это выражение в формулу (5), получаем:
_ цтд
V1 +р2
(б)
Для нахождения значения /3min (/3max) используем уравнение (4). Заменяя в нем Ti, Т2 и FTmax на правые части (1), (2) и (6) и полагая для определенности /3 = /?mjn < тг/4, получаем следующее уравнение для нахождения угла /3min,:
m£/?(cos/3min - sin/3min) =
fimgr x/l + м2
(7)
Проведя в равенстве (7) тригонометрические преобразования:
COS /3min sin/?min — х/2 sin /?min^ >	(8)
получаем
V 2 Sin I — — Anin I — “Г / -•	 9- •
'4	/	4- ^2
Отсюда для Anin находим: _	7Г	1
Anin — — arcsm	.
4 V2
Аналогичное выражение находим для /Зтах:
Л	7Г	1	11Г
Ртах = Т + arCSin —= --.
4 V2 яуТТр*
Заметим, что аргумент арксинуса меньше 1/\/2, т. е.
> /?тах > Р > Anin > О*
64
Задача 61* (88-г)
Полноприводной автомобиль стоит на наклонной плоскости, как покачано на рис. 26. Угол в основании наклонной плоскости а. Коэффициент трения шин автомобиля о плоскость // > tgo. Водитель । рогает машину с места так, что все четыре колеса одновременно на-'iiiiiiHOT проскальзывать. Найдите ускорение автомобиля в начальный момент движения.
Рис. 26. К задаче 61.
Примечание. Полноприводным называется автомобиль, двига-।глI. которого связан со всеми колесами.
Решение. Максимальное значение силы трения покоя равно /)|||1Х = где N —сила нормальной реакции опоры. Из у слонин равновесия'^ела в направлении, перпендикулярном наклонной и носкости, найдем силу реакции опоры
У = тд cos а,
•и куда
^тах = № cos а.
Условие равновесия тела вдоль наклонной плоскости:
тс/sin ct — FT = 0.
Гсло находится в покое, если
Рт < -Ртах
ПЛИ
> tg а.
Но условию задачи д > tga, следовательно до включения мотора нигомобиль неподвижен.
11ри трогании с места все четыре колеса машины начинают про-। кальзывать. При этом на автомобиль действует сила трения скольжения
Fc - Fmax = цтд cos а, направленная вдоль оси автомобиля. Теперь результирующая сини, действующая на автомобиль вдоль наклонной плоскости, равна
65
F = Fc + Fi, где Fi —составляющая силы тяжести вдоль наклонной плоскости, равная Fi = тд sin о. По теореме Пифагора F2 = F24-F2. Отсюда	______________
|F| = m^^/g2 cos2 а + sin2 а.
Ускорение автомобиля в начальный момент движения будет равно а — |F|/?n = gyjр2 cos2 ot 4- sin2 a
и направлено под углом /? к оси автомобиля, так, что
tg 0 =	= Р tg а.
Задача 62* (87-г)
На наклонной плоскости с углом а при основании находится груз, привязанный к одному из концов невесомой нерастяжимой нити. Другой конец нити прикреплен к некоторой точке наклонной плоскости. В начальном положении грузик смещают по плоскости относительно его наинизшего положения на плоскости так, что наг тянутая нить составляет угол /3 с тем направлением, которое она занимает при наинизшем положении грузика (рис. 27). Известно, что грузик остается в покое при /3 < /Зо и начинает скользить по плоскости при (3 > Определите коэффициент трения между поверхностью грузика и плоскостью.
Рис. 27. К задаче 62.
Решение.В предельном случае /? = /3q на грузик действуют следующие силы: тяжести mg, трения FT, реакции опоры N и натяжения нити Т. Предположим, что грузик находится ниже точки крепления нити, т. е. (3q < тг/2. Сила трения покоя принимает свое предельное значение, равное силе трения скольжения:
FT — цтд cos a.	(1)
Определим направление силы трения покоя. Для этого мысленно “выключим” трение и посмотрим, в какую сторону начнет двигаться грузик. Очевидно, что грузик может двигаться только по окружности, радиусом которой служит нить, т. е. в направлении, перпендикулярном нити. Сила трения покоя направлена в сторону, противоположную этому воображаемому движению. Исходя из этих
66
рассуждений, запишем уравнение движения грузика по наклонной плоскости в проекциях на направление нити и перпендикулярное ей направление:
Т = mg sin a cos /?о >
FT = тд sin а sin (3q .
(2)
< сравнивая правые части равенств (1) и (2), получаем
/z = tg a sin/Jo-
Если же /?о > тг/2, то грузик находится выше точки крепления нити. Он может начать движение не по окружности, а вниз по наклонной плоскости. При этом сила натяжения отсутствует, сила трения покоя направлена вверх по наклонной плоскости. В этом случае грузик будет сохранять состояние покоя независимо от направления нити и про коэффициент трения можно сказать только го, что д > tga, так как по условию задачи грузик скользит вдоль наклонной плоскости.
Задача 63 (79-г)
На гладкую наклонную плоскость, составляющую со своим основанием угол а, помещают массивный брусок. С каким ускорением надо двигать в горизонтальном направлении наклонную плоскость, чтобы брусок двигался вертикально вниз с ускорением свободного падения д'?
В«М)
А (хо,О)
Рис. 28. К задаче 63.
Решение. Воспользуемся системой координат, связанной с горизонтальной поверхностью. Ось Ох направим вдоль этой поверхности, а ось Оу —вертикально верх. Пусть наклонная плоскость покоится и занимает в начальный момент положение, показанное на рис. 28. При таком расположении для того, чтобы помещаемый на нее брусок двигался вертикально вниз, саму наклонную плоскость надо смещать влево.
Пусть наклонная плоскость движется влево с ускорением а. Координата х точки пересечения наклонной и горизонтальной плоскостей (Л) меняется с течением времени по закону
а42 x-XQ--t .
(1)
67
Координата у точки пересечения оси Оу наклонной плоскостью (В) связана с координатой х точки А соотношением
2/=ztgo.	(2)
Подставляя в (2) выражение для а: (1) и принимая во внимание, что я о tg а = I/Oj получаем:
fl л 2
У = 2/0 ”	’
где
а = a tg а.	(3)
Равенство (3) показывает, что при перемещении наклонной плоскости влево с постоянным ускорением а точка В перемещается вниз с постоянным ускорением а' (в разные моменты времени точке В соответствуют различные точки наклонной плоскости).
Очевидно, что брусок, помещаемый на наклонную плоскость в точке ее пересечения с осью Оу, будет двигаться вниз с ускорением свободного падения д в том случае, когда
а' > д.	(4)
При а = д брусок находится в постоянном контакте с наклонной плоскостью, скользя по ней без трения и не оказывая на нее давления; если же а > д, то наклонная плоскость “уходит” из под бруска.
Неравенство (4) имеет место всегда, когда
a></ctga.	(5)
Итак, чтобы помещаемый на наклонную плоскость брусок двигался вертикально вниз с ускорением свободного падения д, надо перемещать наклонную плоскость влево с ускорением а, удовлетворяющим условию (5).
Задача 64* (76-г)
Тело массой т расположено на наклонной плоскости. Угол в основании наклонной плоскости а. Наклонная плоскость движется поступательно в горизонтальном направлении с постоянным ускорением а. При каких значениях коэффициента трения тело будет покоиться относительно наклонной плоскости?
68
Рис. 29. К задаче 64.
I’ е ш ен и е. По условию задачи тело покоится относительно циклонной плоскости, т. е. движется горизонтально с ускорением к относительно инерциальной системы отсчета. На тело действуют • и'И.1 (рис. 29): тяжести mg, реакции опоры N, трения покоя FT. По II hi кону Ньютона:
mg 4- N + FT = ma,
нми в проекциях на нормаль к наклонной плоскости и на направление вдоль наклонной плоскости:
—тд cos а + N = та sin а,	(1)
тд sin а ± _FT = та cos а.	(2)
। 'и л п трения покоя не превышает величины , следовательно, из урпвнения (2) имеем
\тд sin а — та cos а| < дДГ.	(3)
Г«чппя совместно (1) и (3), получаем
I<7 sin а — acosal и < 2--:----------L.
“ a sin а 4- д cos a
Задача 65 (78-г)
Клин массой М с углом в основании а движется по гладкой шризонтальной поверхности под действием силы F, параллельной поверхности. На клине расположено тело массой т. Каково ускорение этого тела относительно клина, если трения между ними нет, а • ила тяжести отсутствует?
Решение. На тело т действует только сила N нормальной реакции клина (рис. 30, а). По III закону Ньютона тело действует па клин с силой -N. Кроме того, на клин действуют со стороны । призонтальной поверхности сила нормальной реакции Q и внешняя । ила F. Пусть а —вектор искомого ускорения. В системе отсчета, г вязанной с клином, тело скользит вдоль наклонной грани клина, следовательно, вектор а направлен параллельно этой грани.
69
Вектор ускорения клина относительно инерциальной системы отсчета обозначим через ар Этот вектор направлен параллельно линии горизонта—туда же, куда и сила F.
В инерциальной системе отсчета тело движется с некоторым ускорением а2. Так как на тело действует единственная сила N, то вектор аг направлен, как и эта сила N, по нормали к поверхности клина. Векторы a, ai и аг связаны друг с другом соотношением
а2=а + ах.	(1)
В силу ортогональности векторов ai и аг, из (1) следует (рис. 30, б)
a = aicosa,	(2)
a2 = ai sin a.	(3)
Выпишем уравнения движения клина и тела:
F+Q-N = Ma1>
N = таг •
Складывая почленно эти векторные равенства и проецируя получающееся векторное равенство на горизонтальное направление, с использованием выражения для аг (3) получаем
F = ai(M + msin2 a).	(4)
Из (2) и (4) находим
Feos a a =--------—•
М 4- т sin а
Вектор а, как мы уже показали, направлен вверх вдоль наклонной грани клина.
Задача 66 (83-г)
Клин массой т с углом в вершине а скользит по вертикальной стенке вниз, опираясь на куб массой М (рис. 31, а). Найдите ускорение куба. Трения нет.
70
Рис. 31. К задаче G6.
Решение. При смещении клина вдоль оси у на величину Л// куб перемещается в направлении оси х на величину Дж. Эти инличины связаны между собой равенством
Дж = Дт/ tg ос.
Аналогичным равенством связаны между собой ускорение куба и и горизонтальном направлении и ускорение клина ai в вертикальном направлении:
а = aitg а.	(1)
Ни клин и куб действуют силы тяжести Mg и mg, силы нормальной реакции со стороны горизонтальной и вертикальной плоскостей Ni н N‘2, силы давления клина на куб и куба на клин F и -F, напра-ii'K’inibie по нормали к наклонной грани клина. Уравнения движения ьуба вдоль оси ж и клина вдоль оси у имеют вид
Рсоза — Ма,	(2)
тд — F sin а = таг •	(3)
Принимая во внимание (1), находим из (2) и (3)
т sin a cos а а = $ ~Fr • 2 i 9	’
М sin а + mcos2 a
Задача 67* (79-г)
Па гладком горизонтальном столе покоится прямоугольный клин г углом при основании а. На верхнем ребре клина закреплен невесомый блок, через который перекинута невесомая нерастяжимая нить. К пей прикреплены тела с массами mi и m2 так, как показано на рис. 32, а. В начальный момент тела покоятся. В какую сторону начнет перемещаться клин, если телам и клину позволить свободно двигаться? Трение между всеми соприкасающимися поверхностями и в оси блока отсутствует.
71
Рис. 32. К задаче 67.
Решение. Для решения этой задачи достаточно найти горизонтальную составляющую результирующей сил, действующих на закрепленный клин со стороны нити и привязанных к ней тел. Если эта составляющая отлична от нуля, то после освобождения клина он начнет двигаться в ту же сторону, в которую она направлена.
Все интересующие нас силы показаны на рис. 32, б. На клин действует со стороны тела с массой m2 сила нормального давления N2, со стороны клина на тело — сила нормальной реакции
R2 = -N2.	(1)
Кроме того, на него действуют силы тяжести m2g и натяжения нити Т2. На тело mi действуют силы тяжести mig и натяжения нити Ть На блок со стороны нитей действуют силы натяжения Тх и Т2, а со стороны клина — сила Fi. Сам блок действует на клин с силой
F2 = -Fb	(2)
Решение задачи сводится к нахождению горизонтальной составляющей равнодействующей F сил F2 и N2.
Так как блок вместе с закрепленным клином покоится, то равнодействующая всех приложенных к блоку сил должна быть равна нулю:
Fi. + т; + Т2 = 0.	(3)
Из (2) и (3) для интересующей нас силы F2 находим:
F2 = t'+T2.
В соответствии с этим для составляющей Fx силы F = F2 + N2 можно написать:
Fx = Т2 cos а — N? sin а.	(4)
Для нахождения неизвестных величин Т2 и ДГ2, входящих в уравнение (4), надо использовать уравнение движения обоих тел, а также
72
•	^держащиеся в условии задачи указания о нерастяжимости и не-пегомости нити, о невесомости блока и об отсутствии трения в его
•	нН.
Обозначим через ai ускорение тела mi и через аг —ускорение к’ла тг- Согласно II закону Ньютона
miai = mig + Ti,	(5)
m2a2 = m2g 4-Т2 4-Иг-	(6)
Ьшисывая равенство (5) в проекциях на вертикальное направление, к равенство (6) — на направления, параллельное наклонной плоско-• in клина и нормальное к ней, и учитывая при этом, что в силу пграстяжимости нити |ai| = |а2| = а, получаем следующую систему и । грех уравнений:
тща = Ti — mig,
m2a = m2<7sina — Т2,
О = R>2 — т2д cos а.
Исключая из первых двух уравнений этой системы величину а, приводим к системе из двух уравнений
mi (т2д sin а - Т2) = m2(Ti - ггцд),	(7)
Я2 “m2(j cos а = 0.	(8)
’Гик как нить невесома, то Тх = —Ti, Т2 = Т2, а для длин векторов Т[ = 71, 7*2 = ТУ В силу отсутствия трения в оси блока и его невесомости сумма моментов сил Тх и Т2 относительно оси блока цолжна быть равна нулю, т. е. = Т2. Таким образом,
7ti, = 712=Ti =Т2.	(9)
H i системы уравнений (7)—(9), принимая во внимание, что Т?2 = ЛГ2, получаем
f mim2a(l 4-sin а)	/<лЧ
Т2 =	-	, N2 = m2g cos а.	(10)
mi 4- m2
Подставляя в (4) выражения для Т2 и из равенств (10), находим:
„	mi - m2 sin а
Fx = m20cosa------------.	(11)
mi 4- m2
73
Из равенства (11) следует, что Fx > 0 при т\/тп2 > sin а, т. е. свободный клин будет двигаться вдоль оси Ох. В противном случае начнется движение в противоположную сторону, а при mi = m2 sin а тела и клин останутся неподвижными.
Задача 68* (79-г)
На гладком горизонтальном столе покоится прямоугольный клин с углами при основании а и /3 = тг/2 — а. На верхнем ребре клина закреплен невесомый блок, через который перекинута невесомая нерастяжимая нитка. К нитке прикреплены грузы с массами mi, m2 (рис. 33, а). В начальный момент скорости грузов равны нулю. В какую сторону начнет перемещаться клин, если телам, принадлежащим системе, позволить свободно двигаться? Трение между всеми соприкасающимися поверхностями и в оси блока отсутствует.
Рис. 33. К задаче 68.
Решение. Задачу можно решить аналогично задаче 67. Приведем здесь более простое решение, основанное на использовании закона сохранения импульса.
На систему из клина с грузами действуют четыре внешние силы. Это силы тяжести и сила нормальной реакции со стороны горизонтальной плоскости. Все они не имеют горизонтальных составляющих и поэтому не могут изменить суммарной величины горизонтальных составляющих импульсов клина и грузов, входящих в систему. В начальный момент суммарный импульс системы равен нулю. Затем, если грузы начинают двигаться вправо (влево) вдоль оси х, т. е. приобретают положительные (отрицательные) импульсы, то, в силу действия закона сохранения импульса, клин начинает смещаться влево (вправо), т. е. получает отрицательный (положительный) импульс.
Чтобы определить направление движения грузов, рассмотрим сначала движение грузов при закрепленном клине.
Все действующие на грузы силы показаны на рис. 33, б. Это силы тяжести mig, m2g, силы нормальной реакции клина Ri, R2, силы натяжения нити Ti, Т2. Обозначая через ai, а2 соответственно ускорения грузов с массами mi, m2, выпишем их уравнения движения:
miai = mig + Ri 4-Ti,	(1)
74
m2a2 = m2g + R2 + Т2.
(2)
1'п( (’уждая так же, как и при решении задачи 67, приходим к выводу, •th. |а,| = |а2| = a, |Tj| = |Т2| = Т. Принимая это во внимание и проецируя уравнения (1) и (2) соответственно на направления, ппрпллельные левой и правой наклонным граням клина, получаем
гща = Т—migsina,
(3)
m2a = m2gsinfl-T
(4)
(при проецировании за положительное принято направление слева и и право). Складывая почленно равенства (3) и (4), находим
m2 sin в — mi sin а а =-----------------.
mi 4- т2
II । этого выражения следует, что грузы движутся направо, т. е. а > (I при m2/mi > tg а. Тогда незакрепленный клин будет двигаться и плево. В противном случае клин будет двигаться направо, а при nij/mi = tg а грузы и клин останутся неподвижными.
Задача 69* (74-р)
На гладкой горизонтальной поверхности покоится массивный клип с углами при вершинах а, /?, 7, причем а = ъ/Ъ и /? < 7. На боковые грани клина ставят тела одинаковой массы (рис. 34, а). II какую сторону начинает двигаться клин? Трение между телами и клином отсутствует.
Рис. 34. К задаче 69.
Решение. При решении этой задачи можно воспользоваться как методом, основанным на рассмотрении сил, действующих на закрепленный клин (см. задачу 67), так и методом, основанным на законе сохранения импульса (см. задачу 68).
Здесь мы приведем решение, аналогичное решению задачи 67, оставляя читателю возможность самостоятельно провести решение другим методом и сравнить ответы.
75
Пусть клин удерживается в неподвижном положении и на него ставятся тела, которые свободно соскальзывают по его граням. При этом со стороны тел на боковые грани клина действуют некоторые силы нормального давления Ni и N2 (рис. 34, б). Вполне очевидно, что если перестать удерживать клин, то направление его движения вдоль горизонтальной оси Ох будет определяться величинами проекций сил Ni и N2 на эту ось.
Если |NU| > |N2r I, то клин начинает перемещаться вправо, если наоборот, то — влево; если же iNi^l =	|, то клин сохранит
состояние покоя.
Итак, для ответа на поставленный в задаче вопрос достаточно рассмотреть силы, действующие при движении тел по закрепленному клину.
Со стороны клина на помещенные на него тела действуют силы нормальной реакции Ri и R2. По III закону Ньютона
Ri = -Ni,	(1)
R2 = -N2.	(2)
Кроме сил реакции на каждое тело действуют одинаковые силы тяжести Р = mg. Силы реакции принимают такие значения, что равнодействующие Fi и F2 сил, приложенных к телам, направлены вниз параллельно соответствующим граням клипа. Рассматривая прямоугольные треугольники, катетами которых служат векторы Ri, Fi и R2, F2, для длин векторов Ri и R2 находим:
Ri = тд cos 7 = тд cos(?r/2 — /3) = тд sin /?, R2 = тд cos /?.
Величины проекций	векторов Ri и R2 на ось Ох даются
формулами
= |Ri|sin 7 = |Ri I cos/? = mg cos/?sin/?, (3) 7?2r = |R2|sin/? = mt; cos/? sin/3.	(4)
В силу равенств (1)—(4)
— |3?ir| = m# cos/3 sin/?, 1^2*1 = |^2я?| = m# cos/? sin/?.
Таким образом, |Mr| = Отсюда следует, что свободный клин при помещении на его грани тел одинаковой массы в движение не придет.
76
Зцдача 70** (92-р)
11одвижный клин с прямым углом в вершине и с углом а в осно-1Н1111Ш покоится на гладкой горизонтальной поверхности. На нем । верху укреплен невесомый блок, через который перекинута нера-। ।вжимая нить, соединяющая два различных груза (рис. 35). Треки в пет. Грузы неподвижны. В некоторый момент нить пережигают. II какую сторону начнет двигаться клин?
Рис. 35. К задаче 70.
Решение. До пережигания нити задача аналогична задаче ПН Из решения задачи 68 следует, что равновесие грузов возможна только при выполнении условия m^/mi = tg а, где mi—масса । руза на боковой плоскости клина, составляющей угол а с горизон-hi.hi.io. Следовательно, груз с большей массой должен находиться пн боковой поверхности клина, составляющей меньший угол с гори-н >п ничью.
После пережигания нити задача становится аналогичной задаче 1Ш, по с разными массами грузов. Проекции сил нормального да-I»пения грузов на горизонтальное направление при равных массах ।«казались одинаковыми. В нашем случае эта проекция больше при Польшей массе. Следовательно, незакрепленный клин начнет двинуться в сторону большего угла в основании клина.
Задача 71 (80-р)
Лодку тянут по воде с постоянной силой F. При этом ее скорость постоянна и равна v. Какой путь S пройдет лодка в воде после прекращения действия силы, если сила сопротивления движению иодки пропорциональна ее скорости относительно воды?
Решение. Под действием силы F лодка двигалась равномерно, । ледовательно, по I закону Ньютона, на нее действовала еще сила 11«противления, равная Fc = -F. По условию задачи Fc = -av, где о “коэффициент трения”. Как видим, а = F/v. После прекращения действия силы F лодка двигается замедленно, так как на нее нействует сила сопротивления до тех пор, пока есть скорость относительно воды. Для бесконечно малого промежутка времени Д/ можно записать II закон Ньютона:
mAv = —av№.	(1)
77
Используя определение скорости v = Дя/Д/, перепишем равенство (1) в виде
тДи = —аДя.	(2)
Равенство (2), в отличие от (1), справедливо не только для малых, по и для любых изменений Av и Д/. Полное изменение скорости от на чала торможения (когда скорость лодки была равна v) до остановки равно Av = -v, а полный пройденный путь равен Ах = S. Подста вляя эти значения в (2), получаем соотношение mv = aS = FS/v, откуда
S = mv2/F.
Задача 72 (74-г)
Брусок движется с постоянной скоростью v относительно на к лонной плоскости, составляющей с горизонтом угол а. Сама на клонная плоскость движется относительно земли с постоянным уско рением а, направленным горизонтально. Найдите величину скоро-сти бруска в системе отсчета, связанной с землей. В начальный момент времени t = 0 скорость наклонной плоскости относительно земли равна нулю.
Решение. Будем рассматривать движение бруска и наклонной плоскости в связанной с землей системе прямоугольных декартовых координат (ж, у), ориентируя ось Ох по горизонтали, а ось Оу — вертикально вниз. Скорости бруска и наклонной плоскости в этой системе отсчета обозначим соответственно через и и v'. По закону сложения скоростей
и = v 4- N*
Проецируя это векторное равенство на оси Ох и Оу, имеем
=	+ Uy = Vy + v'y.
Так как по условию задачи
vr=vcosa, vx = at, vy = vsina, vy = 0, то
= v cos о + at, uy = vsina, |u| = ^/u2 4-	= \/v2 4- 2avt cos a + a2t2.
Задача 73* (74-r)
Кубик вылетает из точки А, ударяется о стенку одной из своих граней и попадает в точку В (рис. 36). Лобовой удар кубика о
78
• н ику абсолютно упругий. Коэффициент трения скольжения ме-•i*/iy кубиком и стенкой р < 1. Расстояние между точками А и 1»н|»но а. Расстояние от точек Л и В до стенки равно Ь. Найдите угол и между направлением начальной скорости и нормалью к стенке; * и 'in. тяжести отсутствует.
Рис. 36. К задаче 73.
Решение. Кубик ударяется о стенку под углом а, а отскаки-|нк' г от нее под углом /? так, что
tg /3 = max{tg а - 2/2,0} .	(1)
’ ho нетрудно получить, если учесть, что при ударе о стенку соста-нл н кидая импульса pj_, перпендикулярная стенке, меняет направление па противоположное. За малое время 6t изменение составляет
<5р± = Fift,
и тд все время удара :
Дрх = 2pi =
Под действием силы трения FT = pF± составляющая импульса рц, параллельная стенке, изменяется за время 6t на
<5рц = pFj.6t = p6pL.	(2)
Равенство (2) справедливо до тех пор, пока рц не обратится в ноль.
(Следовательно, полное изменение импульса рц за время удара
Дрц = min{2ppj.,p||} .	(3)
Учитывая, что
.	Р||	* а рН “ *₽11
tg а = — и tg р = —--------,
Р±	Р1
79
получаем результат (1), откуда после несложных геометрических преобразований следует, что
{а а 1 /‘ + йчГ
Кубик в процессе удара скользит вдоль стенки без переворота. Необходимое для этого требование (р, < 1) выполняется по условию задачи.
Задача 74 (78-р)
Тело массой т расположено на наклонной плоскости. Наклонная плоскость движется поступательно в горизонтальном направлении с постоянным ускорением (рис. 37). При каком значении этого ускорения тело будет покоиться относительно наклонной плоскости? Трение отсутствует, начальная скорость тела равна нулю.
Рис. 37. К задаче 74.
Р е ш е н и е. По условию задачи тело покоится относительно наклонной плоскости, т. е. движется горизонтально с некоторым ускорением а. На тело действуют силы тяжести mg и реакции опоры N. По II закону Ньютона
mg 4- N = ma,
или в проекции на направление вдоль наклонной плоскости:
mgsina — та cos а,
откуда
а = тд tg а.
80
3. ДИНАМИКА ВРАЩАТЕЛЬНОГО ДВИЖЕНИЯ
При вращении точки по окружности радиусом R с линейной скоростью v ее центростремительное ускорение ацс направлено к цен-। ру окружности и равно
V2 ацс ~ R ’
Вывести это соотношение можно следующим образом.
За время Д/ радиус-вектор R поворачивается на угол Да, а точки перемещается по дуге, длина которой ДЯ = ЯДа. Скорость этого перемещения определяется соотношением
ДЯ dR da	.
v = lim = — = Я— = Яа>,	(1)
Д/ dt dt	' '
। иг и — угловая скорость точки.
За это же время Д/ вектор v поворачивается на такой же угол До, поскольку v ± R. Конец вектора скорости описывает окруж-н<•< ть, по которой он перемещается на
Дгг = vAa.
Искомое ускорение (скорость изменения вектора скорости) находит-• н по аналогии с формулой (1):
..	Ди	dv	da	_ 9	v2
ацс =	lim	— =	—	=	v— =	vw = Rw	= —.
ц	Д/ •	dt dt	R
I Io II закону Ньютона движение точки массой m с ускорением пцс = возможно только при наличии силы, направленной к цен-। ру окружности и равной
^цс —
mv2
"л"’
(2)
В большинстве задач на вращательное движение необходимо установить источник и величину сил, вызывающих это движение, и свя-тть их уравнением (2) с кинематической характеристикой движения —центростремительным ускорением.
Рассмотрим вращательное движение твердого тела на простейшем примере плоского движения обруча (колеса, вся масса которого равномерно распределена по окружности).
81
Мысленно разобьем обруч на большое число N маленьких эле ментов массой т, каждый. На каждый элемент действует некото рая сила fy, и он движется с некоторой скоростью v, . По II закону Ньютона
Суммируя эти равенства по всем элементам, получаем
N Л
1>‘дГ=г’	<">
1=1
где величина F есть не что иное, как суммарная внешняя сила, действующая на тело (все силы взаимодействия отдельных элементов друг с другом взаимно сокращаются по III закону Ньютона). Выражение в левой части равенства (4) можно переписать в виде Др/At, где введено обозначение р = £2^	— суммарный импульс те-
ла. Из определения центра масс получаем, что р = Mv. Здесь М — масса обруча, v — скорость центра масс. Ускорение центра масс из равенства (4)
“=£ <5>
Теперь спроецируем равенство (3) для каждого из N элементов на направление, касательное к ободу, в системе отсчета, связанной с центром масс:
=	(6)
Суммируя левые части равенств (6) для всех точек обруча и учитывая, что линейные скорости точек обруча относительно центра масс равны по величине, получаем:
Айн
1 = 1
Суммирование правых частей равенств (6) дает
(7)
(8)

— суммарную проекцию внешних сил на направления, касательные к ободу. Приравнивая в соответствии с (6) правые части равенств (7) и (8) получаем нетривиальное соотношение,
Mat ~ Ft
(9)
82
• и ।' ipoe внешне похоже на II закон Ньютона, но имеет другой смысл — •пи • праведливо только для вращения обруча вокруг своей оси.
Исследуем теперь кинетическую энергию вращающегося обруча. II»»’ элементы обруча участвуют в поступательном движении центра и»и с с некоторой скоростью vq и во вращении относительно центра нп«( с одинаковыми по величине, но различными по направлению • ииростями |vf| = Vf Кинетическая энергия обруча запишется в НИЦ»'
.1	(v* + vo)2	vt + vo + 2^0*4 cos
Акин = 2^ mi -----0----- =	----------2----------
(10)
lnrci, введен угол а, между векторами v, и vo. Выполняя суммирование в равенстве (10), учтем, что cos а = — cos(tt - а) —для каждо-|и слагаемого с углом а, найдется слагаемое с углом тг — о,. Сумма • •1ПГПСМЫХ в правой части равенства (10), содержащих косинус угла, • •(•pitищется в ноль, и получаем окончательный результат:
_ Mvl , Mvj
Ъкин 2 + 2 ‘
(11)
Для обруча, катящегося по некоторой поверхности без проскаль-'МШН1ИЯ, скорость центра масс равна линейной скорости точек обручи относительно центра масс = t?o, и кинетические энергии вра-IHIIтельного и поступательного движений оказываются равными:
>®В —	— 2 А^КИН •
(12)
* * *
Задача 75 (75-г)
Два груза одинаковой массы подвешены на невесомой нерастя-•нимой нити, перекинутой через два невесомых блока как показано ни рис. 38, а. В какую сторону начнет двигаться правый груз, если:
Рис. 38. К задаче 75.
83
а)	левому грузу сообщить небольшую скорость в горизонтальном направлении;
б)	левый груз отвести на небольшой угол и отпустить?
Р е ш е н и е. а) Если груз, подвешенный на нити длиной Z, имеег1 горизонтальную скорость v, то он обладает центростремительным ускорением ацс = v2/I. На груз действуют две силы (рис. 38, б); натяжения нити Т и тяжести тд. По II закону Ньютона для левого груза
тд — Т = та — тацс, для правого груза
Т — тд = та.
Из этих двух уравнений находим:
9 Оцс V а=Т=2Г
Таким образом, в результате сообщения левому грузу горизонтальной скорости v правый груз начинает двигаться вверх с ускорением v7(2/).
б)	Так как скорость левого груза в начальный момент равна нулю, то и центростремительное ускорение в начальный момент равно нулю. Запишем II закон Ньютона для левого груза в проекции на направление нити (рис. 38, в):
Т — тд cos а = та,
где а — угол между нитью и вертикалью. Для правого тела в проекции на вертикальное направление:
тд — Т= та.
Разрешая эти уравнения относительно а, получаем:
Таким образом, при отклонении левого тела на угол о правое тело начинает двигаться вниз с ускорением д(1 — cosa)/2.
Задача 76 (75-р)
Маленький шарик массой т, подвешенный на нерастяжимой нити в поле тяжести, вращается в вертикальной плоскости. В верхней точке траектории натяжение нити равно нулю. Найдите натяжение нити в нижней точке траектории.
84
Решение. Поскольку в верхней точке траектории шарика • пип натяжения нити равна нулю, то по II закону Ньютона о mv -7Г = т9-
lin’ri» v — скорость шарика в верхней точке, R—длина нити. Для нижней точки траектории уравнение имеет вид
'Г -1г = Т-тя.
1цегь vj —скорость шарика в нижней точке траектории и Т—натяжение и in и в этой точке. Из закона сохранения энергии следует
mvr mv£ Л
— = — + ZmgR.	(3)
Z	А
Гешпя систему уравнений (1)—(3) относительно Т, находим:
Т = 6 тд.
Зцддча 77 (76-г)
Мотоциклист ездит по внутренней стенке вертикального цилин-ирн, имеющего радиус Я, со скоростью v на постоянной высоте над пипгрхностью земли. Найдите коэффициент трения покоя между и плесами мотоцикла и цилиндром, а также угол наклона мотоци-1« циста по отношению к вертикали, считая, что мотоцикл и мотоци-к цист наклонены под одним и тем же углом.
Решение. На мотоцикл со стороны стенки действует сила R, которую можно разложить на силу нормальной реакции Q и силу । рения FT. Сила R приложена в точке касания колес и направлена нд<nib прямой, проходящей через центр тяжести. Точку приложения • ii.iiы R можно перенести в центр тяжести и сложить с силой тяже-• in mg. Их равнодействующая F должна быть направлена к оси цилиндра. Она и создает центростремительное ускорение. Из приведенного построения нетрудно усмотреть, что сила F равна силе нормальной реакции, F = Q.
Будем считать, что расстояние от стенки до центра тяжести си-• п*мы пренебрежимо мало по сравнению с радиусом цилиндра. Тогда но II закону Ньютона
„	mv2
Q = F = ma=——.	(1)
Л
85
Из указанных построений также ясно, что
tg « = у.
гт
Л = -тд.
Из равенств (1) и (2) следует
V2 а = arctg —.
Сила трения покоя должна удовлетворять неравенству
где — коэффициент трения скольжения между колесами и цилин дром. Из (1), (2) и (3) следует, что
F? _ gR
“ Q “ v2 ‘
Задача 78* (78-г)
Тонкий обруч массой т и радиусом R раскрутили в вертикальной плоскости вокруг его оси до угловой скорости wq и поставили на горизонтальную поверхность так, что в начальный момент скорость центра обруча относительно этой поверхности была равна нулю. Найдите скорость центра обруча относительно поверхности п установившемся режиме. Считать, что есть некоторое трение скольжения, а трения качения нет. Как распределится первоначальная кинетическая энергия обруча к концу переходного режима?
Рис. 39. К задаче 78.
Решение. Рассмотрим сначала кинематику движения обруча (рис. 39). После соприкосновения с горизонтальной поверхностью обруч участвует в двух движениях: вращательном движении около своего геометрического центра О, положение которого совпадает с положением центра масс, и прямолинейном движении центра масс.
86
II < оответствии с этим в любой момент времени t скорость уд(/) про-•I ни мп>пой точки А обруча в неподвижной относительно поверхности • in к‘ме отсчета складывается из линейной скорости вращения ул (/) • и 41 точки в системе центра масс и скорости vq(/) поступательного пни жения:
va(0 =v^(t)+v0(t).
11,	ii н всех точек обруча длина vA вектора ул в данный момент вре-М1Ч1И одна и та же:
v'a =
। цг lj — угловая скорость точек обруча в системе центра масс. Век-।• i|> vq(/) в любой момент времени горизонтален. Ускорение ад точки I в неподвижной системе отчета складывается из:
I)	ускорения ао поступательного движения центра масс (вектор Но всегда горизонтален);
2)	центростремительного ускорения ацс, направленного всегда к центру обруча и равного ацс = vA/R = u?(t)R\
3)	касательного ускорения at, направленного по касательной к • •кружности и равного at = (dw/Л)/?.
Зная ао и at и пользуясь вышеприведенными формулами, можно пнйти величины и направления остальных ускорений и всех скоро-< iHl, т. е. полностью описать движение обруча во все время перекидного режима.
Для нахождения ао и at исследуем динамику процесса. Снача-||п ограничимся качественным рассмотрением. В начальный момент /	0, когда обруч приходит в контакт с горизонтальной поверхно-
। । ыо, поступательного движения еще нет, т. е.
vo(^)lt=o ~ О
и нижняя точка С вращающегося обруча проскальзывает по гори-юнтальной поверхности со скоростью vc = ул. Вследствие этого попил яется постоянная во времени сила сухого трения скольжения FT, in е время направленная против скорости vc нижней точки С. Кроме loro, начинает действовать направленная вверх сила Q нормальной реакции со стороны горизонтальной поверхности, которая уравновешивает силу тяжести mg, приложенную к центру масс. Сила трения 1'\. выполняет двоякую роль. С одной стороны, она ускоряет центр масс обруча, а с другой стороны, “притормаживает” вращательное движение обруча, создавая отрицательный вращательный момент. Возникает касательное ускорение, и уменьшается угловая скорость
87
вращения. Соответствующие уравнения выведены в начале данного раздела — равенства (5) и (7):
FT = mao,
FT = mat.
Скорость движения центра масс обруча будет меняться по закону
vQ(i) = aQi =—i,	(1)
m
а скорость точек обруча относительно его центра — по закону
УАЦ) = ^А(0)-а^ = шоЯ-—1.	(2)
т
Проскальзывание прекратится, и сила трения скольжения в точке С перестанет действовать в момент /о, когда сравняются скорости поступательного и вращательного движений обруча:
Vo(to) = v'A(t0) = V*.	(3)
Подставляя в (3) выражения (1) и (2), получаем момент прекраще-ния проскальзывания:
1 mujoR ° ~ 2 FT ’ В этот момент скорости поступательного и вращательного движений принимают одинаковые значения
* 1 „ v = ^0Я
и далее не меняются.
Поскольку при качении обруча без проскальзывания кинетические энергии поступательного и вращательного движений равны (формула (12) в начале раздела), то полная кинетическая энергия:
„	Лт(и*)2	1
Якин — 2	-	— — TTIWqR .
Начальная кинетическая энергия вращающегося обруча равна
Якин о — 2 ^^оЯ2, т. е. кинетическая энергия в процессе перехода к установившемуся режиму уменьшилась ровно в два раза (половина начальной кинетической энергии перешла в тепло).
88
’ livwia 79 (83-г)
Гонкий обруч массой т скользит без трения по гладкой гори-нц|гнльной поверхности со скоростью vo, не вращаясь. Ось обру-1и । оризонтальна, вектор vo лежит в плоскости обруча. В момент нргмсни t = 0 гладкая поверхность заменяется шероховатой с коэффициентом трения р. Найдите установившуюся скорость центра • •(•руча.
Решение. Как только обруч попадает на шероховатую по-н« |эхиость, на него со стороны поверхности начинает действовать • и ин трения скольжения FT = цтд с моментом М = FTR относи-ii'iii.iio оси обруча. Под действием постоянной силы FT скорость v снижения центра масс обруча уменьшается и может быть найдена ini формуле
v = vq — at = vq — FTt/m — vq — pgt.	(1)
i цругой стороны, момент M сообщает обручу угловое ускорение
P = M/J = fig/R
( I mR2—момент инерции обруча), и обруч начинает вращаться мок руг своей оси с угловой скоростью
w = (It = ngt/R,
। г. все его точки в неподвижной системе отсчета приобретают линейную скорость
v* = WjR = ngt.	(2)
I 'ила трения действует до тех пор, пока скорости v и vf не сравняются, а обруч не начнет катиться без проскальзывания. Из формул (I) и (2) получаем уравнение
v0 - pgt = pgt
имя нахождения момента времени /, когда прекращается действие • илы F . Решая его, получаем
t = уМ-	(3)
Подставляя в формулу (1) величину t из равенства (3), находим । корость установившегося движения центра обруча v = Vq/2*
89
Задача 80 * (77-р)
Тонкий обруч радиусом г скользит по горизонтальной поверх!и сти без трения, одновременно вращаясь вокруг своей вертикальн расположенной оси с угловой скоростью wo- Обруч налетает на вс| тикальную стенку по нормали к ней (рис. 40, а) и отскакивает тш что компонента скорости, нормальная к стенке, остается неизмсп ной и равной uq. Найдите скорость центра обруча после удара, угон под которым он отражается от стенки, и угловую скорость вращении вокруг своей оси после удара. Коэффициент трения между обручем и стенкой д. Величина деформации обруча в процессе удара мноо меньше его радиуса.
Рис. 40. К задаче 80.
Р е ш е н и е. Во время удара, длящегося малый промежуток времени Д/, на обруч действуют сила нормальной реакции N (• стороны стенки и сила трения FT, направленная вдоль стенки. Сили N, по условию задачи, сообщает обручу в направлении нормали (oci Ох) импульс 2muo:
NAt = 2?пг?о,	(I)
где N— среднее значение силы N в процессе удара, а т — массп обруча. В направлении вдоль стенки (ось Оу) под действием силы 1*\ обруч за время удара приобретает некоторый импульс mv —такой, что
FTД/ = mv .	(2)
Здесь v —составляющая скорости вдоль оси Оу после удара. Кро ме того, сила FT создает относительно центра обруча вращающий момент М = FTr, вследствие чего обруч за время Д/ замедляет свог вращение от угловой скорости wq до w , а момент импульса измени ется на величину J(wo —w ), где J = тг2 — момент инерции обруча Изменение момента импульса происходит таким образом, что
J(wq — w ) = МД/, или mr2(wo — w ) = иГтгД/. (3)
Отсюда находим
t FTAd v
W = Wo-------= Wq----
mr	r
(4)
90
Принимая во внимание, что компоненты vx и vy скорости v центра • •priii после удара определяются формулами vy = v и vx = vq, •iN iriiK'M для скорости v после удара:
V = ^v'2 + v%.	(5)
Vhhi о между вектором v и осью Ох:
а = arctg — = arctg —.	(6)
Vq
Почможны два случая.
Л) В процессе удара обруч все время проскальзывает по стенке. При ’>том в каждый момент времени имеет место равенство
FT = fiN.
I III ЦП
v = 2/w0.	(7)
II i ^ответствии с (7) равенства (4)—(6) принимают вид
v = vo \/1 + 4р2,	(9)
а = arctg (2р).	(10)
Полученные результаты справедливы лишь в том случае, если hi время удара обруч не начал катиться по стенке без проскальзы-IUI и и я, т. е. если гш > v , или
rcj0 > 4/wq.	(11)
В) Предположим теперь, что соотношение (И) не выполняется. II -этом случае при ударе обруча о стенку его угловая скорость о?о уменьшается до такого значения w , а его скорость вдоль оси Оу но ।растает до такого значения v , что имеет место равенство
v -Jr.	(12)
Ппчиная с этого момента сила трения на обруч уже не действует, '(вменяя в (4) v на правую часть (12), получаем уравнение для и , и । которого находим
LV = <Jo/2.	(13)
91
С учетом (13) равенство (12) принимает вид
v —
Подставив это выражение в (5) и (6), имеем
v =	4-w^r2/4,	(14)
а = arCtg 2v?	<15'
Итак, если имеет место (11), то ответ на поставленные в условии задачи вопросы дается равенствами (8) — (10). Если же гшо < 4/и>( то ответ — равенства (13) — (15).
Отметим, что в пограничном случае при гшо = 4ди формулы (8) — (10) и (13) — (15) приводят к одинаковым результатам.
Задача 81 (86-р)
Машина проезжает поворот дороги, имеющий вид дуги окружности (угол поворота 90°), с постоянной скоростью v за время t. Какова средняя сила, действующая на машину при повороте, если масса машины равна т?
Решение. Средняя за время t сила Fc вызывает такое же изменение вектора скорости Ду, как и реально действующая сила F(f):
Fc = тДу/t
Учитывая, что вектор скорости за время t повернулся на 90°, оставаясь неизменным по величине, получаем изменение скорости
| Ду| = y/2v.
Величина средней силы
|FC| = \/2mv/i.
Задача 82 (86-р)
Машина проезжает участок дороги, изображенный на рис. 41, с постоянной по величине скоростью. Найдите среднюю силу, действующую при этом на машину.
Рис. 41. К задаче 82.
92
I’ r in е н и е. Решение этой задачи аналогично решению преды-uu'il. Учитывая, что теперь полное изменение скорости Av = О, нм||учасм, что Fc = 0.
’ 1ндача 83 (85-р)
Шарик подвешен на нерастяжимой нити длиной I в поле тяжести । ускорением свободного падения д. С какой скоростью v надо по-инуть точку подвеса в горизонтальном направлении, чтобы шарик • нпгршил полный оборот в вертикальной плоскости?
Р о ш е н и е. От неподвижной перейдем к системе отсчета, жест-ин связанной с точкой подвеса. В этой системе точка подвеса непо->нч1 жна, а шарику сообщается начальная горизонтальная скорость с, в результате чего он приходит во вращательное движение в вер-। ипильной плоскости. При таком вращательном движении сила на-П1ЖГПИЯ нити в любой точке траектории должна быть отлична от пуня, за исключением, может быть, верхней точки траектории, где "ин может обращаться в ноль. В верхней точке на шарик действует • и пи тяжести тд и сила натяжения нити Т, обе направленные вниз. । hni создают центростремительное ускорение где vi —скорость шарика в верхней точке. По II закону Ньютона
vj тд + Т = т-р
I )гсюда
т-т\г~9)-
Можно утверждать, что тело совершит полный оборот, если в верхней точке траектории Т > 0, т. е. если
(1)
Скорость Vi зависит от начальной скорости v. Согласно закону сохранения энергии
_l + 2m^=—.	(2)
H i (2) и (1) находим:
v2 > 5gl.
Итак, чтобы шарик совершил полный оборот, ему, или его точке подвеса, надо сообщить горизонтальную скорость v, удовлетворяющую условию v > y/5gl.
93
Задача 84 * (86-г)
Автомобиль едет по мосту, имеющему форму параболы. Высот» моста Л = 5м, длина по горизонтали I = 60м. Найдите отношеимг силы давления автомобиля на дорогу на вершине моста к его вес) на ровной дороге, если по мосту он едет с постоянной скоростын v — 54 км/ч.
Решение. На автомобиль массой т в верхней точке мост» действует сила тяжести mg, направленная вниз, и сила нормальной! давления со стороны моста N, направленная вверх. Из уравнении движения автомобиля
о
mv	ы
—R - =mg-N,
получаем искомое отношение
N __ 1 v2 тд ~ gR'
где радиус кривизны моста R в верхней точке пока не известен. Определим его из следующих соображений. Пусть с вершины мо* ста в горизонтальном направлении бросили маленькое тело с такой скоростью г>о, чтобы оно летело по параболе, совпадающей с мостом. Нетрудно получить, что
v°=30~' V 8п с
Но в этом случае ускорение тела в верхней точке моста равно ускорению свободного падения и является центростремительным ускорением v$/R = д. Отсюда получаем
Я =
9	8Л’
и искомое отношение равно:
А - 1 _8/11,2 тд ~ дР
0,75.
Задача 85 * (86-г)
Легкая пружина с коэффициентом жесткости к одним концом прикреплена к вертикальной оси, вокруг которой она может свободно вращаться, а другим — к маленькому грузу массой т. Вся система находится на горизонтальном гладком столе, пружина не
94
. v Hiiiy га. Грузу ударом сообщают скорость vo, направленную пер-I» пппкулярно пружине. Найдите минимальное и максимальное рас-• "нпия от груза до оси, если скорость груза при максимальном । i -h iiiui от оси равна vp
Г «’ in е н и е. При максимальном и минимальном расстояниях от и hi до оси скорости груза Vi и V2 направлены перпендикулярно ж ине. При движении груза сохраняется энергия и момент им-|> н.гп:
mug _ mv? k(li - /0)2 _ mv% k(l2 - /о)2	,n
~Г~~+	2	-~Г +	2	’	(1J
muo/o — mvili = mv2h‘	(2)
i. /о—длина нерастянутой пружины; Zi, /2 — максимальное и •♦нппмальное удаления груза от оси.
Покажем сначала,что I2 = 1о> ^2 = «о- Действительно, I2 < /о. I • '”1 /2 < то согласно (2) t>2 > vo, и равенство (1) не может быть ni iiiojiiieHO.
11ч равенств (1), (2) получаем
,	_ /у2т(уо + vi)	у^т^Ур+У!)
2	° у k(vo-vi) ' 1 у fe(vo-vi)
’ liviana 86 * (86-р)
На столе лежит спираль из тонкой жесткой проволоки. Высота • ипрали h, радиус Я, длина проволоки /. На спираль сверху наде-||цю г маленькую бусинку, которая соскальзывает без трения.
а)	Найдите время, за которое бусинка пройдет всю спираль.
6)	Через интервал времени Atf сверху спускают еще одну бусинку. II и Идите максимальное расстояние между бусинками (по спирали).
в)	Найдите силу, с которой действует спираль на бусинку в конце iiv гн, масса бусинки т.
Решение, Силы, которые действуют на бусинку, — это си-un тяжести тд и сила реакции (нормального давления) со стороны проволоки. Под действием этих сил бусинка приобретает ускорение, которое можно разложить на две компоненты: касательное ускорение* at, направленное вдоль проволоки в том месте, где находится пусинка в данный момент, и центростремительное ускорение ап, направленное в горизонтальной плоскости к оси спирали и равное: ип = v^/Я, где vx — горизонтальная составляющая вектора скоро-• гн в данный момент. Мы учли, что при движении бусинки изменяйся только горизонтальная составляющая вектора скорости.
95
а)	Тот факт, что пружина изогнута, никак не скажется на вели чине ускорения at — она такая же, как если бы бусинка спускалш i по прямой длиной I с разностью высот между концами h:
at = sin a = gh/l.
Поэтому время, за которое бусинка пройдет всю спираль, равно
<= у (/sm a
скорость бусинки в конце пути
v = a^t — y/*2lg sina,
а горизонтальная составляющая скорости
vx = v cos a = \/2lg sina cos a.
б)	Скорость через время А/ после начала движения есть
vq = at At = д sin а At,
а путь, пройденный вдоль проволоки, равен
S = at(At)2/2 = ^sin a(At)2/2.
В этот момент спускают вторую бусинку. Так как касательные ускорения бусинок равны, то первая бусинка движется по спирали с постоянной скоростью vq относительно второй. Расстояние между ними увеличивается и становится максимальным, когда первая бусинка достигает нижнего конца спирали; это произойдет спустя время t — At после начала движения второй бусинки. Для расстояния между бусинками получаем при этом
S = vq(1 — At) = a sin a At I \	;----At .
\ у g sin a /
в)	Силу F, с которой спираль действует на бусинку, найдем из уравнения движения бусинки
та = F + mg.
Здесь а = at + ап —полное ускорение бусинки. Направленная к оси пружины горизонтальная составляющая силы F есть
Fp = тап = mv2//?= 2mgt sina(cosa)2//?.
96
• tn гпиляющая этой силы, ортогональная пружине и оси ж, напра-ii'M’iiii под углом а к вертикали и равна Fy = тд cos а. Для величины • ihii.i F в конце пути получаем
F — yj = тд cos ay 1 + sin 2a
' Задача 87 (87-г)
У наклонной плоскости одна половина гладкая, другая — шер-iiiniwiH. Граница их раздела—прямая линия. Если тонкую шайбу и» н гпвить на шершавую половину, она будет скользить вниз парал-н’in,по границе раздела гладкой и шершавой частей. Какова будет ||1нгктория шайбы, если ее положить на границу раздела?
Решение. Центр шайбы будет двигаться вниз параллельно । рнпице раздела гладкой и шершавой половин, а шайба будет вра-нпиъся относительно центра.
Рис. 42. К задаче 87.
Доказательство этого факта следует из того, что сумма сил трепни, действующих на различные участки поверхности шайбы, направлена все время параллельно границе раздела. Покажем, что но действительно так. Предположим, что в некоторый момент времени центр О шайбы движется со скоростью v ъдрлъ границы, и она вращается вокруг центра с угловой скоростью о?.
Рассмотрим два маленьких участка поверхности шайбы А и В • Mi и паковой площади (рис. 42). Силы давления наклонной плоско-• in на них одинаковы, а скорости Уд и у в направлены в разные • । оропы от границы раздела под одинаковым углом /?. Соответствующие силы трения равны по величине и направлены противоположно векторам Уд и у в- Поэтому результирующая сила трения, дей-• шующая на эти участки, направлена вверх вдоль границы раздела. Поскольку всю поверхность шайбы можно разбить на пары А, В, то полученный результат относится и к сумме всех сил трения.
97
Задача 88 (87-г)
На гладком горизонтальном столе лежит невесомый жестки* стержень длиной 21, на который надеты две тяжелые бусинки, рш положенные симметрично относительно центра стержня на расе гол нии I друг от друга. Сначала бусинки жестко закрепляют на стерж не в этих положениях и стержень раскручивают до угловой скорое/! и вокруг вертикальной оси, проходящей через его центр. В некого рый момент времени бусинки освобождают так, что они могут бм трения скользить вдоль стержня. На концах стержня установлены ограничители, соударения бусинок с которыми абсолютно упругие Определите время за которое бусинки вернутся в прежнее поло жение.
Рис. 43. К задаче 88.
Решение. В момент освобождения бусинок (положение /I на рис. 43) их скорости относительно стола vq = w//2. Так как стержень невесомый, то сумма приложенных к нему сил должна быть равна нулю (иначе он двигался бы с “бесконечным” ускоре нием). Кроме того, должна быть равна нулю сумма моментов сил, действующих на стержень (иначе он вращался бы с “бесконечным” угловым ускорением). Отсюда заключаем, что сила, с которой стер жень действует на каждую из бусинок, равна нулю (размеры бу синок много меньше /). Поэтому каждая бусинка движется отпо сительно стола равномерно и прямолинейно со скоростью vo (стер жень при этом поворачивается). Бусинки окажутся у конца стерж ня (положение В), когда расстояние между ними будет равно 21. К этому моменту времени каждая из них переместится на расстояние* у/12 — (Z/2)2 = /д/З/2 относительно стола. Отсюда время движения бусинок до ограничителей равно <о = /х/3/(2^о) ♦ При отражении от ограничителя меняется на противоположную компонента скорости бусинки, направленная вдоль стержня, а компонента, перпендикулярная стержню, остается неизменной. Стержню необходимо повернуться еще на 60°, чтобы бусинки вновь оказались на расстоянии I друг от друга. Так как после удара скорости бусинок г>о и проходи-
98
»н.К’ пути будут теми же, что и до отражения, то на это затратится время /о, т. е. t = 2/q = 2\/3/w.
' 1пдача 89 (87-р)
Два грузика уравновешены на легком разноплечем коромысле и । вязаны нитью (рис. 44). Коромысло раскручивают вокруг вер-нпаипэной оси, проходящей через точку подвеса. В какую сторону । честятся грузики?
~О О—
т*	Рис. 44. К задаче 89.
Решение. В исходном состоянии равновесия
Ш1Г1 = ТП2Г2.	(1)
’11<>бы удержать грузики в этом положении после того, как система придет во вращение, нить должна натянуться с некоторой силой Г, Причем должны выполняться условия
Т = miw2ri, Т = т2<л}2Г2-	(2)
Vi новия (1) и (2) не противоречивы, следовательно, грузики будут пиконться после раскручивания. Однако это положение равновесия неустойчиво. Действительно, если немного сдвинуть систему вдоль • н’ржня (например, увеличив гг и уменьшив ri), то условия рав-пмпесия (2) не будут выполняться одновременно. Сила натяжения ни in Tf (постоянная вдоль всей невесомой нити) будет иметь неспорое промежуточное значение между тщш2Г1 и Ш2^2г2. Второй । ру in к начнет удаляться от оси (для его удержания силы Т' недо-। I«точно), а первый — приближаться к оси.
Задача 90 (81-р)
Горизонтальный диск радиусом R = 10 м вращается вокруг своей • ни с постоянной угловой скоростью w = 0,2рад/с. По краю диска гнет мотоциклист с постоянной по величине скоростью v = 36 км/ч • ппосительно диска в сторону, противоположную направлению вращения диска. Каким должен быть коэффициент трения между шипами мотоцикла и поверхностью диска, чтобы такое движение было III» 1МОЖНО?
99
Решение. Чтобы мотоцикл мог двигаться по окружности радиусом R со скоростью v, к нему должна быть приложена напри вленная к центру диска сила
F=mv2/R,	(I)
где тп — масса мотоцикла с мотоциклистом. Эта формула справед лива только в инерциальной системе отсчета, например связанной с землей. В этой системе скорость мотоцикла v складывается mi скорости мотоцикла относительно диска vq и скорости точек край диска относительно земли и = wR:
v = vq — и = vo - wR.	(2)
Знак минус возникает из-за того, что направление вращения диска противоположно направлению движения мотоцикла. Единственной силой, действующей на мотоцикл с мотоциклистом в горизонталь ном направлении, перпендикулярном скорости мотоцикла, является сила трения покоя между поверхностью шин и диском. Эта сила не может превосходить своего максимального значения
Fmax=p|N|,	(3)
где N — сила нормального давления со стороны диска на мотоцикл с мотоциклистом и д — коэффициент трения. В вертикальном направлении на них действуют лишь две силы: mg и N. Так как мотоцикл в этом направлении не движется, то mg + N = 0, откуда
|N| = тд,	(4)
Учитывая равенства (1)—(4), получаем
F = т(у - wR)2/R < Fmax = ртд-	(5)
Отсюда
(v — w/?)2 р > k	= 0,64.
Итак, чтобы мотоцикл не соскочил с диска, коэффициент трения должен быть не меньше 0,64.
Задача 91 (81-р)
С какой максимальной скоростью vmax может проехать мотоциклист по закруглению дороги с радиусом R = 120 м, если коэффициент трения между шинами мотоцикла и поверхностью земли р = 0,25?
100
Решение. Решение задачи похоже на решение предыдущей. Пиможив ш = 0 в формуле (5) предыдущей задачи, получим нера-н» н< 1 во
mv2
“д- < №>
••туда
v < y/^igR.
I нким образом, для максимального значения скорости, с которой мигоцикл может проехать по заданному закруглению, получаем |'жпм w 17,3 м/с = 62,3 км/ч.
101
4. ЗАКОН ВСЕМИРНОГО ТЯГОТЕНИЯ
Закон всемирного тяготения описывает взаимодействие точеч ных или сферически симметричных тел. На тело массой т, нахо дящееся в поле тяжести Земли на расстоянии R от ее центра, дей ствует сила
FT = -G^^,	(1)
Hr Я
где G = 6,672 • 10"11	— постоянная всемирного тяготения. Это
одна из немногих мировых постоянных, значение которой во многом определяет свойства вселенной.
Работа по переносу тела массой т из бесконечности в точку пн расстояние R от центра Земли равна потенциальной энергии тела:
Приравнивая силу тяжести (1) к величине тд, где д — ускорение свободного падения на поверхности Земли, получаем
и для потенциальной энергии Un можно написать:
С/„(Я) = -тд-^-.	(4)
Я
Для разности потенциальных энергий на высоте Л и на поверхности Земли находим
Д1/п = ^п(Яз + Л) - £/п(Яз) =	(5)
Яз н- л
При Л <$С 2?з можно пренебречь величиной Л в знаменателе правой части равенства (5) и получить для потенциальной энергии на высоте h относительно поверхности Земли ДС7П = mgh.
При движении по окружности радиусом R в поле силы тяжести (1) тело испытывает центростремительное ускорение
аЦс —
_гмз
R т Т R2 ’
(6)
102
• I h V ЛД.
v = (7)
V rC
и in, с учетом (5):
v =	(8)
v /t
При l{& R3 получаем первую космическую скорость:
= V9R3,	(9)
in иЬходимую для движения вокруг Земли по окружности радиусом //.
(’ увеличением радиуса круговой орбиты скорость по формуле |Н) убывает, но это не означает, что запустить спутник на высокую • •|И»иту легче, чем на низкую. Действительно, выражение для пол-||и|| шергии тела, двигающегося в поле тяжести со скоростью v на Iнм ( гоянии R от центра Земли:
Е = Un + £Кин =	•	(Ю)
/г z
11нд( тавляя сюда скорость движения по круговой орбите радиусом II из (8), получаем энергию как функцию расстояния R
1 Др 1
ад =	-хгпд-^- = -Un	(11)
и ’in как функцию скорости движения по круговой орбите
ад =	-^ = -ад	(12)
Несмотря на то, что с увеличением радиуса орбиты скорость (и кинетическая энергия) убывает, потенциальная и полная энергия теин растут. Следовательно, для запуска на более высокую орбиту индо сообщить телу большую энергию, т. е. большую начальную • корость.* *
Кроме того, из выражения для энергии (10) легко получить значения энергии, при которых тело сможет улететь бесконечно далеко. Для этого надо в выражение (10) положить Un = 0, а Якин =
* Из равенств (11) и (12) следует очень важное соотношение, определяющее и курсах по теоретической и квантовой механике многие свойства гравитаци-
• tiiiioro и электромагнитного полей: Un = -2ЕКин- Мы вывели это соотношение кия движения по окружности, но оно справедливо и для средних значений Un п /v’kmh при движении по любым ограниченным траекториям.
103
mv^/2. Получаем, что тело может улететь бесконечно далеко при Е > 0. Это неравенство выполняется, если скорость тела на поверх ности Земли больше второй космической скорости:
= у/ЪдЯз-
(1Я)
* * *
Задача 92 (74-г)
Как влияют притяжение Солнца и вращение Земли на показании пружинных весов, измеряющих вес тела на экваторе в полдень и в полночь? Считать, что ось вращения Земли перпендикулярна к плоскости орбиты.
Решение. На тело, подвешенное на пружинных весах, дей ствуют силы: притяжения Земли тду притяжения Солнца (Rf - расстояние от тела до центра Солнца), натяжения пружины Р (показание весов). Под действием этих сил тело испытывает уско рения, связанные с вращением Земли вокруг своей оси
4?Г2Г	2	(	1	\
Я1 = —у = (т = 1 сут)
и вращением Земли вокруг Солнца
4тг2Л Мс , a2 = ^- = G-^- (Т = 1 г).
Здесь R — радиус орбиты Земли иг — радиус Земли. В полдень (индекс 1) и в полночь (индекс 2) тело, центр Земли и Солнце находятся на одной прямой, поэтому все силы и ускорения направлены по одной оси. По II закону Ньютона получаем:
Mem ( о	Мс\	. .
mg - Pl - G-52- = m |Л - тС-?- ) ,	(1)
jvj	у	/С	/
„	^Мст	(	9	^Мс\	/ЛЧ
тд - Р2 + G- у = ш I ш2г + mG-^- ) ,	(2)
•*^2	\	Е'	/
где Ri = R — г, R2 = R + г — расстояния от тела до центра Солнца в полдень и в полночь соответственно.
104
H i этих уравнений можно получить значения Pi и Рг. При выпи лги ни следует учесть малость радиуса Земли по сравнению с Iни1 нусом ее орбиты:
1	_ 1 /	2г\
(Я±г)2 ~ Л2 V Т R ) '
Ге |ультат вычислений:
/ 4тг2г л4тг2г\
Pi « Р2 « т {д -	- 2-yj-J	(3)
Пиццо, что последним слагаемым в правой части равенства (3) мож-iin пренебречь (Т 3* т).
Ответ:
I (оправки к весу тела, связанные с вращением Земли вокруг сво-|’П »м и и вокруг Солнца, в полдень и в полночь одинаковы и равны
ДР « — 4тг2тгДг « —0,0034 тд.
Задача 93 (74-г)
Почему почти все спутники запускаются с запада на восток? I )цгните скорости, которые необходимо сообщить спутнику для запуска на низкую круговую орбиту, проходящую вдоль экватора, с in и ада на восток и с востока на запад.
Решение. Из-за суточного вращения Земли все точки ее поверхности имеют некоторые скорости, направленные с запада на ши ток. Точки экватора движутся со скоростью v = 2яЯз/Т\ где /'	1 сут — период вращения Земли. Вычисляя эту скорость, полу-
•нк’М и « 0,5 км/с. Следовательно, скорость, которую надо сообщить • путнику для запуска в западном направлении, на 1 км/с больше • корости, необходимой для запуска в восточном направлении.
Задача 94* (75-г)
Некоторая планета целиком состоит из несжимаемой жидкости плотностью р. Температура в глубине планеты постоянна. Найдите твисимость давления от глубины. Радиус планеты Я.
Решение. Расстояние от данной точки до центра планеты пудем характеризовать радиусом г. Требуется найти зависимость цпвления р внутри планеты от г, т. е. функцию р(г).
Рассмотрим узкий столб жидкости, расположенный вдоль радиуса планеты. Одно основание столба находится на расстоянии г от центра планеты, а другое — на поверхности. Площадь поперечного
105
сечения столба обозначим через S. Результирующая сила, действу ющая на этот столб, складывается из силы давления
Л=р(г)5,	(I)
направленной вдоль радиуса к поверхности, и из силы тяжести /‘У направленной в противоположную сторону. Найдем силу тяжести F2. Сложность ее вычисления состоит в учете того, что на разные малые элементы рассматриваемого столба действуют разные силы тяжести.
Выделим небольшой элемент столба, нижнее основание которо го расположено на расстоянии г от центра планеты, а верхнее -на расстоянии г 4- dr. Объем dV этого элемента дается формулой dV = S dr, а масса dm = pdV. На этот элемент столба действуй сила тяжести со стороны жидкости, заключенной в шаре радиусом г с массой М(г) = /?“7гг3. Результирующее действие на массу din со стороны остальных частей планеты равна нулю. Действие шара массой М(г) на массу dm. эквивалентно действию точечной массы М(г), расположенной в центре шара. С учетом этого замечания, для силы dF, действующей со стороны шара с массой М(г) на массу dm, можно написать:
dF = G—Ц------= -тгр2 Srdr.
rz 3
Разделив обе части этого равенства на dr, получим
. dF 4 2 о = * = 6з’'> Sr’
где /(г) — сила тяжести, действующая на элемент столба единичной длины, находящийся на расстоянии г от центра планеты; /(г) есть линейная функция от г. Результирующая сила тяжести F2, действующая на весь рассматриваемый столб, численно равна площади фигуры, ограниченной сверху графиком функции f(r), снизу осью абсцисс, с боков начальной и конечной ординатами:
F2 = Д/(Д) ~ rftr) = с|тгр25(Я2 - г2).	(2)
Z	и
Поскольку рассматриваемый столб жидкости находится в равновесии, то Fi = F2. Приравнивая правые части равенств (1) и (2), находим:
9
Р(Н = G-irp2(R2 - г2).
О
106
Ill’ргходя в этом выражении от переменной г к “глубине” h = R — г, нниучаем
Р(Л) = G^irRp2h fl -	= pgh fl -	.
□	\	Z/1 /	\	Z/I /
Lirri, д = G^irRp — ускорение свободного падения на поверхно-• hi планеты. На малых глубинах (Л R) давление описывается и un i гпым выражением
p(h) « pgh.
fIпиление в центре планеты (при h = R) оказывается равным
р(0) = pgR/2.
Задача 95 (75-р)
Какую энергию необходимо сообщить космическому кораблю мас-• •II I т для того, чтобы он, стартовав с поверхности Земли, вышел ни круговую орбиту радиусом R = 16000 км? Считать, что потенци-•11ЫН1Я энергия тела в поле тяжести Земли на расстоянии от центра ||*мли R > R3 равна Еп = —бтЛ/з/Я, где Мз — масса Земли.
Решение. На круговой орбите полная энергия корабля mv2 тМз e = — -g~r~-
1нееь v — скорость корабля на орбите. По II закону Ньютона mv2 _ тМ3 Rl ^R-G~i<r = ms№
(мы учли, что на поверхности Земли тд =	Выражение для
з нн’ргии можно переписать так:
Р тдЯ-з
Е = "йГ-
Покоящийся на Земле корабль обладает потенциальной энергией
Ео = -G^ = -mgR3.
R3
Гнким образом, для подъема на орбиту кораблю надо сообщить НКфГИЮ
ЛЕ = Е-Е0 = тдЯ3 fl -	» 5 • 101ОДж.
\ ZR)
107
Задача 96 (76-г)
Вычислите первую космическую скорость при старте с поверх ни сти Юпитера, если известно, что один из его спутников вращастгц по почти круговой орбите радиусом г = 10б км с периодом Т = 7, III сут. Радиус Юпитера R = 7 • 104 км.
Решение. Спутник, вращающийся на орбите радиусом г i периодом Т, двигается с центростремительным ускорением
ацс = v2/г = 4тг2 г/Т2.	(I)
Это ускорение создается силой притяжения со стороны планеты. Hi II закона Ньютона следует, что
auc = GM/r2,	(У)
где М — масса Юпитера. Приравнивая правые части (1) и (2), нп ходим:
GM = 4тг2г3/Т2.	(3)
Первая космическая скорость vi — это скорость спутника на орби те, проходящей вблизи планеты радиусом Я, т. е. при г = R. Онл должна удовлетворять условию
vl/R = ацс = GM/R2.	(-1)
Заменяя в (4) произведение GM правой частью (3) и решая затем получающееся уравнение относительно vj, находим:
2тгг Гг Л ,Л«км ,	₽ KM ЛЛЛкм
VI = — J- w 3,32 • 106--= 1,38 • 105 — = 38,3 —.
т у R	сут	4 с
Задача 97 (77-г)
При отсутствии сопротивления воздуха человек на поверхности Земли может толкнуть ядро на I = 22 м. Сможет ли он так толкнуть это же ядро на поверхности астероида, чтобы оно стало спутником? Масса астероида М = 3 • 1013 т, радиус R = 10 км.
Решение. Результатом воздействия человека на ядро явля ется сообщение ему некоторой начальной скорости v. Брошенное с поверхности Земли с начальной скоростью v под углом а к горизонту тело к моменту падения проходит в горизонтальном направлении путь I, определяемый формулой
V2
1 = —sin(2a),	(1)
9
108
н I/ ускорение свободного падения на Земле. Как следует из ill наибольшая дальность полета /тах для данной скорости v донн иг |гя при угле бросания а = 45°. В связи с этим очевидно, что ’	следовательно, v = у/дЦ^. Наибольшей дальности по-
। hi ядра smax отвечает и наибольшее значение начальной скорости, ! |м|нк' человек может сообщить ядру v = vmax. Выполняя вычи-hiiiiii, находим, что vmax = 15 м/с. Обозначим через vj первую । " ми’кткую скорость для астероида, т. е. vj — это минимальная •' » in hi.пая скорость, которую необходимо сообщить телу, находяще-ии| на поверхности астероида, для того чтобы оно превратилось •• • I" спутник. Если vmax < vi, то человек не может толкнуть ядро • •in, чтобы оно превратилось в спутник, если vmax > vj, то может. Ihlbv’M величину vj. Обозначим массу ядра через т. Очевидно, что ни Ьудст вращаться вокруг астероида по окружности радиусом Л, hi 111.нюлняется условие
GmM/R2 = mv2/R.
• »н ц»да получаем
VI = y/GM/R = 14 м/с.
I «и* им образом, vmax > vi, следовательно, человек может толкнуть • 1|н। гак, чтобы оно превратилось в спутник астероида.
' 1пдача 98 (77-г)
11усть имеется шар, радиус которого равен радиусу Земли, а мас-•I массе Земли. На шаре расположено некоторое малое тело. Шар •M'liiiiaeT вращаться вокруг оси, перпендикулярной к радиусу, coin ияющему тело с центром шара. Опишите, что будет происходить н'лом при медленном увеличении угловой скорости шара. Между и"|И’рхпостями тела и шара существует трение.
Решение. Когда шар покоится, на тело действуют уравнове-нннтющие друг друга силы. Это сила тяготения F и сила реакции V (' момента начала вращения шара на тело начинает действовать i|H’riiX сила — сила трения FT, направленная по касательной к по-•м цхпости шара в сторону вращения. Эта сила вначале сообщает н чу угловое ускорение, равное угловому ускорению шара. Благо-nil hi этому тело первое время движется вместе с шаром без про-
• и и л взывания, сохраняя постоянное положение на его поверхности. ’ и un трения, а точнее, сила трения покоя FT (тело относительно ||'||»г|)хности шара неподвижно!) не может принимать сколь угодно ’•м.)||.п1ие значения. Она ограничена значением Fmax = pQ, где р— нмффициент трения, a Q — сила нормального давления, равная по
109
величине силе реакции N (Q = N). Силу нетрудно найти. Пол действием разности сил F и N тело получает центростремительно!' ускорение а = w2r (здесь w—угловая скорость, г — радиус шара) По II закону Ньютона F — N = таг г (т — масса тела). Принимай во внимание, что F = GmM/г2 (G — гравитационная постоянная, М — масса шара), находим:
N = m[G— -w2r).	(I)
\ г J
Из формулы (1) видно, что с увеличением угловой скорости вращс ния шара и сила реакции ЛГ, а следовательно, и сила нормального давления Q тела на шар, и значение максимальной силы трения по коя Fmax уменьшаются. При дальнейшем увеличении угловой ско рости шара тело начнет проскальзывать, т. е. угловая скорость шп ра будет больше угловой скорости тела. И, наконец, при некотором значении угловой скорости тела сила реакции опоры и сила трения обратятся в ноль. Из равенства (1) следует, что
2
wmax 2 ’
При этой угловой скорости тело перестает давить на шар, приобрс тает первую космическую скорость и превращается в спутник, вра щающийся по круговой орбите. В дальнейшем растет только угло вая скорость шара, а угловая скорость тела сохраняет постоянно!’ значение.
Задача 99 (86-г)
Определите радиус г орбиты спутника Земли, который все время находится над одной и той же точкой земной поверхности.
Решение. Орбита, по которой должен двигаться спутник, па ходится в плоскости, проходящей через земной экватор. В против ном случае широта, на которой находится спутник, будет меняться Кроме того, орбита должна быть круговой, так как по эллиптической орбите спутник, в отличие от Земли, вращается неравномерно. Таким образом, надо найти радиус г круговой орбиты в плоскости экватора с периодом обращения спутника Т = 24 ч. Из уравнения движения спутника получаем
v2/r = GMjr\
где v — скорость движения спутника, М — масса Земли, G — гравитационная постоянная. Замечая, что д = GM/R^} получаем для
110
hi |»иода обращения
Т = 2тг- = 2тг
V
II । v того выражения легко находим радиус орбиты спутника
/	07*2	\ 1/3
r = R3 Ab" = 6,4 Я3 = 42,24 103 км. \4тг“пз /
Задача 100 (90-г)
Частица массой m движется под действием некой силы по кру-|мцп|| орбите радиусом R с постоянной скоростью. Потенциальная шгргпя U частицы в поле этой силы зависит только от расстояния Н центра орбиты следующим образом: a) U(R) = kR (k > 0); • •) //(/?) = kR2 (& > 0). Найдите скорость частицы в случаях а) и б).
I* о ш е н и е. Частица массой m движется по окружности радиу-• »»м R с постоянной скоростью v, если на нее действует постоянная пи величине сила F\ направленная в любой момент времени к цен-। ру окружности. Эта сила создает центростремительное ускорение । и по II закону Ньютона
F = mv2/R,
(1)
Формула (1) позволяет определить скорость частицы v, если известии сила F как функция от R:
v — \/RF(R)/m.
(2)
Ннйдем связь функции F(R) с величиной U(Я).
Вудем мысленно перемещать частицу с некоторой постоянной • коростью (чтобы ее кинетическая энергия оставалась неизменной) и поле с потенциальной энергией U{R} вдоль произвольного ради-v и. Для этого на частицу надо воздействовать внешней по отно-ип’нию к системе “частица—поле” силой Я(Л), равной по величине искомой силе и направленной противоположно, т. е. от центра. I 'умпарная сила будет равна нулю, и поэтому движение будет равномерным. Сила Я(Я) на пути ДЯ из точки с радиальной коор-нипатой R в точку с радиальной координатой R + ДЯ совершает рибиту ДД « Я(Я)ДЯ. Это приближенное равенство тем точнее, •им меньше величина ДЯ. Вся работа ДД, совершаемая внешней • илой, идет на пополнение только потенциальной энергии системы "чпстица—поле” (кинетическая энергия постоянна!). В результате
111
совершения этой работы потенциальная энергия системы увеличи вается на величину ДС7 — ДЛ « F(R)AR. Отсюда приближенно F(R) « AU/&R. Чем меньше ДЯ, тем точнее будет это равенство В пределе при ДЯ —> 0 мы получаем точное равенство:
Я(Я) = д1ппо Д17/ДЯ = dU/dR.
Итак, искомая функция F(R) равна производной от потенциальной энергии U(Я) по Я.
Используя общую формулу (3), в случае а) получаем
F(R) = dU/dR = k>
(4)
т. е. F — постоянная, не зависящая от Я величина. Подставляя и (2) выражение (4), находим:
v = \JkR/m.
В случае б):
F[R) = dU/dR = 2kR.
Подставляя в (2) выражение (5), имеем:
v = R\/2k/m.
Задача 101 (94-г)
Сравните потенциальные энергии тела на поверхности Земли и на поверхности Луны. Принять, что ускорение свободного падения на Луне в шесть раз меньше, чем на Земле, а радиус Луны в три раза меньше радиуса Земли.
Решение. Чтобы сравнить две потенциальные энергии, их необходимо отсчитывать от одного и того же уровня. В данном случае удобнее всего взять за нулевой уровень бесконечно далекую точку. Тогда
t/з = -G = -mg3R3)
Я3
ид = _дтМл _ -т^лт?л.
Ял
Видим, что потенциальная энергия тела на Луне больше, чем на Земле, на величину
17
Uji - U3 = т (-gnRji + 03Я3) = —т^зЯз.
112
Задача 102 (81-г)
Космонавт, выйдя в открытый космос, остался связан с кора-'• и м посредством троса, длина которого I = 64 м. Масса космо-пип га. т = 100 кг, масса троса пренебрежимо мала, масса корабля \1 Ют. Найдите силу натяжения троса, считая, что корабль на-мгштся между космонавтом и Землей, а трос все время направлен iiin.ni> прямой, проходящей через центр Земли. Орбита корабля — окружность радиусом R = 6400 км, ускорение свободного падения д пн прбите корабля равно 10 м/с", размеры корабля много меньше I.
т
О
т
-т (л7)
Mg
Рис. 45. К задаче 102.
Решение. Силы, действующие на космонавта и корабль, пока-н|иi.i па рис. 45. На космонавта действуют сила тяжести mg' и сила ппгяжепия троса Т, направленные к центру Земли. На корабль дей-• inyioT сила тяжести Mg, направленная также к центру Земли, и • ила натяжения троса —Т, направленная в противоположную сторону. (Учитываем, что ускорение свободного падения на орбите ко-|ш1 кия и на орбите космонавта отличаются.)
Из условия задачи следует, что космонавт и корабль движутся вокруг Земли с одинаковой угловой скоростью си и обладают цен-।рогтремительными ускорениями си2(Я + /) и urR соответственно. < 'огласно II закону Ньютона движение космонавта и корабля по окружности описывается формулами
+ /) = тд' + Т, Mu2R = Мд - Т.
(1)
(2)
') га система из двух уравнений содержит два неизвестных си и Т. Решая ее относительно Т, получаем
mM[/g + R(g - /)]
MR + m(R + l) ’	W
Оценим величину д — д'.
113
Так как д обратно пропорционально расстоянию R до центра Земли, ~ 1/Я, то можно написать:
/ r2 Г
3 -5(л+02 * * V 2Я/ ’
ИЛИ
д-д'^2д^.	(4)
гС
Видим, что вкладом разности ускорений (4) в формуле (3) пренс бречь нельзя, и получаем
т	„ * 3-4 * з. ю-н. (6|
MR+m(R + l) yR	v '
В процессе решения мы пренебрегли силой притяжения между кораблем и космонавтом F = GmM/I2 « 2 • 10”8Н. Поскольку F <£ Ту это пренебрежение вполне оправдано.
Задача 103 (77-р)
Спутник летает вокруг Земли по круговой орбите радиусом R -25600 км. Во сколько раз скорость спутника отличается от первой космической?
Решение. Центростремительной силой для летающего но круговой орбите спутника является сила тяжести F = GmM/R\ где G — гравитационная постоянная, т и М — массы спутника и Земли соответственно. Эта сила создает центростремительное уско рение v2/R} где v — линейная скорость спутника. Согласно II закону Ньютона (F = та)
GmM/R2 = mv2/R.	(1)
Первая космическая скорость vj — это скорость спутника, движущегося по круговой орбите, радиус которой совпадает с радиусом Земли. Таким образом, имеет место равенство
тМ v?
G~i%=m-R3
(2)
Деля почленно равенство (1) на равенство (2) и извлекая квадрат-
ный корень, получаем:
v/n = y/R^/R = 1/2.
(3)
114
5. МЕХАНИЧЕСКИЕ КОЛЕБАНИЯ
Р<мнение задач на механические колебания сводится обычно к in  игдованию свойств решения обыкновенного дифференциального 11 hi in юпия второго порядка
। Питч! решение этого уравнения можно представить в виде линейной н»мбипации функций sincj/ и cos tut с произвольными постоянными ♦••иффициентами а и 6:
/(/) = a sin cot + frcoswf.	(2)
И и функция описывает гармонические колебания. Она может iihiri» записана в другом виде:
/(f) = A sin(wf + у?о),	(3)
нк’ /I и — произвольные постоянные, имеющие наглядный физи-|к кий смысл амплитуды и начальной фазы колебаний. Действи-н’льно, максимальное и минимальное значения функции (3) равны I и -А соответственно, а ее значение при t = 0 есть Asin^o- Связь им ряжений (2) и (3) устанавливается с помощью тригонометриче-• к их преобразований: А = х/а2 + Ь2 и ^>о = arctg
Исследование решений уравнения (1) и связанных с ним уравнении, описывающих колебания с затуханием (под действием трения), |111лсбаний под действием внешней силы и многих других, это инте-|н | пая и достаточно сложная математическая задача. Обратимся к физической стороне описываемых явлений.
Для существования колебаний необходимо наличие определенных условий. Колебания—это периодические изменения параме-। рои системы в окрестности некоторых средних значений. Следова-н ,|||>но, сама система должна находиться в состоянии устойчивого рппповесия. Для механических систем это означает наличие минимума потенциальной энергии (равновесного положения). При выводе системы из положения устойчивого равновесия возникают си-Н.1, возвращающие ее к начальному состоянию. Обычно физическая чисть задачи и заключается в определении положения устойчивого рппповесия системы и “возвращающей силы”. Степень отклонения
115
системы от положения равновесия описывается некоторым парам!» тром (например, координатой). Гармонические колебания возникп ют в том случае, когда при отклонении системы от положения рпи новесия возникают силы, направленные к положению равновесия и пропорциональные параметру отклонения.
Характерный пример — тело массой т, подвешенное на упругий пружине с жесткостью k. При растяжении пружины на величину A# возникает сила F = —kx. Уравнение движения тела та — ~ki совпадает с уравнением (1) при w2 = k/m.
Определение уравнения движения тела в окрестности положе ния равновесия и приведение его к виду (1) и составляет физической содержание задач на механические колебания.
* * *
Задача 104 (74-р)
На середине сильно натянутой невесомой упругой струны длиной / укреплен маленький шарик массой т. Найдите период малых вер тикальных колебаний шарика около положения равновесия, если натяжение струны F намного больше тд, так что действием силы тяжести можно пренебречь.
Рис. 46. К задаче 104.
Решение. На рис. 46 показаны силы натяжения струны F, действующие на шарик А, отклоненный на расстояние 6х от по ложения равновесия. Величину их равнодействующей Fe нетрудно найти, рассматривая подобные треугольники АВС и АВ'С7:
Fs = 4F6x/l.
Сила F% прямо пропорциональна смещению 6х, коэффициент пропорциональности равен k = 4F/1. Такая сила приводит к гармоническим колебаниям с периодом
Т — Ъя^/т/к — 'Ky/ml/F.
116
Задача 105 (77-г)
Невесомая пружина с жесткостью k имеет в свободном состоянии д,л ину /о. К пружине прикреплены два тела, связанные нитью пипой I (/ < /0). Тела расположены, как показано на рис. 47. Macri ।сл Mi и М?- В некоторый момент нить перерезают. При каком и|||'|('пии I нижнее тело оторвется от земли?
Рис. 47. К задаче 105.
Решение. После разрыва нити верхнее тело под действием си-н.1 упругой деформации пружины начинает совершать гармониче-• кие колебания. В процессе этих колебаний длина пружины может Iнн тянуться настолько, что сила натяжения превысит силу тяже-• in нижнего тела. Следовательно, для решения задачи необходимо определить максимальную длину пружины в процессе колебаний in рхнего тела.
Колебания будут происходить с некоторой амплитудой А отно-• нгельно положения равновесия верхнего тела. В положении равновесия длина пружины li такова, что сила упругой деформации I in ни а весу верхнего тела:
। <*. li = Iq — Mig/k.
Амплитуду колебаний верхнего тела можно найти как разность иачальной длины и длины пружины в положении равновесия
А = li - I = l0 - I - Mig/k.
Максимальная длина пружины /тах при колебаниях верхнего тела равна
/max — / + 2А = 2/i — I — 2/q — I — 2Mig/k.
Нижнее тело оторвется от земли, если будет иметь место неравенство
М2д < кДЛ,
। цг Д/— растяжение пружины. Максимальное растяжение пружины равно
(Д/)тах = /плах ~ /о = /о ~ ~ %Mig/k.
117
Нижнее тело оторвется от земли, если
М2д < Ц!о - 0 - 2М\д,
откуда следует условие на начальную длину пружины
/</о-(М2 + М1)^Д.
Задача 106 (77-г)
Поверхность сплетенной пауком паутины горизонтальна, Корди паук находится в ее центре, паутина прогибается, величина прогибь я о = 1 мм. Муха, попав в паутину, начинает биться. Можно лн по частоте резонансных колебаний определить, превосходит масти мухи массу паука или нет?
Решение. Паутина представляет собой упругую систему с нулевой массой. Считая, что при малых растяжениях паутины действующие в ней силы подчиняются закону Гука, можно написа н, следующее уравнение:
mQg = kxQ.	(I)
Здесь mo — масса паука; д — ускорение свободного падения; к — ко эффициент упругости паутины.
Паутина, нагруженная пауком массой то, и паутина, нагружен ная мухой массой т, представляют собой разные колебательные см стемы. Круговые частоты wq и и собственных (резонансных) коле баний этих систем определяются формулами
о>о = 2тг/7о = у/к/тъ, w = 2тг/Т = у/к/т.	(2)
Здесь То и Т—периоды собственных колебаний систем.
Из системы уравнений (2) нетрудно получить равенство
m/mo=uio/w2,	(3)
содержащее не известную пока величину wq. Последнюю нетрудно найти, исключая с помощью (1) из первого равенства (2) отношение Л/т0:
Wo = х/^/жо-	(4)
Подставляя (4) в (3), находим
m/mo = д/ (®ow2).
Отсюда ясно, что если масса мухи превосходит массу паука, то w < \/д/х0,
118
ни, с учетом численных данных задачи, эквивалентно условию
ш < 100с-1.
Hiпк, но частоте резонансных колебаний паутины с мухой можно • «и । и делить, превосходит масса мухи массу паука или нет.
Задача 107 (77-р)
Шарик массой т, подвешенный на невесомой нерастяжимой ни-। и, вращается по окружности вокруг вертикальной оси с постоянной и повой скоростью ы (эта система носит название “конический Mail ।ник”, рис. 48, а). Где нить конического маятника должна быть прочнее— на Земле или на Луне?
Рис. 48. К задаче 107.
Решение. Обозначим через R радиус АО1 окружности, по ни горой вращается шарик, и через I— длину нити АО. Центростремительная сила
F = mu2R,	(1)
oi.i пявающая вращение, направлена по горизонтали к центру наружности и является равнодействующей силы натяжения нити Г и силы тяжести mg. Из очевидного подобия треугольника сил и । ргугольника AOOf (рис. 48, б) следует равенство:
R/l = F/T.	(2)
Гнная пропорцию (2) относительно Т с учетом (1), находим
Т = тш21.	(3)
Нидпо, что натяжение нити Т не зависит от ускорения свободного ।тления ду которое на Земле и на Луне разное.
Таким образом, нить конического маятника на Земле и на Лупе должна выдерживать одинаковое натяжение Т и, следовательно, пилжна иметь одинаковую прочность.
119
Задача 108 (78-г)
Два одинаковых массивных шарика скреплены невесомой пру жипой. Шарики летят с одинаковой скоростью v, как показано пи рис. 49, а. Сила тяжести отсутствует. Как будут двигаться шарн ки после столкновения со стенкой, если удар о стенку абсолютна упругий и время удара шарика о стенку много меньше периода сой ственпых колебаний системы?
Рис. 49. К задаче 108.
Решение. То, что до столкновения со стенкой шарики летя'1 с одинаковой скоростью v, означает, что во время полета пружина не деформирована. В процессе удара о стенку первого шарика про исходит практически мгновенное изменение его скорости на проти воположное. Так как по условию задачи время удара много меньше периода собственных колебаний системы, то за время удара второй шарик не успевает изменить ни своего положения в пространстве, ни скорости. Картина, складывающаяся сразу же после удара, показа на на рис. 49, б: пружина не сжата, а шарики 1 и 2 имеют скорости —v и v, направленные к центру пружины. Дальше система движет ся под действием внутренних сил, поэтому ее центр масс покоится. Сначала шарики сжимают пружину, а затем она распрямляется до недеформированного состояния. В этот момент скорости шарикои 1 и 2 имеют значения v и —v, и шарик 1 снова приходит в сопри косновепие со стенкой (рис. 49, в). Во время второго удара шарик 1 изменяет свою скорость от значения v до значения -v, а шарик 2 не успевает изменить ни своего положения, ни своей скорости. Таким образом, мы приходим к ситуации, когда пружина не деформирована, а оба шарика имеют одинаковые скорости — v, с которыми и улетают от стенки. Эта последняя фаза процесса показана на рис. 49, г.
Задача 109	(78-г)
Малое тело массой т = 10 г подвешено на нити АВ к пружине с жесткостью k = 10кг/с“ (рис. 50). Пружина и нить невесомы, нить нерастяжима. Система совершает гармонические колебания в
120
и» pшкальном направлении в поле тяжести. Может ли амплитуда А • ।их колебаний быть равна 1см?
иЩЦШШЩШШШ!
Г л в
Рис. 50. К задаче 109.
Решен и е. Обозначим длину пружины в недеформированном • и. |()янии /о- Если система, описанная в условии задачи, находится н । <>1 гоянии равновесия, то пружина растянута силой тд и ее длина / /,> I-Д/о, где
. , mq 0, 01 кг • 9, 8 м/с2
Д/о =	= -2-------  ' = 0,98 см.
к 10 кг/с2
\мн.читуда гармонических колебаний груза не может быть больше \/()( гак как при смещении груза вверх относительно положения |нншовесия на расстояние Д/ > Д^о сила натяжения нити равна пуню. Нить может “тянуть” пружину вниз, растягивая ее, но не м«»жет “толкать” ее вверх, сжимая.
Движение груза, смещающегося от положения равновесия вверх на расстояние больше Д/о, уже не гармоническое — это дви-жriiне в поле силы тяжести с постоянным ускорением д.
Итак, поскольку А = 1см > Д/о = 0,98 см, система не может < «щгршать гармонических колебаний с амплитудой А = 1см.
Задача 110 (78-г)
Найдите все возможные значения угловой частоты w колебаний конического маятника, образованного малым телом массой т, подпиленным на пружине с жесткостью k в поле тяжести. Длина пружины в нерастянутом состоянии /, масса ее пренебрежимо мала.
Решение. Описанный в условии задачи конический маятник • тематически изображен на рис. 48, а (к задаче 107).
Угловая частота w колебаний такого конического маятника • 1111 и 'деляется формулой
w = 2тг/Т,	(1)
। не Т — период колебаний (время, необходимое для прохождения ir.iioM полной окружности). Поскольку Т является функцией угла
121
а, составляемого осью пружины с вертикалью 00' (0 < a < тг/2|, то w также есть функция угла а. Найдем зависимость w(a).
На тело массой т действуют две силы: сила тяжести mg и сили натяжения пружины Т (рис. 48, б). Сумма этих сил F должна бын. направлена к оси ОО' и равна
F = ти2/Я,	(7)
где
R = I sin a	(II)
— радиус окружности, по которой движется тело. Полную окру ж ность тело проходит за время
T=2ttR/v.	(4)
Проецируя силы на горизонтальное направление, получаем
F = mgtga.	(5)
Сила натяжения пружины определяется ее растяжением
T=k(l-lQ),	((I)
где I—длина растянутой пружины. С другой стороны, тело не дви гается в вертикальном направлении, следовательно
T=mg/cosa.	(7)
Из уравнений (1)-(7) находим
w2(a) =------.
m# + kiQ cos а
Принимая во внимание, что 0 < а < тг/2, для искомой области изменения си получаем
^min W < Wmaxj
где
_ /	_ г*
^min — \	, 1 ] > ^тах — \1
у тд + kIq	V т
Значение tvmax равно собственной частоте малых вертикальных ко лебаний пружинного маятника. В действительности частота w нс может достичь значения wmax. При а —> тг/2, из формулы (7) получаем, что Т —> оо. Но сила натяжения пружины не может превосходить некоторой предельной величины, при достижении которой пружина рвется.
122
Задача 111 (79-г)
Два стержня одинаковой длины соединены под прямым углом •• • родней точке и насажены на горизонтальную ось О. На концах и р ж ней закреплены грузы пренебрежимо малых размеров с мас-• ими ?П1, m2, тз, Ш4, равными соответственно 1, 2, 3 и 4 кг. Си-• к му привели в положение, показанное на рис. 51, а, и отпустили. II it Пл ите угловую амплитуду возникающих колебаний. Трение в оси ..11 у гствует.
Рис. 51. К задаче 111.
Решение. Система совершает колебания около положении устойчивого равновесия. В положении устойчивого равновесия in игр тяжести системы находится под осью О на одной вертикали с iirll. Такое положение центра масс будет только в том случае, когда । ру 1Ы с массами mi и m2 расположены точно над грузами и тз и ответственно (рис. 51, б). В это положение система приходит из и< ходкого путем поворота около точки О по часовой стрелке на угол и тг/4. Поскольку в исходном положении момент сил тяжести от-..нтельно оси О действует в этом же направлении, то угол а = к/4. и является угловой амплитудой возникающих в системе колебаний.
Задача 112 (79-г)
Брусок массой т, находящийся на шероховатой поверхности, прикреплен к стене пружиной, жесткость которой к. Коэффициент треки я между бруском и поверхностью //. Брусок сдвигают на рассто-нниг до от положения, при котором пружина недеформирована, и игпускают. Сколько колебаний совершит брусок до полной остановки? Считать, что начальное смещение #о направлено вдоль оси пру-/кппы перпендикулярно к стенке и длина пружины L я0-
Решение. Воспользуемся системой отсчета, ось Ох которой направлена от стенки вдоль оси пружины, и за начало отсчета шнвмем точку О, в которой находится центр тяжести бруска при in деформированной пружине.
Рассмотрим силы, действующие на брусок в горизонтальном направлении. Координата центра тяжести бруска х. На него со сторо
123
ны пружины в направлении оси Ох действует сила
F = -kx.	(I)
Сдвинем брусок от точки О па расстояние Xq. Тогда со стороны пружины на него будет действовать сила Fq = — kxQ. Чтобы пил действием этой силы брусок пришел в движение, надо, чтобы ее аб солютная величина была больше максимальной силы трения покои -Ftmax = т. е. для начала движения необходимо выполнение неравенства
|F0 | > FTmax »	(^)
ИЛИ
|з?о| > ртд/к.	(3)
В противном случае действие силы Fq уравновешивается силой трг ния покоя и брусок в движение не приходит.
На движущийся брусок кроме упругой силы действует сила трения скольжения, равная ртд и направленная в сторону, протн воположную движению. Выпишем уравнение движения бруска:
та = — kx =F ртд.	('I)
Здесь а — ускорение бруска. Верхний знак отвечает движению брус ка от стенки, нижний—движению к стенке. Исследуем характер движения, описываемого уравнением (4).
Для упрощения исследования уравнение (4) удобно записать п виде
та = — к (х ± цтд/к).	(5)
Если в уравнение (5) вместо координаты х ввести новую координату х1 по формуле
х1 = х ± цтд/к,	(6)
то уравнение движения примет вид
та = —кх'.	(7)
Это уравнение гармонических колебаний около положения равнове-сия х1 — 0. При движении бруска по направлению к стене корнем уравнения <г'(я) = 0 является величина
ж* = цт-д/к,	(8)
при движении в противоположном направлении — величина —ж*. Итак, сдвинутый относительно точки х = 0 брусок при движении к
124
н им* совершает в течение полупериода гармоническое колебание •hullo точки х*, а при движении от стенки — также в течение по-ц периода гармоническое колебание около точки —я*. Колебательные движения бруска продолжаются до тех пор, пока его остановки происходят вне промежутка [— аГ, <т*] оси х, т. е. до тех пор, пока ..рдипаты х точек остановки подчиняются неравенству
И>*Л	(9)
П1н.'|огичному неравенству (3). Колебания сразу же прекращаются, ник только очередная остановка бруска происходит внутри промели гка [—жж, я*], т. е. в том случае, когда брусок останавливается в ih'ikc с координатой х, удовлетворяющей условию
|х|<яж.	(10)
цовимся координаты точек остановки бруска после совершения ?-in колебания отмечать индексом i: Xi —это координата бруска после Фопчания первого колебания, — координата бруска после окончи и и я второго колебания и т. д. Установим зависимость координаты I, от номера колебания г. С этой целью более детально проследим hi движением бруска, который стартует из точки с координатой in Для определенности будем считать, что а?о > 0- В этом слупи* брусок начинает двигаться по направлению к стенке и соверша-• । в гонение полупериода колебание около точки ж* с амплитудой li - .го —	. Окончив колебание, брусок останавливается в точке
з?! = xq — 2А1 = —#0 + 2я*.
Hi гонки з?1 брусок начинает двигаться (если |.т| > а?*) в положи-i ильном направлении оси х и совершает колебание около точки — х* । амплитудой А2 = —£Г* — Завершив колебание, брусок остана-п чикается в точке с координатой
Х2 — Х\ + 2А2 — х$ — 4х*.
II процессе совершения третьего колебания брусок движется по направлению к стене и совершает это колебание около точки а?* с амплитудой Лз = Х2 — Xя. Остановка происходит в точке с координатой
Хз = Х2 - 2Аз = -Xо + 6.г*,
и г. д. Использовав метод математической индукции, нетрудно по-игтть, что, завершив колебание с номером г, брусок остановится в ।очке с координатой
Xi = (-1)Джо — 2га?*).	(11)
125
Это и есть искомая зависимость координаты Х{ от номера колебании г. Обозначим через п число колебаний, совершаемых бруском л»' окончательной остановки. Тогда
жп-1 = (-1)” 1 (®о - 2(п — 1)ж*),	(!!!)
хп = (-1)" (®о - 2пх")	(Cl)
суть координаты предпоследней и последней (окончательной) ости новки бруска. Подставляя в (9) х = яп-1 и в (10) х = хп, приходим к следующей системе неравенств:
а?о	1	з?о	1
-------< п <-------—,
2х*	2-	2х*	2’
которая с учетом (8) принимает вид
kXQ 1
2}img 2 ~ П 2цтд 2’
Данные неравенства однозначно определяют искомое целое число п, так как величины, ограничивающие его в (14) сверху и снизу, отличаются друг от друга ровно на единицу.
При получении формулы (14) мы предположили, что яо > 0 Если от этого предположения отказаться, то для нахождения числи п получим следующую систему неравенств:
Л1аг°1	1 „ X- *1®°!	1	им
2цтд	2 ” 2}гтд	2	v
справедливую и для отрицательных Число п может быть вы ражено и через посредство равенств, если воспользоваться примо няемым в математике для обозначения целой части произвольного числа А символом [Л]: если	| есть целое число, то
&|а?о|	1
71 = ------— ,
2ртд 2
в противном случае
’фо| Г
П — о------Ь й •
2^тд 2
Задача 113 (80-г)
Две невесомые пружины имеют длины , I2 и жесткости fci, Одна пружина вставлена в другую. Концы пружин попарно скрс плены. Другими точками пружины друг друга не касаются. Какова жесткость к получившийся пружины?
126
I' r in е н и е. Для определенности будем считать, что /1 > /2. I Hinn / составной пружины удовлетворяет неравенству 1% > I > 1\.
Пусть под действием силы F, направленной вдоль оси, система 'Н’кип растягивается (сжимается) на какую-то величину Дх. По н||»мцу Гука
F = JtAx,	(1)
при пом к первой и второй пружинам оказываются приложенными in in н орне силы Fi, F2, такие, что
F = Fx+F2.	(2)
• и. для определенности Дж > Z2 — / . Тогда обе пружины растяну-। I I и гилы Fi, F2 направлены в одну сторону. Проецируем векторное । mii'iK'TBo на направление оси растянутых пружин:
F = Fi + F2.	(3)
Hi hi. действием сил Fi, F2 пружины растянуты относительно их по-|нжг|1ий равновесия на величины I —4- Дж и /2 — I + Дя соответ-।nriiiio. По закону Гука
Fi = Лх(г - + Дж),	(4)
F2 = fc2(Z2 — / + Дж).	(5)
II । равенств (1)—(4) получаем уравнение
АгДж = А?1(/ — /1) 4- &2(Z2 — I) 4" (ki 4“ &2)Дж,	(6)
hnn»poe должно выполняться при любых значениях Дж, в том числе и при Дж = 0. Это возможно только в том случае, если
Ы*-*1) + Ы*2-П = о.	(7)
1' учетом (6) из (5) находим: к = к\ 4- А?г-
Задача 114 (80-г)
Вертикальный абсолютно жесткий невесомый стержень длиной I in креплен в нижней точке с помощью шарнира. К верхнему концу стержня прикреплена точечная масса т. Система удерживается и положении устойчивого равновесия с помощью двух одинаковых пгпгеомых горизонтальных пружин, имеющих жесткость к (рис. 52, ii) Найдите период малых колебаний, возникающих в системе при |п.1ведении ее из положения равновесия.
127
Рис. 52. К задаче 114.
Реше н и е. Выведем систему из положения равновесия, повгр ну в стержень на угол а. Точечная масса т при этом смещается я горизонтальном и вертикальном направлениях соответственно на
х — Zsina,	(1)
у — 1(1 - cos а).	(2)
Так как по условию задачи угол о мал, то sin а а и cos а ъ 1. ( учетом этого формулы (1) и (2) принимают вид
х « /о,	(3)
У * 0.	(4)
Таким образом, при малых колебаниях смещением точечной маг сы в вертикальном направлении можно пренебречь. Следовательно, можно с достаточно хорошей степенью точности считать, что при малых ос силы Fi и Fo, приложенные к точечной массе т со сторо ны сжатой и растянутой пружин, направлены по горизонтали в одну и ту же сторону. Для величины их равнодействующей Fynp можно написать:
^упр = Л + ^2 = 2А’ж.	(5)
Кроме F на массу т действуют сила тяжести n?g и сила реакции стержня N (рис. 52, б). При малых а в вертикальном направлении тело не смещается, следовательно, проекция на вертикальное на правление результирующей F всех приложенных к массе сил равна нулю. Отсюда
N cos а « N = тд.	(6)
Проекция результирующей на горизонтальное направление
Fx(x) = N sin а - Fynp « A'a — 2kx.	(7)
С помощью уравнений (3) и (6) равенство (7) легко преобразуется к виду
Fx(x) = -l^x,	(8)
128
I не
A’i = 2k — mg/l.	(9)
11 'in 2k > mg/l, то сила Fx(z) направлена к положению равновесия н является гармонической силой. В результате ее действия возникнет малые гармонические колебания с периодом
Т — 2ъ\/т/к\ — 2тгу/т1/(2к1 — тд).
I г ни же 2к < тд/1, то положение равновесия, отвечающее значению |»	0, неустойчиво, поскольку в этом случае сила Fx(x) либо равна
пулю, либо направлена от положения равновесия. В таком случае • ила Fx(x) не является гармонической, и колебания в системе не шиникают.
Задача 115 (81-р)
Лифт поднимается на некоторую высоту, причем первую половину пути он движется равноускоренно, а вторую —равнозамедленно in полной остановки. После этого движение повторяется в обратную • ।ирону. Отстанут или уйдут вперед маятниковые часы, установленные в лифте, по сравнению с такими же часами, покоившимися на нмле? Маятник в часах можно считать математическим и соверши ющим малые колебания, а ускорение лифта меньше ускорения । победного падения.
Решение. Прежде всего исследуем зависимость периода Т Mail мх колебаний математического маятника, подвешенного к потолку кабины лифта, от ускорения лифта. Для этого рассмотрим, как ве-цгт себя свободно падающее тело в инерциальной системе отсчета и п системе отсчета, жестко связанной с ускоренно движущимся лиф-н>м.
Пусть ускорение лифта в инерциальной системы отсчета есть а. Ускорение свободно падающего тела в этой же системе отсчета есть ускорение свободного падения д. Тогда относительно лифта это тело и,нижется с ускорением
а' = д — а.	(1)
Умножая равенство (1) на массу падающего тела т, получаем
та' = т[д — а).	(2)
Введя обозначение
Fi =т(д-а),	(3)
приведем равенство (2) к следующей форме:
a* = F\/m,	(4)
129
не отличающейся от записи II закона Ньютона для инерциальной системы отсчета. Итак, в неинерциальной системе отсчета можно пользоваться уравнением движения тела для инерциальных систем, если, как следует из формул (3) и (4), заменить ускорение свобод ного падения g некоторым “эффективным ускорением свободно! и падения” g — а. При этом для периода Г' малых колебаний матемп тического маятника в неинерциальной системе отсчета получим
т' = 2^077,	(h)
где I — длина математического маятника, а
/=|g-a|.	(0)
В покоящемся лифте маятник колеблется с периодом
Т = ^уДГд.	(7|
С учетом формул (6) и (7) равенство (5) нетрудно привести к ело дующему виду:
т' =	= СГх/p/lg -а|.	(Н)
Если ускорение лифта направлено противоположно ускорению cnci бодного падения g, то
|g — а| =д + а,
где а = |а|, и формула (8) принимает вид
Т' = т^д/(д + а).	(9)
Если направление ускорения лифта совпадает с направлением уско рения свободного падения, то либо (при а < д)
|g-a| =д - а,
либо (при а > д)
|g — а| = а-д.
В первом случае формула (8) принимает вид
Г = Ту/д/[д — а), а < д.	(10)
Во втором случае вектор g' или равен нулю, или направлен вверх, и поэтому маятник, прикрепленный к потолку лифта, вообще н<* может колебаться. Формулы (9), (10) и дают искомую зависимость
130
периода Т* малых колебаний математического маятника, подвешен-Hi'io к потолку лифта, от величины а. Теперь нетрудно ответить и ни вопрос задачи.
Пусть при совершении маятником одного полного колебания секундная стрелка часов поворачивается на угол При совершении шинником п колебаний она поворачивается на угол
^ = п^0>	(И)
z это и есть показание часов.
Рассматривая описанное в условии задачи движение лифта, не-। РУ дно показать, что каждую четверть пути он проходит с одинаковыми по величине (но не по направлению!) ускорениями а за одно и io же время t. Полное время движения лифта
tf =4t.
За время tf маятник покоящихся на земле часов (часы 1), который колеблется с периодом Г, совершает
колебаний, а секундная стрелка поворачивается на угол
^ = п^о = 4ф.	(12)
Найдем теперь показания часов, установленных в лифте (часы ') Первую и третью четверть своего пути лифт движется с ускорением а, направленным вверх, и маятник часов 2 колеблется с перинном (формула (9))
Пюрую и четвертую части пути лифт движется с ускорением а, направленным вниз, и маятник часов 2 колеблется с периодом (формула (10))
Т' = Д = T.I-2-.
]/ д -а
In время 2/, в течение которого совершаются первая и третья части нути, секундная стрелка поворачивается на угол
ft t \	21 Г а
^14-з = П1+з9?0 — ¥>о	+ р J - 'ту 1 + “•
131
За время 2/, в течение которого лифт проходит вторую и четвергу hi четверть пути, стрелка часов 2 поворачивается на угол
ft t \ 2t Г а
<^2+4 = n2+4$Po = Fo +	= ^07771/1 - -•
\Т2	Т47 Ту д
За все время движения лифта стрелка часов 2 поворачивается ни угол
^--^1+3 + ^2+4 — ^0777(1/14---F 1/1--)• (Ь’П
Т \у д у д;
Чтобы сравнить показания часов, разделим почленно (13) на (12) и получим
Возводя последнее равенство в квадрат, получаем
следовательно, р/р' < 1.
т. е. часы, помещенные в лифте, отстают от часов, покоящихся пн земле.
Задача 116 (81-р)
Два шарика с одинаковой массой т соединены невесомой пру ж и ной, жесткость которой к. Эта система, двигаясь со скоростью гц, как показано на рис. 53, сталкивается с точно такой же покоящей ся системой. Происходит абсолютно упругий центральный удар inn риков. Опишите дальнейшее движение систем, считая, что врем и соударения шариков много меньше периода собственных колебаний систем, а длина недеформированной пружины I много больше вели чины vo y/m/k.
Решение. Рассмотрим движение шариков в системе отсчет, связанной с центром масс. В этой системе отсчета обе пары шариком летят навстречу друг другу с одинаковыми скоростями го/2. Из-чп симметрии конфигурации достаточно рассмотреть только одну пару шариков (для определенности левую). После первого столкновении правый шарик изменит направление скорости на противоположна^ и шарики в паре будут сближаться друг с другом с начальными
132
»•••!"( тями t>o/2- По мере сближения их кинетическая энергия будет н. |»« ходить в потенциальную энергию сжатой пружины.
Начальная кинетическая энергия пары шариков Eq = а •Mili niщальная энергия сжатой пружины = k(А/)2/2, где А/ — н 1М1Ч1гпие длины пружины. Максимальное значение А/ будет донн ну го, когда вся начальная кинетическая энергия перейдет в по-н иппальную: [7тах = ^о» Отсюда максимальное изменение длины пружины равно
Д/max = V0 y/m/k.
Ни условию задачи эта величина много меньше начальной длины иру/кипы, так что столкновение шариков в одной паре не произой-|. ।
После наибольшего сближения шарики начнут разлетаться под |. 11г гнием силы упругой деформации, потенциальная энергия пру-» и и и будет переходить в кинетическую энергию шариков. Шарики •»» риу гея в исходное положение со скоростями, равными начальным •и» in личине, но направленными в противоположные стороны. Про-п 1и||л,('т повторное столкновение правого шарика, после которого его i.npiK ть опять изменится на противоположную. Скорости шариков и н/шой паре окажутся одинаковыми не только по величине, но и по ||ннрпвлению. Пары будут разлетаться в противоположные стороны
• •/I и паковыми скоростями Vq.
II исходной системе отсчета после двух столкновений левая пара niiipiiKOB будет покоиться, а правая — двигаться вправо со скоро-II.|П
Vo
Рис. 53. К задаче 116.
V
Рис. 54. К задаче 117.
Задача 117 (82-р)
Три одинаковых шарика массой т каждый надеты на длинную Н1р1г1оптальную штангу, по которой они могут скользить без трения. Iiih из них связаны невесомой пружиной длиной I с коэффициентом упругости fc; оба шарика покоятся. Третий шарик налетает на них и скоростью v, как показано на рис. 54, причем v <<С 1\/к/т. Опиши п4 движение шариков после абсолютно упругого удара третьего
133
шарика о второй. Время удара и размер шариков считать пренс6|м« жимо малыми. Найдите максимальное и минимальное расстоянии между первым и вторым шариками после удара.
Решение. Так как по условию время удара пренебрежим» мало, то длина пружины за время удара не успевает измениться и можно считать, что во время удара пружина на шарики не действ) ет. Поэтому удар происходит так, как если бы просто столкнули» I. шарики 2 и 3. Поскольку массы шариков одинаковы, а удар абсп лютно упругий, то из законов сохранения импульса и энергии слг дует, что в результате удара шарик 3 останавливается, а шарик У приобретает скорость v. Состояние системы сразу же после ударп шарики 1 и 3 покоятся, а шарик 2 движется налево со скоростью г
Дальнейшее движение шариков будем рассматривать в систем» отсчета, связанной с центром масс шариков 1 и 2. Эта система отсче та движется относительно исходной влево с постоянной скоростьи* v/2.
В новой системе отсчета шарики 1 и 2 после удара двигаются замедленно с начальными скоростями v/2, направленными к цен тру пружины, а шарик 3 движется налево с постоянной скоростьн» v/2. Шарики 1 и 2 приближаются к центру пружины до тех пор, п»» ка их скорости, а следовательно, и кинетические энергии не буду i равны нулю. В этот момент расстояние между ними минимально, (/min), их начальная кинетическая энергия m(v/2)2 перешла в потен циальную энергию упругой деформации сжатой пружины, равную Аг(/ — tain)2/2. Так как трения нет, то по закону сохранения механи ческой энергии
откуда находим
Из условия задачи следует, что правая часть равенства (1) положи тельна, следовательно, столкновения шариков 1 и 2 не происходит,
Затем под действием сжатой пружины шарики 1 и 2 приходят н движение, удаляясь от ее центра. К тому моменту, когда длина пру жины снова становится равной Z, шарики, как это следует из закона сохранения энергии, имеют одинаковые по величине скорости, равные v/2 и направленные от центра пружины. Дальнейшее движение шариков растягивает пружину, при этом их скорости падают до тех пор, пока не станут равны нулю. В этот момент расстояние между шариками максимально (Zmax), их начальная кинетическая энергия
134
HHiiiior.Tbio перешла в потенциальную энергию упругой деформации ГФ глпутой пружины fc(/max — J)2/2- Воспользовавшись законом со-|н11|спия энергии, находим
/max = / + Vx/m/(2k}.
II последующем шарики 1 и 2 снова начнут сближаться друг с дру-|нм. Из приведенного анализа следует также, что повторного столк-нн||<ч1ия шариков 2 и 3 не произойдет — скорость шарика 2 не пре-ihн ходит -и/2.
Итак, в движущейся системе отсчета шарики, скрепленные пру-•киной, колеблются около своих начальных положений. Найдем ча-• югу этих колебаний. При смещении шариков 1 и 2 от положения рппповесия на величину х пружина оказывается сжатой (или растя-io юй) до размеров Z 2я:, и на шарики 1 и 2 с ее стороны действует . iii.i F = —2кх. Таким образом, уравнением движения шарика 1 (и шарика 2) служит равенство
та = —2кх.
ho уравнение гармонических колебаний с частотой w = yfak/m. В исходной (неподвижной) системе отчета после удара шарик 3 покошен, а центр пружины с шариками 1 и 2 на концах движется влево • и скоростью и/2, при этом пружина периодически (с частотой и) in сжимается, то растягивается.
Задача 118 (82-р)
Маленький шарик массой т закреплен на конце невесомой нерпе гяжимой нити, другой конец которой закреплен в точке О. Если нимало координат поместить в точку О, а ось Oz направить вертикально вниз, то траекторию движения шарика можно описать уравнениями (x(t})2 -h = Я, z(t) = zq. Найдите натяжение нити и • корость шарика.
Решение. Приведенная в условии задачи система уравнений описывает расположенную в горизонтальной плоскости окружность с радиусом R и центром в точке О1, имеющей координаты ,г у = 0, z = zq. Таким образом, подвешенный на нити шарик движется по окружности (конический маятник). На шарик действуют гилы натяжения Т и тяжести mg. Их равнодействующая F в любой момент направлена к центру О1 окружности и постоянна по величине. Она сообщает шарику центростремительное ускорение v2/R. Но II закону Ньютона
F = mv2/R.	(1)
135
Как и в задаче 107, см. рис. 48, рассмотрим подобие треугольники АОО* и треугольника сил и выпишем пропорцию:
= R/zq,
откуда получаем
F = Rmg/z$.	(2)
Исключая из (1) и (2) величину F, находим линейную скорость шарика
v = Ry/g/zv
и угловую скорость
w = x/q/zq.
Рассмотренные выше треугольники прямоугольные, из чего следует
Т= \/Н2Ш
Подставляя сюда F из (2), находим
Т = mg\J\ 4- (ЛДо)2.
Задача 119 (83-г)
Маленький кубик массой m лежит на горизонтальной поверхно сти. Коэффициент трения между ними ц. С помощью горизонтально расположенной пружины жесткостью k кубик соединяют с верти калькой стенкой и отводят от положения, при котором пружина ш» деформирована, на расстояние А, меньшее длины нерастянутой пру жины, после чего отпускают. Найдите длину пути, который пройде! кубик до остановки.
Решение. Пусть $ — искомая длина пути, проходимого куби ком до окончательной остановки, Дхо > 0 — отклонение кубика oi положения, в котором пружина не деформирована, в момент окоп нательной остановки. Воспользовавшись законом сохранения энер гии, получаем соотношение
kA2/2 - й(Дяо)2/2 = nmgs.	(1)
Здесь kA2/2 — начальная потенциальная энергия пружины; &(Дяо)2/2 —потенциальная энергия пружины после остановки кубика; pmgs — работа силы трения FT = p>mgy совершенная за все время движения кубика. Из (1) для s находим
s = А2-(Джо) 7(26),	(2)
136
। ih* для краткости записи введено обозначение
Ь = р>тд/к.	(3)
Формула (2) позволяет найти з, если известна величина Для ни хождения Д#о рассмотрим движение кубика более подробно.
I (вправим ось Ох от стенки, а начало координат выберем в точке, и шпорой находится кубик при нерастянутой пружине. При движении кубика на него со стороны пружины действуют сила F = — кх и « ила трения FT = цтд. Эта сила в процессе движения направлена против вектора скорости кубика v. Проекция этой силы на ось х lniniia —{гтд, когда кубик движется от стенки, и +ртд при движении к стенке. В связи с этим уравнение движения кубика от стенки имеет вид
та = —кх — цтд.
Снижение кубика к стенке описывается уравнением
та = — кх + цтд,
II них уравнениях а — проекция вектора ускорения пружины на ось । (! использованием обозначения (3) уравнения движения принимают вид
та = —к(х 4- 6),	(4)
та = —к(х — Ь).	(5)
Уравнения (4), (5) показывают, что при движении от стенки кубик н |гчение полупериода совершает “гармоническое колебание” около 1»»чки с координатой х = — Ь. Двигаясь затем к стенке, в течение еле-нющего полу периода кубик совершает “гармоническое колебание” .ihiuio точки с координатой х = Ьу и т. д. Колебательное движение иубика продолжается до тех пор, пока следующие друг за другом •н гановки происходят вне промежутка [—6,6] оси ж, т. е. до тех пор, инка координата х точки остановки подчиняется неравенству
l®l > Ь.	(6)
Колебания сразу же прекращаются, как только очередная останов-пн кубика происходит в пределах промежутка [—&, 6], т. е. когда координата х остановки удовлетворяет условию
И <6.	(7)
137
В самом деле, если имеет место условие (7), то величина ствующей на кубик в точке остановки упругой силы |F| = А’|т| » kb = цтду т. е- меньше максимально возможной силы трения покоя Поэтому при |ж| < 6 упругая сила не может преодолеть силы трении покоя.
Рассмотрим возможные случаи. Для определенности будем счи тать, что А > 0.
1)	Пусть 0 < А < Ъ. В этом случае кубик вообще не сдвинется i места, так как с самого начала находится внутри полосы (7), т. е н данном случае Дяо = А и, следовательно, s = 0.
2)	Пусть теперь А выбрано так:
Ъ<А<ЗЪ.	(Н)
В этом случае неравенство (6) не выполняется, и кубик начинайi двигаться к стенке. Он совершает пол периода гармонического ко лебания около точки х = b с “амплитудой”, равной разности А - Л, скорость кубика обращается в ноль в точке с координатой
Х1 = 6- (Л-6) = 26- А.	(1)|
Величина a?i может быть как положительна, так и отрицательна, но при этом, как нетрудно показать, используя (8), — b < a*i < 6, т.с точка с координатой a?i лежит внутри полосы (7) и, следовательно, является точкой окончательной остановки кубика. Следовательно, в случае (8) Да?о = |zi| = |26 — Л|, и из равенства (2) находим
Л^-(2к-ЛУ=2А_ь
26
3)	Пусть отклонение А удовлетворяет неравенствам
36 < Л < 56.	(10)
Как и в предыдущем случае, кубик останавливается в точке с коор динатой «1, определяемой формулой (9); однако, в отличие от пред шествующего случая, в силу (10)	< —6, т. е. остановка происходи !
вне полосы (7). В связи с этим после остановки начинается движе ние от стенки и при этом совершается пол периода “гармонического колебания” около точки х = — 6 с “амплитудой”, равной разности —6 - xi = А — 36. Очередная остановка происходит в точке с координатой
z2 = —6 + (Л — 36) = А — 46.
138
Hi ппльзуя (10), нетрудно показать, что — Ь < Х2 < 6, и, следователь-нн( п точке с координатой кубик останавливается окончательно. » м’цовательно, в случае (10)
Да?о = |я2| = И - 46|,
и и । (2) получаем
$ = 4(Л-26).
I) Теперь нетрудно рассмотреть и общий случай. Пусть начальник о тклонение кубика А таково, что имеют место неравенства
(2N - 1)6 < А < (2N + 1)6, N = 0,1,...	(И)
II этом случае, поступая аналогично и используя метод математиче-। iioll индукции, находим, что кубик окончательно останавливается и непсе с координатой
= (-1)W(A - 2Ж) Дг = 0,1,-••	(12)
। мгдовательно в случае (И)
Да?о = |sjv| —- И ~ 2N61,
и in (2) находим
s = 2N(j4 —N6).	(13)
Заметим, что число в формулах (И)—(13) не может быть взя-|п сколь угодно большим. В самом деле, по условию задачи рассто-ииие А не может быть больше длины I нерастянутой пружины, что накладывает ограничение на выбор числа N
(2JV + 1)6 < L
кому неравенству эквивалентно следующее:
< I - 6 _ kl — цтд
~~ 26 2р>тд
Итак,
N<NQ)
|Де
_ Ikl — pmg
0 2цтд символом [ ] обозначена целая часть числа.
139
Задача 120 (83-р)
Как известно, при вычислении периода колебаний математичо ского маятника делается замена sin а на а, где а— угол откло нения нити маятника от вертикали. Выясните, меньше или болыш» вычисленный таким образом период по сравнению с реальным. Мп тематический маятник представляет собой точечную массу т, под вешенную на невесомой нерастяжимой нити длиной I.
Решение. При отклонении нити от вертикали на угол п точечная масса т проходит по дуге окружности с радиусом I путь длиной s = /а. Для проекции ускорения на направление касатель ной к траектории в момент времени /, когда нить составляет угол п с вертикалью, имеем
cPs сРа а = ТИ=1~№'
На направление касательной имеет составляющую только сила тя жести тд. В момент времени i эта составляющая равна
F = —тдsin а.
(1)
(2)
Согласно II закону Ньютона получаем
d2a
= -mgsma. at~
При рассмотрении малых колебаний (а <<С 1) в уравнении (1) обычно полагают sin а « а, и оно принимает вид
(Ра _ да 1Р~~Т
(уравнение гармонических колебаний). Так как | sin о| < |а|, то ясно, что при замене sin а на а мы увеличиваем правую часть, а следовательно, и левую часть уравнения (1). Увеличивая в каждой точке траектории угловое ускорение сРа/di2, мы увеличиваем и угловую скорости da/dt. Поэтому действительный период колебаний с заданной амплитудой оказывается больше значения Т = Ъку/Цд, получающегося из приближенного уравнения (2).
Задача 121	(86-г)
Тонкую легкую резину натянули обруч с радиусом г, расположенный в горизонтальной плоскости. В центре обруча к резине прикрепили маленькую гирю массой т. В поле тяжести с ускорением д = 10м/с“ гиря опускается вниз по отношению к плоскости обруча на величину о, а г. Оцените период малых колебаний гири в плоскости обруча.
140
Решение. Разобьем резину на маленькие секторы с углом каждый, при этом	= 2тг. Будем считать, что каждый
и’ктор подчиняется закону Гука, т. е. ведет себя как упругая пружинка. Тогда сектор действует на гирю с силой Д/*< = &Д<р,Дг, где Л/’ - приращение длины сектора. Коэффициент k можно определить, зная величину прогиба:
А:ДгДу>,— = тд. i
1лмсчая, что Дг/а = а/r, получаем
тдг* k= 2^3’
Зная t, вычислим силу, действующую на гирю, отклоненную в некотором горизонтальном направлении х на расстояние Дх < г от центра обруча, если обруч закреплен на горизонтальном столе и прогиба нет.
Рис. 55. К задаче 121.
Из рис. 55 следует, что Дг = Дж cos так как Дг г, Дж <& г. ('ила, действующая на гирю со стороны сектора, расположенного под углом к направлению х, равна
Д/7 = А’Д^ДжСО8$2.
Кс проекция на направление х равна
Д/*1 = — &Д<рДжсоз2 = — &Д^Дж + А:Д^Джсоз2^>/2.
При суммировании по всем углам 0 <	< 2тг второе слагаемое даст
пулевой вклад, и проекция силы, действующей на гирю в направлении смещения, будет равна
2 mgr*
2 а3
F= -
Дх.
141
Таким образом, сила пропорциональна смещению Ах из положении равновесия и направлена в сторону, противоположную смещении» Для периода соответствующих гармонических колебаний получаем
Задача 122 (86-р)
Маленький шарик массой т, прикрепленный к пружине с ко эффициентом жесткости fc, совершает гармонические колебания h направлении оси пружины с амплитудой А, Когда пружина была сжата, на пути шарика поставили второй такой же пружинный ми ятник, причем обе пружины имеют общую ось, а положения равно весия шариков совпадают. Опишите дальнейшее движение системы в следующих случаях:
а)	удар центральный и абсолютно упругий;
б)	удар центральный и абсолютно неупругий (т.е. шарики “ели паются”).
В начальный момент второй шарик покоился в положении pan новесия.
Решение, а) Так как удар абсолютно упругий, а массы шарм ков равны, то первый шарик остановится в положении равновесии, а второй начнет двигаться с той же скоростью, с какой двигался до удара первый. Очевидно, что амплитуда колебаний второго мп ятника также равна А. Второй шарик сместится на расстояние /I от положения равновесия и вернется обратно, столкнувшись в по ложении равновесия с первым шариком. В результате этого стол к новения он останется на месте в положении равновесия, а первый шарик придет в движение, сначала смещаясь на расстояние A oi положения равновесия, затем возвращаясь к нему и вновь ударяясь о второй шарик, и т. д.
б) В начальный момент второй шарик покоится в положении pan новесия. Первый шарик подойдет к положению равновесия со ско ростью tig = Ay/k/m, вычисленной из закона сохранения энергии при гармоническом колебании. После абсолютно неупругого удара шарики слипаются и образуют тело массой 2m, скорость которой» сразу после удара uq/2, что следует из закона сохранения импульса, При смещении этого тела на расстояние х от положения равнопо сия на него действует сила F = — 2kx. Поэтому период колебаний Т* = 2тгу/2т/(2к) сохранится таким же, как у одного пружинного маятника. Амплитуду колебаний системы слипшихся шариков най дем из закона сохранения энергии:
142
Примечание. При ударе шариков половина энергии первого ша-1<н к и переходит в тепло:
Q = т— - 2т—-— = т —.
Задача 123 (87-р)
Крутильные весы представляют собой коромысло длиной I = 1 м, нинпешенное за центральную точку на длинной упругой нити, с »крепленными на концах тяжелыми шарами массой М = 100кг ка-фпый (рис. 56). Известно, что частота крутильных колебаний весов 0,1с”1. Обоим шарам одновременно сообщают импульсы, равные по абсолютной величине р = 5 • 10”2 кг • м/с, противоположные ipvr другу по направлению и перпендикулярные нити и коромыслу. Ний,лите максимальный угол закручивания нити в последующем ко-н i «мольном процессе, если в начальный момент весы покоились.
М М
О——О
Рис. 56. К задаче 123.
Решение. Возникают гармонические колебания, при этом in ре мощение шаров вдоль окружности описывается формулой
/у/
s(t) = — = А(1 - coswt),	(1)
II гкорость — v(/) = Aw sincu^,	(2)
Hie а—угол закручивания нити; А— амплитуда колебаний шара niHMib окружности. Максимальное значение угла а находим из ранет тва (1):
п -2Л
атах ~	(6)
и максимальную скорость — из равенства (2):
^тах — Аш.	(4)
143
По условию задачи начальный импульс шара р =	Из этой»
условия и равенств (3) и (4) находим
2р max - IMw
= 0,1 рад.
Задача 124 (88-г)
В глубокой вертикальной шахте под землей находятся два груза равной массы, соединенные невесомой нерастяжимой нитью, кото рая перекинута через неподвижный блок. Найдите период верти кальных колебаний грузов, если радиус Земли 6400 км, ускорение свободного падения на поверхности 10 м/с2.
Решение. На поверхности Земли сила тяжести, действующая на тело массой т, есть сила всемирного тяготения между этим те лом и Землей. Если считать Землю однородным шаром с радиусом Л, то
m9 = G^~^	U)
где Л/ — масса Земли; G — гравитационная постоянная. В шахте, ко гда оба груза находятся на глубине h под землей, на каждый из них со стороны Земли действует сила
„ mpV 4 „
F = С(яТ^ = з’гС',т(л“ 41
где р —плотность Земли, V = |тг(Л — /г)3 —объем шара радиусом R — h.
Если теперь один из грузов (например, правый) приподнять нп высоту х от положения равновесия, то он окажется на глубине h — x и на него будут действовать сила натяжения нити Т и гравитациош ная сила равная
4
Л1(я) = -7rGpm(R — (Л - я)), и
Уравнение движения правого груза в проекции на направленную вверх координатную ось Ох примет вид
4
mai = — -irGpm(R - h 4- ж) + Т,	(2)
и
где ai —проекция ускорения правого груза на ось Ох. Так как нить нерастяжима, то левый груз опустится на расстояние х от равновесного положения. На левый груз будут действовать направленная
144
вертикально вверх сила натяжения нити Т, так как нить невесома, и гравитационная сила Fq(ж), равная
4
Fi(x) = -irGpm(R - (Л + ж)), о
Уравнение движения левого груза в проекции на ось Ох имеет вид
4
-ma2 = — -irGpm(R — h — я) 4- Т.	(3)
и
Так как нить нерастяжима, то модули ускорений правого и левого грузов равны, а их направления противоположны (ai =	= а).
Вычитая уравнение (3) из уравнения (2), получаем
или та 4- lv23? = 0,	(4)
где введено обозначение
4
W2 = -Л-Ср.	(5)
О
Из уравнения (4) следует, что грузы будут совершать вертикальные трмонические колебания, период которых равен
Т=2тг/ш.	(6)
Подставим в формулу (1) массу Земли, равную М = pV — ^pR3. При этом можно получить выражение для ускорения свободного падения в виде
4
9 = -icGpR.	(7)
о
Пз (5) и (7) получаем частоту
ы = \fgjR
и период колебаний
Т = 2TCy/R/g « 80 мин.
145
6. ЗАКОНЫ СОХРАНЕНИЯ
Из основных законов динамики и из общих свойств нашего мирп следуют так называемые законы сохранения,в которых утверждается факт постоянства некоторых физических величии п процессе развития той или иной физической системы.
В отличие от законов Ньютона законы сохранения не дают воз можности определить зависимость физических величин от времени. Вместо этого они определяют множество всех состояний, в которых может находиться данная система.
Энергия — это физическая величина, являющаяся функцией со стояния системы. Изменение этой величины равно работе, совер шенной внешними силами над системой.
Приведем примеры таких функций состояния:
К = mv2/2— кинетическая энергия тела массой т, движущегося со скоростью v;
U = mgh— потенциальная энергия тела массой находящегося на высоте h (над уровнем, энергия которого принята за нулевую);
U = &(Д/)2/2— потенциальная энергия упругой деформации, Д/—деформация, k — жесткость системы.
В данных примерах различные физические величины имеют одно общее свойство— изменение этих величин связано с совершением работы.
В механических процессах, в которых отсутствуют или пренебрежимо малы силы трения (точнее, мала работа сил трения), сохраняется механическая энергия — сумма потенциальной и кинетической энергий.
Использование законов сохранения импульса и энергии при решении различных задач позволяет устанавливать связь между начальным и конечным состояниями системы даже в тех случаях, когда промежуточные состояния нам неизвестны. Характерный пример— столкновение тел (от элементарных частиц до космических объектов). В процессе столкновения участвуют различные силы и взаимодействия, иногда неизвестные. Тем не менее, законы сохранения дают возможность по начальным данным определить параметры системы после взаимодействия.
При решении задач удобно использовать связь кинетической энер гии с импульсом. Действительно, из равенств К — mv2/2 и р = mv
146
•нтко получить
г——
К — -—, или р = у/2тК.
2т
Закон сохранения энергии — это общий закон природы. В неко-|прых задачах механическая энергия системы тел не сохраняется (например, при совершении работы силами трения). В этих случа-н х механическая энергия макроскопических тел не исчезает, а переходит в тепловую. Тепловая энергия — это энергия хаотического ‘innжения громадного числа элементарных объектов, составляющих макроскопическое тело.
* * *
Задача 125 (74-р)
Снаряд, выпущенный из пушки, установленной под углом 45° к । орнзонту на плоской горизонтальной равнине, разрывается в верхней точке своей траектории на два осколка равной массы. Первый < к колок падает на землю прямо под точкой разрыва снаряда спустя lb е. На каком расстоянии от пушки упадет второй осколок, если с момента выстрела до момента разрыва прошло 15 с? Сопротивлением воздуха пренебречь.
Решение. Так как время падения первого осколка и время подъема снаряда до верхней точки траектории (где вертикальная компонента его скорости обращается в ноль) равны, то начальная скорость падения первого осколка равна нулю.
Пусть /1 —расстояние от пушки до места падения первого осколка, I? —расстояние от места падения первого осколка до места падения второго, тогда искомое расстояние от пушки до места падения нюрого осколка I будет равно:
/ = /1 + ^2-
Разложим начальную скорость снаряда vo на две составляющие: |'|)г — по оси т, направленной горизонтально в сторону перемещения снаряда, и voy— по оси т/, направленной вертикально вверх.
Так как угол равен 45°, то Vq® = Voy; voy находим из условия равенства нулю вертикальной составляющей скорости в верхней точке траектории:
— gti >
147
где ti — время с момента выстрела до момента разрыва снаряди Тогда из равенства /i = Vo^i следует, что /i = giЕ 2. Из закона сохрп нения горизонтальной составляющей импульса
т TTlVQx —
находим горизонтальную составляющую скорости второго осколки V2x = 2г>ог- Из закона сохранения вертикальной составляющей им пульса следует, что вертикальная составляющая скорости второй। осколка равна нулю. Он падает в течение того же времени <i, чти и первый осколок, и пролетает по горизонтали путь /2 = ^2^1 -2Wi = 2gt\.
Итак, I — /1 + /2 =	2 = 6750 м.
Задача 126* (75-г)
Резиновый шнур с пренебрежимо малой массой подвешен за верх ний конец. Шнур продет сквозь массивную шайбу, которая в на чальный момент закреплена в верхней точке шнура. Если шайбу освободить, то она начнет скользить вдоль шнура, причем сила трг ния будет постоянной и равной F. Коэффициент упругости шнура на растяжение равен Л, на сжатие — равен нулю, длина шнура I.
а)	Какое количество тепла выделится при соскальзывании mall бы?
б)	Если на нижнем конце шнура есть зажим, прочно захватыва ющий шайбу, то начнутся колебания шайбы, подвешенной на шнург При каком соотношении между параметрами задачи эти колебании будут гармоническими?
Р е ш е н и е. а) При достижении шайбой нижнего конца шнура его длина становится равной I + х, где х — растяжение шнура, вы званное действием силы трения F, т. е. х = F/k. При скольжении вдоль шнура шайба совершает работу
F2
A = F(l + x) = Fl+—. hf
Часть этой работы переходит в упругую энергию Е шнура:
Е~~’
а остальная часть выделяется в виде тепла:
р2
Q = A-E = Fl+—.
148
(») Если шайба, достигнув нижнего конца шнура, попадает в за-фпм, то начинаются колебания, при которых энергия не теряется, । I’, не переходит в тепло. Эти колебания будут гармоническими, м ин шайба не будет подниматься выше положения, при котором I’liiiia. шнура равна I. Это обусловленно тем, что, по условию зада-III, при подъеме шайбы выше указанного положения сила со стороны шнура на нее не действует (к = 0). Это приводит к нарушению н1рмоничности колебаний. Шайба не будет подниматься выше ука-|ци кого положения, если в процессе ее соскальзывания выделится •ни гаточно большое количество тепла. Будем отсчитывать высоту пид ьема шайбы h относительно положения, при котором длина шну-1«н равна I. Тогда из закона сохранения энергии для верхней точки, ииторую шайба достигает при колебаниях, получаем
mgl = mgh + Q при h > 0, kx^
mgl = mgh + Q H—— при h < 0.
(1)
(2)
I • ли h > 0, то шнур сжат и упругая энергия в нем не запасена Ц 0); если h < 0, то шнур растянут и его упругая энергия kh2/2.
По условию задачи h < 0, т. е. равенство (1) не должно иметь места. Это требование всегда будет выполнено, если
/?2
Q = Fl +	> mgl.
Задача 127 (75-г)
Протон (ядро атома водорода), летящий со скоростью и, много меньшей скорости света, сталкивается с неподвижным атомом ге-•ПО1. При центральном ударе 25% кинетической энергии протона переходит в энергию возбуждения атома гелия. Найдите скорость протона после столкновения. Масса атома гелия примерно в четыре раза больше массы протона. Объясните, почему получается два решения. Какой физический смысл имеет каждое из них?
Решение. Пусть —масса протона, m2 — масса атома гелия (ш-2 = 4mi), Vi и V2—соответственно скорости протона и атома |е,нпя после соударения. Выпишем законы сохранения импульса и
пк’ргии:
7711V = W1V1 + Ш2У2,	(1)
2	2 ।	2 ।	2
miv = mivj 4- m2V^ + am^ v .
(2)
149
Последний член в правой части (2) —это энергия возбуждения пти ма гелия (а = 1/4). С учетом того, что удар центральный и т? • 4mi, система уравнений (1), (2) принимает вид
V = V1 + 41/2, v2(l — а) = v2 + 4и2.
Решая эту систему, находим
Vi = v(l т УЙ)/5, v2 = v(4 ± Л1)/20.
Если в решении взять верхние знаки, то vi < 0 < v?—протон, приведя в движение атом гелия, отскакивает от него обратно.
Если в решении взять нижние знаки, то 0 < V2 < vp Это решенш имеет следующий смысл: протон, приведя в движение атом гелии, пробивает его ядро и летит впереди (или захватывается ядром t п вперед вылетает другой, содержавшийся в ядре гелия, протон).
Задача 128 (76-г)
Тонкий обруч расположен в горизонтальной плоскости. На об руч насажены два шарика, которые могут скользить без трения Ип обручу (на рис. 57— вид сверху). Массы шариков тщ и m2. Рал меры шариков много меньше радиуса обруча. В начальный момеп i скорости шариков Vi и V2- Соударения шариков упругие. Найдите скорости шариков после четвертого столкновения.
Рис. 57. К задаче 128.
Решение. При абсолютно упругом ударе двух шариков с массами mi и m2 и начальными скоростями Vi и V2 конечные ско рости шаров и v2 определяются из законов сохранения энергии и импульса
mi V2 + ^2^2 = ml(vl)2 + m2(^)2>	(1)
mivi + m2V2 = 7Л1 v'i + m2v2.	(2)
150
I iiii как размер шариков много меньше размеров обруча, то удар мнжно считать центральным. При центральном ударе закон сохранения импульса упрощается и принимает вид
mivi + m2V2 = miv{ + m2t/2.	(3)
I Невидно, что система уравнений (1) и (2) имеет два решения, одно и । которых tii =	, i»2 = v2 тривиально и соответствует прохожде-
нию шариков ’’друг сквозь друга” без столкновения. Второе, нетри-IHIильное, решение соответствует реальному физическому процессу • юлкновения шаров. При повторном ударе шары будут сближаться । п скоростями и и2. В соответствии с этим правые части уравнении (1) и (2) будут описывать начальные энергии и импульсы шаров.
В силу единственности нетривиального решения этой системы и ные части уравнений будут давать энергии и импульсы шаров по-। '!<• 2-го столкновения, и т. д. После четного столкновения шаров их • цпрости будут по величине равны первоначальным скоростям.
Задача 129 (76-р)
Две тележки одинаковой массы соединены тонкой натянутой пи-н.ю. Тележки стоят на горизонтальном полу и могут катиться без I рения. На тележках установлены одинаковые двигатели, нить прикреплена к валам двигателей. Диаметр каждого вала d, ось вала перпендикулярна нити. В некоторый момент двигатель на одной из |е.1|(‘жек включается и его вал начинает вращаться с постоянной VI.новой скоростью Нить равномерно наматывается на вал так, •I г<> диаметр витков равен диаметру вала. Каковы будут скорости н*лежек относительно земли при включенном двигателе, если счи-нп ь нить невесомой и нерастяжимой?
Р е ш е н и е. С момента начала вращения вала до момента приобретения им постоянной скорости проходит некоторое время. В к’чение этого времени на тележки со стороны нити действуют неко-юрые силы Fi и F2. Эти силы равны по величине и противоположно направлены (следствие невесомости нити и III закона Ньютона): l<’i = —F2. По II закону Ньютона действие одинаковых сил приводит к одинаковому изменению импульса тележек. Так как массы тележек одинаковы, а начальные скорости равны нулю, то одинаковы (ио противоположно направлены) и скорости в любой момент времени. Нить укорачивается (а тележки сближаются) со скоростью I» - wd/2} следовательно, скорости тележек 14 и v2 относительно |гмли равны: = v2 = v/2 = wd/4.
151
Задача 130 (76-р)
Невесомая нерастяжимая нить перекинута через невесомый блок, который может вращаться без трения. На концах нити подвешены два устройства одинаковой массы. В начальный момент они покп ятся. Внутри устройств имеются одинаковые двигатели, нить при креплена к их валам. Диаметр каждого вала d, ось вала перлон дикулярна нити. При включении двигателя в одном из устройств вал начинает вращаться с постоянной угловой скоростью w, а нить наматывается на вал одинаковыми витками, диаметр которых рп вен диаметру вала. Каковы будут скорости относительно земли при включенном двигателе?
Р е ш е н и е. Отличие этой задачи от предшествующей состои’1 лишь в том, что силы, действующие со стороны нити на прикреплен ные к ней устройства, направлены в одну сторону. Вследствие это г и оба устройства двигаются вверх с одинаковыми скоростями vi и Vy, т. е. |vi| = |vo| = wd/A..
Задача 131* (78-г)
Маленький шарик массой т подвешен на тонком резиновом жгу те, длина которого в нерастянутом состоянии равна /. Масса жгу гп пренебрежимо мала. Если шарик поднять до точки подвеса и отпу стить, то при т = 100 г и I = 5 м жгут не рвется, а при т = 110 г и I = 5 м — рвется. Порвется ли жгут при т = 100 г и I = 6 м?
Решение. Рассмотрим систему из шарика и резинового жгучи в двух состояниях— когда шарик занимает самое верхнее и самое нижнее положения. В этих положениях скорость шарика равна ну лю, Предположим, что жгут еще не рвется. Пусть длина растяпу того жгута равна V. При переходе из первого состояния во вторси» потенциальная энергия шарика в поле силы тяжести уменьшается на величину U = mgV, которая переходит в потенциальную энер гию деформированного жгута. На единицу его длины приходится энергия
E=U/lf = mg.	я (I)
Очевидно, что максимальная сила натяжения жгута определяется только энергией, приходящейся на единицу длины максимально рас* тянутого жгута. Эта энергия, как видно из формулы (1), не зависит от первоначальной длины жгута I и определяется только массой гп привязанного шарика. Отсюда ясно, что, если жгут не рвется при т = 100 г и / = 5 м, то он не рвется и при т — 100 г I = 6 м.
152
Задача 132 (78-г)
Как будет ориентирован относительно Солнца спутник сфериче-|н»|| формы, одна половина которого зеркальная, а другая черная? ' читать, что вращательные колебания спутника быстро затухают.
Решение. Рассмотрим сначала силы, действующие на неболь-himII участок зеркальной сферической поверхности при отражении ин рал дельного пучка световых лучей. В процессе отражения про-IH нодит изменение первоначального направления нормальной к по-iH'lIXпости составляющей количества движения светового потока на ьратное, а касательная составляющая не меняется. В результате к кнждой точке сферической поверхности оказывается приложен-цн|| сила давления света, направленная по радиусу к ее центру О Ipiir. 58, а). Для нас существенно, что сила имеет нулевой враща-ц |Ц|||ый момент относительно любой оси, проходящей через центр • |||Г|)Ы.
Рис. 58. К задаче 132.
Иначе обстоит дело, если параллельный пучок света падает на |г|)пый участок сферической поверхности. Такой участок поглоща-• । пгсь падающий на него свет, а вместе с ним и переносимый им импульс. В результате к участку оказывается приложенной сила F, и и и рп пленная вдоль оси пучка (рис. 58, б). Эта сила создает вра-шпггльный момеш. стремящийся повернуть шарик в направлении, •ипинанном на рис. 58, б изогнутой стрелкой.
Рассмотрим теперь действие параллельного пучка света от Солн-ни па сферический спутник, одна половина которого зеркальная, а и ругая черная. На рис. 58, в спутник изображен таким, каким его пилит свет”, распространяющийся нормально к плоскости рисунка
153
(черная часть спутника заштрихована). На рис. 58, г представ ь вид того же спутника в направлении оси О, а также указаны пич вой поток со стороны Солнца и силы светового давления, дейсни» щие на спутник. Равнодействующая сил давления Т, приложит» к зеркальной части поверхности, направлена к ее геометричггкн< центру и не создает вращательного момента. Равнодействующий »»< давления F, приложенных к черной части поверхности, создасн щ щательный момент относительно оси О и стремится повернуть сщ ник вокруг этой оси в направлении, показанном стрелкой. Сну ни* приходит во вращение, “подставляя” Солнцу зеркальную сторнн и в конце концов он будет обращен к Солнцу только зеркальн стороной — так, как показано на рис. 58, д. В этом положении н|> щательный момент со стороны сил давления равен нулю. При п> большом отклонении от указанного положения, как это следу с i »< изложенного выше, всегда возникает вращательный момент, вег nip щающий спутник в исходное положение. Таким образом, на рш с? представлено положение устойчивого равновесия.
Когда спутник обращен к Солнцу целиком своей черной с*и»р ной (рис. 58, е), сумма всех действующих на него относительно »»• О моментов также равна нулю. Спутник находится в равновсст положении, но такое равновесие неустойчиво. Достаточно сну iих ку чуть отклониться от этого положения, как сразу же появляин» силы, под действием которых он поворачивается к Солнцу ст и» зеркальной стороной.
В наших рассуждениях мы пренебрегли излучением спутник так как возникающие при этом силы “отдачи” имеют нулевой щательный момент и не влияют на ориентацию спутника.
Задача 133 (78-р)
Тяжелый вагон свободно катится по рельсам со скоростью u U трения. В заднюю стенку вагона попадает пуля массой т, летевши* со скоростью v относительно земли, и застревает в стенке. Найдиi количество тепла, выделившееся при ударе, считая, что векторы и и v параллельны и |v| > |u|.
Решение. Пусть масса вагона равна М (М т) и скоро» и вагона после попадания в него пули есть t/i. Так как на систем вагон—пуля внешние силы в горизонтальном направлении не д»»Н ствуют, то можно воспользоваться законом сохранения импулы для движения в этом направлении:
Mud- mv = (М + т)щ.	(1>
154
1 hiiiпн’м закон сохранения энергии, обозначив количество выделив-
..... тепла через Q:
+	= (М +	+ 0	(2)
ih (I) получаем
Ми + mv Ui = ---.
М + т
(3)
•I -.Ф in вив в (2) вместо ui выражение (3), решим полученное урав-|Ц||' относительно Q:
Q =	[ Ми2 + mv2
& X
(М и + mv}2 М + т
тМ (и — и)2
М 4- т 2
(4)
»ю точное решение содержит неизвестную величину М. Исполь-। и неравенство М ?п, можно получить приближенное решение, ф»’ не содержащее М. Для этого преобразуем первый сомножитель при вой части (4) к виду
тМ _ т
М + т 1 + т/М’
(5)
Н< рлпспству М т эквивалентно неравенство т/М «С 1. Поэтому •• нтменателе правой части равенства (5) можно пренебречь слага-• 1ым т/М в сравнении с единицей, т. е. & т, В соответствии ним
m(v — и)2
2
1ндача 134* (79-г)
На правой чаше весов с плечами и I? лежит камень массой т. Itiholl же камень падает на левую чашу весов с высоты hi с нуле-••••II начальной скоростью (рис. 59). На какую высоту поднимется |||ц||н>1Й камень, если удары абсолютно упругие, а весы жесткие и ih массой можно пренебречь?
т
Рис. 59. К задаче 134.
155
Решение. Прежде всего следует отметить, что время аюГнн ного полета правого и левого камней значительно превышает врем* удара камня о чашу весов. Действительно, камни и весы абсолим но жесткие, т. е. любые сколь угодно малые деформации при удпр приводят к появлению бесконечно больших сил упругости. Эти Гинн шие силы, действуя в течение очень малых промежутков времени вызывают большие ускорения и, следовательно, быстрые измошчнн* скорости. Расстояния hi и Аз» на которые опускаются или подпили ются чаши весов за время удара, значительно меньше высоты « и» бодного падения левого камня hi и высоты подъема правого ками* Аг-
За время свободного падения до момента удара левый камин» приобретает скорость vi, которую можно найти из закона сохрлн» ния энергии:
mght =
В процессе удара кинетическая энергия Е\ левого камня меняли« и становится равной
где гц—скорость левого камня после удара. Правый камень шн’Ф удара получает скорость из и кинетическую энергию
~2
^2 = ~22.
Полная энергия в процессе удара не меняется, поэтому
, mvf mv?
mghi = —1 + —(I) Л z
Наличие у правого камня скорости после удара позволяет ему пил няться на высоту Аз:
*2
mgh2 = -г-.	('»)
Z
Из полученных выражений невозможно определить высоту Л2, w как есть еще не определенная величина щ.
Для нахождения учтем, что изменения скорости левого и при вого камней происходят из-за действия сил со стороны весов.
Пусть на камни действуют в процессе удара средние силы 7?| и F2 в течение времени Т. Изменение импульса левого камня за эн» время
mv\ — mvi = Fi Г,	(II)
156
и 1М<’1кч!ие импульса правого камня
?тгг>2 = F2T.	(4)
" них равенствах не учитывается сила тяжести, так как силы Fi, '"К’нь велики и значительно превышают тд. Силы, с которыми 'Мин действуют на весы, равны по величине Fi и F2. Сумма мо-• пни! сил, действующих на весы, должна быть равна нулю, так • I мпсса весов пренебрежимо мала:
Fih = F2l2.	(5)
Hi рпнгнств (3)—(5) получаем:
#1 =	- 7^tJ2.	(6)
ч
♦ in pi. можно из соотношений (1), (2) и (6) определить высоту подъ-м правого камня:
h -	h
2~ (|? + /22)2 11
I h метим, что при Zi / 12 неравенство Л2 < М выполняется неза-н. iiMiioT того, на какой чаше весов лежит этот камень (с коротким • hi илпиным плечом).
1цп,пча 135 (80-г)
I |»пкий невесомый стержень длиной I стоит вертикально на го-. и и hi I ильной поверхности. Нижний его конец скреплен с поверхно-• 1 in г помощью шарнира, на верхнем конце закреплена “точечная” • «и । и т. Легким прикосновением стержень выводят из положения . иии 1ВССИЯ, и он начинает падать. Найдите ускорение массы т не-। । prii,( гвенно перед ударом о поверхность.
Рис. 60. К задаче 135.
I' с in с н и е. Искомое ускорение а может быть представлено в •.Hili' суммы двух взаимно перпендикулярных составляющих ai и а2 i|'iii (И)):
а = ai + а2, ах 1 а2.	(1)
157
Первая составляющая ai вектора а создается силой натяжения гпцм ня Т и направлена к точке О, т. е. ускорение ai является центр" стремительным и равно
ai = v2/I.	(’Л
Здесь v — скорость массы т в момент соприкосновения ее с поверь ностью. Вторая составляющая а2 создается действием силы тяжсст mg и направлена по вертикали вниз. Согласно II закону Ньютона
mg = ma2,
откуда а2 = 9<	(3|
Найдем неизвестную величину v2. Для этого воспользуемся чи коном сохранения энергии. При падении стержня потенциальная энергия точечной массы mgl переходит в ее кинетическую энергии* mv2/2. Отсюда следует:
V2 = 2д1.	(-11
Из формулы (2) с учетом равенства (4) получаем
ai = 2д.	(М
Из (3) и (5) находим ускорение а согласно (1) но формуле
а = у а1 + а2 - З'/Ь-
Задача 136 (81-р)
Какую минимальную работу надо совершить, чтобы передвинуп» лежащий на шероховатой горизонтальной поверхности груз массой т = 10 кг на расстояние I = 1 м с помощью невесомого резиновой» упругого жгута с коэффициентом упругости к = 10 Н/м. Жгут а начальный момент не растянут. Коэффициент трения груза о по верхность д = 0,1.
Решение. Чтобы работа была минимальная, груз надо пере двигать очень медленно. Тогда производимая работа не будет расхо доваться на увеличение кинетической энергии груза, а пойдет толь ко на преодоление действия упругой силы и силы трения. Жгу| при этом должен быть растянут так, чтобы он действовал на груз <’ силой
F = ртд.
Если сила будет меньше F, то груз не сдвинется с места, а если больше, то груз начнет двигаться с ускорением, и его кинетическая энергия будет возрастать.
158
IliHi’iajie (при растяжении жгута) мы действуем с переменной по • hiчине силой, которая изменяется от нуля до F. Жгут при этом । •• i питается на величину
Г'||ц 11 а /11, совершаемая при растяжении жгута, определяется фор-Ч lull
_ &(Д/)2 _ F2 __ /12т2д2
1 “	2	~2к~ 2к ’
При цельнейшем движении и груз и точка приложения силы F пе-> > mi in,лютея на одинаковое расстояние Z, при этом сила F совершает । ц1н 11 у
Л2 = Fl = /imgl.
Н"'|цп>| искомая работа складывается из 41 и
и.2т2д2
А - Л + А2 = —--------h //mgl « 15 Дж.
' 1цидча 137 (81-р)
какую минимальную работу надо совершить, чтобы лежащий •и • юле груз массой т = 1 кг поднять на высоту h = 1 м при •t iMitin,!! привязанного к нему резинового жгута с коэффициентом нрукити к = 10 Н/м? В начальном состоянии жгут не растянут.
Ми» । пй жгута пренебречь.
Решение. Чтобы, как и в предыдущей задаче, работа бы-и минимальной, груз надо поднимать с почти нулевой постоянной |’и|нк’тью. При этом предварительно придется растянуть жгут на-ннп.ко, чтобы он начал действовать на груз с силой
F = тд.
II процессе растягивания жгута внешней силой будет совершена ра-• и in Л ।, равная изменению упругой энергии пружины
F2
A1 ~ 2к
При последующем подъеме груза на высоту h будет совершена pain» । п
А2 = mgh.
Нюшей искомая работа А складывается из Ai и А2:
/тд _ \ = т9 + h)
А — 41 + Л2
« 15 Дж.
159
Задача 138 (81-р)
Какое количество тепла выделится при лобовом ударе двух г инк цовых шаров массой т = 1 кг каждый, скользящих без вращении i абсолютно гладкой горизонтальной поверхности по одной прям'’И одном направлении? Скорость первого шара uj = 10 см/с, скорм». второго V2 = 20 см/с. Удар шаров считать абсолютно неупругим
Решение. Так как удар абсолютно неупругий, то ио» • удара оба шара будут двигаться с одинаковой скоростью v. Сопит н закону сохранения импульса
+ mv? = 2mv.
Отсюда
Vi + V2 v ~	2	
До столкновения шары обладали кинетической энергией mv? mvj
1	2	2’
После удара их кинетическая энергия равна и/ о"™2	+ V2)2
1У2 = 2—=---------------.
В результате удара кинетическая энергия системы уменьшаете л и величину
AW = Wr - W2 = ^(v, - v2)2 = 0,0025 Дж.
Эта энергия и переходит в тепло.
Задача 139 (81-р)
Какое количество тепла выделится при абсолютном неупруи»»-ударе свинцового шара массой т = 1 кг об очень тяжелую степи > движущуюся со скоростью и = 10 см/с? Шар до удара двигался п вращаясь перпендикулярно к стенке со скоростью v = 5 см/с.
Решение. Так как удар абсолютно неупругий, то пси i удара стенка и шар будут продолжать движение вместе с одипли» вой скоростью v'. В процессе удара механическая энергия системе частично переходит в тепловую, полная энергия системы при viih-сохраняется. Обозначая массу стенки через Л/, можно написать
mv2 Ми2 (m + M)vf2
—+ —=	2	+ ^
|l
Здесь WT—тепловая энергия.
160
II процессе удара остается неизменным и импульс системы. Соин по чакону сохранения импульса
Ми — mv = (m 4-	.	(2)
И i. пиная v' из системы уравнений (1) и (2), получаем
_ 1 m(u + v)2
т ~ 2 1 + т/М •
И • \i ловию задачи стенка очень тяжелая, т. е. т/М 1, и
т(и 4- и)2
Wr «	= 0,01125 Дж.
1пдпча 140 (82-г)
I М»руч массой т катится по горизонтальной плоскости со скоро-И.1Н г». Перед ним покоится горка массой М и высотой Н. Обруч не । к пл взывает; горка может скользить по плоскости без трения. При какой минимальной скорости г>о обруч переедет через горку?
I’ г тени е. Предположим, что v < vq. В этом случае обруч, и пгргехав через горку, поднимется на некоторую высоту h < Н. В *। момент он неподвижен относительно горки, а сама горка сколь-♦н по плоскости с некоторой скоростью и. Из законов сохранения •finул пса и энергии следует, что
mv = (m + М)и,	(1)
E=(’" + ")“2+^.	(2)
Л
I'li’i I. = mv2 —кинетическая энергия катящегося обруча (она и 1пд|||вается из кинетической энергии mv2/2 поступательного дви-м1ш>| и кинетической энергии mv2/2 вращательного движения). Il и иючая из равенств (1) и (2) скорость и, находим
_ v2 2М 4- т д 2(М + т)'
• удлинением v величина h растет. Когда v достигает значения vg, •" I и чина h достигает значения Я. Заменяя в полученном равенстве пи и h на Я, получаем
12дН(М + т)
Vq у 2М 4- т
161
Задача 141 (83-г)
Вы стоите, не касаясь земли, на качелях, которые находятся I» покое. Можно ли их раскачать? Если можно, объясните, как эти сделать и какая сила сообщает вам с качелями импульс в горизон тальном направлении.
Решение. Единственная внешняя сила, которая действует ни систему человек—качели (предполагаем, что человек стоит в центр1’ качелей) и которая может иметь горизонтальную составляющую, это сила реакции опоры качелей N. Только с использованием этой силы и можно раскачать качели. Для этого надо действовать так Сначала человек отклоняется в сторону от вертикали, качели при этом отклоняются в противоположную сторону (центр тяжести си стемы человек—качели остается при этом на месте). Затем надо пи некоторое время прекратить воздействие на качели (начать приседа ние). В это время горизонтальная составляющая силы N сообщшч качелям горизонтальный импульс, а горизонтальная составляющий импульса человека практически не меняется. Если приседание ’in кончить и встать в тот момент, когда качели расположены верти калыю (а, следовательно, и сила N направлена вертикально), горизонтальная составляющая импульса системы человек—качели не изменится. В итоге система человек—качели приобретает гори зонтальный импульс. Действуя таким образом в такт с колебаниями качелей, и можно их раскачать.
Задача 142 (83-р)
На гладком горизонтальном столе лежит гантелька, состоянии! из двух маленьких шариков 1 и 2 с массами и mi (mi > m2), си единенных жестким стержнем длиной I (масса стержня пренебрсжн мо мала). На шарик 2 налетает третий шарик с массой m3 = mi-nij скорость которого v направлена перпендикулярно стержню, и ели пается с ним. Опишите дальнейшее движение гантельки.
Решение. После слипания шариков с массами m2 и m3 Пн обоих концах гантельки будут сосредоточены одинаковые массы пь (так как m2 + m3 = mJ. Отсюда ясно, что центр масс получившей! и системы будет находиться в геометрическом центре стержня.
По закону сохранения импульса центр масс образовавшейся пи еле удара системы с массой mi + m2 + m3 = 2mi будет обладпн импульсом 2miu, равным импульсу тзУ, который имел до удар» третий шарик:
2miu = тзУ, или 2miu = (mi — тг)у.
162
I n гI» u — скорость центра масс системы после удара, равная
(1)
I" • < «ударения шариков 2 и 3 шарик 3 обладал моментом импульса нкк ительно геометрического центра гаительки
г I ,	. I
L = m3v- = (mi - m2)v-,
(2)
• mi «менты импульсов шариков 1 и 2 были равны нулю. В силу зако-п>хранения момента импульса гаителька после удара будет вра-j
•hi вся вокруг своего центра масс. Обозначая линейную и угловую »н1|нic/ги вращения масс mi, сосредоточенных на концах гантельки, •• • иг теме покоя центра масс через и' и о/, для суммарного момента •ншульса после удара мы должны написать
L' = 2miv'^ = 2miw (0
(3)
Н • илу закона сохранения момента импульса L' = L. Приравнивая i|HH»i.ie части (2) и (3), находим
GJ —
V
Г
(4)
Итак, после удара гантелька вращается около своего центра масс и новой скоростью ил, определяемой формулой (4), а сам центр и» । движется поступательно со скоростью и, определяемой фор-h «и41 (I), в том же направлении, что и шарик 3 до удара.
1ц||.ача 143 (86-г)
1л ли столкнуть два стеклянных шарика, то разобьется только ши Почему не разобьются два шарика сразу?
Г г ш е н и е. Согласно III закону Ньютона на шарики в процес-। ки1кновения действуют одинаковые силы. Величины этих сил м> 1|ю возрастают от нуля до некоторого максимального значения. •II Ирик и не могут быть абсолютно одинаковыми, поэтому предельно шачения сил, после которых упругая деформация сменяется пиеской и шарики разбиваются, должны хотя бы немного раз-u ни вея. Если силы взаимодействия между шариками достигнут •нпнмального из этих предельных значений, один из шариков раз-•Hhiiчем, а сила взаимодействия резко падает. Так что второй ша-•♦!• «и‘тается целым.
163
Задача 144 (86-р)
Мяч массой т бросают горизонтально с некоторой высоты поверхностью земли так, что его полная энергия равна Е. С кпм» высоты и с какой скоростью надо бросить мяч, чтобы далын» • броска была максимальной? Какова эта максимальная дальней н Потенциальная энергия мяча на поверхности земли равна нулю
Решение.В момент падения мяча на землю его потенциал ьн« энергия равна нулю, следовательно, его скорость в конечной тн'н траектории есть
^кин = х/2-Е1/ш.
Рассмотрим все возможные траектории мяча с энергией Е. Эти 1|ь ектории будут различаться углом акин между направлением ск*»|» сти vKliH и горизонталью. При этом начальная скорость броска
— ^кин COS Окин =	COS Окин,
а начальная высота определяется из закона сохранения энергии
v2
Е = mgh + т-^ = mgh + Ecos2 акин, А
т. е	Е
h =	(1 — COS Окин)*
тд
Каждая такая траектория представляет собой параболу, причем h ка броска будет находиться в вершине этой параболы. Очевидно, щ дальность будет равна половине длины отрезка, который парабн । отсекает от горизонтали.
Если мы обозначим дальность броска £, то L = Lq/2, где длина вышеупомянутого отрезка. Этот отрезок представляет соП. • не что иное, как дальность броска с начальной скоростью г»о = I'm под углом а о = <*кин с поверхности земли. Очевидно, что дальнем i Lo будет максимальной при ао = акин = 45°. При этом Lq = 2Е тд *
Следовательно, искомые величины
Т Е ГЁ , Е
L-—, v = \ —, h=---------.
тд V т 2тд
Задача 145 (87-г)
Мальчик садится на санки в точке А (рис. 61, а), съезжает с го| ки и останавливается в точке В. Определите коэффициент трении• санок о снег. Где остановятся санки, если мальчик начнет съезжю из точки А'? Санки при движении не подпрыгивают.
164
Рис. 61. К задаче 145.
1‘ <‘ ш е н и е. Рассмотрим движение санок на малом участке 1<нгк гории СС длиной Д/. Изменение потенциальной энергии есть
ДЕп = тд(уС' - Ус) = тдЛу.
и IV трения на участке С С' можно считать постоянной и равной , утд cos а, где а— угол наклона отрезка СС* к оси Ох. Работа и 11.1 трения на этом участке есть
АТ = —цтпд соваД/ = —ртд&х.
ili uiKona сохранения энергии следует
— mvc)/2 4- mg&y — —цтд&х.
। нмируя такие уравнения по всем участкам траектории АВ и учи-• пния, что скорость в начале и конце движения равна нулю (уд = и О), получаем
тд(уА ~Ув) = ~1мпд(хА -хв),
। н уда
Р = (уд - Ув)/(хв - хА).
Hi рис. 61, б находим: р 1/9. Заметим, что р = tg/?, где /3— нн| наклона прямой АВ к оси Ох. Поэтому при движении из точ-н I' санки остановятся в такой точке В', что прямая А1 В1 парал-•III.на прямой АВ. Из рис. 61, б находим координаты точки В1 : . *i 24 м, у ъ 4,3 м.
Задача 146 (88-г)
Человек массой М = 100 кг жонглирует десятью гирями массой • I кг каждая, стоя на весах. Гири взлетают на высоту h = 5 м. июню среднее показание весов?
165
Решение. Пренебрегая сопротивлением воздуха, можно оп|и делить скорость гирь в начале и в конце движения, а также вргми полета гири. Из закона сохранения энергии mv2/2 = mgh следу» i что
v = x/2gh « 10 м/с.
Время подъема гири Д/i равно времени ее падения, определяемом» из уравнения ^(Д<1)2/2 = Л, откуда Д/i = y/2h/g = 1с. Поэтом» время полета каждой гири равно Д/ = 2Д/1 = 2 с. Когда жош'лп ловит гирю, а затем подбрасывает ее, происходит изменение импу.ц|. са гири
Др = 2mv = 2mx/2gh « 20кг-м/с.
Это изменение импульса гири равно импульсу действующей на нн силы
Др = FSty
где St — время взаимодействия гири с жонглером. При этом средним сила, действующая со стороны человека на гирю, равна
F -
ЬЛ + St АЛ'
если считать, что Si <& Д/. По III закону Ньютона такая же m величине сила действует на весы со стороны одной гири. Со сторож» jV гирь средняя сила, действующая на весы, в N раз больше и рани»»
F = N^n 100 Н.
Д/
Среднее показание весов складывается из силы F и веса жонглгрм Мд:
Р= Mg + F к 1100Н.
Заметим, что после длительных вычислений мы получили три виальный результат: средний вес жонглера с гирями не зависит »н того, жонглирует он или нет, и равен силе тяжести.
Этот же результат можно получить путем простых рассужд»1 ний: среднее положение центра масс системы “жонглер+гири” IP зависит от времени, следовательно, по II закону Ньютона средн»1» значение суммы внешних сил, действующих на эту систему, рпп но нулю. Внешние силы — это сила тяжести и сила реакции опоры равная, по III закону Ньютона, показанию весов. Таким образом среднее показание весов равно силе тяжести.
166
Задача 147* (88-г)
В салоне реактивного самолета, движущегося со скоростью и = 71)0 м/с, ребенок запускает игрушечную машинку в направлении нпижения самолета. Вначале машинка неподвижна. Затем она разнимется до скорости v = 10 см/с. Масса машинки т = 100 г. Изменение ее кинетической энергии в системе отсчета самолета 6Е = гн №/2 -0 = 5- 10“3 Дж, а в системе отсчета земли ДЕ1 = т(и 4-р)7/2 — ти2/2 = 2 Дж. Объясните различие SE и ДЕ, ведь энергия, и ।расходованная находящейся в машине пружиной, в обоих случаях • ища и та же.
Р е ш е н и е. В системе отсчета, связанной с самолетом, изменение импульса машинки равно импульсу силы трения F, действу-|нЩ(»й на машинку в течение времени движения Д/:
mv = FAi.
(1)
1’пбота силы F идет на увеличение кинетической энергии машинки. II случае постоянной силы движение машинки равноускоренное, и рп(юта силы F с учетом соотношения (1) равна
л „	„	„ Д/ mv2
А = Fs = Fvc = Fv— = —— = 6Е,
&
ni,c «— путь, пройденный машинкой при равноускоренном движении относительно самолета.
В системе отсчета, связанной с землей, путь, пройденный машинной за время Д/, равен
Д/
S = s 4-	= V— 4- иД/.
(2)
1’ибота силы F в этой системе отсчета с учетом формул (1) и (2) рнвиа
Д/	mv2
A = FS = Fv — 4- FuSt = — 4- mvu.
Увеличение кинетической энергии машинки в этой системе отсчета
А „ m(u + v)2	mu2 mv2	_ л
ДЕ = ——-— -----— = —— 4- mvu = оЕ 4- mvu,
Z	Z Z
Гн к им образом, различие величин ДЕ и 6Е в соответствии с теоремой об изменении кинетической энергии объясняется различием рп(ют, производимых действующей на машинку силой F в разных
167
системах отсчета из-за различия путей, проходимых машинкой и этих системах отсчета.
Откуда же берется дополнительная энергия AjE — 6Е = mvu и системе отсчета, связанной с землей? Это не что иное, как допил нительная работа, совершаемая двигателями самолета в процесс»* разгона машинки, равная F(S — s) = mvu.
Задача 148 (89-р)
Снаряд массой т = 10 кг разрывается в полете на две равные части в тот момент, когда его скорость равна Vo = 100 м/с. При разрыве суммарная кинетическая энергия увеличивается на АЛ’ 4105 Дж. Определите максимальный и минимальный из возможных углов между скоростями осколков сразу после разрыва.
Р е ш е н и е. В системе отсчета, связанной с центром масс, с ко рости каждого из осколков в момент взрыва равны v = ±^/Д£/?п ±200 м/с, так как масса одного осколка равна половине массы сип ряда. Из закона сохранения импульса следует, что полный импулы в этой системе отсчета равен нулю, т. е. осколки разлетятся в проти воположные стороны с одинаковыми скоростями. В системе отсчета, связанной с землей, импульсы осколков направлены под некоторым углом /3 друг к другу. Очевидно, что максимальное значение угла будет равно /?тах = я*. Угол /3 будет минимальным, когда вектор v перпендикулярен vq:
. /?гтйп _ V _ I ДЕ
л	~~ \!	2 ’
2	vq	у 771Vq
или
/?min = 2 arctg2.
Задача 149 (92-г)
В вагон с песком, движущийся в составе поезда с постоянной ско ростыо v, падает с некоторой высоты кирпич массой т и застревая i в песке. Скорость кирпича в момент падения в песок была равна и Скорость поезда не изменилась. Определите количество выделив шегося тепла и работу, совершенную двигателями поезда.
Решение. При падении кирпича в движущийся вагон то пло выделяется не только за счет изменения механической энергии системы при неупругом ударе, но и за счет работы, совершенной двигателями поезда. Действительно, кинетическая энергия кирпичи после падения W = mv2/2 может быть и больше, чем его энергия до
168
мн цепня По = zrnr/2, а некоторое количество тепла Q тем не менее •u.i мелится. Недостающую энергию может привнести только двига-н иь, совершивший некоторую работу А для сохранения прежней
I-прости поезда.
Чтобы найти количество выделившегося тепла, удобнее перей-IH к системе отсчета, связанной с поездом. В этой системе отсчета ... |д стоит на месте и его двигатель не совершает никакой работы, h мистическая энергия кирпича до падения в этой системе равняется
т„ m(v + u)2 m(v2 + и2)
W' =-----2----=-------2----’
। пиело падения — нулю. Следовательно, Wi и есть количество вы-|| 'ипппегося тепла Q. Эта величина не зависит от выбора системы и чета. В исходной системе отсчета использование закона сохранении энергии приводит к следующему соотношению:
JVo 4- А = W + Q,
ii'iii
mu2 . mv2 m(v2 4- w2) —+л=—+-НН'
• I куда следует А = mv2.
Задача 150* (87-г)
Ч тобы полностью вставить пробку в горлышко пустой бутылки, •hi цп приложить силу F = 50 Н. Чтобы вытащить ее обратно, невидима сила F = 75 Н. Оцените, на какую высоту h в вакууме in снимется пробка, “выстреливающая” из первоначально закрытой • ц пилки, если давление газа в бутылке начинает медленно расти. Мшта пробки т = 5 г, ее длина I = 5 см.
Решение. Силу трения скольжения, действующую со стороны ||»р.||ышка на движущуюся пробку, можно считать пропорциональ-•|ц|| площади соприкосновения пробки с горлышком: F(x) = F\x/l} ши .г - длина части пробки, находящейся в горлышке. При вытас-I’ннппии пробки в первый момент надо приложить силу F^ > Fi, так । uh максимальная сила трения покоя может быть и больше силы «рения скольжения. Но когда пробка начнет двигаться, на нее бу-|| I действовать уже сила трения скольжения. Полагая объем проб-|>н малым по сравнению с объемом бутылки, можно считать, что in мление газа остается постоянным при “выстреливании” пробки и mi действует на пробку с постоянной силой F%. Постоянная сила
169
давления и переменная сила трения выталкивают пробку с ускорк
нием

совершая при этом работу
&
Эта работа затрачивается на кинетическую энергию пробки, кого рая в процессе подъема переходит в потенциальную энергию: А mgh. Отсюда получаем
.	(2F2-Fi)Z
Л = —---------
2тпд
= 50 м.
7.	СТАТИКА
Положение равновесия механической системы определяется двумя условиями:
— сумма внешних сил, действующих на покоящуюся систему, |твна нулю (это гарантирует сохранение состояния покоя центра мисс):
N
Е> = °;	(1)
i=l
-сумма моментов внешних сил, действующих на систему, равна пулю (это гарантирует отсутствие вращения вокруг какой-либо оси):
ЕМ>=°-	(2)
1=1
Покажем, что при выполнении условия (1) суммарный момент внешних сил не зависит от выбора начала координат.
Момент внешних сил относительно некоторой точки равен
N
M = J2r<xr<.	(3)
1=1
Момент внешних сил М' относительно другой точки, сдвинутой ив некоторый вектор а:
N
М'=^2Е{Хг<.	(4)
1=1
1Ь пользуя условие г' = г, +а, получаем
N	N
м'=^2 F>х г*+52 Fi х а (5)
»=1	i=l
II силу условия (1) второе слагаемое в правой части равенства (5) • |(|рпщается в ноль, и выражение (5) совпадает с (3). Следовательно, мы можем требовать выполнения условия (2) относительно произ-нмлыюй точки, если убедились в выполнении условия (1).
Гораздо более сложная задача — выяснение устойчивости или ш устойчивости равновесия. Если равновесие устойчиво, то его нарушение приводит к появлению сил, возвращающих систему к по-никгнию равновесия.
171
!б
Рис. 62
По определению потенциальной энергии это означает, что устой чивое равновесие соответствует локальному минимуму потенциаль ной энергии. На рис. 62 изображены примеры устойчивого (а), иг устойчивого равновесия (б) и неравновесного состояния (в) системы
♦ * *
Задача 151 (75-г)
Для ориентации космического корабля включают два двигатели, развивающие силу тяги Fi и F2 = —2F1. Можно ли заменить эти два двигателя одним так, чтобы он оказывал такое же воздействие на движение корабля? Если можно, то куда его надо поместить и какую силу тяги он должен развивать? Расположение двигателей, направление сил тяги и размеры корабля указаны на рис. 63, а Положение центра тяжести корабля неизвестно, а = 45°, L = 3<7/2
Рис. 63. К задаче 151.
Решение. Найдем равнодействующую двух антипараллельныя сил Fi и F2, не равных друг другу по величине. Расстояние между направлениями действия этих сил равно I = d/y/2.
Разложим силу F2 = — 2F1, приложенную в точке В (рис. (13, б), на две силы — силу F^ = —FH приложенную в точке А, и силу Fc = —Fi, приложенную в точке С (АВ = ВС). Силы в точке Л компенсируют друг друга. Следовательно, суммарная сила равна —Fi и приложена к произвольной точке, принадлежащей линии, проходящей через точку С. Исходя из геометрии корабля, нетрудно показать, что эта линия проходит через “нос” корабля.
172
Задача 152 (82-г)
Кубический ящик стоит на полу. При каких значениях коэффициента трения между ящиком и полом легче двигать ящик, а не I* и пговать?
Рис. 64. К задаче 152.
Решение. Найдем сначала минимальную силу Fj, которую нiwi.o приложить, чтобы сдвинуть ящик, при условии, что эта сила ♦•••йетвует под некоторым углом а к горизонту. Все силы, действующие при этом на ящик, показаны на рис. 64, а. Помимо силы F •н’П< твуют силы тяжести mg, нормальной реакции пола N, трения l'\. 11 ри F = Fi ящик начинает двигаться вправо с нулевым ускорением, при этом
FT = ^,	(1)
и сумма всех приложенных к ящику сил равна нулю:
Fj 4- N 4- mg 4- FT = О,
(2)
u ni в проекциях на горизонтальное и вертикальное направления:
Fi cos а - FT = О,	(3)
Fi sin а 4- N - тд = 0.	(4)
II । уравнений (1) — (4) находим
Fi = тд----y—:—.	(5)
cos а 4- /х sm а
। il ia Fi принимает минимальное значение Fimin при таком угле а, при котором знаменатель правой части равенства (5) максимален.
Преобразуем выражение в знаменателе. С этой целью введем VIIMl 0, для которого
sin# =	,б)
cos# =	(7)
173
Это можно сделать, так как правые части выражений (б) и (7) вег гда меньше единицы и соотношение sin2 0 4- cos2 0 = 1 выполняется Очевидно, что
cos а 4- sin а = \/1 4- p2(cos а cos 0 4- sin а sin 0) = \/1 4- ^2cos(a — 0)
Это выражение достигает максимального значения >/1 + /22 при а . 0. Таким образом, из (5) находим
Fimin = т9 г—=7’	(Ю
у1 + ^2
Ящик начинает переворачиваться под действием силы F, если она достигает некоторого значения F\\. Чтобы перекантовать ящик с помощью силы Fn, нужно, чтобы создаваемый ею относительна точки А момент F\iL был равен моменту тда/2, создаваемому силой тяжести (рис. 64, б):
FuL = mg^,	(0)
где а — длина ребра ящика; L — плечо силы F. Из соотношения (9) находим
Fn = тв-.
Сила Fn достигает своего наименьшего значения Fnmin при мак симальной длине плеча L, равной диагонали АВ ящика: L — \/2и Таким образом,
Fnmin = 2^'	(10)
Ящик выгоднее двигать, если
Fimin <
Подставляя в это неравенство выражения для F[ min и Fjimin из (8) и (10), получаем
Р < 1 \/1 4- М2 %\/2
Отсюда < 1/\/7. При таких значениях коэффициента трения легче двигать ящик, а при ц > \/у/1 легче кантовать.
Однако надо еще проверить, при каких значениях коэффициент» трения ящик будет переворачиваться.
Чтобы ящик переворачивался, а не скользил, надо, чтобы сумм» сил, действующих на ящик, заведомо равнялась нулю,— условие
174
। 0, которое в нашем случае можно переписать в проекциях на горн читальное и вертикальное направления как
-£= - FT = О, ч/2
F
-у= + N - тд = 0.
1 ьольжение отсутствует при выполнении неравенства
FT < fiN.
(11)
(12)
(13)
Из (11) — (13) получаем неравенство р > 1/3, которое согласует-II г полученным выше результатом, так как 1/3 < 1/\/7.
Задача 153 (83-г)
Однородный стержень длиной I опирается о пол и ступеньку 11*111’. 65, а). Коэффициент трения между стержнем и полом д = 1, • |i*'iiii>i между стержнем и ступенькой нет. При какой высоте сту-•н in.ки стержень может находиться в равновесии, если угол а = </17
Рис. 65. К задаче 153.
Решение. На стержень действуют силы (рис. 65, б): тяжести »нц, реакции пола Ni, реакции ступеньки N2, трения FT, направлен-iiiiii горизонтально. Если стержень покоится, то сумма действующих пн пего сил равна нулю:
rag + Ni + N2 + FT = 0.
Нрнецируя это векторное равенство на горизонтальное и вертикаль-нии направления, получаем
FT — N2sina = 0,	(1)
—тд + N\ + Мг cos а = 0.
(2)
175
С учетом того, что а = 45° и, следовательно, sin а = cos а = l/v ' равенства (I) и (2) принимают вид
Когда стержень находится в равновесии, должна быть равна ну н и сумма моментов этих сил относительно произвольно выбранньН точки. Рассматривая моменты относительно левого конца стержни получаем
- V2hN2 = 0.
2 V 2
Отсюда
Л г = "77“ •	I'11
4 Л
Из системы уравнений (1) — (3) находим
FT = —7Г-, М = тд ( 1------=- .
Учитывая, что при равновесии сила сухого трения FT < pNi = Л( (так как р = 1), получаем условие равновесия стержня в виде
mgl	(1 О
—-=- < тд 1-------=- .
4>/2Л “	\	4х/2Л/
Этому неравенству эквивалентно следующее:
h > -^=. - 2у/2
С другой стороны, в положении равновесия всегда
h < Z sin о = Z/\/2.
Итак, равновесие стержня возможно, если высота ступеньки такопи что выполняются неравенства
2V2 - - V2’
Задача 154 (76-г)
Два одинаковых кубика с длиной ребра Ъ массой т каждый гн» ят на гладком горизонтальном столе на расстоянии b друг от други Между ними помещен рычаг длиной 2у/2Ь с пренебрежимо мало!!
176
-ни mil. Коэффициент трения между поверхностями кубиков и сто-||| /I Коэффициент трения между рычагом и кубиками очень боль-•к"|| и точке А и очень маленький в точке С. В точке В приложена и hi F, направленная перпендикулярно к рычагу, как показано на
। in 66, а. Определите, какой из кубиков сдвинется раньше, если •i n н'пенно увеличивать силу F.
Рис. 66. К задаче 154.
Решение. Рассмотрим сначала силы, действующие на ры-VII при равновесии системы (см. рис. 66, б). В точке В действует и un F, направленная по нормали к рычагу. Так как в точке С коэффициент трения между рычагом и кубиком, по условию задачи, н|ц’ПгГ)режимо мал, то в этой точке к рычагу приложена только си-pi F| реакции кубика, направленная по нормали к рычагу. В точке I ин рычаг действует неопределенная сила F2. Условия равновесия
(•i.i'inra — равенство нулю суммы всех сил и суммы моментов, созда-ii»ii’Mi>ix этими силами, относительно любой точки. Первое из этих н шпий равновесия имеет вид
F + Fi + F2 = 0.
(1)
llinpoe условие запишем, рассматривая моменты всех сил относи-।«'Hi.iio точки Л:*
F1V26 - F •	= О,
* Выбор точки определяется простотой соответствующих формул—момент nn.i Л’а, направление и величина которой априори не известны, относительно । и н. и А равен нулю.
177
или
Fi-2F = 0.	(Л
Из равенства (1) и параллельности сил F и Fi следует, что и с ил я F2 параллельна F. С учетом этого из (1) и (2) находим
Fi = -2F, F2 = F.
Рассмотрим теперь силы, действующие на каждый кубик. Они показаны на рис. 66, в. Силы реакции Qx и Q2 принимают значении
Qi =тд + 2F/V2, Q2 = тд - F/y/2.
Горизонтальные составляющие Ях и /?2 равнодействующих всех си ч приложенных к кубикам, определяются формулами
/?i = V2F-FTi, F2 = F/v/2-Ft2.
При этом, если ^2F < FTimax и F/x/2 < F^max, то R\ = Л2 = •• » оба кубика покоятся. Правый кубик приходит в движение, если
V2F > FTi max = fiQi = fi (mg + \/2f) .	(!l|
Левый кубик двигается, когда
F/a/2 < FT2max = fiQ2 = fi (mg - F/V2) .	(4l
При д > 1 неравенство (3) не имеет места и, следовательно, в движ»* ние приходит левый кубик. При р < 1 неравенства (3) и (4) можп« привести к виду
T2F > Цт3 — > рт9 1 - fi’ \/2	1 + я'
Минимальная сила Fj, необходимая для приведения в движешь правого кубика, выражается формулой
Минимальная сила Fn, необходимая для приведения в движешь левого кубика—
Fu = 72^_	(.1.
1 + fi
Разделим почленно равенство (5) на (б):
178
— = _li£_	(7)
Fn 2(1 - fi)	[)
II । рпненства (7) видно, что левый кубик приходит в движение, если 1+Д	.	1
5?т-7у>1, т. е, при р>-.
। кубика приходят в движение одновременно, если 1+М	.	1
2(ь^) = 1' ’епри ^З'
Hi pniJM начинает двигаться правый кубик, если 1+д	1
т. е. при
1пдача 155* (87-г)
Качели представляют собой легкую доску, укрепленную по сере-•нш’ на горизонтальной оси. На концы доски садятся два человека, ин 11.1 которых mi = 40 кг и m2 = 60 кг. В начальный момент каче-н1 шризонтальны и неподвижны. Найдите силу давления качелей h i iii’i» в начальный момент. Трение отсутствует.
Решение. Так как доска относительно легкая, то сумма сил, и И* снующих на нее, как и сумма моментов этих сил, может быть 'i nnикона равной нулю. Поэтому оба человека давят на качели с hi пиковой неизвестной силой Г, а качели на ось— с силой F = ‘I Качели начинают поворачиваться, и ускорения людей в первый himoiit направлены по вертикали — у легкого вверх, а у тяжелого пни ।
т^а = Т — Ш1<7, т2а = т2д - Т.
И* них уравнений находим
2mim2
mi *т тп2 н / 2Т«9бОН.
1ндача 156 (78-р)
Проволочная рамка, имеющая вид окружности радиусом г = I mi, стянута мыльной пленкой. Найдите силу сжатия проволоки ||ш условии, что коэффициент поверхностного натяжения мыльно-• - рш твора а = 0,04 Н/м.
179
Рис. 67. К задаче 156.
Решение. Мыльная пленка стягивает кольцо, что вычини ет появление в проволоке сил упругой деформации F, котормг I» требуется найти по условию задачи.
Мысленно разрежем нашу систему по диаметру на две равны» части и рассмотрим силы, действующие на правую половину со г и-роны левой (рис. 67). Направо со стороны левого полукольца д» II ствуют искомые силы F и налево — силы поверхностного натяин ния пленки, показанные на рисунке маленькими стрелками. Ош» действуют на обе поверхности пленки. Равнодействующая этих « м .
Fa = 2ad,
где d—диаметр кольца, приложена к центру кольца. Правая и»» ловина кольца с пленкой покоится, т. е. сила Fa уравновешивав и и двумя равными силами F. Отсюда ясно, что F = Fa/2 — ad = 2ar 0,0008 Н.
Задача 157 (79-г)
Тонкая проволочная горизонтально расположенная рамки имеющая форму окружности длиной I = 10 см, стянута мылы1пИ пленкой. Коэффициент поверхностного натяжения а = 10-2 Н/м Найдите наибольшую возможную массу пленки.
Рис. 68. К задаче 157.
Решение. На рис. 68 показано вертикальное поперечное сияние, проведенное через середину проволочного кольца. На мыльиур пленку в этом сечении действуют: сила тяжести mg, направлениин вниз, и силы ДГ поверхностного натяжения, направленные по кати тельной к поверхности пленки в точках контакта пленки с кольцом Направления этих сил составляют некоторый угол а с вертикал Ын
180
Рш'сматривая точки поверхности пленки, расположенные на про-ин»и||оложных концах диаметра кольца, нетрудно сообразить, что • рц читальные составляющие сил Af в этих точках направлены в ||н||пвоположные стороны и уравновешивают друг друга. Поэтому । nu in действующая F всех сил Af, приложенных к пленке со сто-। <ны кольца, будет направлена вверх; она складывается из верти-। <|'п.пых составляющих всех сил Af. Учитывая, что пленка имеет ни’ поверхности, получаем
F = 2сг/ cos о.
I <ih как пленка покоится, то проекция на вертикальное направление । ммы всех приложенных к ней сил равна нулю, т. е.
F — тд = 2al cos о — тд = 0.
и।сюда находим
т = 2alcosa/g.	(1)
II условиях равновесия масса пленки достигает своего максималь-нм| <> значения mmax при а = 0, когда все силы поверхностного нанок опия Af направлены вертикально вверх. Полагая в (1) а = 0, «•ну чаем
nimax = 2<rl/ff = 0,2 г.
11 tin т > mmax, то пленка отрывается от кольца.
Задача 158 (86-г)
Два велосипеда—складной и обычный — выполнены так, что си-III, необходимая для движения, в обоих случаях одинакова. Известии, что у обычного велосипеда радиус колес в 1,2 раза больше, а рпииус большой шестерни, связанной с педалями, в 1,5 раза больше, и’М у складного. Найдите соотношение радиусов маленьких шестеренок, связанных с ведущим колесом. Педали велосипедов считать щипаковыми; потерями на внутреннее трение пренебречь.
Решение. Введем обозначения: р— радиус педалей вело-- инодов; ri и Г2 — радиусы больших шестерен; и г2—радиусы милых шестеренок; R\ и Т?2 — радиусы колес; индекс 1 относится к |«Гц.|чному, а 2 — к складному велосипеду.
(’истема из педалей и больших шестеренок действует как рычаг. 1гли надавить на педаль с силой Fq, по правилу рычага получим пн втором плече — большой шестерне — силу
Fir2 = Fop/ri^
181
В цепной передаче “изменения силы” не происходит — натяжни» вдоль цепи постоянно. Система из малой шестеренки и колеса rn» i. действует как рычаг. Усилие на ободе колеса
_ р ^,2 — го-----—.
П ,2^1,2
А',2 = ^1,2
г1,2
#1,2
||
По условию задачи F{ = F?} т\/т2 — 1,5, Я1/Я2 = 1,2. Исполни» равенство (1), получаем
=	= 1,8.
Г2 Г1
Задача 159 (74-г)
На полу лежит однородная длинная узкая доска массой ?ti. Iv эффициент трения между доской и полом равен /л Поперек досин » ее торцам прикладывают две равные по величине и противополин но направленные горизонтальные силы F. При каком значении / доска начнет поворачиваться?
F
ttttttttttttttt;;................
X
Рис. 69. К задаче 159.
F
Решение. Сила реакции пола равномерно распредол1Н» по доске, следовательно, силы трения распределены как покачни* на рис. 69. Чтобы доска начала поворачиваться, необходимо, Ч’п» бы момент сил F относительно центра доски, равный FI (/ —длин»» доски), стал больше максимально возможного момента сил тропим Участок доски величиной Дж дает вклад в момент сил трения
А „ _	a	A m
ДМ = цхдАтп = /лхд/АХ—,
где х — расстояние от центра доски до участка Дж. Суммируя вклп ды в момент сил трения от всех участков доски, получаем полный момент сил трения М = iimgl/b. Из условия равенства момешпи находим значение F = цтпд/^ при котором доска начнет поворачн ваться.
182
8. КИНЕМАТИКА
Определить положение тела в пространстве и времени можно н»1||,ко по отношению к другим телам и событиям. Для этого вво-HI к*» понятие системы отсчета (сокращенно с. о.)
('истема отсчета—это совокупность базового тела (тела отсчета) в приборов, которыми измеряются расстояния между различными к ci ками и временные интервалы между событиями. Базовое тело итбходимо для построения системы координат—фиксированной ||ц'||1льной точки (начала координат) и фиксированных направлений (осей координат). В декартовой системе координат три оси прями ни нейны и взаимно ортогональны.
При движении тела меняются координаты его точек. Каждая и ci к а движется по своей линии (траектории). В случае поступали итого движения траектории всех точек тела одинаковы. Для •пнеания такого движения достаточно описать движение одной из ih'ICK.
Материальная точка—тело, размеры и форма которого не су-ц|г| гвенны в рассматриваемом движении. Перемещение точки за и|и’мя А/ — вектор Дг, соединяющий положения точки в моменты н|н’мспи t и t + Д<:
Дг = r(t + Д/) - r(f).
Путь Д5, пройденный телом за тот же промежуток времени,— ин длина соответствующего отрезка траектории. Средняя скорость пн пом промежутке v = Дг/Д/.
(Скоростью точки в момент t называется предел средней скорости ни интервале от t до t + Д/ при стремлении Д/ к нулю:
Дг _ dr
V = hm —- = —.	(1)
At-4-о Д* dt	v '
Ih определения скорости следует, что вектор скорости всегда на-нIтвлен по касательной к траектории точки.
Ускорение—это быстрота изменения скорости. Вектор ускорения может быть разложен на две составляющие —по касательной к ||»м'ктории (at) и по нормали к траектории (an). При этом at харак-н’ризует быстроту изменения величины скорости, а an — быстроту и 1менения направления.
При движении со скоростью v по окружности радиусом R (или по криволинейной траектории с кривизной R в данной точке)
183
a.n — v2/R, В проекциях на ось координат Ох выражение (1) ими •
вид
dx
Vx = Tt-
pi
ах, м/с2
Используя это выражение, легко показать, что изменение копр динаты точки от момента времени ti (x(/i) = Xi) до момента /, (2(^2) = ®2) численно равно площади под графиком vr(t) (рис. 70 а). Аналогично, изменение компоненты скорости vx(t) на интерна ле [ti,/2] (vx(/i) = Vxiy ^(^2) = ^2) численно равно площади пин графиком ax(i).
Простейший случай движения — равноускоренное движение ш> прямой (для определенности —по оси х) определяется условием ax(t) — ах — const. Равномерное движение по прямой (аг = 0) частный случай равноускоренного. Для равноускоренного движг ния график ускорения— горизонтальная прямая (рис. 70, б), грп фик скорости — прямая, тангенс угла наклона которой численно рп вен ускорению (рис. 70, в). Вычисляя площади под соответствуют!! ми графиками, получаем
vx2 = Vxl +ax(t2-ti),	(3)
®2 =	— <1) +	- <l)2/2.	('I)
Объединяя аналогичные выражения для равноускоренного движ« ния по остальным направлениям, можно написать в векторном виде
v(<) = vo+at, г(<) = го + vot + а/2/2-	(А)
184
1фч’|» введены обозначения го и vq — радиус-вектор r(t) и скорость v | /) в начальный момент времени (при t = 0).
I’посмотрим движение тела в поле силы тяжести. Направим ось । h iоризонтально, а ось Оу вертикально вверх. Ускорения всех тел и ноле тяжести одинаковы и равны д. Равенства (5) в этом случае ми/к по записать в проекциях на оси координат так:
х = жо + VQxt,	(6)
У = Уо + VOyt - 0t2/2-	(7)
1'д’ли из равенств (6) и (7) исключить время, получим уравнение ||и|гктории в поле силы тяжести:
у(х) = Уо + (ж - Хо)^- -	2^°^ •	(8)
11 'in угол между горизонталью и вектором начальной скорости vo |ииич! а, равенство (8) можно переписать:
ф) = Уо + {X - z0)tga —	(9)
Z l/Q CUo CI
Ггшгпие очень многих задач кинематики сводится к исследованию Iрпвпения (9).
* * *
Задача 160 (74-г)
Известно, что окружающая нас часть Вселенной расширяется. Иидимые с Земли галактики удаляются от нас со скоростями, пропорциональными расстоянию R до Земли, причем коэффициент пропорциональности одинаков для всех галактик: v = HR. Какую кар-। ину расширения Вселенной видят наблюдатели из других галак-I нк?
Решение. Пусть координаты и скорости двух произвольных шлактик А и В в нашей системе отсчета:
уд = HRx, vb = HRb.
Координаты и скорость галактики В в системе координат, связанной • шлактикой А, имеют вид
Rb = Rb-Ra, v'b=vb-va.
185
Отсюда следует, что v'B = HRZB, т. е. картина расширения Вссмн*и ной одинакова для всех наблюдателей.
Задача 161 (74-г)
На горизонтальной равнине установлена пушка, стреляющая п< «и углом а к горизонту. Во сколько раз надо увеличить начальной' скорость i?o снаряда, чтобы:
а)	дальность полета снаряда увеличилась вдвое;
б)	максимальная высота подъема снаряда увеличилась вдвое, в) время полета снаряда увеличилось вдвое?
Решение. Движение тела в поле тяготения Земли на высота много меньших радиуса Земли, можно считать равноускоренным Это ускорение направлено вниз и равное « 10 м/с2. В произволын<|| инерциальной системе отсчета равноускоренное движение описыпи ется уравнениями (5). Для решения задачи выберем начало к<><»|» динат в точке, где установлена пушка. Ось у направим вертикалын' вверх, ось х —горизонтально в направлении полета снаряда. Тоглн уравнения (5) в проекциях на выбранные оси координат примут ним
x(t) = VQxt,	vqx = vq cos a,	(l|
y(/) = vQyt - gt2/2, voy = vo sin a,	(7|
vr(/)=vor,	(**l
vy(t) = vOy - gt,	(4|
Из этих уравнений и найдем дальность, максимальную высоту и время полета снаряда.
Максимальная высота подъема утах определяется из условия
Утах=у(*1), иу(й) = 0.	(hl
Это условие означает, что за время 1\ снаряд достигает высшей том ки траектории, в которой вертикальная составляющая его скорости обращается в ноль. Из условия (5) и уравнений движения (2) и («|| находим ti = VQy/g, откуда
Полное время полета снаряда <2 в два раза больше времени подъ ема до высшей точки:
t2 = 2/1 = Ъиоу/д = 2vo sin а/д.	(7)
186
Дальность полета Z, по определению, равна расстоянию по горн юптали от места падения до места вылета снаряда: I = з?^)-Подставляя выражение (7) в уравнение (1), получаем
I = 2vQxvoy/g = sin 2а.	(8)
II । равенств (6) — (8) следует, что для увеличения дальности и вы-hi ы полета в два раза надо увеличить начальную скорость в у/2
рн I, а для увеличения в два раза времени полета — в два раза.*
Задача 162 (76-г)
Самолет летит на некоторой высоте с постоянным ускорением •I ио горизонтальной траектории. В тот момент, когда его скорость рн in। a v, с него сбрасывают бомбу. Начальная скорость бомбы от-иисптельно самолета равна нулю. Постройте траекторию падения ьммбы с точки зрения земного наблюдателя и с точки зрения летчика. Сопротивлением воздуха пренебречь.
Решение. Неподвижную относительно Земли систему отсчета пыберем так: ось х направлена в сторону движения самолета, ось I/ по вертикали вверх, а начало отсчета расположено на поверх-III и ти Земли под точкой сбрасывания бомбы.
Уравнение движения бомбы в проекциях на оси этой системы координат имеет вид
x = vt, у = ho - gt2/2,	(1)
inc Ло — высота полета самолета. Уравнение траектории бомбы поручаем путем исключения из системы (1) времени t:
2vz
* Для сравнения в Добавлении VI приводим другой способ решения подобных задач, основанный на анализе размерностей.
187
Это уравнение параболы. Ее вершина находится в точке с координн тами х = 0, у = ftp. Нисходящая ветвь параболы пересекает ось пн расстоянии х = vy/2ho/g от начала координат. Траектория бомбы и неподвижной системе координат показана на рис. 71, а.
Найдем траекторию бомбы в системе отсчета, связанной с спм»» летом. Ось х, как и раньше, направим в сторону движения, ось // по вертикали вверх, а начало отсчета поместим в точке, все время находящейся точно под самолетом на уровне Земли. В этой систем» отсчета начальная скорость бомбы равна нулю. Ускорение бомби относительно самолета постоянно. Горизонтальная составляющий этого ускорения направлена в сторону, противоположную ускор» нию самолета, и равна ему по величине, вертикальная составляй» щая направлена вниз и равна ускорению свободного падения //. < учетом этого нетрудно написать уравнение движения бомбы в пр»» екциях на оси новой системы координат:
х — —at2/2} у = ho - gt2/2.
Исключая из этой системы уравнений время, получаем:
У - ho + дх/а.
Это уравнение прямой, пересекающей ось у в точке с координат»»II у = ho и ось х— в точке с координатой х = —aho/g. Траектория бомбы, какой ее видит наблюдатель с самолета, показана на рис. 7 I б и в; рисунок б соответствует случаю а > 0, рисунок в — случаи • а < 0.
Задача 163 (76-р)
Как надо бросить камень с поверхности Земли, чтобы он прол»’ тел через точки с координатами а?! = 10 м, = 10 м и хэ = 711 м, У2 = Ю м? Координаты точки бросания Яо = 0, уо = 0. ()»т. х направлена горизонтально, ось у — вертикально. Ускорение сип бодного падения принять равным 10 м/с2, сопротивлением воздуха пренебречь.
Решение. Любое тело в поле тяготения Земли двигается » ускорением g, направленным вниз. Его положение в любой прои» вольный момент времени t определяется радиус-вектором г, кои» рый с течением времени меняется по закону
r = vot + gt2/2.	(И
где vo —начальная скорость. Проецируя уравнение (1) на оси ко ординат, получаем:
х = vOxt, у = VQyt - gt2/2.
188
III ключая из этих двух уравнений время, находим уравнение тра-< 1< |орИИ
(2)
v0y ff 2 У = -X - 7Г2~Х
luiiopoe содержит искомые величины Vor и voy-
Гак как точки с координатами t/i и я2, у2 по условию задачи ИН1Ж11Ы принадлежать траектории камня, то эти координаты удо-Н1Ц’1воряют уравнению (2), т. е. справедливы следующие равенства:
9 J2,	_	9
У1 —	^1 г> 2	о 2 ®2’
иОя; 2^	2*4
(3)
I’l'iiian эту систему уравнений, находим:
д Х\Х2(Х2 — £1) Vor — \	,
У 2 угх2 -У2^1
v _ д x2yl(x2-xi) | gxi(yjx2 - y2xi)
V°y ~ у 2 xi(yix2 - у2хг) у 2 x2(x2-xi)
Подставляя сюда исходные данные задачи, получаем:
vqx = Ю /, vOy = 15 /.
Задача 164 (76-р)
,Метательный аппарат стартует с поверхности Земли. Первые 10 с 1нн|гта двигатели работают так, что вертикальная составляющая \i корения аппарата относительно Земли направлена вверх вдоль • и и // и равна 10м/с2, а горизонтальная — вдоль оси х и также рав-ни 10 м/с2. Затем в течение 20 с полет происходит с выключенными двигателями. В следующие 10 с двигатели включены; при этом вертикальная составляющая ускорения направлена вверх и равна III м/с2, а горизонтальная — вдоль оси х и равна —10 м/с2. Найдите наибольшее значение скорости и ее составляющих, максимальную высоту и дальность полета, скорость в наивысшей и в конечной тонких полета. Считать, что на протяжении всего полета сопротивление пи viyxa отсутствует, а ускорение свободного падения равно 10 м/с2.
Решение. Движение аппарата в каждый из промежутков времени от 0 до 10 с, от 10 до 30 с и, наконец, от 30 до 40 с описывается г in темами из двух уравнений:
г = Го + VO(/ - to) + а(/ - to)2/2,	(1)
v = vq + а(/ - to).
(2)
189
имеющих одинаковый внешний вид и отличающихся друг от друт лишь значениями входящих в них постоянных. Проецируя уравпг ния (1), (2) на оси координат, получаем:
х	=	#о + vox(t	— ^о) +	o,x(i	—	/q)2/2,	(3)
У	=	Уо + voy(t	- to) +	ay(t	-	to)2/2,	('ll
Vx	=	VOX + Ox(t	~ to),	(f')
Vy	=	vOy + ay(t	- to).	(d)
Исследуем движение на I участке полета, которое осуществляй ся в течение времени 7, удовлетворяющего неравенствам 0 < t < III с. В этом случае <о = 0,а?о — Уо = О,^ = voy = 0 и ах = av 10 м/с2. В соответствии с этим система уравнений (3) — (6) принц мает вид*
х	=	5/2,	(7)
У	=	5/2,	(Н)
Vx	=	10/,	(I))
Vy	=	10/.	(1(1)
Исключив из уравнений (7)—(10) время 7, получим следующее уран нение для траектории I этапа полета:
у = х (0 < х < 500 м).	(II)
Это уравнение прямой, составляющей угол 45° с осью х. Таким обра зом, на I этапе полета аппарат движется вдоль прямой из начала координат до точки А с координатами ха и уд, которые найдем, пп ложив в равенствах (7)—(8) t = 10 с. Вычисление дает хд = Уд 500 м. Значения ухд и ууд составляющих скорости v в точке найдем, положив в равенствах (9)—(10) t = 10 с: ухд = ууд = 100 м/с.
Исследуем движение на II этапе полета, которое осуществляется в течение времени /, удовлетворяющего неравенствам 10 < I < 30 с Начальные данные движения на II этапе полета совпадают с копеч ными данными на I этапе, т. е. я0 = хд — 500 м, уо = уд = 500 м, Vso = ухд = 100 м/с и vyo = иуд = 100 м/с. С учетом этих начальных условий и того, что на II этапе полета ах = 0, ау = —g = — 10m/cj,
* Для упрощения записи во всех нижеследующих равенствах подставлены конкретные числовые значения известных величин в системе СИ. При этом „од разумевается, что остальные величины, имеющие буквенные обозначения, тож»> измеряются в единицах системы СИ.
190
i равнения (3)—(6) принимают вид
х
У
Vx vy
= 500 + 100(< - 10),
= 500 + 100(< — 10) — 5(£ — 10)2,
= 100 = const,
= 100-10(^-10).
(12)
(13)
(14)
(15)
Исключая из системы уравнений (12), (13) время /, получаем сле-ц'ихцее уравнение траектории на II этапе полета:
У = ®	2000°)"	(500 < ж < 2500 м)-
^иии
Iн> уравнение параболы. В момент прохождения ее вершины со-• являющая скорости vy обращается в ноль. Приравнивая правую нк ть равенства (15) нулю и решая полученное уравнение относи-п’льно t, находим момент прохождения верхней точки траектории (|очки В) t = 20 с. Координаты вершины хв и ув найдем, положив н равенствах (12), (13) t = 20 с: хв = 1500 м и ув = 1000 м. В конце II п апа движения аппарат будет находиться в точке С траектории • координатами хс = 2500 м и у с = 500 м. Эти значения получаем, положив в равенствах (12), (13) t = 30 с. Значения vxc и vyc соста-1О1ИЮ1ЦИХ скорости в точке С найдем, положив в уравнениях (14), 11Ь) / = 30 с: vxc = ЮО м/с и vyc = -100 м/с.
Исследуем движение на последнем — III участке полета, которое • нвгршается в течение времени /, удовлетворяющего неравенствам III < t < 40 с. Начальные данные движения на III участке полета • ••впадают с конечными данными движения на II этапе, т. е. /о — III с, д0 = хс = 2500 м, уо = — ус — 500 м, vxo = vxc = Ю0 м/с и - vyc = —ЮО м/с. С учетом этих данных и того, что ускорение пн III этапе полета ах = —10м/с2 и ау = 10 м/с2, уравнения (3)—(6) принимают вид
X = 2500 + 100(< - 30) - 5(/ - 30)2,	(16)
у = 500 - 100(t - 30) + 5(/ - 30)2,	(17)
vx = 400- 10(/-30),	(18)
Vy = —1004-10(Z — 30).	(19)
Исключая из системы уравнений (16)—(17) время t, получаем уравнение траектории III этапа:
у = 3000 - х (2500 < х < 3000 м).
191
Это уравнение прямой, соединяющей точку С с точкой Z), имеют» И координаты хп = 3000 м, у о = 0 м. Прямая составляет угол 41» с осью х. В точку D летательный аппарат приходит в момент н|«» мени t = 40 с. Из уравнений (18), (19) следует, что в этот мом«<ю составляющие скорости имеют значения vxp = 0 и vqd — 0. Тнкиь образом, летательный аппарат совершает “мягкую” посадку в точи» D с нулевой конечной скоростью.
Из систем уравнений (9), (10), (14), (15) и (18), (19) следует, чь наибольшего значения составляющая скорости vy достигает в ни ках А и С : vy = 100 м/с. В этих же точках имеет наиболыпр величину и составляющая скорости vx = 100 м/с. Следователю!!», •• полная скорость аппарата достигает в этих точках своего максиму ма
Umax = y/v2xA+VyA = xfac + ^C » 141 м/с-
Максимальные высота и дальность полета равны соответствии!’ ув — Ю00 м и xd = 3000 м, а скорость в наивысшей точке 100 м/с.
Задача 165 (77-г)
Ракета движется с постоянным по величине и направлению у<* к» рением а = 10 м/с2 в течение одного года. Начальная скорость рн»п ты равна нулю. Можно ли найти ее конечную скорость, пользу и» • формулами классической механики? Ответ обосновать.
Решение. Для вычисления скорости тела v, которую пп> приобретает за время t, двигаясь с постоянным ускорением а, к л ш сическая механика дает формулу
v = vo4-at.	(Ii
Здесь vo — скорость тела в начальный момент времени. По услотн’ задачи
vo = 0, |а| = 10 м/с = 10“2км/с и t = 1 год « 32 • 106 с.
Подставляя эти данные в формулу (1), для искомой скорости рпю ты |v| получаем:
|v| = 320000 км/с.
Эта величина превосходит скорость света в вакууме. Так как <|>h|i мулы классической механики справедливы только для скороенН
192
hi» но меньших скорости света, то и найти скорость ракеты, поль-•уцг|» этими формулами, нельзя.*
Задача 166 (77-р)
Автомобиль едет по шоссе, параллельному железной дороге, с фи гоянным ускорением ai = 1м/с2. В некоторый момент времени ипрость автомобиля равна 72 км/ч. В этот момент в поезде, иду-•iii’M па расстоянии 600 м впереди автомобиля в ту же сторону со и простые» 54 км/ч, машинист включает тормозную систему. Далее ..ид движется с ускорением а,2 = —0,5 м/с2 до остановки. Найди-• • расстояние между поездом и автомобилем в момент остановки фф 1да.
Решение. Отсчет времени t будем вести от момента начала "•рможения поезда, а отсчет расстояния S — от положения авто-• |и(|||.пя в этот момент. Расстояние между автомобилем и поездом в фгюьпый момент обозначим через So, их начальные скорости — I»!••• i vol и г?02, момент остановки поезда — /р В моменты времени • /1 автомобиль и хвост поезда находятся в точках с координатами
5i = vqi/ + ai/2/2,
5з = So -f- V02/ + Д2/2/2.
Гф гтояние AS между ними в момент времени t
AS = S2 — Si = So + (V02 — vqi)/ + (^2 “ di)/2/2.	(1)
• ярость поезда
v% = V02 + <22/-
•I момент остановки поезда V2 = 0, т. e.
0 = V02 4- 02/1 •
• |ц|Ида = —^02/^2- Полагая в (1) / = /1, для искомой разности \ находим:
AS = So 4- — (vi — — V2 ] = -360 м. c2 \ a2 /
* Точное выражение для скорости ракеты (с точки зрения земного наблюда--1н) г учетом постоянства скорости света в различных инерциальных системах i> it'in имеет вид
at v = —	----« 200000 км/с,
\/1 + a2t2/c2
м . и .300000 км/с — скорость света в вакууме.
193
Знак “минус” перед правой частью означает, что в момент остшшн к и поезда его “хвост” находится позади автомобиля на расстоянии 360 м от него.
Задача 167 (77-р)
Спортсмен пробегает стометровку за 10 с. Первые 10 м он беж и» с постоянным ускорением а, остальную часть дистанции — с ноги» янной скоростью v. Найдите ускорение а и скорость v.
Решение. При решении задачи будем пользоваться следу ющими обозначениями: ti —время, за которое спортсмен пробегтч первую часть дистанции 51 = 10м;/2 = Ю с — время, за которое ни преодолевает всю дистанцию 5з = ЮО м. Так как первую часть ну ти спортсмен движется равноускоренно, то имеют место следу loniih равенства:
Si = a/J/2, v = ati.	(Il
Разделим почленно первое равенство (1) на второе и из полученпп|« соотношения находим
v = 2Si/ti.	('«Il
Вторую часть пути спортсмен движется равномерно со скоростьь v, и можно написать:
S2 — Si = v(t2 - ii)-	(''I)
Подставим в уравнение (3) выражение для скорости v, а затем |п шим его относительно ti:
fi = 251/2/(5'1 + 5*2).	(il
Подставляя (4) в (2), получаем
v = (Si + 5г)/<2 = И м/с.	(hl
Из второго равенства (1) находим ускорение а = v/t^ используй выражения для ti (4) и v (5):
а = (Si + S2)2/(25itl) = 6,05 м/с2.
Задача 168 (77-р)
Скоростной поезд на испытаниях движется по окружности рп диусом R = 10 км. Через 100 с после начала движения скорое! г поезда, равномерно возрастая, достигает величины 360 км/ч. Ни II дите зависимость полного ускорения поезда от времени t.
194
Решение. При неравномерном движении по окружности -••и н»|) ускорения а имеет две составляющие: at, направленную по «цельной к окружности, и ап, направленную к ее центру. Каса-• и.пня составляющая at характеризует быстроту изменения ско-। «in в данный момент по величине, а нормальная составляющая 1 по направлению. Величина полного ускорения а в любой мо-п‘11г времени t определяется формулой
а = al + al.	(1)
НнИцгм неизвестные и ап. По условию задачи величина линейной нирости v растет равномерно, т. е.
v = at/,	(2)
• Hi —постоянная, не зависящая от времени величина. При t = \	100 с величина v принимает значение vi = 360 км/ч:
«1 = М1,	(3)
туда получаем at = ^i/h.	(4)
Пндгтавляя выражение (4) в (2), находим
v = vit/ti.	(5)
bill нормального (центростремительного) ускорения с учетом (5) ♦ттем написать:
V2 ап“ R ~
Пнцставляя в (1) выражения для и an (4) и (6), получаем
V[t2 ж-
(6)
•1 учетом числовых данных
a = \/1 + 10-а«4.
Пнльзуясь этой формулой для численных расчетов, время надо вы-||ижать в секундах, тогда ускорение будет получаться в метрах в • •'купду за секунду.
195
Задача 169 (78-р)
Каретка прибора прошла путь S следующим образом: нерпу► половину она двигалась с постоянной скоростью v = 12 м/с, ni* рую — с постоянным отрицательным ускорением, так что в копь пути остановилась. Найдите среднюю скорость движения карими
Решение. Средняя скорость равна v = S/i} где t — в|м м« прохождения всего пути, t = ti 4-/2» Л и /2 —время, за которо каретка прошла соответственно первую и вторую половины пут
<1 = S/(2v).
Ускорение, с которым каретка прошла вторую половину пути, m
рицательно:	а = -v/t2,
и путь	S . atl	vt2 vt2 -=„12+-=„12-т = т.
Отсюда t>2 — S/v, и средняя скорость: v = 2v/3 = 8 м/с.
Задача 170 (79-г)
В углах квадрата ABCD со стороной I находятся черепахи a, A, • В некоторый момент времени они начинают двигаться с постоянном по величине скоростью v так, что в любой момент скорость черешни а направлена к той точке плоскости, где в этот момент находи и « черепаха Ь, скорость черепахи Ь направлена к той точке плоской п где в этот момент находится черепаха с, и т. д. Сколько врсмипн пройдет от начала движения до встречи черепах? Размерами ’К'р» пах пренебречь.
Решение. В силу симметрии задачи траектории всех чг|н пах будут иметь одинаковую форму и при повороте около центцн исходного квадрата на углы, кратные 90°, будут всеми своими том ками накладываться друг на друга. Поскольку черепахи двигаю и и вдоль своих траекторий с одинаковой скоростью, то в любой мом<мн времени /, отсчитываемый от момента начала движения, они бу;ц • находиться в вершинах некоторого квадрата A'B'C'jD' со сторон»*!’ V < I и с тем же центром О. По мере движения черепах квадри» А'В'С'Р' “сжимается” и поворачивается по часовой стрелке okiui» своего центра О. Отсюда ясно, что встреча черепах произойди »• точке О. Для определения момента встречи t = Т достаточно рп» смотреть движение одной из черепах, скажем, черепахи а.
Обозначим через г(/) расстояние О А' черепахи от центра кин драта в произвольный момент времени t. Вектор ее скорости v (/) ••
196
• ин момент направлен вдоль стороны А1 В* квадрата А1 В* CD1. По .’Фишю задачи длина вектора v(t) есть величина постоянная, не •шигищая от t и равная v:
| v(t) | = v = const.
Проекция вектора v(t) на линию, направленную к центру квадрата, I ИНИН
/ \ I f \ I я* и vr(/) = |v(Z)| cos — =
I HhiiM образом, эта проекция является величиной постоянной. Рас-нц|||ие r(i) черепахи от центра с течением времени меняется по
••ninny
(1)
(4\	1	1 Vt
r(t) =r0-Vrt = -у=- -/=
У Li у Z
|||гг|>	= О А = Ijyfi—начальное расстояние черепахи а от цен-
||ц| В момент времени t = Г, когда черепахи встречаются, г = 0.
11и'1пгая в (1) t = Т и г(Т) = 0, получаем уравнение
^ = 0, у/2
I * Unix которое, находим T = l/v.
Злдача 171 (79-г)
В углах расположенного на горизонтальной плоскости правиль-iniiii шестиугольника ABCDEF со стороной I находятся черепахи । А, с, d, ей/. В некоторый момент времени они начинают двигать-•и г постоянной по величине скоростью v так, что в любой момент кироеть черепахи а направлена к той точке плоскости, где в этот момент находится черепаха 6, скорость черепахи Ъ направлена к той 11»чк<! плоскости, где в этот момент находится черепаха с, и т. д. Ка-11 di путь пройдет каждая черепаха до встречи? Размерами черепах пренебречь.
Решение. Поскольку каждая черепаха ползет с постоянной пи величине скоростью v, то длина L проходимого ею за время Т iiviii вдоль некоторой кривой определяются формулой
L = vT.	(1)
I нким образом, решение задачи сводится к нахождению времени Т.
197
В любой момент времени t после начала движения черепахи в ходится в вершинах правильного шестиугольника с центром в -!••••» Оу и встреча черепах также произойдет в точке О.
Далее мы не будем приводить рассуждения, подробно излпям • ные в решении задачи 170, а покажем лишь результат, к которнм они приводят.
Так же, как и в задаче 170, вектор скорости черепахи (мы ри сматриваем черепаху а) имеет постоянную во времени длину в
| v(t) | = v = const.
Проекция вектора v на направление О А равна
vr = v/2.
Расстояние r(t) черепахи от центра с течением времени меняете и в закону
r(i) = tq — vrt = I - vt/2.	I'
Здесь го = АО = I— начальное расстояние черепахи от цеп ip» В момент i = Ту когда черепахи встретятся, г = 0. Положив в р t = Т и г(Т) = 0, имеем уравнение
l — vT/2 = 0.
Решая это уравнение, находим:
T=2lv.	(ili
Исключая из уравнений (1) и (3) время Г, получаем для длины ну и* Ly проходимого черепахой до момента встречи: L = 21.
Таким образом, проходимый черепахами до момента встречи ну и не зависит от скорости их движения v и ровно в два раза превосм» дит длину стороны шестиугольника.
Задача 172 (79-г)
Цель движется горизонтально и прямолинейно на высоте Л - I км со скоростью v = 720 км/ч. По цели с земли производится вы стрел. Начальная скорость снаряда vq = 400 м/с. При каком раса и янии L между орудием и целью должен быть произведен выстрой чтобы время полета снаряда до цели было минимальным? Считан, что траектория цели проходит точно над орудием. Сопротивлением воздуха пренебречь.
198
Решение. Начало системы координат поместим в точку О, где пн ходится орудие; ось х направим вдоль поверхности земли парал-н ii.no прямолинейной траектории АА1 цели, а ось у — вертикально н»н'рх. Отсчет времени будем вести от момента выстрела.
Нетрудно сообразить, что снаряд достигнет высоты h за минимальное время / = /о, если выстрел производится вертикально ши»рх. В этом случае местом встречи снаряда и цели служит точка 11* пересечения прямой АА1 осью у} а движение снаряда подчиняет-и уравнению
y = vQt- "2”
Пинагая в этом равенстве t = to и у — hy получаем уравнение для пнхождения времени to- Решая его, находим
In юрой (больший) корень уравнения мы отбросили, так как он отнимает моменту вторичного прохождения снарядом точки О1 при его возвращении” на землю).
11тобы снаряд попал в цель, стрелять надо в тот момент, когда ur.ui» находится в точке В, отстоящей от точки (У на расстояние
I = ^о-
искомого расстояния L между точками О и В имеем
L = \/h2+l2 =
« 4640 м.
Задача 173 (79-г)
11а ракете установлены два одинаковых двигателя. Конструкция пит ателей такова, что при включении одного из них ракета движет-। и вертикально вверх с постоянным ускорением ai = 5 м/с, а при мд повременной работе двух двигателей — с ускорением аг = Ю м/с. Двигатели можно включать либо одновременно, либо последова-|гльно. После включения каждый из них работает непрерывно в п’чение промежутка времени /1 = 1 мин, а затем отключается. Как и идо включать двигатели, чтобы ракета поднялась на максимальную высоту? Сопротивлением воздуха и изменением ускорения свободного падения с высотой пренебречь.
199
Решение. Возможны два различных способа включоипн двигателей:
а)	оба двигателя включаются одновременно;
б)	второй двигатель включается в момент окончания работы in р вого двигателя.
Рассмотрим движение ракеты при двух способах включения дин гателей. При этом условимся исследовать движение только до мн мента t = /тах достижения ракетой наивысшей точки подъема |1 этот момент скорость ракеты становится равной нулю.
а) Ракета движется вертикально вверх с ускорением на отрг • ке времени [0,<i] и с ускорением —д на отрезке времени [ti, Скорость ракеты v в произвольный момент времени t определяем и уравнением
v(t\ = S a2t	при/ = [0,<1],
U \ «2*1 - g(t-tl) при t = [tl.tmax],
а высота подъема
>/,ч	( a2t2/2	при i = [0,<i] >
U	\ a2tl/2 + a2ti(t-ii)-g(t-ii)2/2 при t = [h, <max] 
СЛ
Время подъема tmax находим из (1), используя условие v(Zmax) - II
. _аг + <7.
fcmax —	И-
<7
Из формулы (2) получаем выражение для максимальной высот подъема ракеты hmax:
> -a2t^A.nf(i t\ „(^max-Й)2 _ a2tj a22tl
“max — г <*24 (*тах — 4 J — g -	—	~ I	'*
б) Ракета движется вертикально вверх с ускорением ai на отрсч ке времени [0,2/J и с ускорением — д на отрезке времени ^1, t^axj Скорость ракеты v в произвольный момент времени t определяется уравнением
, Г ayt	при t = [0,2/J,
V | 2ajti -g(t-2ti) при t = [2ti,4ax] .
а высота подъема
, Г <*1/2/2	при/= [0,2/1],
—2ai<j+2ai/i< - g(t - 2<i)2/2 при t=
200
Ih равенств (4) и (5) можно по аналогии с выводом формулы (3) trill i и выражение для максимальной высоты подъема Лтах:
A™ax = 2a1t? + 2a?t?/<Z.	(6)
Подставляя данные из условия задачи в выражения (3) и (6), нн1|учаем Лтах « 36 км, Лтах « 54км, т. е. при последовательной рнГк > ге двигателей ракета поднимается на большую высоту.
Задача 174 (79-р)
Из точки, находящейся на высоте хо над землей, одновременно • pin juoT два камня с одинаковыми по величине начальными ско-1"п гями v = 10 м/с — один вертикально вверх, второй вертикально ни и I. Второй камень упал на землю через t = 2 с после броска. Ни какой высоте над землей находился в это время второй камень? • • (противлением воздуха пренебречь.
Решение. Ось х направим вертикально вверх от поверхности ••млн. Движение камней описывается уравнениями
= Яо + vt - gt2/2,	(1)
х2 = хо - vt - gt2/2,	(2)
। nr и x2— положение камней в произвольный момент времени t. Пычитая (2) из (1), найдем расстояние между камнями в момент t :
xi — х2 = 2vt.	(3)
Налагая в (3) t = 2 с и х2 = 0, получаем = 2vt = 40 м.
Задача 175 (79-р)
Ракета стартует с поверхности земли вертикально вверх и в теннис t = 10 с поднимается с постоянным ускорением а = 4,9 м/с2. 1пт(‘м двигатели ракеты отключаются. Найдите максимальную вы-• и гу подъема ракеты над поверхностью земли. Сопротивлением воздуха пренебречь. Ускорение свободного падения считать равным I/ 9,8 м/с2.
Р е ш е н и е. За время работы двигателей t ракета наберет Iкорость
vq = at = 49 м/с
и достигнет высоты
ho = at2/2 = 245 м.
201
Скорость ракеты с течением времени меняется по закону
v = vQ - gt,	11
а высота
h = h0 + vot — gt2/2.	| л
Высота h достигает максимума hmax в момент t = <i, когда скорей и ракеты v = 0. Полагая в равенстве (1) t = и v = 0, получаем:
Л = vQ/g.
При t = ti по формуле (2) находим:
V2 hmax —- ho “F т- — 367,5 м. %д
Задача 176 (80-г)
Представьте себе космический город будущего, имеющий вид п*• лого цилиндра с радиусом R = 1 км. Цилиндр вращается вокрр своей оси с постоянной угловой скоростью си, такой, что эффект» ное ускорение “свободного падения” на внутренней поверхности пн линдра д = 10 м/с2. Из некоторой точки А внутренней поверх] к и in цилиндра бросают камень. Начальная скорость камня го в сиснцп отсчета, жестко связанной с цилиндром, направлена “вертикалы» вверх”, т. е. вдоль радиуса цилиндра в сторону его оси, и составлю । 100 м/с. На каком расстоянии от места броска упадет камень? I'm стояние отсчитывается вдоль поверхности цилиндра. С какой сю» ростью и в какую сторону надо бросить камень, чтобы он побыппм на оси цилиндра и вернулся в исходную точку?
Решение. Будем рассматривать движение камня и цилинл|"» в инерциальной системе отсчета, ось z которой совпадает с осы* цилиндра. В этой системе отсчета все точки поверхности цилиндр имеют одинаковую угловую скорость ы вокруг оси z.
Прежде всего определим, при какой скорости w эффектишп»» ускорение “свободного падения” на внутренней поверхности цилип дра равно д. В этом случае покоящийся относительно вращающей! и поверхности цилиндра камень массой т давит на эту поверхность» силой F = тд. По III закону Ньютона с такой же по величине силиН поверхность давит на камень и сообщает ему центростремительно! ускорение а = w2R. По II закону Ньютона
тд = ты2 R,
202
и куда
W = \fg]~R.	(1)
Рассмотрим движение брошенного камня. В нашей инерциаль-п* *11 системе на брошенный из точки А камень после броска никакие । илы не действуют и он движется равномерно и прямолинейно. Скорм* гь камня v, равная начальной скорости в точке А, складывается in скорости vo, которая направлена от точки А к оси цилиндра, и нпк'йной скорости vi точки А поверхности цилиндра, направленной и*» проведенной в точке А касательной к окружности. Для скорости pi имеем:
V! = wR = \/gR « 100 м/с.
I нким образом, vi = vq. В связи с этим ясно, что вектор v = vq + vi • оставляет с вектором Vi угол а = 45° и v =	+ v? = vgy/2.
Прямолинейная траектория камня пересекает цилиндрическую по-п*’рхность в некоторой точке В. Соединяя точки А и В с центром наружности О, получаем (рис. 72, а) треугольник АОВ. Нетрудно показать, что он является прямоугольным с катетами АО = ВО = R и гипотенузой АВ = R\/2, На прохождение пути АВ камень затра-411 нает время
t = AB/v — R/vq.
In это время радиус О А поворачивается на угол
/у R	y/gR	V1
<p = ut = \	=-----= — = 1 рад,
V Rvo vo	vo
и точка А занимает положение А .
С точки зрения наблюдателя, покоящегося относительно поверхности цилиндра, искомое расстояние s от точки бросания до точки падения камня, измеренное вдоль поверхности цилиндра, будет равно длине дуги А В, которая стягивает угол тг/2 — <р. Очевидно, что
s = r(^-<p) =r(^-1) й 570м.
203
Ответим теперь на второй вопрос задачи. Во втором случш' •• нашей инерциальной системе отсчета камень должен двигаться и прямой, пересекающей ось цилиндра, т. е. по прямой АО А (рис. 77. i Для этого ему надо сообщить в точке А такую скорость v, что! и i складываясь с Vi, она давала вектор v , направленный вдоль |»<» диуса АО. Двигаясь со скоростью v , камень будет преодолении расстояние АА за время
За это же время точка цилиндра А, двигаясь по дуге окружное п« с линейной скоростью vi, должна совместиться с точкой А . Дин этого она должна пройти путь, равный длине нечетного числа пн луокружностей, т. е.
ViT = (2п + 1)тгЯ, п = 0,1,...	(3)
Из (2) и (3) находим
/ _ 2vi _ 2y/gR (2п 4- 1)тг (2п 4- 1)тг ’
Это “вертикальная” составляющая вектора v. “Горизонтальная” п» ставляющая равна ui и направлена против вектора Vi. Для искомой величины v имеем:
V = yjvl + v' = Vi ^1 + (2n +	= \JgR (1 + (^ + 1)2jr2)'
Для максимально возможного v, отвечающего значению п = 0, мп сленные расчеты дают v « 120 м/с. Вектор v должен составлять • “вертикалью” такой угол тангенс которого определяется вырп жением
tgP=vi/v =(п4-1/2)тг.
Задача 177 (80-г)
Два дуэлянта стоят на горизонтальном диске радиусом /?, ко торый вращается вокруг вертикальной оси, проходящей через его центр. Один из дуэлянтов расположен в центре диска, второй— пп краю. Куда должен стрелять каждый из дуэлянтов, чтобы попасть в противника? Угловая скорость вращения диска ш. Скорость пули V. Сопротивлением воздуха пренебречь.
204
Решение. Пуля, вылетевшая из центра диска со скоростью v, мн тигнет края диска за время t = R/v. За это время диск повернет-н на угол а = wf = wR/v. Следовательно, первый дуэлянт должен • 1рглять с “упреждением” на угол а = иR/v по отношению к поло-•lu’iiiiio второго дуэлянта. Второй дуэлянт в каждый момент времени движется со скоростью г2 = wR, направленной по касательной о циску. Пуля, вылетевшая из ствола его пистолета, должна лететь и»» радиусу диска к его центру. Это возможно только тогда, когда I» юрой дуэлянт стреляет под таким углом /3 к радиусу диска, что ..in// = V2/V = wR/v. При wR > v пуля вообще не полетит к центру <ц||’ка и второй дуэлянт не сможет поразить цель.
Задача 178 (80-р)
Две автомашины едут по взаимно перпендикулярным дорогам с постоянными, одинаковыми по величине скоростями. В некоторый м»1мепт времени машины находились на расстояниях G = 1 км и 11	3 км от перекрестка. Найдите минимальное расстояние между
мп in инами.
Решение. Направим оси координат вдоль каждой из дорог в • трону движения машин. Поскольку в условии задачи не оговорено, приближаются машины к перекрестку или удаляются от него, к» координаты и уо положения машин в настоящий момент времени t = 0 могут иметь как положительные, так и отрицательные шпчения, т. е. я?о = ±3 км и у0 = ±1 км. Координаты машин х и у в произвольный момент времени t > 0 определяются формулами
x{t) = xq + vt, y(t) = yo + vt.	(1)
Для расстояния l(t) между машинами можно написать:
W = x/^W+Ж	(2)
1пмсняя в (2) я(/) и y(t) правыми частями (1) и возводя затем обе чисти равенства (2) в квадрат, после простых преобразований получаем
2v2t2 + 2иДя0 + Уо) + Xq + 2/0 - I2 = 0.	(3)
Решая это уравнение относительно находим два значения t\ и /2, отвечающие одному и тому же значению Г.
*1,2 =	± 1^0 + Уо)2-2(х2 + У2-Р).	(4)
Машины дважды находятся друг от друга на одинаковом расстоянии I : сначала в процессе сближения, затем в процессе удаления.
205
Именно поэтому уравнение (3) имеет два решения. При I = /min коренное выражение в (4) обращается в ноль, т. е.
(а>о + г/о)2 = 2(o!o + j/o-/min),	(bl
и уравнение (3) имеет уже единственное решение
у	+ 2/о	.
11 — <2 —----й---•	’Н
2v
Из (5) для Zmin находим
.	_ 1*0 - Уо|
min “ Л •
Из (6) ясно, что t = ti = t2 > 0 только в том случае, когда ж о < и т. е. когда = —3 км. При этом t/o может иметь как положительнн» так и отрицательное значения. Если уо = —1 км, то Zmjn = \/2 км если 1/о = 1 км, то Zmin = 2\/2 км. Если xq = 3 км, то, независимо <м того, положительно уо или отрицательно, I минимально в началыи»||1 момент t = 0. В данном случае расчет дает для /mjn следующг« значение: /min =	= л/10 км.
Задача 179 (82-г)
Две машины едут к перекрестку по прямым дорогам, составлю ющим угол а < тг/2 между собой. Скорости машин постоянны, и* отношение равно v\/v2 = cos а. В момент времени t = 0 машины находились на расстояниях /1 и I2 от перекрестка. Найдите мини мальное расстояние между машинами за все время их движения.
Решение. Воспользуемся системой отсчета, связанной <•• второй машиной. В этой системе отсчета машина 2 неподвижна, и машина 1 обладает скоростью
v'l = V! - V2.
Здесь vi и V2— скорости машин 1 и 2 в системе отсчета, нет» движной относительно Земли. Так как, согласно условию задачи, v\/v2 = cos а, то ясно, что вектор направлен по нормали к век тору vi и, следовательно, в используемой системе отсчета траекто рией машины 1 является прямая. Кратчайшее расстояние /min мг жду машинами равно длине перпендикуляра, опущенного из точки, в которой находится машина 2, на траекторию машины 1. Нетрудно сообразить, что
^min = |^1 —/2C0SQf|.
206
Задача 180 (82-р)
Маленький обруч с радиусом г катится по большому обручу с piiuiiycoM R. Центр О маленького обруча вращается относительно •юн гра О' большого обруча с угловой скоростью ш против часовой Iредки. Маленький обруч вращается относительно своего центра при г и в часовой стрелки с угловой скоростью и/ . Найдите угловую impocTb Q вращения большого обруча относительно своего центра. Прпгкальзывания между обручами нет.
Решение. Будем рассматривать движение обручей в “по-нп1жной” системе отсчета, которая вращается относительно точки с угловой скоростью против часовой стрелки. В этой системе • I» чета центр О маленького обруча неподвижен, а его точки движутся против часовой стрелки с линейной скоростью v = о/г. Так mi к проскальзывания нет, то в используемой системе отсчета с та-и. ill же линейной скоростью v вращаются по часовой стрелке вокруг центра О1 точки большого обруча. Отсюда ясно, что в “подвижной” ист(»ме отсчета большой обруч вращается вокруг точки О' с угло-|м»|| скоростью
td" = v/R = (Jr/R.
II исходной “неподвижной” системе отсчета движение большого обручи складывается из вращения его с угловой скоростью cd" против •псовой стрелки и с угловой скоростью cd по часовой стрелке. Ре-|уш>тирующая угловая скорость вращения Q определяется равен-। IIIDM
Q = Cd — Cd" = Cd — wfr/R,
Положительные значения Q отвечают вращению обруча против ча-и»иой стрелки, отрицательные — по часовой стрелке.
Задача 181 (81-г)
Тело свободно падает на землю с некоторой высоты. Начальниц скорость тела равна нулю. За последнюю секунду оно проходит । ретыо часть всего пути. С какой высоты и сколько времени падало и .но? Сопротивлением воздуха пренебречь.
Решение. Обозначим через h высоту, с которой падает тени, и через th —время его падения с этой высоты. Эти величины । ргбуется найти. Они связаны друг с другом соотношением
h = gtH2.	(1)
Чтобы найти Л и надо помимо (1) иметь еще одно уравнение, । вызывающее эти величины между собой. Имея это в виду, заметим,
207
что за время th -t от начала движения тело проходит путь, рппннн g(th — 2)2/2. При t = to = 1 с по условию задачи этот путь рпн» h - Л/3 = 2Л/3. Таким образом,
2Л/3 = g(th - t0)2/2.	I',
Исключая из системы уравнений (1) — (2) величину Л, получи» »• следующее уравнение для th :
tf - 6toth + 3t2o = 0.	(h
Из двух корней этого уравнения мы должны выбрать тот, который удовлетворяет условию th >to- С учетом этого из (3) имеем
th = (3 4- х/б)/о = 5,45 с.
Зная th, по формуле (1) находим
h = |ff(5 + 2x/6)tg = 150 м.
Задача 182 (81-г)
Тело начинает двигаться без начальной скорости с постоянным по величине и направлению ускорением а. В некоторый момечн ускорение меняется на противоположное. Найдите путь, пройдгн ный телом за время to после начала движения, если перемещен и» тела за время to равно нулю.
Решение. Тело будет двигаться по прямой, направлен цн|| вдоль вектора а. Направим ось х вдоль этой прямой, а точку I1 начала отсчета на оси х совместим с положением тела в начальныII момент времени t = 0. Координата х тела — это и есть перемещен и»
Движение тела можно разбить на три этапа:
1)	равноускоренное движение из точки О с координатой х$ ” в направлении оси г, заканчивающееся в некоторой точке А с к<м>р динатой ха, в которой ускорение меняется на противоположное;
2)	равнозамедленное движение в прежнем направлении, закип чивающееся в некоторой точке В с координатой хв, в которой тел" останавливается;
3)	равноускоренное движение, направленное к началу коордтнн
За время /о тело проходит путь
з = 2хв>	(I)
Таким образом, нахождение пути s сводится к определению копр динаты хв точки В.
208
I hi первом этапе движение подчиняется следующим уравнениям: х = at2/2, v = at.
• Инипачая через /д момент времени прохождения телом точки Л, пн re координаты хд и скорости va в этой точке можем написать
ха = а/д/2,	(2)
ид = а/д.	(3)
1 In втором этапе движение описывается формулами
x(t) = xA+vA(t-tA)-a(t-tA)2/2,	(4)
v(t) - vA - a(t-tA).	(5)
I пн как в моментов скорость тела vb = 0, из уравнения (5) находим tB = ^ + tA = 2tA.	(6)
» учетом этого равенства, а также равенств (2), (3) из уравнения 11) получаем
Хв = at а •	(7)
На третьем этапе движение тела описывается уравнениями
®(t) = хв - a(t - Хв)2/2,	(8)
v(t) = —a(t - хв)-	(9)
II момент времени t = to тело имеет координату г о = 0- Из уравнении (8) с учетом равенств (6) и (7) получаем уравнение
t2A-2totA+t%/2 = 0.
Инс интересует лишь тот корень, который удовлетворяет условию 11 < <о, т. е.
/д=/о(1-1М).
Подставляя это выражение в (7), с помощью раевнства (1) получаем
5 = at% (з — 2л/2^.
Задача 183 (81-г)
Два спутника движутся по круговым орбитам, расположенным и одной плоскости, с линейными скоростями vi = 8 км/с И V2 = / ,1) км/с в одну и ту же сторону. Определите интервал времени г, че-I। который оба спутника периодически сближаются на кратчайшее |нп ( тояние. Считать, что Земля — шар с радиусом R3 = 6400 км^а г к прение свободного падения на поверхности Земли дз = 10 м/с2.
209
Решение. Очевидно, что расстояние между спутниками инн меньшее тогда, когда они расположены на прямой, проходящгII >ц рез центр Земли, по одну сторону от нее (рис. 73).
Рис. 73. К задаче 183.
Пусть в начальный момент времени t = 0 спутники находи и »• на кратчайшем расстоянии друг от друга. Обозначим через ^|(0 •• их угловые координаты в произвольный момент времени I.' I• координаты определяются формулами
где wi и о?2— угловые скорости спутников. Очевидно, чо спутпньн будут вновь находиться на кратчайшем расстоянии друг от дру i н ♦. такие моменты времени tn, что
¥>1(*п) - РгЦп) = (wi -w2)tn = 2тгп, п = 1,2,...
Отсюда
_ 2тгп tn =	’
— 0^2
Спутники периодически сближаются на кратчайшее расстояние д|ц • от друга через промежутки времени
2тг
Г = tn-|-l — tn “	•	(11
— CJ2
Выразим угловую скорость спутника w через известные из услоппн задачи данные — v, дз и 7?з. На спутник массой т, движущийся in> орбите с радиусом Я, со стороны Земли действует сила тяготения
Здесь G — гравитационная постоянная и Мз— масса Земли. Если бы спутник покоился на поверхности Земли, на него действовала Г>ы сила
- тМ3
f =	= т^-
210
• । • । равенство позволяет выразить в формуле (2) произведение GM3 pei величины R3 и дз‘.
F = тдз (Я3/Я)2 .
•I »«1 действием силы F спутник получает ускорение
а = F/m = 93 (Я3/Я)2 •	(3)
»••• ускорение является центростремительным, и для него можно •мшгнть:
a = v2/R.	(4)
Hi равенств (3) и (4) находим радиус орбиты
Я = Яз(ЯзЛ)2 •
ну । ник вращается по орбите с угловой скоростью
- v - у3
U~R~g3R3	(5)
||< пользуя формулу (5), из равенства (1) находим
Т =	« 1,4 • 105с w 4ч.
Задача 184 (81-р)
Мишина, трогаясь с места, проходит путь si = 1 км с постоянным ♦ • и прением во = 0,2 м/с2, затем на отрезке пути $2 = 1 км ее ускорение равно нулю. После этого машина двигается равнозамедленно •in полной остановки, затратив на эту часть пути время /3 = 40 с. II к Идите среднюю скорость v движения машины.
I’ с ш е н и е. По определению средняя скорость равна
_	51 + S2 + 53	/1Х
»= 7Г+т
те «з—путь, пройденный машиной за время /3, a ti и /2 —время, и н’чение которого проделан путь и 8% соответственно.
Найдем время /1, используя формулу для пути при равноускоренном движении с нулевой начальной скоростью:
ti = \/2si/ai.	(2)
211
На отрезке пути s2 машина движется с постоянной скоростью i ( равной наибольшей скорости, достигнутой на отрезке пути si, i ••
и2 = aiti = \/2aiSi.
Время движения /2 вдоль отрезка пути s2 находим из формулы рпн номерного движения:
/2 = s2/v2 = s2/\/2aisi.	(.Il
На отрезке пути S3 движение равнозамедленное с начальной скорм стью v2, поэтому
S3 = ^2/з — 03/3/2,	('ll
где аз — неизвестное пока ускорение. Его нетрудно найти из угли вия, что конечная скорость машины равна нулю:
О = V2 - a3t3.
Отсюда аз = v2/*3*
Подставляя это выражение в формулу (4), получаем
«з = ^2/з/2 = a/2gisi /з/2.	(Ь)
Заменяя в определении средней скорости (1) величины Л, /2 и /ц полученными для них выражениями (2), (3) и (5) и используя чи сленные данные задачи, находим
+ s2 + |*зл/2а151 1О.м а км v =	  .----—----------= 12,6 — = 45,4 —.
^/2si/ai 4- s2/v^ai5i + ^з с	4
Задача 185 (81-р)
Камень бросают вертикально вверх с начальной скоростью v0 10 м/с с высоты ho = 10 м от поверхности земли. Через время /о 1 с после этого с поверхности земли вдоль той же самой вертикали бросают вверх другой камень с начальной скоростью и3 = 20 м/< Через какое время и на какой высоте камни столкнутся?
Решение. Будем отсчитывать высоту h от поверхности rw мли, считая положительным направление вверх. Уравнения движг ния первого и второго камней запишутся соответственно в виде
hi(t) = hQ 4- vQt -gt2/2, = uQ(t — to) -g(t -to)2/2.
212
Момент столкновения камней Т определяется из условия
Л1(Т) = Л2(Т), uni
Ло + v0T — дТ2/2 = uq(T — /о) — д(Т ~ ^о)2/2.
и i пода
Ло + uQtQ - gt%/2
1 =-----------------= 1,75 с.
«о - vo + gto
II ।пк, камни столкнутся через 1,75 с после бросания первого на вы-м|е II от поверхности земли:
Н = h^T) = Л2(Т) = Ло + vqT - дТ2/2 = 12,2 м.
Задача 186 (82-р)
Шарик налетает с горизонтальной скоростью vq на наклонную •носкость с углом при основании а. Его удары о плоскость абсолют-пи упругие. Время удара пренебрежимо мало. Найдите максималь-♦♦»•<’ удаление шарика от нормали к плоскости, проведенной через инку падения, время этого удаления и число соударений с плоско-• 11.10.
Решение. Ось х декартовой системы координат направим Kin ии> наклонной плоскости вверх, ось у — перпендикулярно к плос-юн ги через точку падения. В момент перед ударом скорость шарика • (1, <•<» составляющие на оси х и у:
vGx = vo cos а , voy = vo sin a.
При ударе сила нормальной реакции плоскости меняет только у-инмпоненту скорости. Удар абсолютно упругий, поэтому после уда-|Hi = — voy, ^-компонента не меняется: Vgr = vqx. После первого птра:
Vqx = Vo cos a , Vgy = vo sin a.
Ногле каждого следующего удара z-компонента не меняется, у-|н1мпонента только меняет знак.
Скорость шарика v и расстояние г между точками первого и нюрого ударов:
v = v0 + gi, г = v'ot+ g<2/2.
II проекциях на оси х и у:
vx = vo cos а — д sin at, х = vq cos a i — g sin a t2/2,	(1)
vy = vo sin a — g cos at, у = vq sinaf — g cos a/2/2,	(2)
213
Время удара пренебрежимо мало, и я-компонента скорости и» •• время не меняется, поэтому уравнения (1) справедливы нс тичн между первым и вторым ударами, но и во все время движении »• позволяет использовать уравнения (1) для нахождения максими н ного удаления Ятах шарика от оси у и времени /тах, затрачиваем» • на это движение.
В момент времени t = /тах составляющая скорости vx обрати» • ся в ноль:
О = vo cos а - д sin а /тах,
откуда ьтах — etg а.
9
Полагая во втором уравнении (1) t = /тах, получаем
_ 1 Vo ( Ятах — п COSQCtgQ.
2 9
Найдем промежуток времени t между первым и вторым ударим и Для этого воспользуемся вторым уравнением (2). Поскольку уи»ч о наклонную плоскость происходит в точках с у = 0, из втор»н уравнения (2) находим
,	2v0
/1 = —tga.
9
Положив в первом уравнении (2) t = ti, найдем ^-составляют скорости шарика в момент начала второго удара:
= —vq sin a,
откуда следует, что viy = vq^ . Уравнениями (2) можно пользовать» и и для описания движения шарика между последующими ударами о наклонную плоскость, если начинать отсчитывать время от кин ца предшествующего удара. Таким образом, все промежутки пр» мени между любыми двумя последующими ударами о наклоннуь' плоскость равны Т = 2votga/^. Выделяя целую часть отношении ^тах/Т, находим число п' промежутков времени Г, содержащихся н промежутке /тах:
max 1 , 2
Т-] =
Искомое число п ударов шарика о наклонную плоскость на единит больше числа п':
п = п' + 1 =
1	. 2 '
2	ctg °
+ 1.
214
1пдача 187 (83-г)
Ведущие колеса правой и левой гусениц танка вращаются в од-•«•»м направлении с угловыми скоростями и о^2 соответственно । j • o>i). Найдите радиус окружности, по которой движется танк, н угловую скорость его движения. Ширина танка /, радиус веду-•UII х колес г. Середины гусениц не проскальзывают относительно •и шгрхности земли.
Решение. Так как проскальзывания нет, средние части право-• •• и левого бортов танка за малое время Д/ проходят соответственно ••у I I.
Д5 = v&t =	(1)
Д5 = иД/ = и>2гД/.	(2)
Имеете с тем, нетрудно увидеть, что
Д5 = ^(Я-//2)Д/,	(3)
Д5 = ^(Я + //2)Д/,	(4)
щи Н—радиус окружности, по которой движется центр танка, а ч» угловая скорость движения центра танка вокруг центра этой наружности. Из равенств (1)—(4) находим
/	о f(wi+w2)
W-(W2-W1)7,	—у.
I Невидно, что Я > //2, т. е. центр, вокруг которого движется танк, нежит вне его со стороны ведущего колеса, вращающегося с меньшей скоростью..
Задача 188 (83-г)
Два колеса с одинаковым радиусом г катятся навстречу друг и ругу по длинной прямой доске без проскальзывания. В неподвижной системе координат центр правого колеса движется поступательно со скоростью ui, а само колесо вращается около своего центра против часовой стрелки с угловой скоростью и. Центр левого кошта движется поступательно со скоростью г/2- Найдите угловую скорость вращения левого колеса.
Решение. Предположим, для определенности, что в неподвижной системе отсчета доска движется вправо с некоторой скоростью и (такое движение наблюдается при шг > uj. Тогда относительно носки центр правого колеса будет двигаться влево со скоростью
vi = ui + и,	(1)
215
а центр левого — со скоростью
V2 = U2 - и	1'1
вправо, если U2 > и, и влево, если щ < и. С другой стороны, ни как проскальзывания нет, то
V1 = шг и V2 = г.	(.11
Знак плюс отвечает случаю, когда U2 > и и левое колесо вра1цп<ч> и против часовой стрелки, а знак минус — случаю, когда U2 < и и <hi -вращается по часовой стрелке.
Из системы уравнений (1)—(3) находим
w = ± I---------.
\ г /
Итак,
'	«1+^2
—----------Of .
г
Если о; > («1 +u2)/r, то вращение левого колеса происходит в лепут сторону, иначе — в противоположном направлении.
Задача 189 (83-р)
Частица массой т движется по прямой. Ее уравнение движении имеет вид та = —кх + с, где а — ускорение частицы, х — ее коорлн ната, к и с— некоторые постоянные, обладающие соответствующгП размерностью. Что вы можете сказать о движении частицы?
Решение. Входящее в уравнение движения
та = —кх + с	(I)
ускорение а определяется формулой
d2x a=dt^'
Переместим начало отсчета из точки с координатой х = 0 в точку с координатой яо = с/к. В новой системе отсчета координата х' чп стицы будет связана с ее координатой х в старой системе отсчслн формулой
х = я0 + я* — с/к + х*.	(2)
Подставляя (2) в (1) и принимая во внимание, что
_ d2x _ d2x* а~"с№~ ~dt?'
216
.... новое уравнение движения
та = —кх\
। г уравнение гармонических колебаний с периодом
Т = У/ку/т/к
•к । uio точки с координатой х1 = 0 в новой системе отсчета, или около it|*1ки с координатой х = с/к в старой системе отсчета.
Итак, частица с уравнением движения (1) совершает гармониче-ииг колебания с периодом Т около точки с координатой х = с/к. Амплитуда и фаза этих колебаний определяются начальными у сломи ими движения, которых условие задачи не содержит.
Задача 190 (84-г)
(! какой наименьшей скоростью надо бросить мяч, чтобы он пе-рг метел через стену высотой Л, которая находится на расстоянии I • I места броска?
Решение. Мяч, брошеный под углом а к горизонту, будет пин аться по траектории, описываемой уравнением
дх^
у(х) =®tga- 2-2-o-2 q-.	(1)
Zvq cos q
i и»’ у— высота над точкой броска; х — расстояние от точки броска ин горизонтали; vq—начальная скорость. Условие перелета через • iriiy:
у(ж) > h при - х = I.	(2)
Очевидно, что при фиксированном значении угла а минимальная • корость соответствует знаку равенства в условии (2). Переписывая »равнение (1) с учетом условия (2), получаем соотношение
h = Ztga — —,	(3)
2vg cos2 a
Ггшая (3) как квадратное уравнение относительно Vo, определяем и ни меньшее значение vo при заданном значении а. Однако выбрать минимальное значение vq из совокупности решений при различных шипениях угла а — не очень простая математическая задача. По-• мотрим на уравнение (3) иначе. Зафиксируем значение vq и бу-шм искать значения угла а, удовлетворяющие условию (3). С уче-шм тригонометрического тождества 1/ cos2 а ~ 1 Ч-tg 2а перепишем
217
уравнение (3) в виде квадратного уравнения относительно tg п
^te2»-'‘s» + ‘ + ^ = o.
1!
Это уравние, как любое квадратное уравнение, имеет два, одно и н ни одного решения в зависимости от значения дискриминанта. ()ц<м дательный дискриминант соответствует случаю, когда нет решении т. е. мяч не перелетит через стену ни при каком а— скорость тч статочна. Положительный дискриминант—два решения и п । мяч перелетит через стену при ai < а < с*2 (скорость достачнчн велика). Наименьшему значению скорости соответствует обрашени дискриминанта в ноль — единственное значение угла а. Вычигни • дискриминант и приравнивая его нулю, получаем условие
или
Vo -	- д2Р = 0.
Решая это уравнение относительно Vq, получаем
у2 = g(h± \/l2 -f- Л2).
Из двух решений выбираем положительное (знак плюс перед lut нем) и получаем окончательный ответ: наименьшая скорость, iipi которой мяч перелетит через стену, равна
Vo = \/g(h+ \/h2 + l2).
Задача 191 (87-г)
Ракета начинает двигаться с постоянным ускорением = 10 м Через время /1 = 10 с из той же точки в том же направлении начини ет двигаться с постоянным ускорением а2 вторая ракета и догоню • первую за время t2 = 100 с. Найдите ускорение а2.
Решение. Ракеты двигаются равноускоренно, следователь!!»' можно написать для них два уравнения движения:
t2
Xi(t) = a! — ,	(Il
Л
ягЮ =		(’•'»
Z
218
В момент времени t = t\ + /2 ракеты встречаются, т. е. #i(/i 4-IJ 4-$2)- Приравнивая правые части уравнений (1) и (2) при < /| находим значение а2:
(^1 + <г)2	! у 2
а2 = ai -—2	= 12,1 м/с .
t2
Задача 192 (87-г)
Определите время разгона до взлета самолета массой т = 10 т, • •ни он взлетает после достижения скорости vo = 216 км/ч. Сила нп и двигателя растет линейно со временем от нуля до максимально и о значения F = 40000 Н в течение первых to = 15 с, а потом • •• гпется постоянной. Сопротивлением воздуха пренебречь.
Р е ш е н и е. В течение первых t секунд ускорение самолета рнгтет по линейному закону a(f) = at/to, где а = F/m. При t > to и к прение постоянно и равно а. Так как в начале ускорение линейно '1НН1СИТ от времени, то среднее значение ускорения за время to есть и а/2, и к моменту to самолет набирает скорость vi = ato/2. II дальнейшем v(t) = vi + a(t — t0). Время разгона T находим из V» иония v(T) = vq :
Г=^ + ^ = 22,5с. Г Z
Задача 193 (87-р)
Тележка массой т = 100 кг неподвижно стоит на горизонтальных рельсах. На тележке стоит человек массой т = 100 кг. Тележку нннинают толкать с силой F — 100 Н. Спустя t = 15 с обнаруживайся, что тележка движется со скоростью v = 10 м/с, но человека и и пей уже нет. Узнайте, в какой момент человек спрыгнул, если из-иггтпо, что в момент прыжка он толкнул тележку в направлении, перпендикулярном ее движению. Трением тележки и сопротивлением воздуха пренебречь.
Решение. Пусть ti — момент времени, когда человек спрыгнул с тележки. (Время отсчитываем от начала движения.) Вплоть цп угого момента тележка движется с ускорением = F/(2m) и ннбирает скорость vi = ai^i. В момент прыжка скорость тележки не меняется, так как человек толкает тележку в направлении, перпендикулярном ее движению. Остальное время тележка движется • ус корением а2 = F/m и набирает к моменту времени t скорость ’’ ' vi 4- a2(t - / J. Отсюда находим
(1=й_2™ Юс.
F
219
Задача 194* (87-р)
На тяжелой платформе, движущейся по прямолинейному inpu зонтальному участку железной дороги со скоростью v, устанонл» и-орудие. В тот момент, когда орудие поравняется с километре)м.и> столбом, из него должны выстрелить в направлении движения, Никаким углом к горизонту надо наклонить ствол орудия, чтобы рн< стояние между этим столбом и точкой падения снаряда было ннн большим? Считать, что скорость вылетающего снаряда относи м н но платформы не зависит от угла наклона ствола. Сопротивлсши »• воздуха пренебречь.
Решение. Пусть скорость снаряда относительно платформ'» равна vq и скорость платформы относительно земли равна v. В » о стеме отсчета, связанной с землей, проекция скорости снаряди и » горизонтальное направление в начальный момент есть
vo* = vo cos а + v,
а на вертикальное направление
vOy = vo sin а,
где а — угол наклона ствола орудия к горизонту.
Высота подъема снаряда
о/2
— vOy/ ~,	(11
а его удаление от места выстрела по горизонтали
х = vo*t	(Л
Дальность полета в зависимости от угла а определяется из условии падения снаряда на землю в соответствующий момент времени:
Ца) = х при h = 0.
Из этого условия и равенств (1) и (2) находим
. 2v<teVOy 2vg	. . t>o • о < 2r<>v •
IAa) =-------- — ---(1 + cos a) sin a = — sin 2a +---sm n.
9	9	9	9
Искомый угол ao определяется максимальным значением вырпж»< НИЯ
220
11 и нахождения этого максимума приравняем нулю производную
•i:> 
vo cos 2gq + v cos Go = О,
и in, переписывая иначе,
2vq cos2 gq + v cos cto + vo = 0.
Гишня это квадратное уравнение относительно cos ао? получаем
\/v2 + 8vq
cos ао = ——;-----“•
4vq
'h’l Ko показать, что правая часть этого равенства всегда меньше
•шпицы, следовательно, при любых значениях скоростей макси-мильная дальность полета снаряда будет достигнута, если ствол •рудия наклонить к горизонту под углом
у/v2 +
ао = arccos ——------
4vq
221
9. ТЕРМОДИНАМИКА И МОЛЕКУЛЯРНАЯ ФИЗИКА
В большинстве представленных в этом разделе задач рп» сматривается простейшая термодинамическая система— идеальны" газ. Проще всего определить идеальный газ как термодинамнч»' скую систему, параметры которой (р—давление, V— объем, 7’ абсолютная температура, и — число молей) удовлетворяют урапн* нию состояния Клапейрона—Менделеева
pV = vRT,
где R = 8,31 Дж/(К моль)—универсальная газовая постоянная, Однако, чтобы правильно использовать это уравнение, необхн димо понимать, как оно получено, к каким реальным физическим системам относится, какие состояния описывает.
Уравнение (1) можно рассматривать как экспериментальны П факт, относящийся к газам при “нормальных условиях”. Дейстпн тельно, один моль большинства газов при нормальном атмосферном давлении и температуре О °C занимает объем

R • 273,15К
101325 Па
яа 22,4дм3,
т. е. удовлетворяют уравнению состояния (1).
При изменении давления и температуры в широких предел п* уравнение состояния продолжает выполняться. Однако при очгчп низких температурах или очень высоких давлениях газы кондсы и руются (сжижаются) и уравнение состояния перестает выполняй, ся. При сверхвысоких температурах (когда молекулы газа прио6|н-тают скорости, близкие к скорости света) уравнение (1) тоже не пы полняется. Итак, газ удовлетворяет уравнению состояния (1), если он достаточно разрежен, не слишком сильно охлажден и не слишком сильно нагрет.
Теоретический вывод уравнения (1) базируется на основном уравнении молекулярно-кинетической теории, связывающем дапл» ние газа р, концентрацию молекул п (число молекул газа в единиц» объема) и среднюю кинетическую энергию молекул* EKiW:
2 —
Р — и
* Следует помнить, что средняя кинетическая энергия молекул газа 7?иин вычисляется в системе отсчета, в которой центр масс газа покоится, прич»»н
222
111*н выводе этого соотношения в молекулярно-кинетической теории и» пшп.зуются следующие приближения:
I) пренебрегают объемом всех молекул по сравнению с объемом « уип;
У) пренебрегают временем столкновения молекул друг с другом •и । равнению со временем между двумя столкновениями;
3)	пренебрегают взаимодействием молекул на расстоянии । iiHihKo отталкивание при непосредственном соприкосновении).
1'’<’ли сосуд с газом привести в тепловой контакт с термостатом, ♦ .» и,п влепие и температура газа будут изменяться в течение некото-। «I и времени. После установления термодинамического равновесия* нм мнение давления к концентрации молекул газа будет зависеть iHiii.Ko от температуры термостата, т. е. можно записать соотноше-И||Р
р/п = кТ,	(3)
нН' к постоянная Больцмана — величина, переводящая кельвины »• .икнули (к = 1,38 • 10“23 Дж/К.)
('равпивая соотношения (2) и (3), получаем связь макроскопиче-параметра (температуры) с микроскопическим (средней энер-।н. II молекулы):
__	Q
Якин = ^кТ.	(4)
Ингывая эту связь и определение концентрации п (N молекул в illM’MC И)
п = N/V = ^a/V,
। пр /VA = 6 • 1023Monb”L —число молекул в одном моле (постоянная \пигадро), получаем из уравнения (2)
pV = vNAkT.	(5)
^рпвпсния (1) и (5) совпадают, если положить R = kNA.
(’.подует помнить, что уравнение состояния идеального газа (1) имнолняется только для равновесных состояний газа.
Газ, выведенный из состояния термодинамического равновесия, ни «вращается с течением времени к равновесному состоянию (вооб-н|п говоря, другому). Достижение равновесия невозможно без вза
|н>|п ия вращательного и колебательного движения молекул не рассматривает-n 11<»тгому величина Екин представляет собой среднюю кинетическую энергию 'Эпического поступательного движения молекул.
• 'Гермодинамическим равновесием называется такое состояние, в котором ’|>Н1|)()(1копические параметры системы (давление, температура и т. д.) имеют 11||<ч((‘ленные значения, одинаковые для всех ее частей.
223
имодействия молекул. Действительно, представим себе газ, инн няющий некий сосуд и находящийся в таком состоянии, что те и молекулы с энергией выше средней движутся по горизонтали, н ь молекулы с энергией ниже средней — по вертикали. При этом л пн । ние газа на боковые стенки сосуда будет выше давления на нилан* и верхнюю стенки. Это состояние не является равновесным. ( • чением времени восстановится равновесие — максимальная хшня • ность на микроскопическом уровне (одинаковое давление на май р скопическом уровне). Восстановление хаотичности невозможно!, взаимодействия молекул. Таким образом, пренебрегать взаимна» и ствием молекул (т. е. использовать модель идеального газа) mouhi только при наличии термодинамического равновесия.
Рассмотрим другой пример нарушения равновесия. Пусть ии> альный газ заполняет половину объема сосуда, разделенного и« городкой. Если перегородка мгновенно исчезнет, газ заполнит п» • • сосуд. Уравнения состояния (1) недостаточно для определения »• нечных значений давления и температуры газа. Необходимо доп»» • нительно предположить, что в процессе расширения в вакуум м* лекулы газа не сталкивались ни с чем и не меняли своей кинг пи. ской энергии, так как взаимодействием молекул на расстоянии ми пренебрегаем. Следовательно, температура газа в этом процесс г и изменилась, а давление уменьшилось в два раза.
Можно ли использовать уравнение состояния для описания н|>< цессов, в которых изменяются макроскопические параметры cm и мы? Ведь любые изменения этих параметров приводят к нарупнчпи-термодинамического равновесия.
Использовать уравнение состояния можно, если изменения мп кроскопических параметров происходят достаточно медленно п< сравнению с характерными временами восстановления равношч ни в системе. Подобные процессы называются квазиравновесными. Нп пример, сдвиг поршня, закрывающего сосуд с газом, происходит к пн зиравновесно, если скорость поршня много меньше скорости мош кул.
Изменение параметров системы (объем, давление, температура в большинстве задач происходит квазиравновесно, и уравнение i •• стояния выполняется в каждый момент. Сами процессы описи пи ются некоторыми уравнениями (типа F(p, V, Т) = 0) и часто прен ставляются графиками на плоскости р, V или р,Г, или V,T. Дли построения этих графиков надо наряду с уравнением процесса ш пользовать уравнение состояния. Исключая один параметр из эти* уравнений, получаем связь двух оставшихся параметров, которая и представляется в виде графика.
224
I («'равновесные процессы не могут быть представлены графика-||| ।нк как при отсутствии равновесия не только не выполняются I |||||<‘пия состояния, но теряют смысл и сами макроскопические • •1|П1МГТрЫ ру V,T.
На жпое свойство квазиравновесных процессов— их обрати-•... вели газ, расширяясь в вакуум из половины объема сосуда,
• И1и н и нет весь сосуд, то вернуть его назад можно только с помощью н <| пни я. Сам газ не вернется в исходное состояние. Если же мы по-..ими газу расширяться, медленно сдвигая поршень, то можно нн же медленно двигать его обратно, возвращая газ в исходное со-и и1 и не.
Идеальный газ представляет собой простую термодинамическую in 1гму прежде всего потому, что потенциальная энергия взаимо-п lb гния молекул газа пренебрежимо мала по сравнению с кинети-•• • кий. Энергетические соотношения для такой системы достаточно и।•• и гы. Рассмотрим их.
Первое начало термодинамики (закон сохранения энергии) • нн рждает, что механическая работа над системой и передача ей и iiiin приводят к эквивалентным (равным, при соответствующем I'l iiiupe системы единиц) изменениям внутренней энергии этой си-н мы Д[7 = Q + A*. Здесь U — внутренняя энергия, Q — переданное ш н'ме количество теплоты, А1— работа, совершенная над систе-iiiilL Вводя А = —Л' — работу, совершенную системой над окружавшими телами, получаем более обычную и удобную запись первого и и'| пл а термодинамики
<? = Д£/ + А.	(6)
11 • it закон справедлив для всех систем и всех процессов (равновесных и неравновесных). В этой универсальности его преимущество in ।и’Д другими соотношениями (уравнением состояния).
11 случае идеального газа наиболее просто описывается внутренний энергия системы. Действительно, из соотношения (4) следует, пи кинетическая энергия хаотического поступательного движения иннскул газа есть Uq = NEKKH = ^vRT. Поступательное движе-iiiii’ - это движение по трем направлениям в пространстве. Оно име-। । три степени свободы. Вводя число степеней свободы i вместо трех и пыражение для {7о, получаем более общее выражение
и = l^RT,	(7)
А
• нраведливое и для многоатомных газов с числом степеней свободы iiiuibiue трех (для двухатомного газа г = 5).
225
Совершаемая газом под давлением р элементарная рабсл и Л ’ при малом изменении объема Д V равна
ДА = рДУ.	I"
Складывая элементарные работы (8) в некотором процессе, пнч. чаем работу, численно равную площади под графиком соотвеи ни ющего процесса на плоскости р, V. Эта работа положительна, г» и-объем газа увеличивается (рис. 74, а), и отрицательна, если оГн.м. уменьшается (рис. 74, б). Для циклического процесса, график и-торого имеет вид замкнутой кривой (рис. 74, в), работа числгин равна (с учетом знака) площади фигуры, ограниченной графикам
При подводе к системе бесконечно малого количества теплин-Д<2 внутренняя энергия (7) меняется на Д/7 = 1//ЯДТ1. С учинм совершенной системой работы ДА получаем из закона сохранении энергии в форме (6) соотношение
ДС? = ^i/ЛДТ + рДИ	(Ui
Разделив обе части равенства (9) на ДТ, получаем общее выражстн для теплоемкости
ДО i ДУ
Для определения теплоемкости в некотором процессе необходим»' установить связь малых изменений объема ДУ и температуры Л/ и воспользоваться равенством (10).*
* Уравнение состояния идеального газа (1) позволяет связать малые изменения параметров р, У, Т:
рДУ + УДр = i/RAT.	(II)
Из уравнения конкретного процесса устанавливается еще одна связь малых и • менений параметров системы. Учет этой связи и равенства (11) позволяет iinllni зависимость ДУ от ДТ.
226
I • плоемкость—важная физическая характеристика системы. И» и и ни теплоемкость чаще всего определяется в экспериментах. В пн»' книги (см. Добавления VII, VIII) подробно рассмотрены про-• и, в которых теплоемкость отрицательна. Это приводит к не-।•||»плыюсти системы — равновесное термодинамическое состояние • 'импиется неустойчивым к бесконечно малым флуктуациям па-•м» ipoB (см. задачу 198).
Положительность теплоемкости — еще одно условие на квази-< «нш тесные обратимые процессы. Не все мыслимые уравнения или нфнки процессов на плоскости р, V можно осуществить на прак-и|»г Этот факт редко учитывается при составлении задач.
Определение теплоемкости (10) и формула (9) позволяют легко н пк лить теплоемкость в простейших случаях.
А Изохорический процесс: V = const, AV — 0. Из (9) сразу • ••IV’IIICM
Cv = Z-vR.
z
Г». Изобарический процесс: р = const, Др = 0. Из (11) следует iHMi. рДИ = i/ЯДТ, из определения (10) следует, что
i 4- 2 cf =
II Адиабатический* процесс: AQ = 0. Теплоемкость Cq = 0.
Г. Изотермический процесс: Т = const, АТ = 0. Из определения I III) следует бесконечность теплоемкости в изотермическом процессе.
II । полученных выражений для теплоемкостей легко вывести iHUidioe соотношение
Ср — Cv = vR.
I к । соотношение, справедливое для идеального газа, является прямым следствием закона сохранения энергии. Именно это соотношение, установленное экспериментально в середине XIX века, послужило основанием для формулировки закона сохранения энергии.
Для вычисления коэффициента полезного действия (КПД) те-п'|п1и)й машины, работающей по замкнутому циклу, необходимо и ншавливать знак выражения AQ. По определению, КПД т/ есть и । ношение совершенной работы к затраченному (подведенному к си-• н’ме) теплу. Для вычисления затраченного тепла надо складывать ы г положительные значения элементарных величин AQ (формула |11)). При решении конкретных задач могут возникать проблемы с
• В добавлении VIII выводится уравнение адиабаты— кривой на плоскости |i, V, описывающей адиабатический процесс (уравнение (4)).
227
определением знака величины Дф на тех участках замкнутой! ин кла, где два слагаемых в правой части равенства (9) имеют рп ин* знаки. В этом случае необходимо сравнивать абсолютные величин* этих слагаемых.
* * *
Задача 195 (80-р)
В теплоизолированном сосуде заключен одноатомный идгн'и ный газ, характеризуемый параметрами ро> Vb и Tq. Сверху « закрыт тяжелым поршнем массой М. В некоторый момент поршни отпускают, и он начинает падать. Найдите значения pi, Vi и /, • тот момент, когда ускорение поршня равно нулю. Площадь порши равна S, давлением атмосферы можно пренебречь.
Решение. Сначала рассмотрим силы, действующие на поршни в произвольный момент времени после того как поршень отпусти чн Он опускается под действием силы тяжести тд) преодолевая пн. давления газа, равную pS.
Второй закон Ньютона в этом случае имеет вид
Мд — pS = Ма.
Когда ускорение а равно нулю, это уравнение переходит в Мд - /ч ' откуда находим
Pi = Мд/S.	(11
Для того чтобы найти объем Vi и температуру Ti, учтем, »ц. сжатие газа происходит без теплообмена с окружающей средой, I » Д<Э = 0, и вся работа, которую совершает поршень над газом, и.ш * на увеличение его внутренней энергии.
Работа, совершаемая поршнем над газом при изменении объем*» газа на малую величину ДУ, равна
ДЛ = -рДУ.	(71
Знак минус показывает, что положительная работа над газом при водит к уменьшению его объема.
Изменение внутренней энергии идеального одноатомного газп
3
ДС7 = -1/ЯДТ.	(31
Приравнивая правые части равенств (2) и (3), получаем связь изм»« нений объема газа и его температуры:
3
рДУ =--1/ЯДТ.	(4)
228
Используя уравнение состояния идеального газа
pV = 1/ЯТ,	(5)
"'нм/ium для малых изменений объема, давления и температуры сони инение
рДУ + VAp=vRAT.	(6)
III4ражая ДГ из равенства (6) и подставляя полученное выра-*рн1н* в равенство (4), устанавливаем связь изменений давления и ь i.rMa:
Др _ 5 р
Д? “ “3 V'
hn соотношение имеет место только при условии
s
pV* = const,
inii) с учетом (5),
TV* = const, Tp~* = const.
» псдовательно, зная ро, Vo и То, получаем величины V и Т в зави-। пмости от р:
с = Мр/ро)-1,	(7)
Т = Т0(р/р0)>.	(8)
Подставляя в (7), (8) давление газа из формулы (1), находим иско-М14с объем и температуру:
(PoSA 6	(Ма\
v1 = vo , Ti=r0 R .
\MgJ	\pQS J
Примечание. Поршень в процессе движения приобретает некоторую скоростью, и-иорую легко найти из закона сохранения энергии
(9)
1цг<*ь в левой части равенства стоит изменение потенциальной энергии поршня, пер-«|нн слагаемое в правой части — изменение внутренней энергии газа, второе слагае-ниг -кинетическая энергия поршня (кинетической энергией газа пренебрегаем, счи-1нл( что его масса много меньше массы поршня). Подставляя сюда вместо V и Т пмПденные выше значения и Т>, получаем скорость поршня в момент прохождения им равновесного положения:
„ _ LsVo Г /роМ*] . „P.Vo Г
\ \Мя) \	~М~[ \p^s) 
229
Проходя по инерции положение равновесия, поршень продолжает движение • новки, совершая работу и увеличивая внутреннюю энергию (и температур) • Поршень останавливается в тот момент, когда его скорость обращается п ш-начинает движение в обратную сторону. Возникают колебания (не гармоннч» < । > Если в системе есть трение, эти колебания со временем затухают и вся киненгн • энергия поршня переходит во внутреннюю энергию газа. В конце концов н»«|>н остановится. После остановки давление газа р2 будет удовлетворять услоншн । повесил поршня (1): pa = Mg/S. Температуру Та и объем газа после о< н»п • можно найти, положив скорость поршня равной нулю в уравнении (9)
и учитывая начальные условия в уравнении состояния идеального газа
РзУа PoVo Т2 “ То
Решая уравнения (10) и (11) относительно V2 и Та, находим объем и темпгрши газа после остановки поршня:
Задача 196 (83-р)
Две тепловые машины работают по циклам ABCDA и АВ( "/’ i (рис. 75). Рабочее тело — идеальный газ. У какой машины болi.ш
КПД?
в с с'
Л' Di D'i
j_____;_____1	У
2 И0 ЗИ0 И Рис. 75. К задаче 196.
Решение. Как известно, КПД тепловой машины равен они-шению работы, совершенной рабочим телом, к количеству теплоiи QH) полученному от нагревателя: т/ = A/Qn.
За один период в цикле ABC(DfA совершается в два рпи большая работа, чем в цикле ABCDA:
Aadcwa = ^AaBCDAi
(II
так как Sadc‘dia — 2Sabcda-
Сравним количество теплоты, получаемое газом в этих цикл пл
Газ получает тепло: на участке АВ, на участке ВС — в цикли* ABCDA, на участке ВС" — в цикле ABC'D'A.
На остальных участках каждого цикла газ отдает тепло холи дильнику.
230
Покажем, что
Qbc1 = ^Qbc-	(2)
I riuio, подведенное к газу, расходуется на совершение работы и • <М1’11гние внутренней энергии:
Qbc = АВс 4- &Ubc> Qbc1 = Авс1 + AUbc-
Н'мгпение внутренней энергии идеального газа пропорционально «нмгпснию температуры:
Ювс ~ {Тс - 7Ъ), 6UBci ~ (Тс> - Тв).
II । уравнения состояния идеального газа и рис. 75 получаем: Т& — 2(Тс — Тв)- Следовательно,
&Ubc’=^UBc.	(3)
Гнвгпства (1) и (3) доказывают справедливость соотношения (2).
II гак, количество теплоты, получаемое газом в процессе \Н( 'ЧУА, равно
Qabc'D'a = Qab 4- 2Qbc>
(4)
। и процессе ABCDA —
Qabcda = Qab 4- Qbc*
(6)
(5)
И » определения КПД с учетом равенств (1), (4) и (5) получаем
_ Адв&Р’А _ ЪАдвсра f]ABC р a Qab + QBC, Qab -р 2Qbc ’
Аавсра WCDt = Qab + Qbc'
• рнвнивая правые части равенств (6) и (7), видим, что t]abcd'A > »/4/Н7М-
(7)
Задача 197 (83-p)
()дин моль идеального газа находится в сосуде между двумя мае-• ннными поршнями, связанными легкой тонкой нерастяжимой ни-н.ю (рис. 76).
Поршни могут перемещаться без трения вдоль оси сосуда. Наружное давление отсутствует. Площади поршней Si и Sg, массы— hi। и m2, длина нити L. Найдите давление газа в сосуде и предельные температуры, при которых возможно существование такой си-। ЮМЫ.
231
IW/////////////7////////A
НШШ
Рис. 76. К задаче 197.
Решение. Условия равновесия верхнего и нижнего поринь »• имеют вид
+Т -pSi =0,	11'
т2д +pS2 - Т = 0,	| и
где Т — сила натяжения нити, ар— искомое давление газа в со< у и» Исключая из уравнений (1), (2) неизвестную величину 7’, iiiim дим
(mi 4- тп2)д
Р~ Si-Si •
Максимально возможный объем газа равен Vmax = LSi, а мини мальный — /т1П = LS2, Из уравнения состояния одного моля ни» ального газа RT — pV получаем предельно допустимые темпера ц ры:
= S=i = (m1+ra!)sL^4-, гС	D1—D2
Tmin =	= (пи + m2)gL q q 
Л,	О1 — 02
Задача 198 (83-г)
Пусть состояние некоторого количества идеального одноатомип го газа характеризуется на плоскости р, V точкой А с координатами (ро, Vb). Постройте на этой плоскости график процесса, проходящий через точку Л, при котором теплоемкость газа была бы отрицатель ной. Возможно ли реализовать такой процесс на практике?
Решение. Определением теплоемкости С газа служит равен ство
С = &Q/AT,
где Д<2— количество теплоты, сообщаемое газу при изменении его температуры на малую величину ДТ. Нам надо построить график такого процесса, для которого Дф и ДТ имеют разные знаки и, следовательно, теплоемкость которого отрицательна.
Проведем через заданную точку А произвольную прямую. Они описывается формулой
p = p0 + A'(V- Vo),	(1)
232
и h постоянная, имеющая смысл тангенса угла наклона прямой . 11мм|)ике.
'I ЧП одного моля идеального газа напишем уравнение состояния
рУ = vRT.	(2)
И»।iniM'iicTB (1) и (2) получаем соотношения между бесконечно машин и вменениями объема, давления и температуры
Др = /<ДУ,
(3)
рД V + С Др = ЯДТ.	(4)
||. и|цо’1ая Др из равенств (3) и (4) и используя уравнение прямой 11), пнходим связь изменений объема и температуры
ДУ =
----
Ро 4- KVQ
(5)
(>н|)сделим теплоемкость газа в процессе (1). По закону сохранении шсргии
д<Э = дс/ + да = |ядг + рДУ.	(6)
IImik гавив выражение для ДУ из (5) в формулу (6) и разделив обе ни in равенства на ДТ, получаем теплоемкость
Ск(р) дт 27?+po + AzVo’
"нюрая зависит от давления и от параметра К. Вблизи интересую-п|г|| нас точки р « ро
Ск(ро) = Ск-
lb следование знака теплоемкости С к в зависимости от К показы-IKK' г, что она отрицательна при следующих значениях К:
_5ро г _Ро
3 Уо Уо ’
Гик им образом, любой процесс с одноатомным идеальным газом бучит иметь отрицательную теплоемкость, если угол а наклона графика процесса на плоскости р, У удовлетворяет неравенству
5 ро + Ро
3Vo < ёа< К»’
233
Покажем, что реализация процесса с отрицательной теплоемки стью невозможна.
Допустим, что такая система существует. Приведем ее в равноп» сие с термостатом с температурой То = PqVq/R. В результате малы флуктуаций система может получить некоторое количество тепл»» ты AQ. При этом из-за отрицательной теплоемкости ее температур» уменьшится. Поскольку тепло переходит от более нагретого тела к менее нагретому, начнется переход тепла от термостата к систем»1 что приведет к еще большему необратимому* увеличению темпер»» туры. Процесс будет происходить до тех пор, пока система пс ш рейдет из исходного равновесного, но не стабильного состоянии и другое, тоже равновесное, но уже стабильное состояние с положи тельной теплоемкостью. Следовательно, система с отрицателыи»И теплоемкостью неустойчива— любые малые отклонения от раин»» весия приводят к переходу системы в другое состояние.
Задача 199 (83-г)
Найдите КПД тепловой машины, работающей по циклу 1-2- 3 4—1, изображенному на рис. 77. Рабочим телом является идеалы! i.ill газ.
2Г0 3^0
Рис. 77. К задаче 199.
Решение. Работа, совершенная газом в этом цикле, численп»» равна сумме площади треугольника 1—2—0, взятой со знаком пл ин (обход по часовой стрелке), и площади треугольника 0 — 3 — 4, в »п той со знаком минус (обход против часовой стрелки). Из графики очевидно, что площади этих треугольников равны, следователь!м», работа равна нулю. Количество теплоты QHf получаемое систем» ill от нагревателя, отлично от нуля (на участках 1 — 2 и 3 — 4 газ го вершает положительную работу и его температура увеличивается) По определению КПД т] = A/Qn получаем ответ: т] = 0.
* Необратимость процесса связана с тем, что тепло не может самопрон» вольно переходить от менее нагретого тела к более нагретому.
234
! Ьщача 200 (82-p)
В процессе 1—2—3 (рис. 78) газ получает количество теплоты Q. НпНднте количество теплоты Qf, получаемое им в процессе 1—4—3. Н« сказанные на рисунке величины Др и ДУ считать заданными.
Рис. 78. К задаче 200.
Решение. По первому началу термодинамики
Q= Д{7 + Л123,	(1)
Q' = ДС/' + Л14з,	(2)
। цг /1123 и Л143— совершенная газом работа в процессах 1 — 2—3 и I 4-3.
'Гак как начальные и конечные состояния газа в обоих случаях ।«/illкаковы, то очевидно, что
Д£/ = Д(Л	(3)
I ‘ учетом (3) из (1) и (2) находим
Q' = Q - М123 - >114з)•	(4)
Обозначим символом А341 работу, производимую газом в процес-н* 3-4—1. Принимая во внимание, что при изменении направления процесса производимая газом работа меняет свой знак, можно написать:
>1143 = “>1з41-
(‘ учетом этого равенство (4) принимает вид
Q' = Q “ (>1123 + >1з41)»	(5)
а /1123 + >1з41 = >112341 —работа, совершаемая газом во время замкнутого цикла 1-2-3-4-1. Эта работа, как известно, численно равна площади фигуры, ограниченной линией графика цикла. В нашем случае это параллелограмм 1-2-3-4, и мы можем написать, что /112341 = >1123 + А341 = ДрД У С учетом этого равенства из формулы (Ь) находим ответ Q( = Q — ДрДУ.
235
Задача 201 (94-г)
Найдите КПД цикла, составленного из двух изобар и /пи участков процесса, отвечающих уравнению состояния pV2 = <»т-» (рис. 79). Рабочим телом является идеальный одноатомный riri
Рис. 79. К задаче 201.
Решение. Так как рабочее тело — идеальный одноатом и нН газ, то изменение его внутренней энергии при изменении тем1м*|м туры на ДГ задается уравнением Д[/ = |р/?ДТ и выполняем»" уравнение Клапейрона—Менделеева pV = рЯТ, где р — количсч- ни молей газа.
Из условия pV2 — const выразим и V3 через Vi:
Pi Vi2 = 4Р1V22,
отсюда V2 = Vi/2, аналогично из
P1V32 =^P1V1
следует V3 = 2Ц.
Подсчитаем количество теплоты, полученное газом на каждом участке цикла. Воспользуемся первым началом термодинамики Д(? = Д£/ 4- А, где AQ— полученное количество теплоты, Л работа газа. Отрицательное Дф означает, что газ отдает тепло.
Рассмотрим процессы, в которых pV2 = const. Найдем Дфг;:
V?	v2
Д<Э12 =	+ У pdV = -Д(рУ)12 4-PiV?У =
Vj	V,
3	/ 9	1 \	1
=	-PiVi) — piV2 ( — — — )= -Р1Ц.
z	\ Vi	Hi /	z
Аналогично определяется Д(?з4:
3	/ 1	1 \
Д<?34 = z(2P1Vi - 4P1V!) - 4piVi — - - = -Р1 Vi.
Z	\Zvi	И1 /
236
кнгрь рассмотрим изобарические процессы. Найдем сначала iiJi । ( гак как процесс изобарический, то р выносится из под знака  inn рала и знака Д):
AQ23= |д(рУ)2з + I pdV =
V2
л	\ /гл 71 \\ г т/
= 4pi т V1 - — + ( 71 - —	= 5pi7i.
Аналогично
/3	\	5
Д<?41 = Pi x(Vi - 2Vi) + (Ц - 2Ц) = --Р1Ц. \	У	"
11 данном цикле газ получает тепло на участках 1-2 и 2-3. Полное ^• iii'iecTBo теплоты, полученное газом от нагревателя за один цикл:
Qh = AQ12 + Д$23 = llpiVi/2.
1'tifnна, совершенная газом за один цикл, равна
А = Д<Э12 4- Д<?23 + Д<2з4 + Д^41 = 2piVi.
I цким образом, КПД цикла т] = A/Qn = 4/11.
* 1|М1.ача 202 (82-г)
Найдите КПД цикла, приведенного на диаграмме р, V на рн< 80, а. Рабочим телом служит один моль идеального одноатом-ци|п газа.
Рис. 80. К задаче 202.
237
Решение. КПД цикла определяется отношением рпГтн. Л, совершаемой газом за время цикла, к количеству теплоты подведенному к газу от нагревателя за это время:
r] = A/QH.	(И
Работа А численно равна площади изображенного на рис. 80, б i|»> угольника АВС:
л 1
А ~
Чтобы найти QHJ рассмотрим превращения энергии, происходя пип на участках АВ, ВС и С А замкнутого цикла АВС А. Испольци уравнение состояния газа pV = RT, находим температуры газа 7 < Тв и Тс в состояниях, отвечающих точкам А, В и С цикла:
Та = PqVq/R, Тв = Тс — 2poVo/R.
На участке АВ идет изохорический процесс. Газ работы не coiwp шает, а его внутренняя энергия изменяется на величину
3	3
&иАВ = ^R(TB - ТА) = -PQVQ > 0.
Согласно первому началу термодинамики это увеличение внутри! ней энергии газа происходит за счет подвода некоторого количеств теплоты
3 Qab = &Uab = nPoVo > 0.
&
На участке С А идет изобарический процесс. Газ сжимается и си вершает отрицательную работу
Аса = PoAV = -poVb,
а его внутренняя энергия изменяется на величину
Q
MJca = ^PoVQ < 0.
Таким образом, на участке С А имеет место неравенство Аса I AUca < 0 и, следовательно, газ отдает тепло (Qca < 0).
Выясним теперь, что происходит на участке ВО, который пред ставляет собой часть участка ВС. Точка О (см. рис. 80, б) текущая точка участка ВС. Обозначим параметры газа (давление, объем и температуру), отвечающие точке О, буквами р, V и Т, они
238
•hi hiiii.i уравнением состояния. На участке ВО внутренняя энергия • »•• и (меняется на величину
Q	Q
^Ubo = -R(T-TB) = -(pV- 2poVo).
При пом газ совершает работу Аво, численно равную площади за-•I । |'Н хованной на рис. 80, б трапеции:
Аво = (Ро + р/2) (V - Vb) •
• ’нк-по первому началу термодинамики на изменение внутренней •и» |н пи газа Д(7во и совершение работы Аво системе должно быть •»• ।нVi,пно количество теплоты
Quo = AUbo + Аво = -z(pV “ 2poVb) 4- (ро 4- (V - Vb). (3)
И «I участке ВС (в который входит и участок ВО) объем газа V и |ч давление р, как следует из графика, приведенного на рис. 80, б, ihi uniы следующей зависимостью:
р = ро (3 — V/Vo)-	(4)
II-иг гавляя в (3) выражение для р (4), имеем
Qbo = ~^ 4—-15V + 11Vo •	(5)
Z \ Ио	/
III (5) следует, что зависимость Qbo от V представляется парабо-i-.ll. пересекающей ось V в точках с координатами
Vi = Vo И V2 = ~rVo = 2,75Vo.
4
lli’lипине параболы отвечает значение
Цо время процесса ВО при изменении V от значения Vb до значении Vp количество теплоты, переданное газу, монотонно растет, и к моменту, когда точка О совмещается с точкой Л, оно составляет
Qbd = Qbo(V)I = ^|poVq.
I V — VD
239
Когда точка О находится в пределах участка DC и приближав i. * точке С, величина Qbo монотонно уменьшается. Это означшч, <• на участке DC газ отдает тепло.
Итак, система получает тепло от нагревателя только на учи» 11 АВ к BD \
97
Qh = QaB + QbD =	I»
Из (1), (2) и (6) находим: т/ = || « 16,5%.
Задача 203 (76-р)
Какая работа совершается во время одного термодинамики ••••• цикла, изображенного на рис. 81, а? Каков КПД тепловой машинг работающей по этому циклу? Рабочим телом является один м»* •• идеального газа.
Рис. 81. К задаче 203.
Решение. Изобразим приведенный на рис. 81, а графт процесса на плоскости V—Т в график на плоскости р—V.
На участке АВ объем сохраняется, т. е. процесс является и ш^ рическим. В соответствии с этим в координатах р—V он изобрпли ется вертикальным отрезком, идущим вверх (при повышении н м пературы давление растет) от pi в точке А до р% в точке В, прим» м из уравнения состояния одного моля идеального газа следует, чн pi = RTijVi, а р2 = 2RTi/Vi = 2pi (рис. 81, б).
На участке ВС объем изменяется прямо пропорционально ап солютной температуре. Это говорит о том, что процесс являем и изобарическим, т. е. в координатах р—V он изображается отрезком параллельным оси V.
На участке CD процесс является изохорическим. Объем газа и» изменяется, т. е. Vc = Vd = 2Vp В координатах р—V он изобрн жается вертикальным отрезком, идущим из точки С вниз, так давление с понижением температуры уменьшается.
На участке DA объем газа убывает прямо пропорционально солютной температуре (прямая проходит через начало коордишн
240
I ) 'О го означает, что мы имеем дело с изобарическим процес-• I I ип.гм изобарически меняется от значения 2V1 до значения Vi, ьпординатах р—V процесс DA изображается отрезком прямой /ч. параллельной оси V.
НпИлгм работу, совершаемую газом, и изменение его внутренней • I'liii, па участках АВ, ВС, CD, DA:
Аав = 0,	(1)
MJAB = Cv(TB-TA) = CvTly
Авс = p(Vc - VB) = R(TC -Тв) = 2RTi,	(2)
Ливс = Cv(Tc-TB) = 2CvTly
Acd = 0,	(3)
MJcd = CV(TD -Tc) = -2CvTly
Ada=P(Va -Vd) = R(Ta -Td) = -ЯТ1}	(4)
MJDA = Cv (7л - TD) = -CvT^
и них формулах Cv = SR/Z, Cp = 5/1/2.
11<мюзная работа, производимая газом за один цикл, равна сумме । чниг (1)—(4), производимых на отдельных участках цикла:
А = Аав + АВс + Acd 4- Аол = RTi >	(5)
I niiii’ircTBO теплоты, получаемое газом на отдельных участках ци-• nt, определяется из первого начала термодинамики Q = AU + А:
Qab	=	MJAB	=	CvTi > 0,	(6)
Qbc	=	AUBc	+	АВс = 2(Су 4- R)Ti	=	2Cp7i	>	0,	(7)
Qcd	=	&Ucd	=	-2Cv7\ < 0,	(8)
Qda	=	AUda	+	Ada = — (Cv 4-	R)(TA	- Td)	=	—Cp7\ < 0. (9)
I In ап м, что количество теплоты положительно на участках АВ, Н(' и отрицательно на других участках* Следовательно, на участке \Н(' газ получает тепло от нагревателя, причем соответствующее |ин|пчество теплоты надо рассматривать как затраченную энергию ||/м и определении КПД. В нашем случае оно равно сумме величин III) и (7):
Q« = Qab + Qbc =(10)
Используя определение КПД т) = A/QH, согласно (5) и (10) окончи и-льно получаем г) = 2/13.
241
Задача 204 (76-р)
Какая работа совершается во время одного термодинамич<ч иы цикла, изображенного на рис. 82, а? Каков КПД тепловой мпшннм работающей по этому циклу? Рабочим телом является один м»• •• идеального газа.
Рис. 82. К задаче 204.
Решение. Задача аналогична задаче 203, но в ходе сг р шения выясняется интересная деталь — если отобразить диаграмм* данной задачи, представленную на плоскости р—Т, на плоско* и V—Т, то получим диаграмму процесса из задачи 203, пущенном * *• обратную сторону.
Что изменится в рассуждениях из решения предыдущей задачи Во-первых, работа, совершаемая за один цикл, изменит знак
a = -rt\.
Во-вторых, количество теплоты и изменение внутренней энергии пи отдельных участках процесса тоже изменят знак. Следователыт количество теплоты, полученное системой, будет теперь равно сум ме правых частей равенств (8) и (9) предыдущей задачи с обратным знаком:
Qn = (Ср + 2Cy)Ti =
Коэффициент полезного действия, определяемый обычным образом, оказывается отрицательным:
ri = A/QH = -2/n.
Этот результат означает, что данная тепловая машина работает как холодильник — работа совершается внешними силами над рабочим телом машины. В результате процесса некоторое количество тепло ты забирается рабочим телом в одном тепловом резервуаре и переда ется в другой (от резервуара с меньшей температурой к резервуару с большей).
242
' 1адача 205 (76-г)
Найдите КПД тепловой машины, работающей по циклу, изобра-м иному на рис. 83. Рабочим телом является идеальный газ.
Рис. 83. К задаче 205.
1'г шение, Состояния А и В находятся на изохоре, а состояния fl к (' — на изобаре, причем на изохоре давление, а на изобаре объем • нгпичиваются в два раза:
Рв=Рс=2рА>	(1)
Vc = 2Va=2Vb-	(2)
I ihu’iictbo (1) следует непосредственно из условия задачи, равенство । J) из пропорциональности объема и давления на участке С А.
Искомый КПД Tj определяется формулой
T1 = A/Qtii	(3)
him А — работа, совершаемая газом за один цикл, QH— количество ♦ • -и ноты, полученное газом за один цикл. Работа А численно равна нннцади, ограниченной графиком процесса на диаграмме р—V:
А= -(рв -Ра)(Ус ~ Va) = t^PaVa-	(4)
ПпПдем QH. Как известно, тепло, переданное рабочему телу, идет на • шсршение работы и на увеличение внутренней энергии:
Дф = ДА + ДСЛ	(5)
Iин определенности будем считать, что мы имеем дело с одноатомным газом. При изменении температуры на ДТ внутренняя энергия । молей этого газа изменится на величину
Q
Д[/ = ^i/ЯДТ.	(6)
243
Используя уравнение состояния идеального газа и формулы (11 и (2), находим ДТ на каждом из участков цикла:

= 7в - Тд = Va&Pab - PaVa vR vR ’
ир Рв^Увс %PaVa
=	~ ±в —----б— =---5— •
vR vR
|/i
(Hl
Складывая почленно равенства (7) и (8), имеем:
	= ТА-Тс = -3^ф-. vR	I'll
Используя формулы (6)—	-(9), получаем:	
лиАВ --	3	3 = -уЯДТдв = тРаУа,	(Illi
&Ubc -	з = -vRATbc = ^PaVa, La	(III
Ы/са --	3	9 = -uJIATca = -xPaVa-Z	Z	(ГЛ
На участке АВ газ не совершает работы: ДЛдв = 0. На учаль» ВС газ при расширении совершает положительную работу Д/1/ь равную площади прямоугольника, расположенного под отреткпм прямой ВС на диаграмме р—V:
ДАвс = Рв(Ув - Va) = %PaVa>
(СИ
На участке С А. газ совершает отрицательную работу (Д/сл 0). Поскольку и изменение внутренней энергии газа на этом учаль» отрицательно (см. (12)), то полное количество теплоты по форму и» (5) на этом участке отрицательно. Это означает, что на участке (’ I тепло переходит от рабочего тела к холодильнику.
На участках АВ и ВС тепло переходит к рабочему телу от пи гревателя:
QH = Дфдв + Д<2вс-	(ИI
Используя формулы (5) и (10)—(14), находим:
3	13
Qh = пРаИд + 5paVa = ~tPaVa-	(1Ы
Л	А
Подставляя в (3) выражения, стоящие в правых частях форму । (4) и (15), находим КПД тепловой машины: г/ = 1/13.
244
Задача 206 (77-р)
Инд некоторым количеством идеального газа совершается рабо-• *» пи циклу, представленному на рис. 84. Найдите отношение мак-пмпльного и минимального объемов газа в этом процессе.
2Г, Т
Рис. 84. К задаче 206.
Рис. 85. К задаче 207.
Решение. Выпишем четыре уравнения состояния идеального ♦ hi, отвечающие точкам 1, 2, 3 и 4 диаграммы. Условимся обозна-Ц| и. объемы, отвечающие состояниям 1, 2, 3 и 4, соответственно
i Ц, 1^2, V3 и V4:
PiV^vRT^	(1)
2P1V2 = vRTly	(2)
2р1/3 = 2^Я7ь	(3)
P1V4 = 2vRT\.	(4)
• рнвпивая правые части равенств (1)—(4), видим, что
V4 = 2Ц = 2V3 = 4У2.	(5)
Iiiiuim образом, газ занимает минимальный объем в состоянии 2 и минимальный—в состоянии 4. В соответствии с равенствами (5)
Vmax/Vmin = V4/V2=4.
Задача 207 (77-р)
Над некоторым количеством идеального газа совершается рабо-1н по циклу, представленному на рис. 85. Найдите отношение мак-• нмального и минимального давлений газа в этом процессе.
Решение. Выпишем четыре уравнения состояния идеального in in, отвечающие точкам 1, 2, 3 и 4 диаграммы. Условимся обозна-IIIи» давления, отвечающие состояниям 1, 2, 3 и 4, соответственно
Pi, Р2, Рз и р4:
PiVi = vRTif	(1)
2p2V1 — vRT\,	(2)
2p3Vi = 2vRT\,	(3)
p4Vi = 2vRT1.	(4)
245
Сравнивая правые части равенств (1)—(4), видим, что
р4 = 2pi = 4р2-
Таким образом, газ имеет максимальное давление в состоянии I минимальное — в состоянии 2. В соответствии с этим
Pmax/Pmin = Р$/Р2 = 4.
Задача 208 (75-г)
Представим себе, что в далеком космосе, вдали от звезд и влпи» • оказалась капля воды. Температура капли О °C. Что будет прои< * дить с этой водой? Замерзнет она или испарится?
Решение. Более быстрые молекулы воды будут вылета и. н капли. Так как внешнее давление не препятствует этому npoiuu •, то вылетевшие молекулы назад не возвращаются и капля охлажп* ется. Поскольку начальная температура капли равна О °C, то ми -замерзает. Таким образом, часть капли испаряется, другая ее чн« н обращается в лед.
Задача 209 (77-г)
Сколько молекул заключено в одном литре воздуха при том in ратуре Т = 300 К и давлении р = 1 атм? Постоянная Больцмпни k = 1,4 • 10-23 Дж/град.
Решение. Зная, что газовая постоянная равна R = A/Va где Na—число Авогадро (число молекул в одном моле), и сити» воздух идеальным газом, запишем уравнение его состояния в сжуй ющем виде:
pV = vNkkT.
Здесь V — объем газа, у— число молей газа в объеме V, Вводим обозначение
лг _pV П vNk кТ'
Выясним физический смысл величины п. Так как у— число молоП a Na — число молекул в одном моле, то п — искомое число молоку и воздуха, занимающего при температуре Т и давлении v объем V.
Принимая во внимание, что р = 1 атм = 105Н/м , V = 1 л 10”3м3, получаем:
„ =	105H/M<10-»M°	= 2 4 10„
1,4 • 10-23Дж/град • 300 град ’
246
1ни,пча 210 (77-р)
II н'плоизолированном сосуде находится 1 кг расплавленного ••инпи при температуре плавления (600 К). В сосуд бросают кусочек и >Н1 массой 100 г при температуре 0°С. Найдите температуру в со-( и’ после установления теплового равновесия. В каком агрегатном • ।пинии будет находиться вещество в сосуде? Теплоемкостью сте-п • огуда пренебречь. Удельная теплота плавления свинца гс = 2,4• нН Дж/кг; удельная теплоемкость свинца Сс = 130 Дж/(кг • град); . н ’И.пая теплоемкость воды Св = 4,2 • 103 Дж/(кг • град); удель-•••♦н и'плота плавления льда гл = 3,4 • 105 Дж/кг; удельная теплота •ш|иобразования воды L3 = 2,3 • 106 Дж/кг.
Решение. Обозначим массу свинца через тс и массу льда через •»i,| В процессе отвердевании свинца при температуре плавления и । игляется количество теплоты
Qi = тсгс = 2,4 • 104 Дж.
При остывании отвердевшего свинца от 600 К до 100°C его темпе-1»н ура изменяется на величину Д71 = 600 - (100 + 273) = 227 град. II и ом процессе выделяется количество теплоты
Q2 = тсСс • ДТ1 = 3,0 • 104 Дж.
При плавлении льда поглощается количество теплоты
Оз = тлгл = 3,4 • 104 Дж.
При нагревании 100 г воды, образовавшейся в результате таяния и.и,а, от 0 до 100 °C поглощается количество теплоты
Q4 = тлС3 • ДТг = 4,2 • 104 Дж.
Мы видим, что количества теплоты 01 + О2 = 5,4 • 104 Дж, выделен-mu о при отвердевании свинца и остывании его до 100 °C, достаточно •I’I>i того, чтобы расплавить лед (01 + Q2 > Оз), но недостаточно для того, чтобы появившуюся при этом воду нагреть до 100°C (Oi + Q2 < Оз+ 04)- Отсюда ясно, что равновесие устанавливается при какой-то температуре Т, удовлетворяющей неравенству
0°С<Т< 100 °C.
Температуру Т можно найти, воспользовавшись уравнением теплового баланса
тпсгс + 7псСс(Т1 - Т) = т„г„ + т„Сл(Т - Т2).	(1)
247
Здесь нами введены обозначения: Т\ — 600 К, Тъ = 273 К. Рсчннм уравнение (1) относительно Т, находим:
Т =	~	~ ГвГг) ед 332 к = 59 °C.
777 С С/с “Ь 771/1
Итак, равновесие устанавливается при Т ъ 59 °C. При этом свпнгн будет находиться в твердом агрегатном состоянии, вода— в жидким
Задача 211 (77-р)
В теплоизолированном сосуде находится 2 л воды при тем in-ратуре 10 °C. В сосуд наливают 0,8 кг жидкого азота при тем пературе кипения (77 К). Найдите температуру и агрегатное стояние вещества в сосуде после установления теплового равнин»-сия. Теплоемкостью стенок сосуда пренебречь. Удельная тепло к» парообразования азота £а = 2 • 105 Дж/кг; удельная теплоем кость газообразного азота Са = 103 Дж/(кг град); удельная и плота плавления льда гл = 3,4 • 1О5 Дж/кг; удельная теплorм кость воды С*в = 4,2 • 103 Дж/(кг град); удельная теплоемкость льд»» Сл = 2,1 • 103 Дж/(кгград).
Решение. Обозначим первоначальную массу жидкой воды через тпь и массу жидкого азота через та. При охлаждении воды <н 10 до О °C выделяется количество теплоты
Qi = твСв • ДТ1 = 0,84 • 105 Дж.
При замерзании всей массы воды выделяется количество теплоты
= твгл = 6,8 • 105 Дж.
Чтобы азот перешел при температуре кипения из жидкого состоя ния в газообразное, требуется количество теплоты
Q3 =	= 1,6 • 105 Дж.
При нагревании всей массы газообразного азота от температуры кипения до 0°C его температура меняется на величину ДТ2 • 273° — 77° = 196°. Чтобы этот процесс прошел, азоту надо сообщить количество теплоты
Q4 = maCa • ДТ2 = 1,6 • 105Дж.
Мы видим, что, с одной стороны, количество теплоты Qi, выдг лившееся при остывании воды, недостаточно для превращения всей
248
пн • ы жидкого азота в газообразное состояние (Qi < Q3) и, с дру-• •<11 । тороны, количества теплоты Qi +<22, отдаваемого водой при • । ывании и полном превращении в лед, слишком много, чтобы пе-। < нести весь азот в газообразное состояние и нагреть его затем до •» ч ’ (Qi + Qz > Q3 + <?4)- Отсюда ясно, что равновесие наступит "ini О °C, однако вода при этом будет существовать одновременно в тух агрегатных состояниях — в жидком и твердом. Массу т„ во-п.1, которая перейдет в твердое состояние, нетрудно найти, исходя н । следующего уравнения теплового баланса:
твСв * Д71 + шлгл = maLa + maCa • ДУУ
Г« 1иая настоящее уравнение относительно тл, находим:
тл = (ma(La + С*а • ДТг) - твСв • ДТ1)/гл = 0,7 кг.
Задача 212 (77-г)
11 горизонтально расположенной тонкой пластинке имеется затяну гое пленкой отверстие площадью S. Пленка обладает свойством пропускать все молекулы, падающие на нее сверху, и лишь часть мо-Н’кул, падающих на нее снизу. Вероятность прохождения молекул < низу вверх, определяемая как отношение числа прошедших через и челку молекул к достаточно большому числу падающих, известии и равна W. Все молекулы, прошедшие через пленку, не меняют • кпрости, а не прошедшие отражаются абсолютно упруго. Найдите ниньемную силу пластинки при нормальном атмосферном давлении.
Решение. Давление газа на некоторую поверхность обусловле-П" взаимодействием молекул газа с этой поверхностью. При ударе молекулы о поверхность на нее со стороны поверхности кратковременно действует нормальная к поверхности сила, которая вызывает и 'мснение импульса молекулы. Согласно III закону Ньютона, такая >ке по величине и противоположная по направлению сила действует in стороны молекулы на поверхность. Усредненное по времени дей-। гвие сил со стороны многих соударяющихся с поверхностью молекул на единицу ее площади мы воспринимаем как давление р.
По условию задачи все молекулы, падающие на пленку сверху. проходят через нее без взаимодействия. Поэтому сила давления < верху на пленку равна нулю. Вероятность прохождения молекул сквозь пленку снизу вверх по условию задачи равна W. Это означает, что из п молекул, попадающих в единицу времени на пленку площадью S, лишь Wn молекул проходят через нее без взаимодей-с । вия, а остальные (1 —PV)n молекул упруго отражаются. Давление,
249
оказываемое на пленку, пропорционально числу молекул, взлимн действующих с участком пленки единичной площади. Если бы irui имодействовали все п молекул, то давление на пленку снизу бн>п бы равно атмосферному и определялось бы формулой
po = kn)	(I)
где k— некоторый коэффициент пропорциональности. В нашем ин случае с пленкой взаимодействует (1 — W)n молекул. Они создай и давление
р = £(1 — IV)n.	(7)
Исключая из (1) и (2) неизвестный коэффициент fc, окончательна получаем
Р = Ро(1 - W).
Таким образом, снизу на пленку действует сила
F = pS = pQS(l-W),	(3)
На остальную часть пластинки силы давления сверху и снизу урвв новешивают друг друга (мы пренебрегаем архимедовой силой). II" этому результирующая сила, действующая на всю пластинку, ранни силе, действующей на затянутое пленкой отверстие, и определяе м и формулой (3).
Задача 213 (78-г)
Два одинаковых герметических сосуда соединены тонкой корт кой прямой трубкой. В сосудах находится некоторое количеств" сильно разреженного идеального газа. В одном сосуде поддержи вп ется постоянная температура , в другом 7V Оцените соотношение давлений газа в сосудах.
Решение. Будем считать, что газ, заполняющий сосуды, нахи дится в равновесном состоянии. Тогда в каждом сосуде число моле кул (п01 и пог), приходящееся на единицу объема, поддерживаете и постоянным. Найдем число молекул пь перелетающих в единицу времени из первого сосуда во второй, считая, что площадь попг речного сечения соединительной трубки есть S, а средняя скорость движения молекул в первом сосуде — vi. Для упрощения расчетов будем считать, что одна треть всех молекул газа движется в верти кальном направлении, другая треть —горизонтально в направле нии оси трубки и, наконец, последняя треть — в направлении, пер пендикулярном к первым двум. В этом случае в единицу времени
250
irpri поперечное сечение соединительной трубки пройдет 1/6 мо-н кул, содержащихся внутри цилиндра с площадью основания 5 и • •(•рнзующей /i = t>i, т. е.
П1 = |nOiSt>i.	(1)
о
Аналогичным образом для числа частиц П2, перелетающих в единицу времени из второго сосуда в первый, находим
n2 = ^n02Sv2.	(2)
Гпк как газ в каждом сосуде находится в равновесном состоянии, ниц = пг» С учетом этого из (1) и (2) получаем
П01/П02 = V!/V2.	(3)
Гпк как абсолютная температура газа пропорциональна средней кинетической энергии молекул, то
vi/v2 = х/Т2/Т\.	(4)
Найдем теперь отношение noi/no2- В газе с массой т, молекулы которого имеют массу то, содержится п = т/тпо частиц. Если этот I in занимает объем У, то в единице объема содержится no = n/V = щ/(гпоУ) частиц. Отсюда ясно, что
прх _ miV2
П02	77*2 Vi
Используя уравнение состояния идеального газа, находим ™х _ pi т2 _ р2
Vi ^RTi' V2 ~{1RT2
11 соответствии с этим
П01	P1^2
— = ТТг'	(5)
7*02	P211
Подставляя в (3) отношение vi/v2 (4) и отношение пох/пог (5), получаем
Р1/Р2 = x/Tl/^2-
Задача 214 (78-г)
На плите греется чайник. Как изменится время до закипания, гели в чайник долить горячей воды?
251
Решение. Передаваемое от нагревателя тепло частпчн» расходуется на нагревание чайника с водой и частично переднен и от чайника окружающей среде. Предположим, что на доведение’ и-закипания недолитого чайника требуется количество теплоты ДЦ Оно складывается из количества теплоты AQi, идущего непос|м'н ственно на нагревание чайника с водой, и количества теплоты AQ. отдаваемого при этом в окружающую среду.
Пусть теперь чайник доливают горячей водой. Теперь на narpi’ вание чайника и всей массы воды необходимо затратить количесием теплоты
AQi — AQi 4* AQi,
где AQi — количество теплоты, необходимое для доведения до к и пения долитой воды. Очевидно, что AQj > AQi (равенство имен место, если чайник доливают кипящей водой).
Попробуем теперь ответить на вопрос: какое количество тепли ты AQ2 будет отдано в окружающую среду при нагревании долит го чайника? Естественно предположить, что теплоотдача чайники с водой в окружающую среду, в основном, зависит от его формы размеров и температуры и слабо зависит от количества воды в пом При этом теплоотдача пропорциональна разности температур чай ника и окружающей среды на достаточно большом расстоянии ni него. Поскольку при доливании горячей воды в чайник его темпера тура увеличивается, то увеличивается и теплоотдача: AQ2 > AQj Следует отметить, что это неравенство имеет место и в том случш», когда чайник доливают кипящей водой.
Итак, полное количество теплоты
AQ = AQ1 + AQ2 >	4“
Отсюда ясно, что при доливании чайника горячей водой время, иг обходимое для его закипания, увеличивается.
Задача 215 (80-г)
Воздушный шарик, содержащий v молей идеального газа при температуре Т, помещен в пустой теплоизолированный сосуд объ емом V. Шарик лопнул, и газ заполнил весь объем. Найдите конеч ное давление в сосуде.
Решение. После того как шарик лопнул, газ, содержавший ся в нем, расширяется в вакуум. Это означает, что внешние силы на него не действуют, и поэтому газ при расширении не совершав i работы. Тепло к газу извне не подводится. Р1з этих двух факторе и
252
•И’цует, что внутренняя энергия газа, а следовательно, и его темпе-I mi ура при расширении в вакуум не меняются. Зная температуру Т н н(>'||('м газа V в конечном состоянии, определяем его давление из । |и1В11ения Клапейрона—Менделеева: р = vRT/V.
Задача 216 (80-г)
В комнате объемом V в течение некоторого времени был вклю-••••II нагреватель. В результате температура воздуха увеличилась от /| до ТУ Давление в комнате не менялось. Найдите изменение вну-ipHinett энергии воздуха в комнате, считая воздух идеальным двухтомным газом.
Решение. При нагревании воздуха в комнате объемом V при постоянном давлении р произведение pV остается неизменным. Принимая это во внимание и обращаясь к уравнению состояния иде-•1'нлюго газа
pV=-RT	(1)
№
I in масса газа, р— молярная масса, R—газовая постоянная и / абсолютная температура), нетрудно сообразить, что при ука-ншных условиях при повышении температуры Т в комнате масса содержащегося в ней газа тп уменьшается, произведение тТ при этом постоянное. Внутренняя энергия газа U складывается из кинетиче-• iuэнергии содержащихся в нем молекул. Средняя кинетическая ц|гргия одной молекулы газа, как известно, пропорциональна его им церату ре Т. Отсюда ясно, что внутренняя энергия газа U пропорциональна температуре газа Т и, кроме того, пропорциональна числу содержащихся в нем молекул, т. е. пропорциональна массе hi in m. Таким образом, можно написать, что
U = СтпТ,	(2)
। и,(' С— коэффициент пропорциональности, определяемый строением молекул газа (если газ одноатомный, то С = ЗЯ/(2М); в случае ппухатомного газа С = 5/?/(2М)). Так как произведение тТ есть пгличина постоянная, то внутренняя энергия U воздуха в комнате иля любого момента времени есть величина постоянная. Сообщаемая воздуху при нагревании дополнительная энергия полностью уносится с той частью воздуха, которая просачивается из комна-ц.| вовне. Итак, если воздух в комнате нагревается при постоянном ппилении р, то внутренняя энергия воздуха в ней не меняется.
Задача 217 (80-р)
Горизонтально расположенный цилиндр разделен поршнем на /нк‘ части. В одной находится идеальный газ, в другой — вакуум.
253
Поршень медленно сдвигают так, что газ заполняет весь цилиндр Рассмотрите этот процесс в следующих двух случаях: 1) цилиндр теплоизолирован; 2) цилиндр помещен в термостат. В каком слупи газ при расширении совершает большую работу?
Решение. В случае 1) газ при движении поршня, расширял» г совершает работу за счет убыли своей внутренней энергии, сопропи ждающейся уменьшением температуры газа. При расширении гп »н в случае 2) его температура и внутренняя энергия остаются пей» менными — газ совершает работу за счет тепла, подводимого изтп»
Рассмотрим элементарную (достаточно малую) работу, совершп емую газом в процессе движения поршня от точки с координатой л до точки с координатой х 4- Дж. Если площадь поперечного сечении поршня 5, то при смещении поршня на Дж объем газа изменится пн величину ДУ = SAx и газ совершает работу
ДА = р(ж)Д V = р(ж)5Дж,
здесь р(ж)—давление газа в цилиндре, когда горизонтальная ко»ц» дината поршня равна ж. Используя уравнение состояния идеал ын» го газа р(ж) = р/?Т(ж)/У(ж), где р — число молей газа в цилиндре R—газовая постоянная, Т(х) и У (а?) — соответственно температурь и объем газа в цилиндре в тот момент, когда поршень находится и точке с координатой ж, и принимая во внимание, что У (ж) = Уо +	,
где Уо —объем газа в момент, когда ж = 0, получаем
=	=	1'1
Так как в случае 2) температура газа неизменна, то при любых .i выполняется Т(х) = Т(0). В случае же 1) Г(ж) с ростом ж монотон но уменьшается, поэтому при ж > 0 всегда Т(ж) < Т(0). С учетом этого из (1) следует, что при смещении поршня от положения с кп ординатой ж в положение с координатой ж + Дж в случае 2) всегда совершается большая работа ДА, чем в случае 1). Складывая и»ъ элементарные работы, совершаемые газом при перемещении порш ня, получаем следующий ответ: Ai < Аг, где Ai и Аг, —работы, совершаемые газом в процессах 1) и 2) соответственно.
Задача 218 (81-г)
Над одним молем идеального одноатомного газа совершает про цесс, описываемый уравнением У(р + ро) = const. Определите коли чество теплоты Дф, которое во время этого процесса получает (или отдает) газ при увеличении его объема на ДУ = 0,01 л. В начальном состоянии давление газа = ро = 1 атм, объем У1 = 22,4 л.
254
Решение. За счет количества теплоты AQ, которое сообщают IM iv и которое надо найти, он при расширении совершает некоторую рнЬиту А, а его внутренняя энергия увеличивается на Д1/ = |7?ДТ, » г
Q
ДЗ = А + |/?ДТ.	(1)
Прежде всего найдем величину ДГ. Используя уравнение состо-miiiui одного моля идеального газа pV=RT, нетрудно преобразовать ipiiniiciiHe процесса V(p + po) = const к следующему виду:
RT 4- роV — const.	(2)
Hi уравнения (2) следует, что изменение температуры газа ДГ во прем я процесса связано с изменением объема газа ДУ соотношением
ЯДТ4-роДУ=0.
< ||гюда
ДГ=-^ДК	(3)
1цпк ‘‘минус” указывает на то, что с увеличением объема, занима-»мпго газом, его температура падает.
Найдем работу А, совершаемую газом. С хорошей степенью точ-инг ги
Л = Р1ДУ-^ДрДУ= (1-^)роД^«роДУ.	(4)
z	\	2 ро /
1|иЦ( тавляя правые части равенств (3) и (4) в соотношение (1), по-IV'IIICM
Д<Э = -|р0ДК
Задача 219 (81-г)
11ад одним молем идеального одноатомного газа совершают про-..г, описываемый уравнением У(р4-ро) = const. Определите ко-ni’ircTBO теплоты Дф, которое необходимо сообщить газу, чтобы уличить его температуру на ДГ = 1°С во время этого процес-• п. Давление газа в начальный момент pi = ро = 1 атм, а объем Г, :22,4 л.
Решение. Так как газ находится при нормальном давлении и nt и и мает такой объем, какой занимает грамм-молекула идеального । и in при нормальных условиях, то начальная температура газа Т\ = ЛЗ К. Таким образом, ДТ/71	1.
255
За счет сообщения газу тепловой энергии Дф увеличивается иг внутренняя энергия на величину |ЯДТ (R— газовая постоянная) н газ, расширяясь, совершает некоторую работу А. Согласно первом! началу термодинамики
Д<Э = |яДТ + Л.	(И
С хорошей степенью точности работу можно найти, считая ди вление газа в этом процессе приближенно постоянным:
А«р1Д/.	(7)
Учет изменения давления газа приведет к поправкам поряди»» ДрДИ Эти поправки малы, так как из уравнения процесса следу»'» что ДУ Др
V ~ р + ро'	11
а из уравнения состояния идеального газа имеем
Д7 Др ДТ
~ + Т = -т-	<"
Из равенств (3) и (4) получаем
Др = ДТ р + ро
Р Т ро и ДУ	ДТ р
= <<L У	Т ро
Используя уравнение состояния для одного моля идеального i в за, которое имеет вид
pV = RT\	(Ь)
нетрудно преобразовать уравнение процесса У(р4-ро) = const к ел» дующей форме:
RT 4-р0У = const.
Из полученного равенства следует, что изменение температуры rnni ДТ во время процесса связано с изменением объема газа ДУ cool ношением
ЯДТ + роДУ = 0, откуда следует
ДУ = -ЯДТ/ро.
256
। учетом этого равенства формула (2) принимает вид
А = -р1ЯДТУро = -ЯДТ.	(6)
Мы видим, что А < 0. Это означает, что во время процесса V(p + />п) const работу над газом совершают внешние силы, сжимая его IЛГ < 0).
Подставляя в (1) вместо А выражение (6), получаем
AQ = |яАТ«4,16 Дж.
Задача 220 (81-р)
Над одним молем идеального газа совершают процесс, во время ьигорого его давление и объем связаны соотношением
(V 4- Vo)(p + Po) = С,
। lie lz0 = 25 л, ро = 1 атм, С = 81 атм-л. Определите максимальную н’мпгратуру газа во время этого процесса.
Решение. Введем для удобства следующие обозначения:
_ Р _ V _ RT _ С
7Г=—,	Г = ——,	(7 = ——.	(1)
Ро Vo Ро Vo роРЬ
11г,||ичины тг, w и т можно назвать безразмерными давлением, объемом и температурой; ст—безразмерная постоянная. В этих новых iи ременных уравнение Клапейрона—Менделеева для одного моля нпгального газа pV = RT и уравнение процесса (p + po)(V + Vb) = С принимают вид
7ГШ = т,	(2)
(тг + l)(w 4-1) = а.	хз)
Кривая, описываемая на плоскости тг, от уравнением (3), явля-। и л гиперболой, асимптотами которой служат прямые тг = — 1 и —1. Ветви гиперболы симметричны относительно биссектрисы первого квадранта. Реальному процессу отвечают положительные ипвление и объем, т. е. положительные безразмерные давление тг и • И п»см w. Поэтому реальный процесс изображается лишь той частью пшерболы, которая находится в первом квадранте.
Графики изотерм Т = const (или, что то же самое, г = const) 1пкже гиперболы, но их асимптотами служат положительные полу-»»ги тг и ш. Изотермы симметричны относительно биссектрисы пер-ппго квадранта. Основная гипербола, характеризующая интересующий нас процесс, если пересекает данную изотерму, то пересека-• I дважды. Этим точкам процесса отвечает одинаковая температура Т. Точки пересечения изотерм с графиком нашего процесса
257
симметричны относительно биссектрисы первого квадранта. Точки пересечения графика процесса с изотермами большей температуры приближаются друг к другу, оставаясь симметричными относитслi. но упомянутой выше биссектрисы. Существует предельное значенш температуры Т = Ттах, отвечающее тому случаю, когда точки ио ресечения, попадая на биссектрису первого квадранта, сливают! и друг с другом. Отсюда ясно, что искомой максимальной темпера ту ре процесса, описываемого формулой (3), отвечает точка основной гиперболы, расположенная на ее оси симметрии. Таким образом в тот момент, когда температура процесса Т = Ттах (т. е. когдн т = ттах), имеет место равенство тг = w. Полагая в формуле (J) тг —	= х, получаем уравнение для нахождения значения (а тан
же и тг), отвечающего значению Ттах. Решая полученное уравнепш относительно х, имеем
х =	— 1.
При тг = w = х в формуле (2) т принимает значение ттах. Отскин» ясно, что
T"max ~ X —	~ 1) •
Воспользовавшись формулами (1), находим
I)2 = i (VC- ^л)2 = 195К
Задача 221 (82-р)
Цилиндрический сосуд разделен свободно скользящим портном на две части. Стенки сосуда и поршень тепла не проводят. В < hi ной части находятся щ молей гелия, в другой — vz молей аргони Поршень быстро вынимают. Найдите установившиеся температур! и давление. Объем сосуда V, давление на поршень в состоянии рнк новесия р. Газ считать идеальным. При вынимании поршня раСхин не производится.
Решение. Вначале, поскольку поршень в сосуде занимает |жп новесное положение, оба газа находятся при одинаковом давлении р, занимая при этом, вообще говоря, разные объемы Ц и У2 и им<»м разные температуры и Т2. Эти величины связаны уравнениями состояния
pVi = Rv\T\ и pVz =	(11
где R — газовая постоянная. После удаления поршня газы персме шиваются, их температуры выравниваются и смесь приходит в рпп новесное состояние. После наступления равновесия параметры гм< си газов (давление р', объем V и температура Т) связаны уравнен и ем состояния
p'V = (pi +	(У|
258
I пи как сосуд тепла не проводит и при вынимании поршня работа не н|н’изводится, то суммарная внутренняя энергия газов не меняется. При пи мая во внимание, что внутренняя энергия одного моля любого •ннпатомного идеального газа равна (3/2)/?Т, мы должны написать, "П	3	3	3
- v\RT\ + -V2RT2 = 5(^1 +
Пиму равенству эквивалентно следующее:
+ ^2^2 = (^1 4" ^2)^	(3)
lb пользуя полученные соотношения, найдем давление р1 и темпера-цру Т смеси газов. Складывая почленно равенства (1) и учитывая 1.1), получаем
pV = RfviTi 4- V2T2) =	+ ^2)Т.	(4)
। I hi впивая равенства (2) и (4), находим р1 = р, т. е. смешение газов не приводит к изменению давления в сосуде. Из равенства (4) находим Н'мпературу смеси:
т_ рУ
(^i + ^2) R
Задача 222 (82-р)
Горизонтально расположенный цилиндрический сосуд разделен • ппбодно скользящим поршнем на две части. Объем правой части 11, объем левой— У2. Стенки сосуда и поршень тепла не проводят. В припой части сосуда находятся Pi молей гелия, в левой —1/2 молей нрюна. Поршень быстро вынимают. Температура образовавшейся • меги Т. Найдите температуры Т\ и Т2, которые имели гелий и аргон in смешивания. Газы считать идеальными. При вынимании поршня рцГю га не производится.
Р е ш е н и е. В задаче рассматривается тот же процесс, что и в шцпче 221. В связи с этим имеют место все уравнения, полученные при ее решении. Из уравнений (1) задачи 221 имеем
№>2
II । уравнения (4) той же задачи следует, что
Р = И +1/2)	.	(2)
VI + V2
259
Заменяя в равенствах (1) р на правую часть (2), находим
_ (1/1 + vz)ViT	_ (1/1 4- ^2)^2^
1	” Р1(Ц + 72) ’	2“ p2(Vi 4- V2) ’
Задача 223 (83-р)
Один моль идеального газа находится в сосуде между двум и поршнями, соединенными пружиной. Поршни могут перемещать и без трения вдоль оси сосуда, за поршнями вакуум. Длина пру ж и ны в нерастянутом состоянии /, ее жесткость А, площадь поршни 5, температура газа Т. Определите удлинение пружины и давлешо газа в положении равновесия.
Решение. Обозначим искомое удлинение пружины Д/ и дн вление газа в состоянии равновесия р. В состоянии равновесия |н> зультирующая сила, действующая на каждый поршень, равна нули»
pS — АД/ = О,
откУда	р=АД//5.	(I)
Воспользуемся уравнением состояния для одного моля идеалынн» газа
p = RT/V}	(71
где V — объем газа, R— газовая постоянная. Приравнивая праньн части (1) и (2) с учетом того, что в состоянии равновесия ra:i ш нимает объем V = S(l 4- Д/), получаем квадратное уравнение дмн нахождения Д/ :
k£l(l 4- Д/) = RT.	(.1)
Учитывая, что Д/ > 0, из уравнения (3) найдем удлинение пружн ны Д/:
А. 1 ( . Г9 4RT\ д' = Ц-' + \/'2 + —
Подставив этот результат в равенство (1), находим
”=2s(1-'+r+—J
Задача 224 (82-г)
Две стеклянные пластинки расположены вертикально и парил лельно, расстояние между ними очень мало. Будут ли они притяш ваться или отталкиваться, если их частично погрузить в ртуть?
260
Решение. Ртуть — несмачивающая стекло жидкость. Поэтому п результате действия сил поверхностного натяжения ее поверх-||иг|ь между близко расположенными стеклянными пластинками iiiyiiiena по сравнению с общей поверхностью. В результате силы •ьмик'ния в жидкости, действующие на внешние поверхности пла-• 1Н1ЮК, не полностью скомпенсированы силами давления, действующими на их внутренние стороны. Отсюда ясно, что опущенные в I»I V'ri> стеклянные пластинки будут притягиваться друг к другу.
Задача 225 (84-г)
Почему чайник шумит сильнее перед тем как закипеть?
Решение. Перед закипанием в наиболее горячих областях чайники, например около стенок, начинают образовываться пузырьки Нира. Однако в результате соприкосновения с более холодными мас-• ими воды температура стенок пузырьков может оказаться недоста-11fiной для создания в пузырьках атмосферного давления. Поэтому инц>ко что образовавшиеся пузырьки схлопываются, что и вызыва-• । « ильный шум.
Задача 226 (84-г)
Правое колено U-образной трубки закрыто краном, а левое— • икрыто. В трубку налита вода. Поверх воды в правом колене имейся капля ртути высотой h = 0,5 см. Высота столба воды в правом иплене hi = 4 см, в левом колене Лг = Ю см. Поднимется или опу-< инея капля ртути, если кран открыть? Плотность ртути в 13,6 раз Польше плотности воды.
Решение. При открывании крана давление воздуха в лепим и правом коленах трубки выравнивается. Предположим, что при этом вода в правом колене опускается на АЛ. Найдем величину ДА (если, в действительности, вода в правом колене не опустилась, и поднялась, то для величины АЛ получится отрицательное значение)- С этой целью выпишем условие равновесия жидкостей после открытия крана:
(Л2 4- ДЛ)рв = (hi - Ah)pB + Лррт.	(1)
1д<’сь рв — плотность воды, ррт —плотность ртути. Из равенства (1) иии АЛ получаем выражение
АЛ = 77 ( -^—h + hi — Л2 | = 0,4см > 0,
2	\ Рв	)
। г, капля ртути опускается.
261
Задача 227 (85-р)
Экспериментальная установка имеет рабочий объем ,7-1 i . находится в термостате с температурой to = О °C. Перед ипчич эксперимента давление в рабочем объеме ро = 0,3 атм. В ходи •» перимента герметичность нарушилась, а затем ее восстановили 11| • этом масса воздуха в рабочем объеме увеличилась в два раза, в и ♦> пература —на ДТ = 110 К.
1) Объясните, почему повышается температура воздуха и pin чем объеме установки.
2) Найдите давление воздуха в установке сразу после вос< tun вления герметичности (pi) и через большой промежуток вргмпн после него (рг)-
Решение. Повышение температуры газа объясняется тем, и атмосферное давление совершает работу над разреженным пгшм
Для вычисления давления pi сразу после восстановления ij метичности надо использовать уравнение состояния, учитывая, ч« масса воздуха возросла в два раза, объем не изменился, а теми» । • тура возросла на ДТ. В результате получаем
р, = 2р0 (1 + ДГ/Т) « 0,81 атм « 8,1 • 104 П.
Давление рг в установке спустя большое время можно опркн» лить из уравнения состояния идеального газа, учитывая, что мш • . воздуха увеличилась в два раза, объем не изменился, а темпг|ти ра вернулась к прежнему значению to- Поэтому р2 = 2ро = 0,(1 я • 6 - 104П.
Задача 228 (86-р)
Металлический баллон со сжатым аргоном = 0,04 кг/мии । объемом V = 44,8 л находится на улице. Температура окружшппя го воздуха t = 0 °C. Начальное давление в баллоне ро = 15 атм И некоторый момент кран баллона ненадолго открывают и снови и» крывают. При этом из баллона успевает вытечь Am = 400 г га in, и внутренняя энергия газа, оставшегося в баллоне, оказывается в пян раза меньше начальной энергии газа.
1) Какова температура газа в баллоне сразу после закрыпнпнм крана?
2) Найдите давление газа в баллоне сразу после закрывания крн на (pi) и через большой промежуток времени после этого (рз).
Решение. Уравнения состояния газа в баллоне до открыли крана и сразу после его закрытия запишем в следующем виде:
Pov = — RT0,	рг V = —ЯТ1.	(11
262
ipti. mi—новая масса газа, То — абсолютная температура, соот-fi-it 1 нукидая О °C. Внутренние энергии газа в баллоне до открытия • । tun и и сразу после его закрытия соответственно равны:
В. = ^ЯГ, = Т- (2>
Ничнльную массу газа то находим из (1): то = 1,2 кг.
II । (2) получаем:
То _ 2пи _ 2 Л _ Ат\ _ 4
Ti то \ то у 3
I iihiiM образом, Ti = |То—температура газа, пока кран открыт, , мрныцается.
II । (1), (2) находим теперь давление pi*.
Р1 = Ро/2 = 7,5 атм « 7,5 • 105 П.
Через достаточно долгое время температура газа в сосуде вновь Itiner равной Tq, поэтому установится давление
mi RT0	^щбгт
р =-----— = 10 атм « 10 П.
Р У
* 1ндача 229 (87-р)
II экспериментальной камере, наполненной идеальным газом, к пшовлены манометр и термометр. В исходном состоянии ро = 1 К)5 Па, То = 400 К. С некоторого момента давление и температу-|ч1 начинают возрастать и устанавливаются равными pi = 3,6 * 105 ||Н| 71 = 600 К. Какой эксперимент проводится в камере? Камеру • in гать герметичной, ее объем — постоянным.
Решение. Уравнение состояния идеального газа pV = vRT. dun увеличения температуры необходимо подводить энергию извне. При этом давление должно возрастать пропорционально темпера-। у ре, если не меняются объем и число молей. Поскольку по ус лоцию задачи объем не меняется, a pi > poTi/To = 3-10° Па, следует предположить, что во время процесса увеличилось число молей в pi7о/(роТ1) = 1,2 раза. Это могло произойти либо в результате увеличения массы газа (путем “возгонки” какого-либо твердого веще-• нт), либо в результате уменьшения его средней молярной массы (<п счет реакции диссоциации). Возможно, что действовали обе эти причины.
263
Задача 230 (86-г)
В кастрюле находится вода при температуре 60 °C. Кастрюль» закрывают крышкой, масса которой т = 5 кг, площадь S = 100 см' Кастрюлю медленно нагревают до 70°C. Сколько раз подпрыпнч крышка за это время, если давление насыщенных паров при 60 И1 равно pi — 2 • 104 Па, при 70°C Р2 — 3,1 • 104 Па, атмосфера» давление ро = 105 Па.
Решение. Когда кастрюлю закрыли крышкой, давление инн крышкой, равное в начальный момент атмосферному давлению складывается из давления насыщенных паров воды pi и давлении воздуха под крышкой рв = р0 - Pi = 8 • 104 Па. При нагревании дн вление под крышкой повышается до р0 = р0 + mg/S = 1,05 • 105 Ни При дальнейшем нагревании крышка подпрыгивает, давление пни ней падает до атмосферногоро, а часть воздуха и пара уходит. Ос in точное давление воздуха р^ = рв • ро/ро = 7,6 • 104 Па, остаточшн давление пара р^ = ро — Рв^ = 2,4 • 104 Па. В следующий раз крыш ка подпрыгнет, когда давление под ней снова превысит р0. При э тим давление воздуха под ней останется примерно равным р^, так кпи при повышении температуры на 10 °C давление воздуха менян геи всего на 3%. Давление пара перед вторым подпрыгиванием буд<ч равно р^2) = Pq - р^1) = 2,9 • 104 Па. Как и в первый раз, находим остаточное давление воздуха р^ = Рв^Ро/Ро = 7,2 • 104 Па. В тре тий раз крышка подпрыгнет, когда давление пара станет равным Рп3> = Pq — Рв2) = 3,3 • 104 Па, что больше р2, т. е. это может произоП ти только при нагреве до температуры, превышающей 70°C. Итак, при нагревании от 60 до 70 °C крышка подпрыгнет два раза.
Задача 231 (86-г)
В большую бочку с водой бросают раскаленные металлический шарики одинаковой температуры. Известно, что шарик радиусом П = 0,5 см нагревает воду на Д/i = 0,1 °C, радиусом г2 = 1 см на Д/2 = 1,2 °C. Оцените изменение температуры воды для шарика радиусом гз = 1,5 см.
Решение. В бочке будут происходить два процесса: нагревание воды и ее быстрое испарение при соприкосновении с поверхностью шарика, пока шарик достаточно раскален. Количество теплоты, за траченное на нагревание, пропорционально изменению температуры воды, Q1 ~ Ы- Это же количество теплоты можно представить как Qi = Q — Q2, где Q — количество теплоты, запасенное в шарике, оно пропорционально кубу его радиуса, Q2— количество теплоты,
264
н I рнчепное на испарение воды, оно пропорционально квадрату ра-|||v< а шарика. Для изменения температуры воды можно написать . рнинения:
Д/i =	+ Br\, Д/2 = Лг| + Brlj
• н' /1 и В — неизвестные коэффициенты пропорциональности. Решим относительно этих коэффициентов систему уравнений, получа-• м /I = 1,6град/см3, В = —0,4 град/см2. Теперь для третьего шара, •пни Л и В, найдем Д/з:
Д/3 =	+	= 4,5’С.
'йщача 232 (87-г)
11 два одинаковых закрытых литровых сосуда помещают по кусну льда, каждый массой т = 490 г. Сосуды соединяют в верх-||| II части трубкой. В первом сосуде поддерживают температуру /1	290 К, в другом — Т2 = 310 К. Оцените разницу масс ДМ со-
• улов с их содержимым спустя большой промежуток времени после ич соединения. Что при этом будет находиться в сосудах?
Решение. Оба куска льда, во-первых, растают, и, во-вторых, in иле таяния льда в сосудах будет вода и пар над ней. Поскольку ллвлечше насыщенного пара при большей температуре больше, пар вудет переходить из сосуда с большей температурой в сосуд с меньшей температурой и там конденсироваться. В итоге спустя большой промежуток времени (достаточный для установления равновесия) в • «н уде с большей температурой воды не останется, там будет только водяной пар (ненасыщенный) с температурой и давлением, равным давлению насыщенного пара при температуре Ti. В другом • ••«•уде будет вода и пар (насыщенный) с температурой Очевидно, что массой паров воды можно пренебречь по сравнению с массой поды в жидком состоянии (они занимают примерно тот же объем, а плотность примерно в 1000 раз меньше). Поэтому ДМ « 2m = 980 г.
Задача 233 (87-г)
В вертикальном теплоизолированном цилиндрическом сосуде с площадью основания S под поршнем массой М находится 1 моль одноатомного идеального газа. В газе расположена проволочная спираль сопротивлением г, подсоединенная к источнику тока с ЭДС Определите скорость v подъема поршня. Атмосферное давление над поршнем равно ро. Внутренним сопротивлением источника пренебречь.
Р е ш е н и е. За малое время Д/ к газу подводится тепло
Д(?= —Д/.
г
265
Газ расширяется при постоянном давлении р = Ро + Mg/S,
В соответствии с первым началом термодинамики
Q
Д£= -ЯДТ+рДИ,
где ДИ— изменение объема газа. Из уравнения состояния газа ни лучаем
рД V = RAT.
Теперь, учитывая, что ДУ = 5Дж, где Дж — смещение поршня и время Д/, находим
V = Ьх/Ы = 2£2[5r(M<7 +p0S)].
Задача 234 (88-г)
В сосуде высотой h находится очень вязкая жидкость с плотин стью р. От дна сосуда отрывается и медленно всплывает маленьким пузырек воздуха с начальным объемом Ц. Какое количество тепли ты получит жидкость за время подъема пузырька? Атмосферное ди вление равно ро - Теплообменом со стенками сосуда и окружающим воздухом пренебречь,
Решение. Так как пузырек маленький, т. е. объем жидком н очень велик, то при всплытии пузырька температура жидкости изменяется. Система равновесна, поэтому и температура пузырим будет постоянна. Кинетической энергией пузырька и окружающий слоев жидкости можно пренебречь, так как, по условию задачи, ну зырек всплывает медленно. При подъеме пузырька потенциалы ши энергия жидкости уменьшается на
AU = pghV1.	(II
Эта энергия переходит в тепло.
В тепло переходит также работа пузырька по расширению. Hull дем ее.
При подъеме пузырька давление воздуха в нем изменяется пн Др = —pgh (знак минус говорит о том, что давление воздуха в ну зырьке при подъеме уменьшается), а его объем увеличивается пн ДИ
Так как процесс изотермический, по закону Бойля—Марио и п имеем
pV = (ро + pgh) Vi = po(Vi + AV).
266
• 11гюда находим изменение объема пузырька
AV =
Ро
При малом расширении пузырька на 6V при давлении р совершатся элементарная работа
6A = p6V= y(p0 + pgh)SV.
। уммируя элементарные работы на всем изменении объема от Ц до In. получаем
ДА = Vipgh |	f1+ I wV/v, v
= Vipgh (	k pgh J Vi
= Vipgh |	Y и k pgh J \ PQ J
Добавляя к работе (2) изменение потенциальной энергии жидко-• in (1), находим полное выделившееся тепло
<2 = Vipgh [1+ (1+-Ц-) In (1+^)
\	Р9<4	\ Ро /
(3)
В нормальных условиях высота сосуда мала (pgh ро)- В этом • цучае можно в формуле (3) рассмотреть предел при h —> 0:
Q —2Vipgh.
Видим, что в пределе оба рассмотренных выше эффекта (уменьшение потенциальной энергии жидкости и расширение пузырька) при-||| |дят к одинаковым результатам и пренебрегать одним из эффектов по сравнению с другим в данном случае нельзя.
Противоположный предел при h -> оо дает следующий резуль-hit:
Q -> V1Pgh (1 + In .
\ Ро /
В в этом случае нельзя пренебречь единицей по сравнению с логарифмом в круглых скобках, так как логарифм — медленно растущая функция. Для подъема пузырька воздуха в воде со дна самой глубокой впадины Мирового океана (10 км) второе (логарифмическое) слагаемое в круглых скобках равно всего 3.
267
10. ЭЛЕКТРОСТАТИКА
Основной закон электростатики — закон Кулона — описыши» i взаимодействие покоящихся точечных электрических зарядов:
F =	|h
47Г£о Т* Г
При решении задач следует помнить, что формула (1) описыши i взаимодействие точечных зарядов. Точечными зарядами hivim вают сферически симметрично заряженные тела, размеры которы» много меньше расстояния между их центрами.
В этом определении очень важна сферическая симметрия зпри жепных тел. Действительно, при строгой сферической симметрии закон (1) оказывается справедлив для тел конечных размеров (српн нимых с расстоянием между центрами зарядов). При отсутствии сферической симметрии заряженные тела взаимодействуют бол< > сложным образом. Это взаимодействие нельзя уже описать толь к» • на языке сил (см. Добавление III).
Коэффициент в формуле (1) связан с выбором системы одн ниц (СИ). Действительно, наличие в системе СИ основной размер ной величины силы тока ампер (А) и производной величины элнж трического заряда кулон (Кл) приводит к необходимости связывпн< механические величины (ньютон-метр в квадрате) и электрически» (кулон в квадрате). Никакого особого физического смысла у этом» коэффициента нет.
От силы (F) в формуле (1) переходят обычно к напряженное i и поля (Е)—силе, действующей на единичный заряд со стороны in ряда q:
„	1	<7 г
Е = -л—4-.
4тгб?о г г
Электростатические поля точечных зарядов по определению (У) являются центральными. В силу принципа суперпозиции централi. ны и любые электростатические поля. Это позволяет ввести понятиг потенциала.
Потенциал поля точечного заряда (или поля сферически сим метрично распределенного заряда) q равен
Этот потенциал определен с точностью до значения на бесконсч ности, которое положено равным нулю в выражении (3). Принцип
PI
268
I п<‘рпозиции позволяет вычислить потенциалы любых распределении зарядов. Следует особо отметить, что только наличие электри-ниснх зарядов приводит к появлению напряженности электриче-hiiio поля и, как следствие, возникновению разности потенциалов мюкду различными точками пространства.
()уличительная черта закона Кулона (и во многом аналогичного • му закона всемирного тяготения) — в зависимости напряженности и* hi л от расстояния по закону обратных квадратов. Такая зависи-И1Н гь приводит к тому, что количество силовых линий, пересекающих любую замкнутую поверхность (выходящие линии считают со •инком плюс, входящие — со знаком минус), пропорционально зарину внутри этой поверхности. Коэффициент пропорциональности hi висит от определения силовой линии и выбора системы единиц. 1И»ы’1но плотность силовых линий делают равной напряженности iih.idi в данной точке. Тогда в системе СИ число силовых линий, ныходящих из заряда q, численно равно д/е^.
* * *
Задача 235 (89-р)
Три точечных заряда одного знака могут свободно скользить по окружности. Два заряда находятся на одинаковом расстоянии от |р<*тьего. При каких значениях зарядов возможно равновесие?
Рис. 86. К задаче 235.
Решение. Пусть заряды дв и qc находятся соответственно в точках А, В и С окружности с центром в точке О (рис. 86). При этом АВ = ВС. Для равновесия системы необходимо, чтобы равнодействующие сил, приложенных к каждому заряду, были направлены вдоль прямых, соединяющих данный заряд с центром
269
окружности. Так как геометрия расположения зарядов симметрии на относительно прямой, проходящей через точки В и О, то ясин что равновесие системы будет достигаться лишь при зарядовой сим метрии относительно этой же прямой, т. е. для равновесия системы необходимо выполнение равенства
Ча = Цс-	(Il
Нетрудно сообразить, что равенство (1) является достаточным угли вием равновесия заряда q. Это следует из того, что при выполнении (1) векторы, изображающие силы Fi и F2, приложенные к зари,in $в, расположены симметрично относительно прямой ВО. Их рати» действующая Ri направлена вдоль этой прямой. Найдем услоши при соблюдении которого находится в равновесии заряд дд (а следи вательно, в силу симметрии, и заряд qc\ Если заряд дд находи I» и в равновесии, то сумма R2 сил F3 и F4, действующих на него и-стороны двух других зарядов, направлена вдоль прямой О А, а при екции F3 sin а и F4 sin /3 этих сил на направление, перпендикулярно» к прямой ОА, равны по величине, т. е.
Fasina = Fjsin/J	(71
(о и (3— углы, образуемые векторами F3 и F4 с направлением О А | Величины сил F3 и F4 определяются равенствами
F. = 1 длдв F<= 1 дАЯС	CI)
3 4tteq (АВ)2 ’	4 4тге0 (АС)2 ’	'
Расстояния АВ и АС не независимы и связаны соотношением
АС/2 = ABsina.	(»1|
Также не независимы и углы а и /3. Нетрудно сообразить, что
/? = тг/2 — 2а.	(h)
С помощью системы равенств (3)—(5) условие равновесия (2) легко преобразуется к виду
Яв _ sinQr/2 - 2а)
Яа 4 sin3 а
Выясним, существует ли такое значение угла а, при котором рн венство (6) имеет место. Так как по условию задачи заряды дд н дв одного знака, то левая часть (6) всегда положительна. Право
270
♦•и 11. положительна при a < тг/4. При изменении угла а от 0 до тг/4 инчипа правой части (6) монотонно изменяется от +оо до 0. Отсю-I» игпо, что какова бы ни была величина отношения Цв/ЦАу среди множества значений угла а, принадлежащих промежутку [0, тг/4], .... /ui найдется такое оо> при котором равенство (6) имеет место. Пит, равновесие возможно при любых 9д и цвУ если qc = 9д.
Ьщача 236 (89-р)
Может ли система из трех точечных зарядов находиться в рав-ЦНПГСИИ?
Решение. Чтобы система из трех свободных зарядов наудилась в равновесии, необходимо, чтобы силы, действующие на никлый заряд со стороны двух других, были равны по величине и нригивоположны по направлению. Отсюда ясно, что в равновесном • н юянии (если оно вообще возможно) заряды должны находиться н«1 одной прямой. Пусть величины зарядов 91, 92, 9з- Очевидно, что I«г/|,пий заряд 92 может покоиться только в том случае, если заряды h и (/з одного знака. Только при выполнении этого условия силы, •u'lli гвующие на средний заряд, будут направлены противоположим и могут уравновесить друг друга. Когда крайние заряды имеют •иппаковые знаки, между ними действуют силы отталкивания; дей-• । пне этих сил может быть уравновешено только силой притяжения • и г тороны среднего заряда. Поэтому для равновесия крайних заря-ф«и необходимо, чтобы знак среднего заряда 92 был противоположен шику крайних зарядов 91 и 93.
Вудем считать, что условие, накладываемое требованием рав-нипссия на знаки зарядов, выполнено. Тогда на каждый из зарядов и Пствуют две противоположно направленные силы. Требование ра-lu iK тва этих сил приводит к соотношениям
3102Л1 =	+ ^)2,	(1)
= ?2^/^2)	(2)
Q3Q2/I2 = <739i/Gi + ^)2,	(3)
ннорым при равновесии должны удовлетворять величины зарядов '/I. 9з и расстояния Zi, /2 между ними. Нетрудно заметить, что •побое из уравнений (1)—(3) является следствием остальных двух. Погрому реальное ограничение на пять величин 91, 92, 93, Л, h на-к ||цдывают лишь два уравнения системы (1)—(3). Это означает, что при любых наперед заданных значениях трех из пяти величин рав-инпосие системы всегда может быть обеспечено путем специального подбора остальных двух. Так, если произвольно заданы расстояния
271
/1, I2 и заряд gi, то, как следует из уравнений (2), (3), равнинен будет обеспечено, если $2 и дз имеют следующие значения:
п - qi п - la-
92 — /11 \э ’	93 — q j
(1 + ??)2 Г
где т/ = /х//2.
Итак, система из трех свободных точечных зарядов можи пм ходиться в равновесии. Легко показать, что это равновесие бул* • неустойчивым.
Задача 237 (89-г)
Три маленьких тела имеют одноименные заряды gi, g2 и Цл »* связаны нитями. Длина нити, связывающей первое и второе ’пчи равна /3, второе и третье тела — /1, третье и первое — /2- Определи)' силу натяжения каждой нити, если система находится в равнопи ин (сила тяжести отсутствует).
Решение. Сумма электростатических сил, действующих и > заряд д? со стороны двух других зарядов (Fi и F2), равна и ишци» влена противоположно сумме сил натяжения двух нитей (Ti и Т J связанных с данной точкой:
Fi + F2 = -(T1+T2).	(Il
Направления нитей (и сил их натяжения) противоположны па при влениям электростатических сил:
Т1 = —aiFi,	Т2 = — «2^2,	Pl
где ai, 02 — некоторые положительные числа. Подставляя вырпж» ния (2) в (1), получаем
(1 — ai)Fi 4- (1 — ^2)^2 — 0.	(.11
Поскольку силы Fi и F2 по условию не параллельны, выполнени! равенства (3) возможно только при обращении в ноль коэффициец тов перед силами: Oi = 1, 02 = 1. Следовательно, силы натяжении нитей равны по величине силе взаимодействия тех зарядов, которы» они соединяют:
гр _ 71^2 гр _ 919з гр _ 929з
12	47Г£о^з ’ 13	4^60/2’ 23	47Г£о/2^
272
Задача 238 (84-р)
Гри одинаковых металлических шарика расположены так, что ••• nipi.i их находятся на одной прямой. Расстояния от центра сред-•м|н шарика до центров крайних шариков одинаковы. Заряд цен-рнньного шарика равен нулю, заряды крайних шариков 4-Q и — Q. НиИдите напряженность электрического поля в точках, равноуда-»| иных от центров крайних шариков и находящихся у самой по-•н рч пости центрального шарика.
Решение. Сначала возьмем два точечных заряда равной iii'iinibi, но противоположного знака. Рассмотрим плоскость, про-'иншую через середину отрезка, соединяющего эти заряды, nep-in иникулярно этому отрезку. Легко установить, производя вектор-|||и' сложение напряженностей полей точечных зарядов, что вектор ц ц|ряженности суммарного поля в любой точке такой плоскости нцнривлен перпендикулярно ей.
Распределение зарядов на шариках обладает, как нетрудно по-i iiiub, следующим свойством симметрии относительно плоскости, приходящей через центр среднего шарика и перпендикулярной от-I » шу, соединяющему центры шариков. Точки, в которых требуется нргделить направление напряженности поля, лежат в этой плос-| |и ги. Именно, если в каком-то объеме AV, лежащем по одну столицу плоскости, находится заряд Д<7, то в равном объеме по дру-’II'» сторону от нее, получающемся зеркальным отражением А1/ в ниЙ плоскости, находится заряд — Д</. Используя это наблюдение и принцип суперпозиции, приходим к заключению, что вектор напря-ф пн пости электрического поля в точках рассматриваемой плоскости •пикет быть направлен только перпендикулярно плоскости. Вместе н'М, вектор напряженности электрического поля вблизи поверх-тити проводника может быть направлен только перпендикуляр-||| । поверхности проводника, иначе свободные заряды в проводнике при шли бы в движение. Поэтому напряженность поля в точках не-|ц и родственно над поверхностью центрального шара, равноудаленных от центров крайних шариков (эти точки лежат в рассмотренной и |ц>скости), равна нулю.
Задача 239 (80-г)
Два изолированных металлических тела, имеющих заряды q и соединяют тонким проводом, после чего их заряды становятся • ।'ответственно 2q и q. Какие заряды приобретут тела после соединения проводом, если вначале им сообщить заряды bq и — 2q? Взаимное рш положение тел неизменно.
273
Решение. Емкостью тонкой проволочки в сравнении с емксм i н ми Ci и (?2 соединяемых ею тел можно пренебречь. Это позво.ин» считать, что заряды размещаются только на телах. В силу зпкин сохранения зарядов имеем
я\ + я\ =	+ 921',
„II 1 „II _ „ If I „ If
<71 + <7г — <71	+ <72 )
где q\t — начальные заряды тел в первом случае, gj1, q™ — во ih« ром; величины со штрихами обозначают конечные заряды тех тел.
После соединения тел проводником их потенциалы (771 в псрв»• случае и Uu во втором) становятся одинаковыми. Полная емко» •• системы после соединения
г _ Qi + Яз _ 711 + 9г1	। *
° - ~7Л~ " "ТТЛ--
По условию задачи полный заряд системы в первом и во вт< »| *•«»« случаях одинаков (g{ + q^ = g*1 + g^1). При этом из формулы (li следует равенство конечных потенциалов: 771 = С7П. Распределенн-зарядов по поверхности проводников после соединения не завш ih от их начального распределения, следовательно gi11 = gi1 ‘J у 92П' = 921' = q-
Задача 240 (80-г)
Два изолированных металлических тела, имеющих заряды </ »< 2g, соединяют тонким проводом, после чего их заряды становии равными 2g и д. Какие заряды приобретут тела после соединении проводом, если вначале им сообщить заряды 4g и 2g? Взаимное ।не положение тел неизменно.
Решение. Все отличие от задачи 239 заключается в лр' гих значениях начальных зарядов д}1 и д^1. Действительно, полный заряд во втором случае (д}1 + д^1 = 6д) в два раза больше, чем в ш р вом случае (д* + q\ — Зд). Следовательно, в силу линейности в» • • уравнений (принципа суперпозиции), в два раза большими стану i н все электрические параметры после соединения тел: потенциал, пи пряженность поля, поверхностные плотности зарядов и заряды и * каждом из тел. Окончательный ответ в два раза превышает отви ь задаче 239: gi11' = gi1' = 4g, g2II? = q?1' = 2q.
Задача 241 (79-r)
Точечный заряд g находится на расстоянии I от центра по и» ряженного металлического шара радиусом г (7 > г). Определи и
274
•|и|0цциал на поверхности шара, считая, что на бесконечности по-нпципл равен нулю.
Решение. При равновесии зарядов электростатическое поле в • •нищ» металла отсутствует и поэтому все точки шара находятся при ••шинковом потенциале. Таким образом, потенциал на поверхности и при не отличается от потенциала в его центре, и задача сводится * ли хождению потенциала в центре шара.
Потенциал у? в центре шара, согласно принципу суперпозиции, и индывается из потенциала
4тгг0/
"питаемоготочечным зарядом д, и потенциалов Дд?,, создаваемых щридами Дд>, расположенными на настолько малых участках поточности шара, что их линейными размерами в сравнении с ради-н им шара г можно пренебречь. Каждый такой заряд Дд^ создает в ш и । ре шара потенциал
д^» =
&qj
4тгб()Г ’
Hi и тому для искомого потенциала (р можно написать:
р = уч + 52 Д(^ -	+ т~— 52
4тгбо< 4тгбог
1ик как шар не заряжен и Дд» — индуцированные заряды, то • Ад, = 0. Таким образом, потенциал в центре шара, а следова-н in.по, и на его поверхности определяется равенством
4тео/
Задача 242 (79-г)
Две частицы с одинаковой массой имеют заряды gi = е и /I -2е. Частицы находятся в однородном постоянном внешнем нюктрическом поле с напряженностью Е. Определите взаимное расшит жение этих частиц, если их ускорения одинаковы.
Решение. Взаимное положение частиц определяется неиз-uiH iiihiM вектором R, соединяющим центры зарядов gi и g2.
Чтобы частицы с одинаковой массой получали одинаковые по |ш»1ичине и направлению ускорения, необходимо, чтобы на них дей-• ни шали одинаковые по величине и направлению результирующие IHII.I.
275
Рассмотрим силы взаимодействия частиц друг с другом и с шнчи ним полем. На частицы с зарядами qi и q2 со стороны внешнего ши. действуют силы Fi и Fo, величины и направления которых опрги» ляются по формулам
Fi = ^1Е, F2 = Q2&.
Для сил взаимодействия F'x и F2 зарядов получаем из закона hi лона:
тр/ _ тр/ _ Я1Я2 R
2	1 я2 я*
Результирующие сил, действующих на каждое из тел, есть
F1 + F, =	(И
F2 + F2 = 92Е+^^ —•	I’’1
Ускорения зарядов одинаковы по величине, если равны действующие на них силы. В нашем случае правые части уравнений (1) и (71 должны быть равны друг другу:
?1Е_9Ц^ = д2Е+^-)
V R2 R 42 R2 R
или
= (?1 - ?2)Е.	( I
Из равенства (3) находим длину и направление искомого iu ь тора R:	_________
о_ / |<?1<?2|	.
V |91 -?2|^’
R _ Е gi-g2 gi<?2	)h|
R Е I?! - g2| I9192I’
В нашем случае по условию задачи qi = е и q^ — — 2е. Подстаплин эти значения в формулы (4) и (5), получаем окончательный отшч
R=JIb, *	в
V з	R	Е
Задача 243 (83-р)
Металлический стержень длиной I расположен посередине м> жду двумя равными точечными зарядами Q противоположного чин ка вдоль линии, соединяющей заряды. Расстояние между заряди мн L. Мысленно разделим стержень пополам. Оцените заряд кажди|| половины стержня.
276
Решение. Пусть для определенности положительный заряд •и «Iмдится слева от стержня, а отрицательный—справа. Вследствие и к । ростатической индукции левая половина стержня приобретает <1 рннательный заряд (—д), а правая — положительный (д).
Можно считать, что заряд не распределен по поверхности стержни, и сосредоточен на его концах. Очевидно, что создаваемое заря-ммн Ьд поле в центре стержня Е = 2&д/(//2)2 должно компенси-|иф||п. поле Е = 2fcQ/(L/2)2, создаваемое зарядами ±Q. Требуя •'Ы1км11!сния равенства Е = Е, находим g = Q(l/L)2.
’ 1ццача 244 (82-г)
Маленькая бусинка массой т может свободно скользить по оси, приходящей через центр кольца перпендикулярно его плоскости. Рашу г кольца R. Кольцо заряжено с постоянной линейной плотно-н.|о А. Бусинка несет на себе заряд д. Определите период малых • н>|Н|апий бусинки около центра кольца.
Решение. Кольцо можно разбить на малые участки длиной \/ - /£. Каждый такой участок несет на себе заряд AQ = АД/, и in можно рассматривать как точечный по отношению к бусинке.
11 а Идем силу f, с которой заряд Дф действует на бусинку. Пусть Ч» ника расположена на оси в произвольной точке с координатой г, и читываемой от центра кольца. Она находится на расстоянии г = , /Р |- х2 от любого заряда Дф, и по закону Кулона для величины • и 'I । .1 f можно написать
f । f |_	_ дАД/
4тгео»'2 4тгео(Я2 + я2)
• и .му f можно разложить на две составляющие: fx вдоль оси х' и нирмальную к ней составляющую /п. Действие второй составляю-uit'll всегда будет уравновешено действием со стороны другого заря-ц| AQ, расположенного на другом конце диаметра кольца. Поэтому ни цо учесть только действие составляющей
А = /сова = /- = Г
дАяД/ 4тг£о(Я2 + #2)^
In угол между направлениями оси х и силы f). Суммируя по in гм участкам кольца Д/, получаем величину результирующей си-Н.1, действующей на заряд д:
qAxR
2е0(Д2 + я2)Г
(1)
277
Если qX > 0, то сила F направлена от центра кольца и колебании возникнуть не могут; если же qX < 0, то сила, действующая на Гц синку, всегда направлена к центру кольца. При малых х (х <£ //) и знаменателе (1) в сумме х2 + R2 слагаемым х2 можно пренебречь и писать
F = -кх,	('Л
где
2г0Яг 2с0Я2'
Как известно, сила вида (2), приложенная к телу массой т, загт вляет его совершать малые гармонические колебания с периодом
Т = 2тгу/^/к.	(1|
Подставляя в (4) выражение для к (3), находим
T = 2kR.I^.
V I «АI
Задача 245 (81-г)
Незаряженная металлическая сфера помещена в однородна электростатическое поле, напряженность Е которого направл<М1и вдоль “оси” сферы. Известно, что при этом на “полюсах” сферы плотность а электрического заряда оказывается равной ctq — inil1 причем величина а не зависит от напряженности поля Е. Найди н плотность зарядов сг на произвольной “широте” т. е. функции*
Решение. Так как ось сферы является осью симметрии пол н. то ясно, что во всех точках сферы, расположенных на одной широк* плотность зарядов сг будет одинакова, т. е. сг есть функция толь ко от <р\ сг = На “полюсах” S и N сферы, которым отвечайй широты ±|, согласно условию задачи,
сг (±тг/2) = ±аЕ.
Нетрудно сообразить, что в силу симметрии задачи относителыи* экваториальной плоскости плотность зарядов на “экваторе” = 0) равна нулю: <т(0) = 0.
Плотность зарядов в точке А сферы, расположенной на проич вольной широте найдем, воспользовавшись принципом суперпогш ции. Однородное поле Е, в которое помещена сфера, можно предста вить как сумму двух взаимно перпендикулярных однородных пол<41
278
пппряженностями Ei и Е2, таких, что вектор Ei направлен по нор-Фнш, а вектор Е2 — по касательной к поверхности сферы в точке I, г. е. Е = Ei + Ег- Величины этих векторов равны Ei = Esin у?, / 1 Ecosy?. Поверхностная плотность заряда сг(у?), индуцировании о полем Е в точке Д(у?), есть сумма плотностей cri и 0*2 зарядов, ни купированных полями Ei и Е2 по отдельности:
= <Т1 + СГ2.	(1)
Ни отношению к полю Ei точка А лежит на оси и является “полю-• им", поэтому
<71 = a|Ei | = аЕ sin у?.	(2)
Ни отношению к полю Е2 точка А лежит на “экваторе”, т. е.
<72 = 0.	(3)
Принимая во внимание (2) и (3), из (1) получаем cr(y>) = аЕ sin 9?.
Задача 246 (77-р)
Голо массой т = 5 г подвешено с помощью тонких невесомых пн гей длиной L = 1 м каждая к двум наполненным гелием шарикам, мнорые несут на себе одинаковые электрические заряды. Система, in виснув в воздухе, находится в равновесии. Расстояние I между m и грами шариков равно 40 см и много больше их радиусов. Найдите щряды на шариках.
Рис. 87. К задаче 246.
279
Решение. На тело действует сила тяжести mg и силы натяни ния нитей Ti и Т2, которые, в силу симметрии системы относите и но вертикали, проходящей через точку А подвеса груза, равны и величине (|Ti| = |Т2| = Т) и составляют с вертикалью одинаконьь углы а (рис. 87). Так как груз находится в равновесии, то
Ti +Т2 + mg = 0.
Проецируя это векторное равенство на вертикальное направлопт получаем
2Tcosa — mg = 0.	(Il
На шарики действуют архимедова сила Fa, сила тяжести Р, сини натяжения нитей Т3 и Т4 и, наконец, электрические силы отталин вания Fi и F2 —такие, что Fi = — F2, т. е. |FX| = |F21 = F. Так im нити невесомы, то Т3 = -Ti и Т4 = -Т2. В силу симметрии сиги мы |Тз| = IT4I = Т. Поскольку шарики находятся в равновесии, м суммы всех приложенных к ним сил равны нулю. Так, для пернон шарика
FA + Р 4-Т3 + Fi = 0.
Проецируя это векторное равенство на горизонтальное направленш получаем
F — Tsina = 0.	(71
Из рассмотрения треугольника О АВ следует, что
sina=±.	(!1)
Исключая из системы трех уравнений (1), (2) и (3) неизвестные* 7' и а и решая затем получающееся уравнение относительно F, находим
mgl 2V4L2 - Z2 ’
Н)
F—это величина силы электрического взаимодействия шариков Обозначим величину зарядов шариков через q. Из закона Кулон и следует, что
F = q2/l2.	(h|
Приравнивая правые части (4) и (5) и решая получающееся уранп»’ ние относительно д, получаем
q = ld	900 ед. СГСЭ « 3 • 10"7Кл.
у 2y4Lz — I2
280
Задача 247 (76-г)
На одной из пластин тонкого конденсатора имеется маленькое • шерстив радиусом г, затянутое мыльной пленкой. Коэффициент поверхностного натяжения пленки сг. Конденсатор заряжен до раз-noi • ги потенциалов U. Расстояние d между пластинами конденсатора мало по сравнению с линейными размерами пластин. Оцените прогиб h пленки внутрь конденсатора, считая его малым, т. е. рас-• мптривая случай слабого поля.
Решение. Силы поверхностного натяжения уравновешивают-• и электрическими силами. Найдем вначале силы поверхностного натяжения.
11риближенно можно считать, что пленка принимает форму сферы радиусом R.
Рис. 88. К задаче 247.
На каждый малый элемент границы поверхности пленки длиной Л/ действует сила поверхностного натяжения Д/ = сгД/. Так как пленка имеет две поверхности, то сила ДР, приложенная к элементу ||пшицы пленки, равна 2Д/:
ДР = 2Д/= 2<тД/.
' >ти силы направлены по касательным к поверхности пленки перпендикулярно отрезку границы Д/ (рис. 88). Силу ДЕ можно разложить на две взаимно перпендикулярные составляющие: одну—в плоскости пластины, а другую— в направлении силовых линий. Обозначим величину последней составляющей через ДРП. Очевидно, что
ДРП = APsina,
где а — угол между пластиной и направлением силы ДЕ. Обраща-нсь к приведенному на рис. 88 чертежу, замечаем, что sina = r/R. Радиус R поверхности связан с радиусом г отверстия и прогибом h формулой
(R — h)2 + r2 = R2,
281
откуда R = (г2 4- A2)/(2/i). Считая, что h г, приближенно но.»п чаем R « г2/(2Л). Таким образом, sin а « 2Л/г и
ДГП « AaAlh/r.
Учитывая, что длина границы пленки I = 2тгг, для равнодейстпун* щей Fn всех сил ДРП получаем
Fn = 8тг<тЛ.	(I»
Равнодействующая Fn должна быть уравновешена силой при i м жения пленки к противоположной пластине конденсатора.
Найдем величину F3n действующих на пленку электрически* сил.
На элементе поверхности пленки площадью Д5 размещен элт трический заряд АД5 (А —поверхностная плотность зарядов). Э н и заряд находится в поле конденсатора с напряженностью F, причем половина этой напряженности создается зарядами одной пластины половина —зарядами другой. В соответствии с этим на заряд АЛ Л действует сила Д^эл = А Д6’Е'/2. Результирующую силу Езл найдем суммируя силы Д^эл по всей площади S поверхности пленки. 'Гии как S « тгг2, то
Гэл = пХг2Е/2.	(?)
В формуле (2) содержатся не известные пока величины Е и \ Напряженность поля Е в конденсаторе определяется формулой Е = U/d. Заряды размещены с той же поверхностной плотностын А, что и заряды на пластинах. Обозначим заряд пластины кондеи сатора через Q и площадь ее поверхности через S. Тогда
X = Q/S.	(3)
Заряд Q можно выразить через емкость конденсатора и приложен! ную к нему разность потенциалов: Q = CU. Площадь S находите’и из формулы емкости плоского конденсатора: S = e$Cd. Подставлял эти выражения в формулу (3), получаем
А- —= — “ eQCd~ eQd'
Из формулы (2) следует
282
Приравнивая Fn и F3Jl из формул (1) и (4), окончательно получаем
_ U2r2 16so<rd2
Задача 248 (77-г)
Металлический стержень А подвешен на непроводящей нити; М1»| пллический стержень В закреплен на непроводящей подставке. Гшттояние между соседними концами стержней очень мало. Что произойдет со стержнем А, если к противоположному концу стержни В поднести положительно заряженное тело С?
7/////////////.
А	В С
'ШШнМПЪ Рис. 89. К задаче 248.
Решение. Вследствие электростатической индукции на концах металлических стержней возникают индуцированные заряды, знаки которых показаны на рис. 89. Очевидно, что силы притяжения между разноименными зарядами соседних концов стержней явля-иигя преобладающими. В результате стержень А притягивается к < к’ржню В.
Задача 249 (89-р)
Два небольших шарика Л и В с зарядами —д и +2д насажены на гладкую длинную прямую непроводящую спицу и связаны ||(’гкой нерастяжимой диэлектрической нитью. Система помещена в неоднородное электрическое поле (рис. 90), так что спица является iH'i.io симметрии. При этом система зарядов оказалась в равновесии, и нить натянута.
а)	Определите отношение напряженностей внешнего поля в точках А и В.
б)	Является ли положение равновесия системы устойчивым?
Рис. 90. К задаче 249.
283
Р е ш е н и е. а) Закон Ньютона дает: та = Ед + Fb, где F ।
—<?Ед, Fb = 2дЕв-
Система находится в равновесии, поэтому a — 0:
О — Ед + Fb ,
или
О = —(/Ед + 2</Ев,
откуда получаем: Ед/Ев = 2.
б) Рассмотрим небольшой сдвиг системы вправо. Тогда на Л (ц дет действовать большая, чем раньше, сила Ед, направленная пл» во, а на В — меньшая, чем раньше, сила Ев, направленная впрмн»» Таким образом, суммарная сила будет отлична от нуля и iiaii|ni влена к исходному положению* системы. Аналогичные рассуждении справедливы и при сдвиге системы влево. Следовательно, пололи ние равновесия устойчивое.
Задача 250 (77-г)
Массивная заряженная частица помещается в неоднороднм» электростатическое поле. Скорость частицы в начальный момнн равна нулю. Будет ли частица двигаться вдоль силовых линий им ля? Ответ обоснуйте.
Решение. Силовые линии, характеризующие неоднородно» электрическое поле, —кривые. Предположим, что частица движи ся вдоль силовой линии. Тогда вектор ее скорости направлен по кп сательной к силовой линии, а вектор ускорения составляет с век и» ром скорости некоторый отличный от нуля угол, ибо только в этом случае траектория движения будет искривлена.
Другой результат можно получить, исходя из уравнения дни жения заряженной частицы в неоднородном электрическом пол»’, которое имеет вид
дЕ = та.
Здесь Е — напряженность поля, g, т, а — заряд, масса, ускоренш частицы. Отсюда видно, что направления векторов а и Е всегда со впадают. Это означает, что вектор а, как и вектор Е, в любой точк» силовой линии направлен по касательной к ней. Теперь ясно, что рп нее сделанное предположение неверно. Частица не может двигаты н вдоль силовой линии.
Задача 251 (77-р)
Два одинаково заряженных шарика подвешены к одной точке на невесомых нитях длиной L = 1 м каждая. Расстояние I между
284
ф и грпми шариков равно 4 см и много больше их радиусов. Найдите •ирнды шариков, если их массы одинаковы и равны т = 5 г.
Решение. Все силы, действующие на шарики, показаны на pin 91. Картина симметрична относительно вертикали, проходящей и рг । точку подвеса. В силу симметрии |Fi| = |F2| = F и |Ti| = |Г|| - Г.
Рис. 91. К задаче 251.
Так как система находится в равновесии, то суммы всех сил, приложенных к каждому шарику, равны нулю. Так, для первого ширика
Fi + mg + Ti = 0.
Проецируя это векторное равенство на горизонтальное и вертикаль-1Пи* направления, получаем
—F + Tsino = 0,
(1)
Г cos а — тд = 0.
(2)
Из рассмотрения прямоугольного треугольника О АВ:
//2 sina = jL
(3)
Исключая из системы трех уравнений (1), (2) и (3) неизвестные Т и а и решая получающееся уравнение относительно F, находим
mgl \/4L2-l2'
Но закону Кулона
F = q2/l\
(5)
285
где q— заряд шариков. Приравнивая правые части (4) и (5), псмц чаем уравнение относительно q, решая которое, находим
9 = lj , mgl к 40 ед. СГСЭ « 13 • 10“9 Кл.
У V4L2 -I2
Задача 252 (81-р)
Четыре тонкие металлические пластины расположены парк.*! лельно друг другу. Расстояние между пластинами много мепыи» их линейных размеров. Пластины изолированы друг от друга. Если внутренним пластинам сообщить заряды +7 и —<?, то между ними установится разность потенциалов U. Как изменится величина I’ если внешние пластины соединить между собой проводником?
Решение. Для определенности расположим пластины верш кально и пронумеруем слева направо (рис. 92). Пусть заряд пласш ны 2 равен +д, а заряд пластины 1 равен — q. Разность U потенции лов 9?2 и 9?з второй и третьей пластин связана с напряженностью / существующего между ними поля формулой
U = Ed,	(II
где d— расстояние между пластинами. Отсюда ясно, что, ответик на вопрос, как изменится напряженность поля Е при замыкании ключа К, мы одновременно ответим и на вопрос, как изменится pin ность потенциалов U.
+ q -я
3 4
—------ Рис. 92. К задаче 252.
К
Пока внешние пластины (1 и 4) не соединены друг с другом, прим тически все силовые линии поля, выходящие из зарядов +д, размс щенных на внутренней поверхности пластины 2, заканчиваются ни зарядах —д, размещенных на внутренней поверхности пластины 3 Мы имеем дело с обычным плоским конденсатором, образованным пластинами 2 и 3. Так как поле слева от пластины 2 практически отсутствует, то все точки этого полупространства, не слишком ди лекие от пластины 2 (в том числе и пластина 1), имеют одинаковый положительный потенциал
286
Аналогичным образом все точки, расположенные справа от пла-। f ины 3 и близкие к ней (в том числе и пластина 4), имеют одинаковый отрицательный потенциал 923. Между пластинами 1 и 4 । у шествует та же самая разность потенциалов U =	что и
между пластинами 2 и 3.
После соединения пластин 1 и 4 начнется перетекание с первой и настины на четвертую некоторого положительного заряда +q и ни потенциалы выровняются. При этом на внутренней поверхности и настины 1 окажется распределенным заряд —q , а на внутренней ппверхности пластины 4 — заряд +q . Под действием этих зарядов и и внешних поверхностях пластин 2 и 3 возникают индуцированные in ряды 4*7 и —7 соответственно, перетекающие с внутренних поверхностей пластин 2 и 3, на которых остаются заряды 4-7 и —7 '"ответственно. Так как полный заряд пластины 2 остается неизменным, то q + q = q. Отсюда следует, что q < 7.
Так как теперь на внутренних поверхностях пластин 2 и 3 насидятся меньшие по абсолютной величине заряды, то и количество • иловых линий между этими пластинами, а значит, и напряженность существующего между ними поля меньше. Уменьшение же напряженности свидетельствует об уменьшении разности потенци-пнов между пластинами 2 и 3 в соответствии с формулой (1).
Задача 253 (79-р)
Два очень маленьких одинаковых металлических шарика, имеющих заряды 71 и 72 = 271, подвешены на невесомых нитях, закрепленных в одной точке. Расстояние между шариками I = 2 см, длина каждой нити L I, Шарики сводят так, чтобы они коснулись друг ируга, и отпускают. На каком новом расстоянии друг от друга они установятся?
Р е ш е н и е. По условию задачи размер шариков мал в сравнении । расстоянием /, т. е. шарики можно рассматривать как точечные тряды.
Силы, действующие на шарики, показаны на рис. 91 к задаче 251.
Так как шарики покоятся, то суммы приложенных к ним сил равны нулю:
Pi 4- mg 4- Ti = О, F2 + mg 4* Т2 = 0.
Проецируя эти векторные равенства на горизонтальную и вертикальную оси и учитывая при этом, что Fi = Р2 и Ti = Т2 = F, получаем два уравнения
Tsina = 717г/^2,	(1)
287
Tcoso = тд.
Деля почленно уравнения (1) и (2), находим
.	_ Щ2
g mgl2'
Кроме того, из рассмотрения треугольника АО В следует, что
//2 tga = —
PI
Так как по условию задачи L то вторым членом разности содержащейся под знаком радикала, можно пренебречь в сравнении с первым, и мы получаем
tSQfs2Z'	(Ч
Приравнивая правые части равенств (3) и (4) и решая получающего при этом уравнение относительно находим
l= j2L<h<12 тд
После соприкосновения шарики несут на себе одинаковые ui ряды
'	1 Q1 4" (72	3	.
Q1 - 02 = —2— = 291
и устанавливаются друг от друга на некотором расстоянии I , кото рое будет определяться формулой, аналогичной формуле (5):
> _ э/21/^2 ^2
” V тз
(V)
Разделив почленно равенство (7) на равенство (5) и решая новое равенство относительно I , имеем
V
(8)
Заменяя в (8) qlt q2 по формуле (б), окончательно получаем
2
288
Задача 254 (79-р)
,Цва маленьких одинаково заряженных металлических шарика пнпвсшены на невесомых нитях одинаковой длины, закрепленных в ••иной точке. Расстояние между шариками 1 = 1 см. Один из шариков 1'н фяжают. Каким будет новое расстояние между шариками?
Решение. Воспользуемся обозначениями задачи 253. Искомое |ц|| < тояние V определяется формулой (8) задачи 253:
= (1) V 9192
В нашем случае по условию задачи
91 = 92 = 9-	(2)
После того как один из шариков разряжают, шарики начинают ипнгаться навстречу друг другу и соприкасаются. После соприкосновения они несут на себе заряды и q2, причем
Q1 — 42 — я/%-	(3)
Заменяя в формуле (1) gi, дг5 41 и Я2 правыми частями формул (2) и (3), для V находим
I' = 1/^4 = 0,63 см.
Задача 255 (76-р)
Две заряженные металлические сферы находятся на большом расстоянии друг от друга. Радиусы сфер п и гэ. Сферы соединены гонким металлическим проводом. Найдите отношение напряженностей на поверхностях сфер. Расстояние L между сферами много Польше радиусов сфер.
Решение. Потенциал поверхности изолированной металлической сферы радиусом г с зарядом Q определяется формулой
Q ф — -----•
4тгеоГ
Поверхности сфер разных радиусов п и Т2 соединены тонким проводником, т. е. имеют одинаковый потенциал, но несут на себе ризные заряды Q\ и Q%. В силу равенства потенциалов сфер имеет место следующее соотношение:
Ql/П = (?2/?*2-	(1)
289
Напряженность поля у поверхности сферы радиусом г с зарядом определяется формулой
ip. М 4яе0г2’
В соответствии с этим
£*1 _ Qi /
^2 Q2 \г1/ ’
или с учетом формулы (1)
^1/^2 = гз/п*	(’Л
При решении задачи мы пренебрегали влиянием сфер друг ни друга. Оценим это влияние. Сфера с зарядом Q\ создает на расстп янии L поле с напряженностью
iei = J01L.
1 1	4m-
Влиянием этого поля можно пренебречь, если его величина мною меньше величины поля, создаваемого второй сферой около своей поверхности, т. е. в том случае, если
Q1 Q2
4тгео£2	47Г60Г2’
откуда
42 » %-rl	СИ
Q2
Рассматривая поле второй сферы у поверхности первой, получа ем условие
L2 » ^г2.	(4)
Используя формулу (1), легко получить следующее условие:
L2 » Г1Г2.	(5)
Таким образом, полученный ответ (2) справедлив только при вы полнении условия (5).
290
Задача 256 (76-р)
} Цзе металлические сферы с радиусами и и г% находятся в ваку-на расстоянии L друг от друга. Сферы имеют заряды 01 и Q? ♦ ^ответственно. Затем сферы соединяют тонким проводником. Наймите заряды и потенциалы сфер до и после соединения. Расстояние между сферами много больше радиусов сфер.
Решение. Потенциалы и сфер до соединения их тонким проводником определяется формулами:
01	0г
V’l — -----,	9^2 — -----.
47Г£оП	47Г£оГ2
Мы пренебрегаем влиянием сфер друг на друга. Такое пренебрежение справедливо, если расстояние L между сферами удовлетворяет иловиям (3) и (4) из задачи 255.
После соединения заряды перераспределяются таким образом, •I го потенциалы сфер становятся равными друг другу, и вследствие и ого имеют место равенства
<2'1 /п = <2'2/г2 = 47TEOSP-	(1)
1цгсь у?— потенциал сфер после соединения их проводником. Так как полный заряд сохраняется, то
0i + 01 — 01 + 0г-	(2)
1,(‘1пая систему, состоящую из трех уравнений (1) и (2), относительно неизвестных 9?, Qi и 01 > находим
1 01 + О2
92 ~~-----------,
4те0 Г1 4- г2
Q'i =	+
ri 4- г2
Г1 + г2
Задача 257 (78-р)
Одинаковые заряды q расположены в вершинах квадрата. Какой тряд 0 надо поместить в центр квадрата, чтобы силы, действующие на каждый заряд со стороны остальных зарядов, были в сумме рпвны нулю?
Решение. Прежде всего рассмотрим силы, действующие на нносимый заряд Q со стороны остальных зарядов. Эти силы напра-ппены по диагоналям квадрата, равны друг другу по величине и
291
попарно противоположны. Поэтому их равнодействующая, незнпн симо от величины и знака вносимого заряда, всегда равна нулю
Заряды q совершенно равноправны. Поэтому, если один из и и» находится в равновесии, то в равновесии будут находиться и цн» остальных, и нам достаточно рассмотреть силы, действующие н»< любой из зарядов q.
Силы, действующие на заряд д, расположенный в правом вер* нем углу квадрата, показаны на рис. 93. Их величины, согласно зн кону Кулона, определяются формулами
Fi = Ъ =	= /j, F4 =	(Il
а2 2а2	а2
где а—длина стороны квадрата. Условие равновесия заряда q
Fi + F2 + F3 + F4 = 0.
Проецируя это векторное равенство на направление диагонали, ни которой находится заряд q, получаем следующее равенство:
(Fi + /*2) cos 45° + F3 + jRj = 0.
Заменяя здесь Fi, F2, F3 и F4 правыми частями (1) и решая эан’М получающееся равенство относительно Q, находим
Рис. 93. К задаче 257.
Рис. 94. К задаче 258.
Задача 258 (78-р)
Одинаковые заряды q расположены в вершинах правильном । треугольника. Какой заряд Q надо поместить в центр треугольники чтобы суммарная сила, действующая на каждый заряд со стороны остальных зарядов, была равна нулю?
292
Решение. Правильным треугольником называют равносторонний треугольник, а его центром — точку пересечения медиан. Центр hi г< шт от любой из вершин треугольника на расстоянии b = । Hi’ а -длина его стороны.
( Начала рассмотрим силы, действующие на центральный заряд Q Они равны по величине и составляют друг с другом угол 120°. Иг трудно показать, что при любой величине и любом знаке заряда Q равнодействующая этих сил равна нулю.
Заряды q совершенно равноправны. Поэтому, если один из них находится в равновесии, то в равновесии находятся и два других. II г вязи с этим нам достаточно рассмотреть силы, действующие на пибой из зарядов q. Силы, действующие на заряд qy расположенный и верхней вершине треугольника, показаны на рис. 94, их величины • •иргделяются формулами
л = = £ Ъ=$ = з£.	(1)
Условие равновесия заряда q
Fi 4- F2 4- F3 — 0.
Проецируя его на вертикальное направление, получаем
(Fi + F2) cos 30° 4- F3 = 0.
1пменяя в последнем равенстве Fi,F2, и F3 правыми частями (1) и решая затем получающееся равенство относительно Q, находим
Q = -д/у/З.
Задача 259 (83-г)
Из четырех одинаковых пластин площадью S каждая, которые нельзя придвигать друг к другу на расстояние меньше d (d <£ л/S), нужно сделать конденсатор максимально возможной емкости. Как иля этого надо расположить пластины?
Решение. Наибольшую емкость С системы из двух пластин площадью S получим, сблизив их на максимально возможное расстояние d. Она определяется формулой
С = esoS/d,
Здесь бо — электрическая постоянная и г —диэлектрическая проницаемость среды.
293
Очевидно, что для получения системы с наибольшей емкп< и* из четырех пластин надо, во-первых, расположить их на рш« о •• нии d друг от друга и, во-вторых, так соединить между собой, ч i»•• • получить систему из возможно большего числа параллельно ...
ненных конденсаторов с емкостью С (параллельное соединенна личивает емкость системы). Располагая пластины на расстоянии друг от друга стопкой и соединяя между собой нечетные по пори<н> расположения пластины с одной стороны и четные с другой, шип чн ем систему, эквивалентную системе из трех конденсаторов емки. । (7, соединенных параллельно. Эта система обладает максимюнн возможной емкостью
(7тах — 3(7 —	/ d.
Задача 260 (81-г)
В однородное электростатическое поле с напряженностью / м местили легкий жесткий стержень, на концах которого закреп и* ш маленькие шарики массой т каждый. Шарики заряжены зарюкю. +q и —q. В начальный момент стержень ориентирован по полю III рики получают скорости +vq и —vq, направленные перпендикуюн но стержню (рис. 95, а). На какой максимальный угол поверю i стержень? Длина стержня 2/, сила тяжести отсутствует.
Рис. 95. К задаче 260.
Решение. Для решения задачи воспользуемся законом • хранения энергии. При вращении стержня около его центра долги м-сохраняться сумма потенциальной энергии стержня U(а) в элек i рм ческом поле и его кинетической энергии nw2(a) (последняя < и дывается из кинетических энергий масс, расположенных на khiiim стержня). Через v(a) мы обозначили линейную скорость зарядом »
294
• mi момент, когда стержень составляет с направлением поля угол । Итак,
mv2(a) 4- {7(a) = mv2(0) 4- (7(0) = mv2 4- {/о-	(1)
I'H’i и (7o = (7(0) — потенциальная энергия стержня в начальный • к»М1чгг, когда а = 0.
При повороте стержня на угол а заряды 4-7 и —q перемещаются mi и тек А и G с потенциалами и в точки В и D с потен-MiifuuiMH <рв и (рис. 95, б). Потенциальная энергия системы при о нм меняется на величину
г(н) - и0 = д(<рв -	- <pg) = д(<рв - <ра) + q&G - <pd}-
Принимая во внимание, что разности потенциалов
|инн1Ы и потенциалы срв и <рс точек В и С одинаковы, получаем
U(a)-U0 = 2q&C-<PA).	(2)
* 'шгывая, что
<РС -<РА = Ех,
• .|Г
х = 1(1 — cos а) = 21 sin2 —
расстояние между точками А и (7, из (2) находим
и (а) = Uo + 4g£7sin2 у.	(3)
II* (I) и (3) получаем следующее уравнение:
mv2(а) 4- 4qEl sin2	= mv2.	(4)
z
Максимальному углу amax отклонения стержня от начального пм'южения отвечает очевидное условие v(amax) = 0. Принимая это ни внимание, из (4) находим
2 . 2 ®niax ^0 sln “Г" “ 4^ЁГ
(5)
li пи в уравнении (5) mv2/(4qEl) > 1, то оно не имеет решения. В <||>м случае стержень, не останавливаясь, вращается в одном направлении. Если же mv^/(4qEl) < 1, то возникают колебания, при
ним
С* max
= 2 arcsin
VQ / т
2 V QEl
295
(два корня соответствуют двум крайним положениям стержни) Если mv2/(4qEl) = 1, то атах =	— это положение неустойчп
вого равновесия стержня.
Задача 261 (77-р)
Тонкое металлическое кольцо радиусом R = 30 см имеет зарил Q = 10 ед. СГСЭ. Найдите напряженность поля на оси кольца пи расстоянии d = 40 см от плоскости кольца.
Решение. Линейная плотность заряда кольца
А = _0_
2тгй’
На отрезке кольца длиной А/ размещается заряд
_ ЧА; QM Дф = АД/ =	.
Поместим в точку О, в которой требуется найти напряженность по ля, положительный пробный заряд величиной q и найдем действу ющие на него силы ДЕ1 и ДЕг со стороны электрических зари дов, заключенных на отрезках кольца длиной Д/, расположенных на противоположных концах его диаметра. Очевидно, что эти силы направлены вдоль прямых, соединяющих точку О с отрезками Д/ кольца (рис. 96), и |AFj| = |ДЕз| = ДЕ, где
д F	__ gQ&l	/11
d2 + R2 27гЯ(с/2+Я2)‘	V 1
296
Результирующая AF сил AFi и AF2 направлена вдоль оси кольца. Ее величина AF определяется формулой
AF = 2AFcosa,	(2)
где а — угол между осью кольца и направлениями сил AFi и AF2.
Рассматривая треугольник ОАВУ нетрудно сообразить, что
d
cos а = .	(3)
V<? + R2
Заменяя в (2) AF и cos а по формулам (1), (3), для AF имеем
____qQdM , .
тгЛ(</2 + я2)3/2
Величину F полной силы, действующей со стороны всего кольца на заряд д, получим, суммируя все силы AF, действующие со г тороны всех образующих кольцо элементов AZ:
г _ V д р - V	_ _qQd__ у- Л.
Z^^(d2 + J?2)3/2 - ^jR(rf2+7?2)3/2Z^£il-
Так как £2 А/ = ^Ry то
F =___________
(d2 + Л2)3/2
Напряженность поля
Е = - = .	« 0,0032 ед. СГСЭ « 96В/м.
q (а2 + я2)3'2
Задача 262 (79-р)
В схеме, изображенной на рис. 97, конденсаторы Ci и С2 имеют одинаковую емкость С. В начальный момент ключи Ki и К 2 разомкнуты, а конденсаторы Ci и С2 заряжены до разности потенциалов U\ и U2 соответственно, причем знаки зарядов, размещающихся на пластинах конденсаторов, показаны на рисунке. Какой заряд пройдет через сопротивление R при одновременном замыкании ключей?
Рис. 97. К задаче 262.
297
Решение. До замыкания ключей заряды конденсаторов ('| и С*2 равны соответственно qi = CU\ и 72 = — CU2, а суммарный эп|hi<1 <ЭВ на верхних обкладках обоих конденсаторов равен сумме q\ | 1/, таким образом,
Qb = 91 + 92 = C(U1 — 1/2)-
Суммарный заряд QH на нижних пластинах обоих конденсатор» и» имеет противоположный знак, т. е.
Qh = -Qb = C(U2-U1).
Далее, для определенности, будем считать, что U2 > СЛ- Touiti Qh > 0 и QB < 0.
После замыкания ключей через сопротивления Л, Я1 и Т?2 булм течь ток до тех пор, пока все заряды не скомпенсируются. Зарин Qh, существовавший до замыкания ключей на нижних пластиши может быть скомпенсирован только зарядом QB, существовавшим на верхних пластинах. Таким образом, после замыкания ключ» II через сопротивление R пройдет отрицательный заряд
QB = C(Ui - U2)
(носителями тока в металлах служат отрицательно заряжен и i а» электроны).
Задача 263 (75-р)
Найдите емкость Со системы, составленной из одинаковых коп денсаторов емкостью С каждый по схеме на рис. 98, а.
Рис. 98. К задаче 263.
Решение. Схему, показанную на рис. 98, а, можно перерисо вать так, как показано на рис. 98, б. Перемычку АВ можно убрать, поскольку потенциалы точек А и В равны как в схеме с этой и»’ ремычкой, так и без нее. Емкость получившейся схемы считаете и просто и равна емкости первоначальной схемы:
С0 = 1с.
298
Задача 264 (82-г)
Имеется N конденсаторов с емкостями Сд, •> Сдг. По одному и । выводов конденсаторов соединены вместе. Конденсаторы не зари жены. Затем свободные выводы конденсаторов подсоединяют со-«овгтственно к N клеммам с потенциалами <pif 923, ..., (рис. 99). • )цределите заряд на каждом конденсаторе.
Рис. 99. К задаче 264.
Решение. Пусть <ро —потенциал общей точки. Заряд qi внешней пластины г-го конденсатора определяется равенством

(1)
In ряд его внутренней пластины равен — qi. Поскольку сумма заря-ннв всех внутренних пластин конденсаторов равна нулю, то равна пулю сумма зарядов внешних пластин, а в силу этого имеет место равенство
N
- <Po)Ci = 0.
1 = 1
Отсюда находим потенциал общей точки
Е fiQ 2=1
Е Ci 2=1
Подставляя это значение в (1), получаем
Qi =
299
Задача 265 (82-г)
Как изменится емкость сферического конденсатора, состоящем' из сфер с радиусами п и гз (ri < гз), центры которых расположены в одной точке, если между ними поместить тонкую металлическую сферу с радиусом Г2?
Решение. В сферическом конденсаторе между его обкладки ми существует поле, эквипотенциальными поверхностями которин' являются концентрические сферы. Поэтому размещение между <><> кладками конденсатора тонкой концентрической (по отношению и обкладкам) металлической сферы не изменит распределения полк и конденсаторе, а вместе с этим не изменит распределения заряда пн обкладках и разности потенциалов между ними. Вследствие этом» емкость конденсатора останется неизменной.
Задача 266 (83-г)
Имеется три тонких коаксиальных металлических цилиндри Малый цилиндр песет на себе заряд —д, два других несут на гг бе одинаковые заряды д/2. Большой и средний цилиндры соедини ют тонкой проволокой. Найдите заряды цилиндров после того как процесс перераспределения зарядов кончится.
Решение. До соединения цилиндров проволокой заряд —q pan номерно распределен по внешней поверхности малого цилиндра. Ин внутренней поверхности среднего цилиндра распределен заряд -I г/, а по его внешней поверхности — заряд —q/2 (так что полный зарлл среднего цилиндра q' = q — q/2 — q/ty. Заряд q/2 большого цилиндра расположен на его внутренней поверхности. Электрическое поле су ществует только в пространстве между цилиндрами. Силовые линии поля между малым и средним цилиндрами начинаются на зарядах расположенных на внутренней поверхности среднего цилиндра, и кончаются на зарядах —7, расположенных на внешней поверхш» сти малого цилиндра. Силовые линии поля в пространстве между средним и большим цилиндрами начинаются на зарядах + q/2, кп торые несет на себе внутренняя поверхность большого цилиндра, и кончаются на зарядах —q/2, которые несет внешняя поверхнос н. среднего цилиндра.
Как только металлическая проволока соединит средний и боль шой цилиндр, по ней сразу же начнет течь ток, который продол жится до тех пор, пока все поле в пространстве между средним и большим цилиндрами не станет равным нулю. К этому моменту вечч. заряд +q/2 с внутренней стороны большого цилиндра перетечет па внешнюю сторону среднего. В результате большой цилиндр окаты
300
ппгтся незаряженным, а полный заряд среднего цилиндра — равным 7' I- <?/2 = д.
Задача 267 (77-г)
Плоский конденсатор емкостью С заряжен до разности потенциалов U. Как изменится разность потенциалов, если заряд одной из обкладок увеличить в два раза, а заряд второй обкладки не менять?
Решение. Разность потенциалов U между пластинами любого (в том числе и плоского) конденсатора емкостью С с зарядом Q определяется формулой
U = Q/C.
Рассмотрим более подробно, как возникает эта разность потенциа-юв в случае плоского конденсатора.
Заряженный плоский конденсатор представляет собой электрическую систему из двух параллельных пластин, на одной из которых равномерно распределен заряд Q, а на другой заряд — Q. Так кпк расстояние между пластинами много меньше линейных размеров пластин, то можно считать, что поле, создаваемое в зазоре между пластинами конденсатора, тождественно полю двух равномерно заряженных бесконечных плоскостей. Обозначим напряженность этого поля через Е. Каждая из этих двух положительно и отрицательно заряженных плоскостей создает в пространстве между ними направленные в одну сторону поля с напряженностью /'7*2 каждое. Напряженность поля пропорциональна поверхностной плотности зарядов.
Увеличивая в два раза заряд на одной из пластин конденсатора, мыв два раза увеличиваем поверхностную плотность заряда на ней. Гюсконечная пластина с увеличенной в два раза плотностью заряда । издает в пространстве между пластинами поле в два раза большей напряженности Е. Полная напряженность поля Е' складывается из напряженности поля Е/2 одной пластины и Е другой:
1	3
Е'=-Е + Е=-Е.	(1)
А	и
Разности потенциалов U и U' между пластинами определяются формулами:
U = E/d, U1 = Е'/а>	(2)
где d—ширина зазора между пластинами. Из равенств (1) и (2) получаем
и' = 1и-
301
Задача 268 (85-г)
Имеется схема, состоящая из большого числа произвольно соеди ненных конденсаторов. Увеличится или уменьшится емкость схемы если один из проводов в ней разрезать?
Решение. Предположим, что заряд этой системы равен Q Соединим две произвольные точки проводником. При этом произой дет такое перераспределение зарядов, что полная энергия системы уменьшится (заряды двигаются только в том случае, если это при водит к уменьшению полной энергии). Поскольку заряд системы конденсаторов остается неизменным, а ее энергия IV, определяемо формулой W = CU2/2 = (?2/(2С), уменьшается, то ясно, что поди» единение проводника увеличивает емкость. И наоборот: размыкаине одного из проводов схемы уменьшает ее емкость.
Задача 269 (85-г)
Имеется бесконечная последовательность изолированных ком центрических полусфер, радиусы которых связаны соотношением Ri = 2Ri_i. Радиус самой маленькой полусферы R\. На каждую по лусферу помещают заряд Q, а на расстоянии г от центра полусфер помещают заряд q. Найдите потенциал в центре полусфер, если по тенциал на бесконечности равен нулю.
Решение. Согласно принципу суперпозиции, потенциал у? п искомой точке представляется суммой потенциалов создаваемым зарядами Q полусфер, и потенциала создаваемого точечным рядом q (у>о — q/rY По тому же принципу, потенциал, создаваемый протяженным заряженным телом, представляется суммой потенци алов, создаваемых зарядом каждой точки тела. Так как у полусфер все точки находятся на одинаковом расстоянии от центра, то, шпп висимо от того, как распределен заряд Q по поверхности данной полусферы, <pi = Q/Ri. Нетрудно сообразить, что Ri = 21"1/?!. (' учетом этого получаем
а 2£
г 7?1’
где проведено суммирование бесконечной геометрической прогрсг сии
= 2.
302
Задача 270 (85-р)
Металлическая пластина массой т и площадью S лежит свобод-in» на горизонтальной поверхности стола. Параллельно ей на высоте i/, малой в сравнении с линейными размерами пластины, укреплена 11»'|по такая же пластина. Между пластинами создается некоторая рпшость потенциалов ЛСЛ При каком значении нижняя пла-। шла оторвется от стола? Считать, что заряды на пластинах равны нп величине и противоположны по знаку.
Решение. Так как по условию задачи расстояние между пла-• Iинами d мало по сравнению с их линейными размерами, то поле между пластинами можно считать однородным. Его напряженность
Е = &U/d.
In ряд Q каждой пластины равномерно распределен по ее поверхно-• । и и может быть подсчитан с помощью формулы
Q = CAU,
(1)
те С — емкость плоского воздушного конденсатора, образованного плпстинами. Для емкости С имеем
С = €QS/d,
(2)
। дг t’o — электрическая постоянная. Поле между пластинами созда-|'К'Я в одинаковой степени как зарядами верхней, так и зарядами нижней пластины. В соответствии с этим для напряженности Ei 1Н».пя, создаваемого только зарядом верхней пластины, можно напи-
। п гь:
(3)
In ряд Q нижней пластины находится в этом поле, и на него со стороны поля действует сила
F = QEU
нпправленная вверх. С учетом равенств (1), (2) и (3) для F находим
Исли величина силы F больше силы тяжести тд} т. е. если имеет мес то неравенство

зоз
то нижняя пластина отрывается от стола. Данному неравенств эквивалентно следующее:
д« > a/W-
V
Задача 271 (85-р)
Найдите силу взаимодействия пластин плоского воздушного кон денсатора емкостью С, имеющего заряд Q, при расстоянии межщ пластинами d.
Решение. Обозначим через Д[/ разность потенциалов междъ пластинами конденсатора. Для Д(7 можно написать:
Д[/ = Q/C.
Считая поле внутри конденсатора однородным, для его напряжен ности Е имеем
Е~ d Cd'
Это поле создается зарядами, расположенными на каждой из плп стин. Для напряженности Е\ поля, создаваемого зарядом одной или стины, имеем
F _ЛЕ=о_
E1 2Е 2Cd
Каждая пластина находится в поле, создаваемом другой пластиноП, и со стороны этого поля на равномерно размещенный по каждой пластине заряд Q действует сила
Q2 F = (ЭЕ, =
4 1 ‘2Cd
Задача 272 (75-р)
Система из двух одинаковых последовательно соединенных коп денсаторов подключена к источнику постоянного напряжения • ЭДС 8. Емкость каждого конденсатора С. Обкладки одного из коп денсаторов закорачивают. Найдите изменение энергии системы коп денсаторов и работу источника. Равны ли эти величины? Если hoi, то почему?
Решение. Напряжение на каждом конденсаторе до закори чивания одного из них равно U = 8/2, следовательно, суммарном энергия конденсаторов
С82
4
304
I li и’ле закорачивания одного из конденсаторов напряжение на нем • ипювится равным нулю, а на другом конденсаторе — суммарной энергия конденсаторов при этом Е2 = С£2/2. Изменение энер-HIII конденсаторов ДЯ = Е2 - 5*1 = С£2/4. Работа источника I - £Дд, где Дд — изменение заряда конденсаторов. Начальный hi ряд конденсаторов £1 = С£/2У конечный — ^2 = С£. Следователь-пи, Дд = q2 — qx = С£/2 и работа источника А = С£2/2.
'Таким образом, работа источника в два раза больше изменения шгргии конденсаторов. Ровно половина работы источника пошла на пыдсление тепла в проводах.
305
11. ЭЛЕКТРИЧЕСКИЙ ТОК
Электрический ток в проводниках — это направленное движгпн' электрических зарядов. Оно вызывается внешним по отношении' ♦ движущимся зарядам электрическим полем (электродвижущей » и лой). Конкретные свойства среды приводят к определенным нпи номерностям— соотношениям между силой тока и приложенным внешним полем (разностью потенциалов). Чаще всего в задачах рп* сматриваются металлические проводники, ток в которых подчини ется закону Ома
I=U/R.	(И
Этот закон является следствием очень сложных физических пр> цессов в металлах, происходящих под воздействием внешних шин II Так, разность потенциалов U на концах линейного проводника ;i in ной I означает существование в проводнике электрического ikmiii напряженностью Е = U/1. Электрическое поле действует на зариим с определенной силой, что приводит к их ускоренному движеншп |. направлении поля. Сила тока связана с количеством зарядов, проч», дящих через поперечное сечение проводника, а это количество н| порционально скорости зарядов. Постоянный ток — направлспп'" движение зарядов с постоянной скоростью. При этом внешнее пик воздействует на заряды и совершает работу над ними, а их cpejui н н скорость и энергия не меняются.
Строго описать процессы, происходящие в проводниках под л« II ствием внешнего поля, можно только в терминах квантовой теории На классическом языке удается сделать вывод, что наличие гони • проводниках, подчиняющихся закону Ома, приводит к выделешн’ тепла. Действительно, если по проводнику течет ток 7, а разно» и потенциалов на концах проводника равна 17, внешнее электрически, поле совершает над зарядами в единицу времени, работу 717, koi»» рая и выделяется в виде тепла. Если закон Ома (1) выполняете л, о количество теплоты, выделяющееся в проводнике в единицу В|м»м» ни, равно N = I2R (закон Джоуля—Ленца).
Движущиеся электрические заряды создают не только элекчрн ческое, но и магнитное поле. Электрическое поле действует нп ш • заряды, а магнитное поле — на движущиеся заряды. Электрически' поле с напряженностью Е и магнитное поле с индукцией В дейгпц ют на заряд qy движущийся со скоростью v, с лоренцевой силой
F = gE + g[vB].	|’’i
306
Формула (2) записана в системе единиц СИ. Как видно из (2), отпишите величин Е и В в этой системе имеет размерность скоро-III В физике существует единственная универсальная постоянная ..шчина, имеющая размерность скорости (см. Добавление X) — корость света с. Если вместо индукции В ввести для описания мпгнитного поля величину сВ, размерность которой совпадает с раз-н'рностью Е, то характеристикой степени воздействия магнитного id и >i на движущийся заряд будет не его скорость v, а отношение v/<•. Это подчеркивает существенно релятивистский характер воз-и Пгтвия магнитного поля на движущийся заряд.
Эго свойство магнитного поля проявляется и при вычислении мпгнитного поля, создаваемого движущимся зарядом. Это поле пропорционально величине заряда и его скорости. По тем же соображе-ппнм размерности, что и при обсуждении формулы (2), эффективным параметром будет не абсолютная скорость заряда, а его ско-pm-гь по отношению к скорости света с.
В так называемом нерелятивистском (классическом), пределе р > оо) движущиеся заряды не создают магнитного поля, магнитит’ поле не воздействует на движущиеся заряды. Это утверждение ни первый взгляд неверно. Проводники с токами создают магнитит’ ноле, которое проявляется в действии на другие проводники, на ншнитные опилки и т. д. Известно также, что скорость движения шрядов в проводниках обычно очень мала (миллиметры в секунду!). ||’.но в том, что движущихся зарядов в макроскопических проводниках очень много (« 1022 в кубическом сантиметре). Из-за этого i n hi много числа источников и появляются заметные магнитные по-
III постоянных токов.
При изменении магнитного поля в проводниках возникает элек-||н движущая сила (эффект электромагнитной индукции). При дви-|||'пии проводников в магнитном поле в них также возникает элек-ц к движущая сила. Эти эффекты имеют совершенно различную природу, но описываются одинаковыми формулами
£ = -ДФ/Д/.	(3)
liK'c.b ДФ — изменение потока магнитной индукции через некоторый контур за время Д£, £ — ЭДС индукции, возникающая в этом кип гуре. Знак минус в формуле (3) показывает, что ЭДС в контуре направлена так, чтобы препятствовать изменению магнитного пото-hii через контур (правило Ленца).
При решении задач для того, чтобы правильно использовать формулу (3), необходимо ясно понимать происхождение ЭДС индукции. При изменении магнитного поля в любом контуре, поток
307
через который меняется, возникает вихревое электрическое пот Ток (движение зарядов) появляется, если рассматриваемый koiihi проходит в проводящей среде. Причиной возникновения ЭДС ii| « изменении формы контура (т. е. при движении проводника в м»п нитном поле) является лоренцева сила (2), действующая на зарин» • проводника. Эта ЭДС может появиться только при наличии своПн< ных носителей зарядов.
* * *
Задача 273 (80-г)
Имеются два источника тока с ЭДС f = 1 В и внутренним • • противлением г = 1 Ом каждый и два одинаковых сопротивлении нагрузки R = 1 Ом. Какую замкнутую цепь надо собрать из ин« элементов, чтобы:
а)	коэффициент полезного действия источника был максимален б) количество теплоты, выделяющееся на нагрузке, было маю и мальным?
Рис. 100. К задаче 273.
Решение. При включении нагрузки с сопротивлением /{н и цепь с ЭДС £q и внутренним сопротивлением источника го ио ш ин течет ток
1=— Го + Ян
308
ii мощность на нагрузке
wu = I2Rh =
gp2 Дн
(r0 + Ян)2 ’
(1)
Мощность источника тока
Г2
W = I£q = Гр + Дм
Щмффициент полезного действия цепи
= Ж = Дн =	1
4 W r0 + RH 1 + го/Ян'
Источники тока и сопротивления нагрузки можно соединять по-|гцовательно или параллельно. При этом можно построить четыре (и| мшчные схемы, изображенные на рис. 100, а—г.
Из формулы (2) видно, что максимальный КПД достигается при минимальном значении отношения гр/Дн- При включении по схеме нц рис. 100, а сопротивление гр = г/2 минимально, а Ян = 2R мак-нмпльно, КПД при этом равен
Для ответа на второй вопрос надо вычислить мощности на на-।РУ же во всех четырех случаях. Используем формулу (1) и учтем, ин ио условию задачи г = R.
п) Источники тока соединены параллельно, сопротивления — ши исдовательно. В этом случае
г	Я £2
£о = £, го = -, /?н = 2Я, Wa = — —.
fir
6)	Источники и сопротивления соединены последовательно:
£0 = 25, г0 = 2r, RH = 2Я, W6 =
Z fl
и) Источники и сопротивления соединены параллельно:
г _ с _ г р _ R тл/ _ 8 £2 5о-5, r0-2, Rn- 2, We_ 25 R
309
г) Источники соединены последовательно, сопротивления параллельно:
р	о с2
£о = 2£, г0 = 2г, Ян = ^, №г=- — .
Итак,
W6=We>Wa = W2)
т. е. мощность на нагрузке максимальна, когда включение произпг дено по схемам б или в.
Задача 274 (79-г)
Схема, изображенная на рис. 101, состоит из четырех активны* сопротивлений (J?i = 100 Ом и R? = 20 Ом), конденсатора емкое ты»» С = 10 мкФ и идеального диода. Схему включают в сеть перемен ного тока с напряжением U = Uocoswi (€7q = 120 В). Конденсатор н начальный момент разряжен. Найдите его заряд в установившего и режиме.
°	Рис. 101. К задаче 274.
Решение. Ток через идеальный диод может идти только в одном направлении— от точки С к точке D. Вследствие этин* заряд на обкладках конденсатора может только увеличиваться. ‘In ряд будет увеличиваться до тех пор, ока напряжение на диоде ш станет таким, что диод будет закрыт на протяжении всего периол п колебаний напряжения в сети. С этого момента ток через диод те‘п не будет, и заряд Q конденсатора останется неизменным.
При отсутствии тока через диод (в установившемся режиме) иг трудно найти силу тока, протекающего через каждое из conpoiii влений:
т U*
1 — -----— cos ап.
Ri + Я2
По закону Ома находим падения напряжений на каждом из сопри тивлений:
гг	гт	U R1	..
Ui =uad = Д1 + д2 cos^;	О'
ТГ	ГТ	UR*	.
^2 = иАС =	+ д2 cosevt	(,')
310
Нппряжение (разность потенциалов) между точками С \\ D равно UCD= UAD~ UAC-	(3)
ho напряжение складывается из падения напряжения на конден-
• игоре UK = Q/C * и на диоде С/д:
uCd = Q/c+ua.	(4)
Приравнивая правые части равенств (3) и (4) с учетом равенств |1) и (2), получаем
__	С/0(Я1-Я2)	, Q
U* = R,+R, ^"‘-5-
Го, что в установившемся режиме диод заперт, означает, что напря-«к<»ние на диоде в любой момент времени ид < 0, следовательно,
Q > С/0(Я1-Я2)
С ~ Ri + Т?2
>ю неравенство будет выполняться при достижении зарядом кон-игпсатора значения
После этого заряд меняться не будет.
Задача 275 (78-р)
Схема, изображенная на рис. 102, включена в сеть с переменным напряжением. В начальный момент конденсаторы разряжены, и иод идеальный. Найдите напряжения на конденсаторах в установи ищемся режиме.
Решение. Будем называть “зарядом точки А” сумму зарядов пи правой обкладке конденсатора Ci и на левой обкладке конденса-н»ра С2. Так как диод пропускает ток только в одном направлении, io после подключения схемы к сети в точке А начинает накапливаться отрицательный заряд, а потенциал точки А уменьшается. Нот переходный процесс продолжается до тех пор, пока потенциал пики А на сравняется с минимально возможным потенциалом точки 13. Начиная с этого момента ток через диод не проходит и заряд |п'1ки А не изменяется.
• Следует отметить, что при данном выборе направления падения напряжения (от точки С к точке D) под зарядом конденсатора понимается заряд его пн жней обкладки.
311
Пусть в установившемся режиме падение напряжения на кпи денсаторе С\ есть Ui (/), а на конденсаторе С2 — Тогда ими । место равенство
Ui(t) +	= Uq coswt	(11
Величины зарядов Qi(t) и C?2(t) на левых пластинах конденсатор)«в C*i и С2 определяются формулами
Q1(i) = C1C/1(<), Q2(t) = C2U2(t).
Заряд точки А постоянен и равен
Q =	+ Q2(t) = C2U2(t) - CiU^t).
Из равенств (1) и (2) находим
=
U2(t) =
—Q + C2Uq cos u>t C1+C2 Q + CiUq coswt
Ci + C2
(VI
(31
(•II
Режим будет установившимся, когда потенциал точки А стан<ч меньше потенциала точки В, т. е. {УгСО < 0- Это условие будет вы полниться при достижении заряда Q = — Cii/o- Подставляя это зни чение Q в равенства (3) и (4), находим
«.(«) =
01 + С2
„г(() =
Ь1 + 02
Рис. 102. К задаче 275. Рис. 103. К задаче 276.
Задача 276 (78-р)
Схема, изображенная на рис. 103> включена в сеть с перемен ным напряжением. В начальный момент конденсаторы разряжены, диод идеальный. Найдите напряжения на конденсаторах в устало вившемся режиме.
312
Решение. Отличие данной задачи от задачи 275 заключается .. шпке установившегося заряда Q :
Q = C1Uq.
I|h/i< тавляя это значение в формулы (3) и (4) предыдущей задачи, ипмучаем ответ
t/i(0 = UQ
—Ci 4- С2 cos си/ Ci+C2
U2(t) = tfo
C*i(l 4-coscu/) Ci+C2
Задача 277 (75-p)
Найдите энергию, запасенную в конденсаторах, и разности по-нчтиалов на их обкладках, если конденсаторы включены по схеме ни рис. 104. Сначала замыкается ключ Ki, затем замыкается и размыкается КЛЮЧ 1<2-
С С Рис. 104. К задаче 277.
Решение. После замыкания ключа напряжение на компенсаторах Ui = £/2 и запасенная в них энергия
После замыкания ключа К2 напряжение на одном из конденсаторов н его энергия равны нулю, а на другом U2 = £ и
CU% С£2 2	2	2
II результате последующего размыкания ключа К 2 никаких элек-। рических процессов в схеме не происходит.
Задача 278 (75-р)
Конденсатор емкостью С заряжен до разности потенциалов U. К нему параллельно подсоединяют незаряженный конденсатор той /ке емкости. Найдите установившуюся разность потенциалов и изменение энергии системы.
313
Решение. Так как суммарный заряд на пластинах конденса т ров остается неизменным, то установившееся напряжение Uf = U/7
До подключения второго конденсатора энергия электростатичг ского поля была равна Е = CU2/2. После подключения емкое и. увеличилась в два раза, а напряжение уменьшилось в два раза, еле довательно, новое значение энергии Е1 = CU2/4 = Е/2. Изменение энергии ДЕ1 = Е1 — Е = — Е/2 — половина первоначальной энергии выделилась в виде тепла в проводах в процессе перезарядки кон денсаторов.
Задача 279 (79-р)
В схему включены последовательно идеальный амперметр, пока зываюгций действующее значение тока, и неизвестный элемент А При включении схемы в сеть переменного тока с действующим знп чепием напряжения Ui = 100 В и частотой и = 300с"1 показании амперметра Д = 0,8 А. При включении схемы в сеть постоянного тока с напряжением U2 = ЮО В показания амперметра /2 = 1 А Предполагая, что неизвестный элемент состоит из последователь но включенных активного и реактивного сопротивлений, найди н их значения и количество теплоты, выделяющееся на элементе А’ и единицу времени в обоих случаях.
Решение. Так как при включении элемента X в цепь постоян пого тока через него протекает ток, отличный от нуля, то ясно, чти он не содержит в себе емкости и состоит из последовательно вклю ченных активного сопротивления R и катушки с индуктивностью L Эти неизвестные величины и найдем.
Применяя закон Ома для цепи переменного тока и для цепи И" стоянного тока, получаем
U1 = hVR2 + u2L2, U2 = bR-
Решая эту систему уравнений относительно неизвестных R и Е, им ходим
R =	= 100 Ом, L =	- (^} = 0,25Гн.
12	W у \ Zi J \ 12 J
Количество теплоты И7, выделяющееся на сопротивлении R в еди ницу времени, находится из закона Джоуля—Ленца
IVj = I2R = 64 Вт, Ж2 = I2R = 100 Вт.
314
Задача 280 (91-г)
В опытах Резерфорда а-частицы (ядра атома гелия) со скоро-• ii.io v = 2 • 107 м/с попадали на тонкую золотую фольгу. Некоторые из них отражались назад. Оцените радиус ядра. Число Авога-иро Na = 6 • 10 23 моль-1, элементарный заряд е = 1,6 • 10”19 Кл, молярная масса гелия р. = 4 • 10“3 кг/моль, гелий и золото имеют 2-II и 79-й номера в периодической системе элементов, электрическая постоянная sq = 8,8 • 10“12 Ф/м.
Решение. Ядро гелия, налетая на ядро золота, может остановиться и повернуть назад, если начальная кинетическая энергия ндра гелия полностью перейдет в потенциальную энергию взаимодействия с ядром золота. Оценим потенциальную энергию их взаимодействия, считая ядро гелия точечным зарядом, а ядро золота— шаром с радиусом R:
и = ^~, r> R,
ц _ 3gig2 _ 91<?2Г2
8те0/? Зтгеой3’
|де г — расстояние между центрами ядер, qi и д2— их заряды.
Максимальное значение эта энергия приобретает в центре ядра шлота:
_ 3gig2 втгсоЯ’
(1)
Приравнивая начальную кинетическую энергию ядра гелия потенциальной энергии (1), находим
R=	2>
(2)
Iде т— масса ядра гелия.
Учитывая, что т = gi = Zie, g2 = Z2e (Zi и Z2— номера i<’лия и золота в периодической системе элементов), получаем из равенства (2)
я Иа^й7.10-14м.
’ )гот результат поразил Резерфорда, так как в то время существовала модель, по которой размеры атомного ядра были сравнимы с размерами атома (1О“10 м).
315
Задача 281 (86-г)
В ванночке с водой (уровень воды h, площадь ванночки S’, Л2 •« S) образовался раствор тяжелых ионов (заряд каждого иона q, мш са — т, количество ионов N). Раствор перемешали. Опишите качг ственно и дайте количественные оценки распределения ионов в водг после установления равновесия. Как это распределение зависит ni температуры раствора?
Решение. Ионы будут отталкиваться друг от друга и стрг миться расположиться по граничным поверхностям воды, образуя две заряженные плоскости (площадь боковых стенок мала, и их мы не учитываем). Этому препятствует тепловое движение ионов, роль которого возрастает с температурой; при высокой температуре ионы распределятся более равномерно по всей высоте столба воды. Дли грубой оценки предположим, что распределение зарядов предстн вляет собой две заряженные плоскости. В равновесном состоянии пробный ион, помещенный в воду, должен находиться в покое, т, г должно быть выполнено условие
где ai, erg — поверхностные плотности зарядов на верхней и пи ж ней плоскостях. Пусть ni, П2 — число ионов на верхней и нижней плоскостях:
ni + n2 = N.	(?)
Очевидно, что cri = qni/S, аг = qn^/S. Подставив в формулу (1) miii выражения, из (1) и (2) получим
ni = N/2 - mge$S/q\ п2 = N/2 + mge^S/q2.
Задача 282 (79-г)
Имеется конденсатор емкостью С и десять аккумуляторов с ЭД(' £ = 1 В каждый. Как следует заряжать конденсатор до напряже ния U — 10 В, чтобы работа источников была минимальной? Какой КПД в оптимальном режиме зарядки? Индуктивностью, емкостью и активным сопротивлением соединительных проводов по сравнению с внутренним сопротивлением аккумуляторов можно пренебречь.
Решение. Рассмотрим сначала вспомогательную задачу. Пус | ь имеется один источник с ЭДС £ и конденсатор с емкостью С. Опре делим, какую работу совершает источник в процессе зарядки коп денсатора и какая доля этой работы переходит в энергию поля п конденсаторе, т. е. каков в данном случае КПД источника.
316
После подключения к источнику конденсатор заряжается до разности потенциалов U = £ и на его пластинах накапливается заряд:
Q = C£.
При этом в электрическом поле заряженного конденсатора запасайся энергия
WE = С£2/2.
При зарядке конденсатора источник тока совершает работу
А = Q£ = С£2 = 2WE-
Гвким образом, КПД источника в данном случае
?? = 1УеМ = 0)5.
Мы видим, что лишь половина совершенной источником работы пцст на создание поля в конденсаторе; вторая половина затрачи-нпгтся на нагревание проводов и самого источника тока.
Вернемся к основной задаче.
При зарядке конденсатора до разности потенциалов I/ = 10£ на «’го пластинах накапливается заряд
Q = CU = 10С£
и в его электрическом поле накапливается энергия
WE = CU2/2 = 50С£2.	(1)
Величины Q и We имеют фиксированные значения, не зависящие от способа зарядки. Поэтому увеличение КПД при сообщении конденсатору заданного заряда и заданной энергии можно достичь только путем уменьшения работы, совершаемой используемыми для это-Н) источниками. Путь достижения указанной цели очевиден: надо постараться не весь заряд Q пропускать через все источники. Так, первую порцию заряда величиной O,1Q можно сообщить конденса-юру, пропустив ее только через один источник тока, который при п ом совершает работу
Л, = 0,1<Э£ = СЕ2.
Вторую порцию заряда той же величины можно сообщить конденсатору, пропустив ее только через два последовательно включенных источника тока, которые при этом совершают работу
Л2 = (O,1Q)(2£) = 0,2QE = 2С£\
и т. д.
317
При предлагаемом способе зарядки источники совершают рп(и»н
ю
А = С£2 к = 55С£2.	I '.'i
А: = 1
Разделив (1) на (2), получаем следующее значение КПД:
т} = 50/55 « 0,91.
Таким образом, предложенная нами схема существенно повынн! ет КПД использования источников тока при зарядке конденсатор! m
Задача 283 (87-г)
При одновременном подключении двух вольтметров послсдонп тельно к источнику тока показание первого U\ = 6 В, вторпН' U2 = 8 В. При параллельном подключении тех же вольтметров и* показания U$ = 12 В. Определите ЭДС источника.
Решение.В общем случае закон Ома
£ = I(R + г) = IR 4- 7г,
где IR = U — полное падение напряжения во внешней цепи (пн вольтметрах), R — сопротивление внешней цепи, г— внутреннее • • » противление.
Обозначим сопротивления вольтметров Ri и Запишем зпкнн Ома для случаев последовательного и параллельного подключении вольтметров, используя величины со штрихом для первого случим и величины с двумя штрихами—для второго.
В случае последовательного подключения вольтметров падсп ш напряжения во внешней цепи равно
U1 = U. + 1/2,
следовательно,
£ = Щ 4- U2 + I'r.
А так как
Г = Ч1.=Ч1	(II
Я, й2’ получаем равенство
£=U2 + Ui fl + ^Y	('?)
318
В случае параллельного подключения вольтметров имеем
U" = и3,
• чсдовательно
г = и3 + i"r,
пли
М' + уН	(3)
ин* введено обозначение R11— сопротивление параллельно включенных вольтметров:
В!' = f1Rl .	(4)
Ri 4- R2
Перепишем равенство (3) с учетом соотношения (4):
£ = С73(1 + -2—(Я1 + Я2)).	(5)
Приравняем правые части равенств (2) и (5):
U2 4" Ui (14- — ] = t/з (1 + р р 4- R2)) >
\	/11 /	\	/4/I2	/
• нкуда с учетом (1) следует
г = С72(С72 4-	- t/з)
Я1 UiU3 + U2U3 — UiU2'
Подставляя (6) в (2), получаем
£ = U2 4- lh
+ UiU3 + U2U3 — UiU2) '
M(^4-t/i)2-t/it/2) ВД 4- U2) - UiU2
Задача 284 (87-г)
При одновременном подключении двух одинаковых вольтметров последовательно к источнику ЭДС показания каждого из вольтме-1ров Ui = 6 В. При параллельном подключении их к тому же ис-ючпику ЭДС их показания U2 = 10 В. Определите величину ЭДС.
319
Решение. Рассмотрим первый случай, когда вольтммрм соединены последовательно:
£ = /(Я+г),
где R— сопротивление цепи, г— внутреннее сопротивление иппч ника, IR = U — падение напряжения на вольтметрах. Так hiu вольтметры соединены последовательно, то общее напряжение п и* пи равно сумме их напряжений:
i/ = 2J71,
откуда
£ = 2Ui + 7г.
(И
В случае параллельного подключения вольтметров
£ = U2 + I'r,
(’Л
где Г— суммарный ток в цепи. Приравнивая (1) и (2), получаем
2t/i - U2 г =-----------
Г — I
(•'II
Легко видеть, что I = U\/Rw Г = 2U2/R, где R — сопротивлении вольтметров. Подставляя эти выражения для токов в уравнение (3) получаем
_ 2^! - и2 о r 1U2 - Ui R’ и, наконец, подставляя (4) в (1), получаем
£ = 2671 + Ui
2Щ - U2\
2U2-uJ
WJ2
2U2 - (7i
« 12,86 B.
Задача 285 (86-p)
Имеется постоянное сопротивление R = 300 Ом, резистор с пере менным сопротивлением 0 4- 1500 Ом и идеальный источник напри жения. При последовательном подключении сопротивлений к истом нику и сопротивлении резистора го = 600 Ом на нем за одну секунду выделяется количество теплоты N = 150 Дж. Существует ли cnocoli получения той же мощности при меньшем потреблении энергии oi источника? Если существует, опишите его, если нет, докажите эго
320
Решение. Ток в цепи I = £/(R 4- го), а мощность выделения и1 ii.ua на резисторе
7У=£2го/(Я + го)2.	(1)
При подключении резистора с сопротивлением гх напрямую к in ।очник}7 в цепи будет идти ток I = ZfrXy а мощность на резисторе |н||ша
* = f2/rr.	(2)
сравнивая формулы (1) и (2), видим, что для получения той же мощности N к источнику надо подключить резистор с сопротивлением тх = (?’о 4- /?)2/го = 1350 Ом. При этом вся потребляемая от иг гочника энергия выделяется на резисторе.
Найдем еще одно значение сопротивления резистора г, которое нк г требуемое значение мощности
N = £2г/(Я4-г)2.	(3)
Сравнивая выражения (1) и (3), получаем для г квадратное уравнение
r2r0 - r(R2 4- Го) 4- г0Я2 = 0.
Один корень этого уравнения г = го, а второй корень г = R2jr^ = 150 Ом, но при таком значении переменного сопротивления энергия, потребляемая от источника, больше.
Задача 286 (86-р)
Имеются последовательно включенные постоянное сопротивление R = 100 Ом, резистор с переменным сопротивлением 0 4- 300 Ом и идеальный источник напряжения. При сопротивлении резистора го — 50 Ом на нем выделяется за одну секунду количество тепло-н.| N = 50 Дж. Существует ли способ получения той же мощности при меньшем потреблении энергии от источника? Если существует, опишите его, если нет, докажите это.
Решение. Ток в цепи / = £/(/? 4- г0), а мощность выделения |гпла на резисторе
N = £2г0/(Я + го)2.	(1)
Первая бросающаяся в глаза возможность—подключить резистор прямо к источнику — к сожалению, не подходит. Действительно, при сопротивлении резистора тх ток в цепи I — а мощность на резисторе
N = £2/гх.	(2)
321
Сравнивая выражения (1) и (2), получаем значение сопротивлспп-резистора гх = (?’о + Я)2/го = 450 Ом, а это больше максимален»г сопротивления резистора.
Но есть еще одно значение сопротивления резистора г, котор» дает требуемое значение N при его последовательном соединении сопротивлением R. Действительно, сравнивая выражение (1) с Л' £2г/(г 4- Я)2, получаем для г квадратное уравнение
г2г0 - r(R2 4- Го) + го^2 = 0.
Один корень этого уравнения г = го, а второй корень г = 7?2/гц 200 Ом. При таком значении сопротивления резистора на нем им деляется в единицу времени такое же количество теплоты. Полин-сопротивление цепи R+r увеличивается в два раза, а следователып в два раза уменьшается энергия, потребляемая от источника.
Задача 287 (86-г)
Квадратная проволочная рамка вращается вокруг одной из сам их сторон с угловой скоростью w в постоянном магнитном поле И перпендикулярном оси вращения. Как изменится количество тепл»* ты, выделяющееся в рамке, если середины противоположных сторин рамки замкнуть проволокой из того же материала?
Решение. Рассмотрим два контура, на которые перемычка л« лит рамку. Поток, проходящий через рамку, также делится пополам и согласно закону электромагнитной индукции, в каждом Koiriyp возбуждается ЭДС £/2, где 8— ЭДС, которая возбуждалась в кип туре до постановки перемычки. Два образовавшихся контура один» ковы, и в каждом течет одинаковый ток. На рамке эти токи имени одно направление, а па перемычке их направления противополнт ны. А так как величины этих токов одинаковы, через перемычм ток не течет. Следовательно, количество теплоты, выделяющее!’и и рамке, не изменится при постановке перемычки. При этом все рнпни какие из противоположных сторон соединять перемычкой.
Задача 288* (86-г)
В кристалле существует выделенная ось Ох, вдоль которой при водимость равна 0*1. Это означает, что если вырезать из кристалл»! прямой провод с осью, параллельной Ох, то сопротивление таи»* го провода будет равно R =	где I и S — длина и площшп
поперечного сечения провода. Проводимость по осям Оу и Oz ран на (T2t 04 / сг2- Чему будет равна проводимость прямого пронс nui вырезанного под углом а к оси Ох?
322
Замечание. Если вектор напряженности электрического поля в »1*п< галле направлен по оси Ох, то ток течет вдоль этой оси. То же прннедливо для осей Оу и Oz.
Решение. Пусть в кристалле существует электрическое поле I’,, направленное вдоль оси Ох. Ток тогда тоже течет вдоль этой и 11усть AS— площадь основания, а Аж —высота цилиндра. Ток \/, протекающий через основания цилиндра, есть А/ = AU/R, где \lf = ЕхАх, a R = Дж/(сг1А5). Отсюда получим для плотности jx = AI/&S = (Г1ЕХ (плотность тока — это заряд, протекаю-hiiiII в единицу времени через площадку единичной площади). Ана-itn и'пю, если электрическое поле Еп направлено вдоль некоторого нгк гора п, перпендикулярного оси Ох} то ток течет вдоль этого век-itipa; соответствующая плотность тока дается формулой jn = а^Е^. II пбщем случае поле Е в кристалле представим в виде суммы двух н< ь торов:
Е = Ед; 4- Еп.
' Ь’рс'з единичную площадку, перпендикулярную оси Ож, протекает »п|< с плотностью jx = V\EXi а через единичную площадку, перпендикулярную направлению п, — ток с плотностью Jn = с^^п-
Результирующее направление тока совпадает, как нетрудно ви-•Н' гь, с направлением вектора j = jz 4-jnJ а через единичную плотинку, перпендикулярную вектору j, протекает в единицу времени •нряд j =	4- Jn- Таким образом, в анизотропном кристалле с
н|сводимостью ai ф (Уг направления тока и напряженности поля, нтб|це говоря, не совпадают.
Обратимся теперь к решению поставленной задачи. Если к концам длинного тонкого провода приложена разность потенциалов <7, in но нему течет ток
I = ^US/l.
нк’ (Tq — проводимость, которую надо определить. Ток I направлен пнпль провода; при этом вектор напряженности Е внутри провода и и правлен под углом а к его оси. Перпендикулярная к оси соста-нниющая возникает из-за того, что поверхность провода соответ-• ।кующим образом заряжается. Представим ток в виде I = joS, где /и плотность тока. Заметим, что U = 1Ео, где Ео— составляющая иск гора Е вдоль оси провода. Тогда получим, что <т0 = jo/Eq.
Нетрудно видеть, что
jx = jo COS Of,	jn = jo sin a.	(1)
Имеете с тем
jx — <TiEX}	jn = o"2 En.	(2)
323
Из равенств (1) и (2) получаем
__	. cos а _	. sin а
<71	(72
Учтем, что
Eq = Ех cos а + Еп sin а,	(I)
и подставляя в равенства (4) выражения (3), получаем после при стых алгебраических преобразований плотность тока
. _	(71(72^0
(cos а)1 2 *<72 4- (sin cv)2 cri
и проводимость
_ _________<^1^2_______
(72(cosa)2 4- <71 (sin а)2 ’
Задача 289 (81-р)
В сеть переменного тока с частотой f = 50 Гц и действующим значением напряжения U = 220 В включены последовательно ш> тивное сопротивление R = 100 Ом и индуктивность L = 1 Гн. К ши «И емкости С надо включить конденсатор вместо индуктивности, *ци бы мощность джоулева тепла в цепи не изменилась? Чему равна нн мощность?
Решение. По закону Ома для участка цепи переменного 'шн с последовательно включенными R, L и С действующее знанииm силы тока равно
Здесь w = 2тг/ — круговая частота. По закону Джоуля—Ленца ни активном сопротивлении R выделяется в единицу времени коли*и ство теплоты (мощность джоулева тепла)
n = i2r =
U2R
Чтобы при замене L на С эта величина не менялась, необходим! выполнение равенства
т 1 wL = -.
шС
Отсюда
1	1
С=-ТГ = Л 2/2г2 ^Ю-5Ф.
ш2Ь 41г2/2Ь2
324
I|i комая мощность равна
U2R
г/2 р
N =-------------—-------———-----45 Вт
R2 + 4"2/2£2	/?2 + 4TW3
Задача 290 (80-р)
От источника с напряжением U = 220 В необходимо передать потребителю в единицу времени энергию N± = 1 кВт. Какое наи-«oMibuiee сопротивление Ятах может иметь линия передачи, чтобы потери энергии не превысили 10%?
Решение. Обозначим сопротивление линии электропередачи •к рез Я, сопротивление нагрузки — Яр Полное сопротивление цепи равно Я 4- Яр и по цепи протекает ток
Я + Я1
Мощность нагрузки
Nt = 12Иг =
U2Ri
(Я+Я1)2'
(1)
Потери N в линии электропередачи определяются формулой
N = I2R =
U2R (R+Ri)2’
(2)
II । уравнений (1) и (2) получаем
N/Ni = R/Ri.
(3)
Если потери не превышают 10 %, то N/Ni < 0,1. В этом случае, как следует из (3),
Я/Л1 < п.	(4)
Наибольшему возможному сопротивлению линии 7?тах отвечает шик равенства в (4). При этом, как следует из (3), максимальному R отвечает и максимальное N, т. е.
Nmax =	= nNx.
Я1
Принимая это во внимание, вместо равенства (2) имеем
£/2Ятах (Ятах 4" Я1)2
= nNi.
(5)
325
Заменяя в равенстве (5) Ri на Ятах/л и решая получающей''и • результате подстановки уравнение относительно Ятах, находим к» комое сопротивление:
U2 п
^тах — “77“ 7 i = 40м.
М (п 4" I)2
Задача 291 (80-р)
С какой наибольшей мощностью N\ можно передать электри «и скую энергию от источника напряжения U = 220 В потребить по линии передачи с сопротивлением R = 10 Ом, если допускаемы» потери не должны превышать 10 %?
Решение. Обозначим сопротивление нагрузки через Яр IЬ и ное сопротивление цепи равно R + Ri, и по ней протекает ток
/=-^. Я + Я1
Потери мощности в линии электропередачи
у Пр U2R (R+R1)2'
Мощность нагрузки М определяется формулой
N1 1 R1 (Я + Л1)2'
Из уравнений (1) и (2) имеем
N/Nx = Я/Я1.
(’Л
Обозначим
N/Nx = п.
Тогда, как следует из (3),
Ri = R/n.
Из (1) и (5) находим
Путем исключения из системы двух равенств (4) и (6) неизвестно! о N для полезной мощности находим
U2 п
N1~~R (1 + п)2’	(/1
326
I нким образом, полезная мощность ЛГг при заданных U и R зави-нг только от п. Условие задачи требует найти наибольшее зна-юпие (М)гпах функции М(п) на промежутке 0 < п < 0,1. Для и ого исследуем функцию М(п). Прежде всего замечаем, что она при всех положительных п неотрицательна и равна нулю на конник промежутка [0,оо]. Исследование функции М(п) на экстремум показывает, что внутри промежутка (0,оо) имеется всего одна экс-||м'мальная точка, отвечающая значению п = 1. При п = 1 функция V| (п) достигает своего максимального значения. А раз так, то ясно, но при значениях п, принадлежащих промежутку (0; 0,1), функция Vi(n) достигает своего наибольшего значения (М)тах на конце это-ю промежутка, т. е. при п = 0,1. Полагая в (7) п = 0,1, находим |N| )max — 400 Вт.
Задача 292 (78-р)
Проволочный квадрат помещают в переменное магнитное поле, вектор магнитной индукции которого перпендикулярен к плоскости кпадрата и изменяется по закону В = Bq cos cut. Во сколько раз изменится количество теплоты, выделяющееся в проволоке, если и । нее сделать не квадрат, а окружность, оставляя длину проволоки неизменной?
Решение. По закону Джоуля—Ленца количество теплоты ЛГИ, выделяющееся в рамке за малое время Д/, определяется формулой
AIV = I2RAt,
। де I — сила тока в рамке, R — сопротивление рамки. Так как длины круглой и квадратной рамок одинаковы, то одинаковы и их сопротивления. Вследствие этого отношение количества тепла Д Wi и ЛГИ2, выделяемых в квадратной и круглой рамках, равно отношению квадратов протекающих в них токов:
aiVi/aw2 = (Л//2)2.	(1)
При равных сопротивлениях рамок отношение протекающих в них юков равно отношению действующих в них ЭДС:
i1/i2 = s1/s2.	(2)
Так как скорость изменения магнитной индукции в обеих случаях одинакова, то различие в действующих ЭДС объясняется только различием площадей рамок. Возникающие ЭДС Si прямо пропорциональны площадям рамок Si (г = 1,2), поэтому
fi/f2 = 5i/S2.	(3)
327
С учетом (2) и (3) равенство (1) принимает вид
AIVi/AW2 = (Si/S2)2.	(И
Пусть длина проволоки есть Z. Нетрудно показать, что плонищи изготовленных из нее кольца и квадрата даются формулами
Z2	Z2
51 = —;	52 = —.	(hl
4тг	16
Подставляя в (4) значения Si и S2 (5), окончательно получаем
ДИЛ/ДИ^ = (4/тг)2.
Итак, количество теплоты, выделяющееся в круглой рамке( i« (4/тг)2 раз больше количества теплоты, выделяющегося в квадрш ной рамке.
Задача 293 (78-р)
Имеется проволока квадратного сечения. Для того чтобы ее рп* плавить, по ней надо пропустить ток I = 10 А. Какой ток nnji" пропустить, чтобы расплавить проволоку круглого сечения с •ц»Н же площадью сечения? Количество теплоты Q, уходящее в окружи ющую среду в единицу времени, подчиняется закону Q = где S — площадь поверхности проволоки, Т — ее температура, температура окружающей среды на большом расстоянии от upoiv* локи, к — коэффициент, одинаковый в обоих случаях.
Решение. Чтобы проволока, нагретая до температуры пли вления, не плавилась и все время оставалась при этой температур! надо, чтобы количество теплоты, выделяющейся в ней при пром» ждении тока, в точности равнялось количеству теплоты, отдавш мой проволокой в окружающую среду. Уравнение теплового балnin и имеет вид
I2R= kS(T„ - Тс).
Здесь I — сила тока, необходимого для поддержания теплового (>н ланса, R — сопротивление проволоки при температуре плавлении Тп. Если сила тока будет хоть немного больше этого значения, про волока начнет плавиться.
Две проволоки, имеющие одинаковые длину Z и площадь пот речного сечения сг, но разные профили, обладают одинаковыми <••» противлениями Я, но площади Si и S2 их поверхностей различны Поэтому для поддержания теплового баланса через них надо при пускать разные токи Ц и 72. Написав для каждой проволоки cihh
328
уравнение теплового баланса и разделив затем почленно одно на пру гое, получаем
(/i//2)2 = Si/S2.	(1)
Пусть а — сторона квадрата проволоки квадратного сечения и li радиус проволоки круглого сечения. Очевидно, что
а = у/сг, Ь = а/сг/тг.
Площади поверхностей проволок подсчитываются по формулам
Si = 4al — 4ly/a, S2 = 2тг6/ = 2/х/тга.	(2)
Подставляя (2) в (1), получаем: (ЛДг)2 = 2/х/тг. Отсюда
(3)
Здесь Л и I2—токи, необходимые для поддержания теплового баланса.
Из неравенства д/тт < у/2 следует, что /2 < Л < Взяв в формуле (3) и качестве ток I = 10 А, который плавит проволоку квадратного сечения, найдем ток, который будет плавить круглую проволоку: /. = 9,4 А.
Задача 294 (77-г)
Индукция магнитного поля в циклотроне В = 1 Т. Поле постоянно и однородно. Из размещенного в центральной части межполюсного зазора источника ионов вылетает протон со скоростью Vo = 10 км/с. За один оборот он два раза проходит между ускоряющими электродами, разность потенциалов U на которых равна 1000 В. За какое время скорость протона возрастет в 1000 раз? Величина k отношения заряда протона к его массе равна 108 К л/кг.
Решение. Частица в магнитном поле, перпендикулярном ее начальной скорости, движется под действием силы F — qvB равномерно по окружности. Ее радиус вычисляется из условия а = v2//?, где а — центростремительное ускорение. Подставляя а = F/m, получаем
" qB'
Угловая скорость определяется формулой
и? — v/R = qB/m.
329
Таким образом, величина ш не зависит от скорости, а значит, одним и тем же будет время одного оборота
_ 2тг _ 2тгт w qB
(Il
Проходя между ускоряющими электродами, протон увеличили • свою энергию на qU. Соответственно за один оборот его энергия пн» растает на 2qU. Число полных оборотов протона за то время, ikuiи его скорость не достигнет заданного значения v, очевидно, равно hi ношению полного приращения энергии Е — Eq = (mv2 — mvg)/2 к < < приращению ДЕ за один оборот:
п =
Е — Eq Ш / 2	mVn
-”s) = IS7
2
- 1
(V)
Тогда искомое время равно
t = пТ.	(3|
Подставляя в (3) величины (1), (2) и учитывая, что v/vq = 101)11 получаем искомое время t ъ 1,6 • 10“ 5с.
Задача 295 (76-р)
В сеть переменного тока с действующим значением напряжении Ед = 200 В включены последовательно соединенные активное сопри тивление R = 100 Ом, индуктивное сопротивление Rl = 1000 Ом и емкостное сопротивление Rc = 1000 Ом. Найдите ток, падение ни пряжения на каждом из сопротивлений, мощность джоулева тепл»» на каждом из сопротивлений. Оцените, во сколько раз измени и и эти величины, если частота тока в сети увеличится в два раза.
Решение. Закон Ома для участка цепи переменного т<пи» с полным активным сопротивлением Я, индуктивным Rl и емми i ным Rc имеет вид
1Д= , Un	(II
УЯ2+(/?ь-Яс)2
Падения напряжения Ед, Еь, Ес на последовательно соединенпы* элементах цепи определяются формулами
Ед = /дЯ, UL = IARL, Uc = I*Rc-	(2)
Тепло в цепи выделяется только на активном сопротивлении. Дли подсчета мощности джоулева тепла служит формула
W = I2aR.	(3)
330
Подставляя данные задачи в формулы (1)—(3), получаем
= 2 A, UR = 200 В, UL = 2000 В, Uc = 2000 В, W = 400 Вт.
При увеличении частоты в два раза индуктивное сопротивление ширастает также в два раза и емкостное уменьшается в два раза. При увеличенной частоте их новые значения будут: RL = 2000 Ом и Н,. = 500 Ом. Выполняя вычисления по формулам (1)—(3) с новыми ншчениями индуктивного и емкостного сопротивлений, находим
= 0,13 A, UR = 13В, U'L = 260В, U'c = 65В, w‘ = 1,69Вт.
I иким образом,
« 15,	7^ «15,	^«7,5, к 30,	^7^ 235.
/д	UR	UL	Uc	W
Задача 296 (76-г)
В сеть с напряжением U = Uq cos art включены последовательно активное сопротивление Я, конденсатор емкостью О’, идеальный ди-• III, (сопротивление которого в одном направлении равно нулю, а в прутом— бесконечности). В момент включения заряд конденсатора ривон нулю. Найдите зависимость тока, напряжения на конденса-nipe и напряжения на диоде от времени в установившемся режиме.
Решение. Сразу же после включения цепи в сеть под напряжением U конденсатор начинает заряжаться через диод. К моменту получения конденсатором максимального заряда ток в цепи прекращается. Разряжаться конденсатор не может, так как в обратном направлении диод тока не пропускает. Таким образом, в установившемся режиме ток в цепи не течет, конденсатор заряжен до некоторого постоянного напряжения, диод закрыт.
Напряжение на диоде находится из условия
Ur + Uc + Ur = UqcosuI.	(1)
В установившемся режиме ток в цепи отсутствует и поэтому IIн — 0. Напряжение на конденсаторе сохраняет постоянное значение Uc - С учетом этого равенство (1) принимает вид
Uc + UA = i/ocosu^,
или
ид = -Uc + IZocoswt	(2)
331
Условие того, что конденсатор больше не заряжается, т. е. ток чг| диод не идет (диод “закрыт”), можно представить в виде (7Д • а Из формулы (2) следует, что это условие будет выполнено в<> и» моменты времени при минимальном значении напряжения на и пн денсаторе, равном [7q, так что напряжение на диоде
ид — -J7o(l ~ cosw/).
Задача 297 (75-р)
Сто две точки соединены попарно (каждая с каждой) одинпн выми сопротивлениями R = 10 Ом каждое. Найдите сопротивлгпн между двумя произвольными точками.
Решение. Выберем из ста двух точек две произвольны' А и В. Из соображений симметрии ясно, что остальные 100 тпчи равноправны; при подключении любого источника к точкам А и /’ потенциалы остальных точек будут одинаковыми. Поэтому вес н|« вода (а также сопротивления), соединяющие эти 100 точек момн собой, можно убрать —ток по ним все равно не идет. В резулинн получается такая схема: к точкам А и В подключены параллн'п но сто одинаковых ветвей сопротивлением 2R каждая и одна псин сопротивлением R. Общее сопротивление этой схемы:
1	_ R _ Ю п
Я°- £ + 100 - 51 - 51 Ом-
Задача 298 (74-г)
Каждое из N различных сопротивлений Ri^.^R^ подсос;инь но одним из своих концов к общей клемме Ао- Другие концы противлений подсоединены к’клеммам А1,...,Адг с потенциалами [71, ...,(7yv относительно нуля. Найдите потенциал точки Aq.
Решение. Обозначим искомый потенциал точки До w К По первому правилу Кирхгофа сумма токов, втекающих в точку I. равна сумме вытекающих, т. е.
Z> = o.	(и
/с=1
Для каждого сопротивления напишем закон Ома: Uk — Uo = R lb, Выражая отсюда Д и подставляя результат в соотношение (1), И" лучаем
~	— л
332
Ilur.iic простых преобразований находим искомую величину Uq :
N Г Uk/Rk
/с = 1
Задача 299* (76-г)
Л,на длинных вертикальных провода соединены в верхней ча-1И посредством индуктивности L. Между проводами горизонтально |нц-||оложен металлический стержень, который может, касаясь пропинов, без трения скользить под действием силы тяжести (рис. 105). Масса этого стержня т. Длина стержня и расстояние между прово-1ПМИ I. Определите характер движения стержня при существовании магнитного поля с индукцией В, направленного по нормали к системе. Сопротивлением стержня и проводов можно пренебречь.
L т
*— I —
Рис. 105. К задаче 299.
Решение. Пусть в начальный момент скорость стержня и гок в контуре равны нулю. Направим ось Ох вертикально вниз и начало отсчета на ней поместим против положения, занимаемого • гсржнем в момент t = 0. Под действием силы тяжести стержень приходит в движение. За малое время А/ стержень проходит путь Ах = х. При этом в контуре возникает ЭДС индукции
АФ _ BAS _ BlSx
St “ А/ “ А/
и ио цепи начинает течь ток. На индуктивности L падает напряже
ние как сопротивления стержня и проводов пренебрежимо малы, то U = £. Приравнивая правые части (1) и (2), получаем
BlSx _L^I
St ~ st'
333
или
Bl Ах = LAI.
На стержень с током действует сила Ампера Fa = Bl AL
Из (3) и (4) находим
Fa = B2l2x/L.
При смещении стержня от начального положения вниз сила /• д дет направлена вверх. Это следует из закона сохранения энер1нн При движении стержня вниз его потенциальная энергия частпчп' переходит в кинетическую и частично— в энергию магнитного ппчи катушки индуктивности. Это означает, что скорость падения стер»*, ня в магнитном поле меньше скорости его падения только в in ми силы тяжести. Следовательно, сила Fa совершает работу пронн* силы тяжести и направлена вверх. Для результирующей силы / действующей на стержень, имеем
Координату #0] определяемую формулой
можно трактовать как координату некоторого “равновесного” и.
жения стержня. Из формулы (5) видно, что при отклонении стер ж ня от этого положения на небольшое расстояние Ах = х — ,гц н « него действует сила F, прямо пропорциональная величине оттп» нения Ах и направленная в сторону положения “равновесия”. К«н известно, под действием силы F — — к Ах на тело массой т шпнн кают гармонические колебания с периодом
Т = 2тг\/т/к.
В нашем случае к = B2l2/L. Таким образом, стержень будп вершать около точки с координатой (6) малые гармонич<ч пн. колебания с периодом
В момент прохождения положения “равновесия” скорость стержни максимальна. Нулевую скорость он имеет в начальном положении (я = 0). Амплитуда колебаний стержня
334
Задача 300 (78-р)
('хема, изображенная на рис. 106, состоит из последовательно in. шочснных сопротивления R = 100 Ом и источника питания с
1( ’ £ — 20 В. Найдите полную мощность, потребляемую схемой, и мощность джоулева тепла, считая, что потенциал точки Л на 10 В »i| и’Восходит потенциал точки В.
Р е ш е н и е. По закону Ома для участка цепи, содержаще-источник ЭДС £, на концах которого поддерживается разность нпн'ициалов Uаву сила тока I определяется формулой
I=-ABr g = -0,lA.
Inn к минус указывает на то, что ток идет от точки В к точке А. Энергия, потребляемая цепью в единицу времени, равна
WO = UABI = -1 Вт.	(1)
Inц|< минус показывает, что энергия передается во внешнюю цепь. 1\н.п и чество теплоты, выделяющееся в единицу времени на сопроти-|| Ц’||ии R:
=/2Я= 1Вт.	(2)
Hi точник совершает в единицу времени работу
IV2 = -£/ = 2Bt.	(3)
I пкнм образом, энергия (3) источника расходуется на выделение и ила (2) и на передачу энергии во внешнюю цепь (1).
8 д	——। +J |~ cf р
11	Рис. 106. К задачам 300 и 301.
Задача 301 (78-р)
('хема, изображенная на рис. 106, состоит из последовательно пи точенных сопротивления R = 100 Ом и источника питания с ЭДС /	20 В. Найдите полную потребляемую мощность и мощность
окоулева тепла, считая, что потенциал точки А на 30 В превосходит цп|епциал точки В.
Р е ш е н и е. По аналогии с задачей 300 сила тока I в цепи ••прсделяется формулой
/ = £-z£ = Oiia.
R
Imч’ь ток идет от точки А к точке В.
335
Энергия, потребляемая цепью в единицу времени, равна
JV0 = [7ЛВ/= 3 Вт.
Количество теплоты, выделяющееся на сопротивлении R в единит времени:
1У1 = I2R = 1Вт.
Источник совершает в единицу времени работу
IV2 = —£/= —2Вт.
Знак минус указывает на то, что работа совершается против ЭД(' например идет зарядка аккумулятора.
Таким образом, мощность внешнего источника (3 Вт) расходуй ся на выделение тепла (1 Вт) и на зарядку аккумулятора (2 Вт)
Задача 302 (74-р)
Шесть одинаковых лампочек включены в цепь по Схеме, изобрн женной на рис. 107. Какая из них горит ярче всех? Какая слаГкч ' Почему?
Решение. Через лампочку 4 течет весь ток, через ламноч ки 5 и 6 — по половине тока, через лампочки 1, 2, 3 — по третьей ч« сти общего тока. Следовательно, ярче всех горит лампочка 4, слп(’н»« всех — лампочки 1, 2, 3.
Рис. 107. К задаче 302.
Задача 303 (74-р)
Пять одинаковых лампочек включают в цепь по схеме, изобрн женной на рис. 108. Какая из лампочек горит ярче всех, а как пи слабее всех? Почему?
Решение. Участок цепи с лампочками 1 и 2 имеет больна • сопротивление, чем участок с лампочками 3, 4, 5, следовательно ни» через лампочку 3 больше, чем через лампочки 1 и 2, т. е. лампочки 3 горит ярче лампочек 1 и 2, имеющих одинаковую яркость.
336
Сопротивление участка с лампочками 4, 5 меньше, чем сопро-।пиление лампочки 3, следовательно, на лампочках 4 и 5 падает меньшее напряжение, чем на лампочках 1 и 2, т. е. лампочки 4 и 5 и»рят слабее, чем лампочки 1 и 2.
Итак, ярче всех горит лампочка 3, слабее всех — лампочки 4 и 5.
Задача 304 (76-р)
В сеть постоянного тока включены последовательно сопротивление и стабилизатор напряжения. Вольт-амперная характеристика • шбилизатора приведена на рис. 109, а. Постройте график зависимости тока, проходящего через сопротивление, от напряжения во писшней сети.
в	б
Рис. 109. К задаче 304.
Решение. Обозначим через U напряжение во внешней сети, к рез Ur— падение напряжения на сопротивлении R и через Uc — нпдение напряжения на стабилизаторе. Эти величины связаны ме-л,у собой соотношением:
и = Ur + Uc
(1)
Применяя закон Ома к участку цепи, состоящему из сопротивления /I, имеем:
I = Ur/R.
(2)
II । приведенной на рис. 109, а вольт-амперной характеристики ста-ьплизатора следует, что при Uc < Uq ток / = 0 и, следовательно, Uи = 0 и Uc = U. Таким образом, при U <Uq ток в цепи отсутству-
Пусть теперь U > Uq. Так как всегда Uc < Uq} то очевидно, что и случае U > Uq на сопротивлении R падает какое-то напряжение и, следовательно, в цепи протекает ток I > 0. При этом, согласно in>льт-амперной характеристики стабилизатора, Uc — Uq. Восполь-идавшись формулой (1), для Ur находим: Ur = U — Uc = U — Uq. I|iвставляя это значение Ur в формулу (2), получаем:
U -Uq R
(3)
337
График зависимости I(U), отвечающий формуле (3), предстали н на рис. 109, б. На этом рисунке у? — arctg
Задача 305 (76-р)
В сеть постоянного тока включены последовательно соединенны, сопротивление R и стабилизатор тока. Вольт-амперная характер!* стика стабилизатора представлена на рис. ПО, а. Постройте грш|ш ки зависимости тока в цепи и падения напряжения UQ на стабнли заторе от напряжения во внешней цепи.
Рис. НО. К задаче 305.
Решение. Обозначим напряжение во внешней сети чор<’ . / и падение напряжения на сопротивлении R через Ur. Всегда мм.» । место равенство
Uc + Ur = U.	(И
Применяя закон Ома для участка цепи, содержащего только сопротивление Я, для тока I имеем:
I = Ur/R,	Pi
при этом обязательно
—/о	( О
Пусть ток в цепи удовлетворяет неравенству
0</<Zo.	(Il
Тогда, как видно из приведенной на рис. ПО, а вольт-амперпоП мн рактеристики стабилизатора, Uc = 0 и, следовательно, Ur = Я. 1н меняя в (2) Ur на (7, получаем
I = UR.	(hl
В соответствии с этим, неравенство (4) может быть написано в пнф О <	< /о, или, что то же самое,
0<U<IqR.	(«И
338
Итак, если имеет место неравенство (6), то сила тока в цепи опре-!»• ’пются формулой (5) и UQ = 0.
Пусть I = /о- Тогда, независимо от значения U, всегда Uc > 0 и I к - IqR. Заменяя в (1) Ur на правую часть последнего равенства и |н'1иая получившееся уравнение относительно t/c, находим:
Uc = U-IqR.	(7)
1пк как Uc > 0, то
и > IQR.	(8)
Ib piio и обратное: при выполнении неравенства (8) всегда / = /о и имеет место формула (7).
Аналогичные результаты получаются при рассмотрении случаев
/и < I < 0 и I = —-fo-
lia рис. ПО, б приведен график зависимости тока в цепи от напряжения сети и на рис. 110, в—график зависимости падения напряжения на стабилизаторе от напряжения в сети. При построении |рнфмков использовано обозначение Uq — IqR-
Задача 306 (76-р)
Как зависят яркости двух одинаковых ламп от положения движки реостата в схемах, показанных на рис. 111, а, б? Напряжение в • ги поддерживается постоянным. Рассмотрите крайние положения чнпжка.
Рис. 111. К задаче 306.
Решение. Лампочки горят тем ярче, чем больше проходящий •П’рез них ток.
Рассмотрим сначала случай, показанный на рис. 111, а. В крайнем левом положении движка реостата лампы включены парал-Н’льно и, следовательно, по ним идут одинаковые токи. Поэтому ।прят они с одинаковой яркостью.
При крайнем правом положении движка реостата лампочка 1 нключена параллельно цепи из последовательно включенных сопро-।явления R и лампочки 2, следовательно, через лампочку 1 идет больший ток, чем через лампочку 2, и лампочка 1 горит ярче.
Рассмотрим схему, показанную на рис. 111, б. При крайнем лепим положении движка реостата лампочка 2 вообще не горит; горит
339
только лампочка 1. При смещении движка в крайнее правое* ни г жение общее сопротивление цепи уменьшается, и поэтому силл i"i через лампочку 1 увеличивается, и она начинает гореть ярче. почка 2 при этом горит менее ярко, поскольку через нее промни только часть тока, проходящего через лампочку 1.
Задача 307 (88-р)
Заряженная частица движется в однородном магнитном ни-и индукцией В и пробивает тонкую фольгу. Траектория частицы при ведена на рис. 112, а. Определите: а) знак заряда частицы; б) в кт* направлении двигалась частица.
Рис. 112. К задаче 307.
Решение. Для пробивания фольги частица потратили чш о своей кинетической энергии, следовательно, ее скорость yMriii.iiiu лась, а из-за этого уменьшился радиус кривизны ее траектории
2
у-1	V	„	Vm
та = гл => т— = qvB =} R =	.	111
R	Bq
Таким образом, частица летела слева направо.
Знак заряда определяется направлением силы Лоренца, гкир сти и индукции магнитного поля с помощью формулы F.n = q\v  h! по правилу правого винта: векторы v, В и Ел образуют праву и»» и стему координат при д > 0 и левую систему координат при у • в Из рис. 112, б видно, что эти векторы образуют правую гисим координат, т. е. д > 0.
Задача 308 (78-г)
Какой заряд пройдет через резистор R в схеме, изображении!! и . рис. 113, а, если замкнуть ключ К?
Решение. Будем называть “зарядом точки А” суммарный ряд правой пластины конденсатора Ci и левой пластины конд»’н» • тора б?2> Аналогично “зарядом точки В” будем называть сумма | ни <н
340
ч|»|д правой пластины конденсатора CS и левой пластины конден-•пора 0'1.
Рис. 113. К задаче 308.
Исли считать, что ключ К ранее не замыкался и не размыкал-н и заряды извне в точки Л и В не переносились, то заряды этих in'irK до замыкания ключа К равны нулю. Это означает, что в на-юьпый момент пластины конденсаторов и С2, соединенные с ।никой А, несут на себе одинаковые по величине заряды q разного шика. То же самое можно сказать и о пластинах конденсаторов Ci и ( 2, соединенных с точкой В. При этом напряжения на конденса-»нрах разные:
Ui = q/Ci) U2 = q/C2>
• 'К'довательно, потенциалы <ра и рв точек А и В оказываются раз-ш'|ными. Принимая потенциал точки D равным нулю, имеем
Ч>А = U\ - q/Ci, <РВ =U2 = q/C2-
При замыкании ключа К через сопротивление R начинает течь и ж в направлении от точки А к точке В, если Ci < С2 (рл > ^в)-1ик существует до тех пор, пока потенциалы этих точек не сравняйся. После установления электрического равновесия схему можно представить в виде, изображенном на рис. ИЗ, б. Из симметрии этой • хсмы следует, что
РА = фВ = ^/2,
। напряжение на всех конденсаторах одинаково U = £/2. Учитывая »и>, для зарядов qi и q2 конденсаторов Ci и С2 находим:
qi = CiU = Ci£/2, q2 = C2U = С2£/2.
Определим заряд точки В. На правой пластине конденсатора С2 п’нерь заряд 4-92? на левой пластине конденсатора С\ —заряд — qi. Суммарный заряд qB точки В равен
<7в = <72 - <71 = (С*2 — Ci)£/2.
341
Так как до замыкания ключа заряд точки В был равен нулю. • именно заряд цв и прошел через сопротивление R после зам ыь пн и ключа.
Задача 309 (81-г)
Две одинаковые катушки имеют общий сердечник и соедит н* с двумя одинаковыми конденсаторами по схеме, изображеппнП и рис. 114, а. Какие электрические колебания возможны в такой ю стеме? Емкость конденсатора С, индуктивность отдельной ктин ки Z, активное сопротивление катушек и соединительных пропин.. пренебрежимо малы.
Рис. 114. К задаче 309.
Решение. Пусть в некоторый момент t через катушки п|м. текают токи, мгновенные значения которых Ii(t) и а на кон денсаторах сосредоточены заряды Qi(t) и Qztt). Для определении сти эти токи и заряды будем считать положительными, если опн имеют направления и полярности, показанные на рис. 114, а. При тивоположным направлениям тока и полярностям отвечают их hi рицательные значения. Мгновенные значения магнитных поток» и» пронизывающих катушки, обозначим через Ф1 и Фг.
В любой момент времени падения напряжения Uli и Ui2 иа м тушках и падения напряжения Uci и Uc2 на подсоединенных пн раллельно к ним конденсаторах равны, т. е. имеют место равенс им
Ul\=Uci, Ul2 = Uc2-	(И
Так как
П d$1	ГТ - d$2	ГТ - <51	ТТ -
UL1 = -~dT'	UL2-—dT'	Uci~~C’	Uc2~~C'
342
н» равенства (1) принимают вид
ЛФ1 _ Qi d$2 _ Q2	/п\
dt ~ С ’ dt ~ С '	U
катушки одинаковы и намотаны на общем сердечнике, поэтому пронизывающие их потоки Ф1 и Ф2 равны по абсолютной величине:
Ф2 = ±Ф1,	(3)
Если обе катушки намотаны в одну сторону, то в равенстве (3) знак п люс, в противном случае знак минус.
С учетом равенства (3) из формул (2) получаем, что
<31 = ±(?2.	(4)
Из рис. 114, а видно, что положительным значениям токов Д и /ч отвечают положительные приращения зарядов dQi и dQ2 , т. е.
Имея в виду равенство (4), из (5) находим, что
Л = ±/2.	(6)
Потоки магнитной индукции Ф! и Ф2, создаваемые каждой катушкой по отдельности, определяются формулами
Ф^^Л, Ф2 = М2.
Потоки Ф1 и Ф2, пронизывающие катушки, являются алгебраическими суммами величин Фх и Ф2:
фх = ±ф2 = ф' ± ф'2 =	± Д) = ^LIi = ±2Ы2.	(7)
С учетом соотношений (7) равенства (2) принимают вид
-2^ = 21,	-2^ = 2?.	(8)
Дифференцируя равенства (9) по t и принимая во внимание равенства (5), нетрудно получить уравнения
d2h   h	d2I3   h_
dt2 ~ 2LC’	dt2 ~ 2LC
343
Каждое уравнение описывает колебательный процесс; их репнчпт имеют вид
Л = /01 COS (ivt +	,	/2 = /02 COS (о>/ + ^>2) •
Здесьw = 1/л/2ЛС — круговая частота, /01 и /02—амплитуд,им» значения токов, и —начальные фазы. Ясно, что амплитуднм» значения токов в катушках совпадают: Zqi = /02, а фазы или и» впадают (у?2 = для случая одинаковой намотки катушек), ii'in различаются на тг (^2 =	+ тг для случая противоположной ни
мотки катушек).
Итак, электрические колебания в этой системе можно пр<*д< iu вить в виде суммы колебаний в двух одинаковых контурах с “ >»|« фективной” индуктивностью 2L и емкостью С каждый, совершнн» щихся или в “фазе” (рис. 114, б), или в “противофазе” (рис. Ill о).
Задача 310“ (88-г)
Катушка изготовлена из проволоки с малым омическим и» противлением, намотанной на длинный цилиндрический сердспин-с очень большой магнитной проницаемостью. К точкам А и I (рис. 115, а) подключен источник синусоидального напряжении • амплитудой Um = 100 В.
Рис. 115. К задаче 310.
а)	Определите амплитуду напряжения между точками А и В
б)	Между точками А и В включают резистор с сопротивлением R = 10 кОм. Найдите амплитуду тока, протекающего через истом ник напряжения.
Точка С находится точно посередине катушки (одинаковое чиси» витков между точками А и С и между точками В и С). Индуктпм ность катушки L. Внутренним сопротивлением катушки пренебрсп.
Р е ш е н и е. а) Обозначим напряжение между точками А и ( как Uq, между точками С и В как Uy. Очевидно, что Uab = % •+
344
Но условию в катушку вставлен сердечник с очень большой магнитной проницаемостью, следовательно поток магнитной индукции н катушке будет один и тот же на всем ее протяжении, т. е. потоки Ф1 через нижнюю часть катушки и Фо через ее верхнюю часть одинаковы. Из равенства потоков и закона индукции U = следует равенство напряжений: Uq — U\. Таким образом, Uab — %Uo с амплитудным значением 2Um.
б) Обозначим величины и направления токов в различных участках цепи, как показано на рис. 115, б.
Поток магнитной индукции, создаваемый нижней частью катушки, равен IiL/2} а поток, создаваемый верхней частью, равен hL/2^ поэтому
_ L(dh dl2\
UAB~~2\lt+lt)-	(1)
Кроме того, Uab = Uq + Ui = 2f7o > откуда
I2=UAb/R = 1Uo/R'	(2)
По закону Кирхгофа /2 =	+ Л, или Д = -Iq + 2U$/R. Из (1) и (2)
следует
<3)
2 у at гС clt ]
Перепишем равенство (3) в виде
d7o = 4t/o 4rf£o	f4x
dt L + R dt ’	U
Представляя в (4) Uq в виде Um sin wt, получаем
din	4 rT	4wrr	,
-j- = -UmSinwt + —UmCOSUt.	(5)
at	L	rt
Решая дифференциальное уравнение (5), находим ток в установившемся режиме
4	4
/о = - — Umcosut 4- —[7msinwZ,
Lu	R
амплитудное значение которого 1т равно
г — AU™ J-1-----
m R2 + (М2‘
345
Задача 311 (74-г)
К клеммам А и В (рис. 116) прикладывают разность потенции лов Ду> так, что один раз <рА > а второй раз <рв > Ра- Найди р отношение мощностей тепловыделения в первом и во втором случи ях. Характеристики диода считать идеальными, Ri > R?.
D
с	Рис. 116. К задаче 311.
Решение. Сначала рассмотрим случай, когда ср а > Рв- гГмк как	, то при закрытом диоде на резисторе Ri будет бблышч
напряжение и потенциал точки С будет больше потенциала точки D. Напряжение для диода будет прямым, и он будет пропуски и. ток, т. е. потенциалы точек С и D будут одинаковыми.
Количество теплоты, выделяющееся в цепи в единицу времени определяется законом Джоуля—Ленца W = I2Ry который можпе переписать в удобном для данной задачи виде:
W = U2/R,
где U— разность потенциалов Д^>. Сопротивление R всей схемы равно
д Ri “И R?
а мощность тепловыделения в этом случае
-	2Я,Я,	
В случае, когда рв > Ра> потенциал точки D больше потенциала точки С. Напряжение для диода будет обратным, и ток он пропуг кать не будет. Сопротивление схемы в этом случае R = (Л1 + Яо)/У Мощность тепловыделения в даннохМ случае равна
Для отношения мощностей получаем
I-h = !72(Я1 + Л2)(Я1 + Я2) = (7?1 + Я2)2
W2 ~ 2RiR22U2	4/?j R2
346
Задача 312 (79-р)
Схема состоит из четырех сопротивлений и амперметра, вну-грсннее сопротивление которого равно нулю (рис. 117). Клеммы А и В подключены к сети с напряжением U — 100 В, Ri = 50 Ом, !{•> = 10 Ом. Найдите ток, текущий через амперметр.
Рис. 117. К задаче 312.
Решение. Обозначим через /д величину тока, текущего через амперметр. По закону Ома напряжение Ucd между точками С и D схемы дается формулой
Ucd = IaRa-
Поскольку сопротивление идеального амперметра Яд = 0, то Ucd = 0, значит точки С и D находятся при одинаковых потенциалах. Вследствие этого
Uac = Uad	и	Ucb = Udb<	(1)
Из симметрии схемы следует, что
Uac = Udb	и	Uad — Ucb-	(2)
Сравнивая (1) и (2), получаем
илс = UCB = UAD = UDB = U/2.
Применяя закон Ома к участкам цепи АС и СВ, находим токи 1\ и Д, протекающие через эти участки:
h =
Uac Ri
U 2Л1
= 1А, 12 =
Ucb <7
Л о	2/?2
Так как сумма токов, притекающих к точке С, должна быть равна сумме вытекающих токов, то Д + /д = Д- Отсюда
/А = /2_/1 = ^(±
2 \п,2
1 \
R1)
= 4 А.
347
Задача 313 (82-р)
Найдите сопротивление изображенной на рис. 118 цепочки, » • ставленной из бесконечно большого числа одинаковых звеньев
Решение. Основой решения большинства задач с бескопечiii.i ми цепочками однородных элементов служит следующее сообрпж» ние: добавление к бесконечной цепочке еще одного звена не измени» i ее параметров.
Рис. 118. К задаче 313.
Заменим приведенную на рис. 118 схему другой, ей эквивалсчи ной. Заметим, что часть схемы справа от точек С и D тождествен ih« исходной. Обозначив искомое сопротивление всей схемы Rx> зим» ним бесконечную цепочку сопротивлений справа от точек Си I) ни одно сопротивление величиной Rx. Искомое сопротивление можни теперь записать в виде
Rx — R-\—i-----j—.
ая +
После несложных алгебраических преобразований находим:
Rx = ЗЯ.
Задача 314 (80-р)
Определите показания приборов в схеме, изображенной нм рис. 119. На клеммы А и В подано напряжение U = 120 В. Buy треннее сопротивление амперметра Яд = 0,5 Ом, сопротивлении вольтметров Ry = 2000 Ом, Ri = 10 Ом, Я? = 30 Ом. Точное н< измерения показаний составляет для вольтметров = 1 В, дли амперметра А/ = 0,005 А.
Рис. 119. К задаче 314.
348
Решение. Определим сначала показания приборов, пренебре-।пн сопротивлением амперметра, а затем оценим поправку, которую иtiдо внести с учетом его сопротивления.
При нулевом сопротивлении амперметра полное сопротивление 1нчп1 складывается из сопротивлений двух последовательно соединенных блоков, каждый из которых состоит из двух параллельно шгдиненных сопротивлений Ry и R. или Т?2- Блоки обладают сопротивлениями
/ _ RiRy I _ Д2Ду
1 “ Ri + Лу ’	2 “ Л2 + Лу '
(1)
Напряжения Ui и U2 па блоках, с одной стороны, пропорциональны их сопротивлениям, т. е.
г другой стороны, их сумма равна поданному на клеммы напряжению U:
Ui 4- U2 = U.	(3)
Решая систему, составленную из уравнений (2) и (3), относительно неизвестных Ui и U2 и используя равенства (1), для J7i и U2 находим
Ri (R2 + Ry)U (Д1 4" Д2)Ду 4“ 2Я1Д
Д2(Д! 4- Ry)U (Д1 4" R2)Ry 4- 27?i7?2
Разность величин токов Л = Ui/Ri и I2 = U2/R2 дает величину тока /д, протекающего через амперметр с нулевым сопротивлением:
1А = Д - 12 =	« 0,03 А.
(Л1 4- /С2)/Су 4-
Через реальный амперметр с конечным сопротивлением Да Должен течь меньший ток /А, поэтому на амперметре будет падать напряжение
(7а — Ла Да < Ла Да = 0,015 В.
Обозначим реальные падения напряжения на вольтметрах и сопротивлениях Ri и R2 соответственно 17V1, [/V2,	U2. Очевидно, что
U. + UA 4- Uy2 = Uy. -UA + U'2 = U.	(4)
349
В случае же идеального амперметра имеем
U1+Uv2 = Uv1+U2 = U.	(Г))
Из сравнения равенств (4) и (5) следует, что замена идеальною амперметра на реальный приводит к изменениям падения напряжг пия па вольтметрах, не превышающим Ua « 0,015 В, т, е. меньшим точности измерения вольтметров. Итак, с точностью до 1 В пока*in пия вольтметров: (7V1 = 30 В и (7V2 = 90 В.
Пе больше, чем на величину (7а, изменяются падения напряжг пия па сопротивлениях 7?i и что приводит к изменениям токи через них, меньшим, чем
ДЛ = Ua/Ri к 0,0015 А, Д/2 = С/а/^2 « 0,0005 А.
Это означает, что замена идеального амперметра на реальный при водит к изменению тока через пего па величину, меньшую Д/а 0,002 Л, которая, в свою очередь, меньше точности показаний ам перметра. В связи с этим очевидно, что в пределах точности измг рений амперметр будет показывать, что через него протекает тон 4 = 0,03 А.
Задача 315 (77-р)
В представленной на рис. 120 схеме конденсаторы имеют заряд Q каждый. После замыкания ключа ki в схеме начинаются незн тухающие колебания. В тот момент, когда заряд на конденсаторах равен нулю, замыкают ключ к2. Как в результате этого изменятся амплитудные значения зарядов на конденсаторах и тока в контуре?
Рис. 120. К задаче 315.
Решение. Заряженные конденсаторы обладают энергией
Wi = Ci Uf/2, W2 = CiUl/2.
(1)
Здесь (7i, U2— напряжения на конденсаторах с емкостями C*i и С2 соответственно. Напряжение U на любом конденсаторе связано с его
350
шрядом Q и емкостью С общей формулой Q — CU. В соответствии < этим в нашем случае
Ui=Q/C\, U2=Q/C2.
(2)
Подставляя в (1) вместо U\ и U2 выражения (2), получаем
Q2
2CV
W-> - —. "	2С2
Полная энергия W, запасенная в обоих конденсаторах перед включением их с помощью ключа ki в состав контура, определяется формулой
w = Wi + iv2 =	+ А-У
Z \С1 С2 /
В процессе возникающих после замыкания ключа ki колебаний н момент, когда заряд па конденсаторах становится равным нулю, нся энергия заключена в магнитном поле катушки и равна LI2/2, где L — индуктивность катушки и I — амплитудное значение тока. В силу закона сохранения энергии электромагнитного поля
LZ2 _ Q2 J_\
2 “ 2 \Ci + Сг/ ’
(3)
Замыкание ключа к2 в момент отсутствия заряда на конденсаторах не меняет энергию системы (изменяется только емкость контура и связанная с нею частота колебаний). Энергия магнитного поля в катушке и ток через катушку не меняются, следовательно амплитудное значение тока остается прежним. Разность потенциалов и заряд на обкладках конденсатора С2 после замыкания ключа 1<2 останутся равными нулю.
Через четверть периода после замыкания ключа к2 вся электромагнитная энергия контура будет сосредоточена в электрическом поле конденсатора Ci, который приобретет некоторый заряд Q*. По закону сохранения энергии
LI2 _ Q"2
2 “ 2С1 ’
(4)
Так как левые части равенств (3) и (4) одинаковы, то их правые части должны быть равны, т. е.
Q' = QV1 + Ci/C2.
Таково амплитудное значение заряда конденсатора с емкостью С\ после замыкания ключа к2 (если С\ = С2, то Q* = \/2Q).
351
Задача 316 (87-р)
В колебательном контуре с индуктивностью L = 1 Гн и емкостью С = 100 мкФ возбуждены колебания. Каждый раз, когда заряд ни конденсаторе максимален (независимо от знака), к колебательному контуру на время Д/ — 0,001 с присоединяют сопротивление Н 1000 Ом (рис. 121). Опишите процессы, которые будут происходить в системе.
Р е ш е н и е. Период собственных колебаний контура Т 2tt\/LC 0,06с	Д/, поэтому подключение сопротивления при
водит в “скачкообразному” изменению напряжения Uo на конденсп торе на величину Д(7. Изменение заряда на его обкладках за время Д/ оцепим, считая ток разрядки постоянным:
Дд = СДУ - ^ДУ
R
Отсюда находим относительное изменение напряжения:
ДУ	Л
— = _сз0,01«1.
Поскольку за один период сопротивление подключают два раза, то амплитуда колебаний напряжения на конденсаторе за период уменьшается примерно на 2%. Таким образом, в контуре происходя i довольно слабо затухающие колебания.
Рис. 121. К задачам 316 и 317.
Задача 317 (87-р)
В колебательном контуре с индуктивностью L = 1 Гн и емкостью С — 2 • 10“° Ф возбуждены колебания. Каждый раз, когда заряд ни конденсаторе максимален (независимо от знака), к колебательному контуру на время Д/ = 10~3 с присоединяют сопротивление Л ~ 50 Ом (рис. 121). Опишите процессы, которые будут происходить ь системе.
Р е ш е н и е. Период собственных колебаний контура Т -2я\/ЬС « 0,03с	Д/, поэтому подключение сопротивления при
водит к “скачкообразному” падению напряжения на конденсаторе.
352
Оценивая изменение заряда на обкладках конденсатора, как в задаче 316 (считая ток разрядки постоянным), получаем относительное изменение напряжения
ЛЦ _ А/ UQ ~ RC
'Такая большая величина этого отношения говорит о том, что сделанное нами приближение несправедливо (ток разрядки нельзя считать постоянным). Но на основании этой оценки можно утверждать, что “скачкообразное” изменение напряжения Ли — величина того же порядка, что и начальное напряжение на конденсаторе 1/q. Поэтому колебания в контуре сильно затухают — они практически прекратятся уже после нескольких подключений резистора.
Задача 318 (86-г)
В схеме, изображенной на рис. 122, металлический шар А соединен проводом с электрической цепью, заряд на шаре равен q. Каким будет заряд на шаре, если поменять местами сопротивления «1 и Я2?
Ла
Рис. 122. К задаче 318.
Решение. Примем потенциал “земли” равным нулю. Тогда заряд на шаре есть q = (7(7,.где U — разность потенциалов между шаром и “землей”, С — емкость шара.
От перестановки сопротивлений ток I в цепи не изменится, а падение напряжения на участке между “землей” и шаром изменится от величины U = IR2 до U = IRi. При этом заряд на шаре станет равным q = CU .
Собирая вместе приведенные выше формулы, получаем
Задача 319 (87-р)
В схеме, изображенной на рис. 123, определите величину и направление тока во всех резисторах. Внутренним сопротивлением источников ЭДС пренебречь.
353
Решение. Через нижний резистор R ток течет влево, н величина I = S/R. Через резистор 2R ток не течет. Через рсзш |»<| R ток течет влево, его величина I = £/R,
Рис. 123. К задаче 319.
Рис. 124. К задаче 320.
Задача 320 (87-р)
В схеме, изображенной на рис. 124, определите величину и ин правление тока во всех резисторах. Внутренним сопротивлением н> точников ЭДС пренебречь.
Решение. Через верхний резистор R ток течет влево, » о величина I = £/(2R). Через нижний резистор R ток течет впрввм его величина I = £/(2Я). В резисторе 2R тока нет.
Задача 321 (75-р)
Пять одинаковых лампочек включены по схеме на рис. 12b, и Какие лампочки горят ярче, какие слабее? Как изменится яркое и лампочек, если вывернуть: а) лампочку 5; б) лампочку 1.
Рис. 125. К задаче 321.
Решение. В первоначальной схеме лампочка 5 не гори i. так как напряжение на ней всегда равно нулю. Если ее вывернуть, ничего не изменится. Если вывернуть лампочку 1, то схема примем вид, изображенный на рис. 125, б.
В исходной схеме напряжение на лампочках 1, 2, 3, 4 одинаков» • и равно (7/2, где U — подаваемое на схему напряжение. В схеме* пи рис. 125, б напряжение на лампе 3 больше, чем (7/2, а на лампе 4 меньше,чем (7/2. Следовательно, лампа 3 будет гореть ярче, чем раньше, а лампа 4 — слабее. Лампы 2 и 5 будут гореть с одинаковой яркостью, меньшей, чем яркость лампы 4.
354
Задача 322 (75-р)
Семь одинаковых сопротивлений соединены так, как показано пп рис. 126, а. Найдите сопротивление между точками А и В, если Н -- 1 Ом.
Рис. 126. К задаче 322.
Решение. Перерисуем схему в виде, изображенном на рис. 126, б. Напряжение между точками С и D равно нулю, поэтому если разомкнуть соединяющий эти точки проводник, то никаких гцектрических изменений в схеме не произойдет. Сама схема примет при этом вид простого параллельно-последовательного соединения. ('оиротивление между точками А и В схемы равно
о	1	8п 8 А
Вав — 1	1	— _ Л — _ Ом.
2R + 2Я+|Н	'	'
Задача 323 (79-р)
Схема, изображенная на рис. 127, состоит из четырех сопротивлений и вольтметра с бесконечно большим сопротивлением. Клеммы А и В подключены к сети с напряжением U = 180 В, = 10 Ом, Ц2 = 50 Ом. Определите показание вольтметра.
Рис. 127. К задаче 323.
Решение. Так как сопротивление вольтметра бесконечно велико, ток по нему не течет, и напряжение между точками С и D такое же, как и при неприсоединенном вольтметре.
355
Через все сопротивления протекает одинаковый ток
Ri + R-2
Применяя закон Ома для напряжений Uac и Uad> существуюиш* между точками Л и Си точками А и D схемы, находим
Uac = IRi =
URX
Ri 4" R2
Uad = IR'i =
UR2
Ri + R2
(H
Вместе с тем
Uac = <Pa ~ <Pc, Uad — ^a — ^d,
Pl
где tpA j 4>d — соответственно потенциалы точек А, С и D, Искомое напряжение Ucd> которое показывает вольтмнр
можно представить в виде
Ucd = ч>с ~ 4>d-
Вычитая почленно из второго равенства (2) первое и принимая н« внимание равенства (1), получаем
Ucd = UAD - UAC =	= 120 B’
Я 2 4“ Jti
Задача 324 (80-г)
Имеется воздушный конденсатор с вертикальными плоско и я раллельными пластинами, расположенными друг от друга на pin стоянии d, много меньшим линейных размеров пластин, между к*• торыми поддерживается постоянная разность потенциалов [/о. Мн ленький металлический шарик массой т подвешен на непроводяш» II нити посередине между пластинами (рис. 128). Длина нити I мши" больше d. Шарик отклоняют от положения равновесия до сопри косновения с одной из пластин и отпускают. При соприкосновении с пластиной его заряд становится равным д$. Найдите средний ни* через конденсатор при условии, что удары шарика о пластины нН солютно неупругие, потенциалы шарика и пластины за время удпрн успевают сравняться, время удара много меньше времени движ» ния шарика между пластинами. Считать, что величина mgd/(qidfu) не очень велика.
356
ипптппп.
Рис. 128. К задаче 324.
Решение. Наибольший угол отклонения от вертикали атах находится из равенства
Чэ amax —	•
Так как, по условию задачи, величина отношения d/l «С 1, то с достаточной степенью точности
d
О'max ~ 8Ш Qmax	tg Ct max —
Ввиду малости угла атах справедливо допущение о том, что движение шарика осуществляется в горизонтальной плоскости.
Покажем, что для определения параметров движения шарика цостаточно учитывать только силу, действующую на него со стороны электрического поля конденсатора:
р _ и0 Г ЭЛ —
По II закону Ньютона уравнение движения шарика имеет вид
та = Еэл + Т 4-mg,	(1)
1дс Т — сила натяжения нити. Учтем, что из-за малости угла а горизонтальная Тх и вертикальная Ту составляющие силы Т определяются формулами
Тх = Тsin а Та, Ту =Тcos
Проецируя уравнение (1) на вертикальную ось получаем Ту = ту « Т, так как считаем, что движение происходит только в гори-юптальной плоскости. Проецируя же уравнение (1) на горизонтальную ось, получаем
Uq । Uq max - qQ— 4-Tr « qQ — 4-Та, a	a
357
или
qoUQ q0UQ ’
Так как amax = d/(2l) <C 1, то вторым слагаемым в правой чш н» уравнения движения (2) можно пренебречь?
С учетом сделанного приближения
/7о ax~q°md-
Итак, движение шарика между пластинами равноускоренное, ни этому
_ axt2 _ д0Цр12
Х 2 2md
Отсюда находим время одного прохода шарика между пластин ими конденсатора (в конце прохода х = d):
t- l2md2
~ V ах ~ У QqUq *
Таким образом,	_____
, _ Qo _ / 9pUo t V 2md2’
Задача 325* (76-г)
Две заряженные частицы имели первоначально одинаковые И" величине и направлению скорости. После того как на некоторн время было включено однородное электростатическое поле век тр скорости одной из частиц повернулся на 60°, а вектор скорейm другой — на 90°. Величина скорости первой частицы уменьшила»i вдвое. Во сколько раз изменилась величина скорости второй час i и цы? Определите отношение заряда к массе второй частицы, если для первой это отношение равно ki.
Решение. Будем считать, что частицы находятся далек»» друг от друга, а потому друг с другом не взаимодействуют. Тогл»» поворот вектора скорости и изменение его по модулю обуслопло ны только однородным электростатическим полем. Таким обрачом изменение импульса частицы равно импульсу силы, равной прои» ведению заряда на напряженность:
ДР = ЕД/ = Eq Д/.	(I)
* Это можно сделать, если величина отношения mgd/(qQUo)	'»»•'
соответствует условию задачи. В противном случае надо использовать точно» решение уравнения (2).
358
Направим ось х вдоль начальной скорости, а ось у вдоль скорости агорой частицы (рис. 129).
Рис. 129. К задаче 325.
Проецируя равенство (1) на оси х и у, получаем
Рх “ Ръх = ExqM, Ру - Роу = EyqAt.
Для первой частицы проекция (1) на ось я:
Р1х - Рат = mi у cos 60° - mit»0 =	--v0 = ExAtki]
для второй:
P2x - Рох = 0 - m2vQ = ExAtq2, -v0 = Ex&tk2,
где fci = gi/mi, a k2 = q2/m2. Отсюда для к2 получаем:
3	4
—Ex&tk2 = ExAtkit k2 = 4	О
Для первой частицы проекция (1) на ось у
Ply ~ Роу = mi —• sin 60° — 0 = EyAtqi, -	= EyAiki;
Z	4
для второй:
P2y - Роу = пг21>2 = EyAiq2, v2 = ЕуДЛк2 = V°.^ к2 = v°^\ 4A?i	о
Таким образом, скорость второй частицы уменьшилась в \/3 раз.
Задача 326 (74-г)
Некоторая область в вакууме заполнена однородными постоянными электрическими и магнитными полями с напряженностью Е и В соответственно, причем Е ± В. С какой скоростью v должна лететь частица с зарядом q и массой т, чтобы вектор скорости не менялся при полете в этом поле?
359
Решение. На частицу со стороны электрического поля jHI ствует сила
Ь'эл —
а со стороны магнитного поля — сила
Fm = tf[v х В].
Постоянство вектора скорости эквивалентно отсутствию ускорении т. е.
Гэл + FM = О, или
Е + [v х В] = 0.	(11
В проекциях па оси координат ж, z равенство (1) имеет вид
Ех 4- vyB2 -vzBy —	0,	(','1
Еу 4- vzBx - vxBz =	0,	(.И
Ez 4" vxBy — VyBx —	0.	(I)
Пусть вектор E параллелен оси z, вектор В	параллелен оси х. При
этом система уравнений (2)-(4) принимает вид
vzB = 0,	(Ь|
VyB = Е.	(П|
Решение системы уравнений (5)—(6) относительно компонент сю» рости:
vz — 0, vy = Е/В} vx —любая.
Дополнительное условие: 4- Vy < с2, где с — скорость света, т. v2 < с2 — Е2/В2, а задача в целом имеет решение только при иы полнении условия Е/В < с.
Задана 327* (87-г)
Заряженный шарик привязан к одному из концов невесомой им растяжимой нити. Другой конец нити закреплен. Шарик движет» п по окружности в плоскости, перпендикулярной магнитному полю (нить тоже находится в этой плоскости). Частота вращения таю» ва, что сила натяжения нити равна нулю. Как изменится часто и» вращения шарика, если индукцию поля медленно уменьшить в три раза? Тяготение отсутствует.
360
Решение. Поскольку в исходном состоянии сила натяжения ниги равна нулю, то шарик вращается по окружности радиусом I |олько под действием силы со стороны магнитного поля:
= q^olB,
пли
mwo — qB
(cvo — исходная частота вращения; qt т — заряд и масса шарика; I — длина нити).
Направление вращения шарика таково, что его собственное магнитное поле уменьшает полный магнитный поток через круг радиусом I. При изменении индукции внешнего магнитного поля возникает вихревое электрическое поле. Это поле направлено так, чтобы изменение скорости частицы порождало изменение собственного магнитного поля, препятствующее изменению полного магнитного потока (правило Ленца). В нашем случае это означает, что при уменьшении индукции магнитного поля частота вращения шарика будет уменьшаться.
Если поле меняется медленно, то приращение кинетической шергии SK шарика за малое время St есть SK = — qSwStгде f — ЭДС индукции в контуре с радиусом I:
<5Ф _ nl2SB St ~ “ St *
Вместе с тем, изменение кинетической энергии есть
SK = ^.m[(cj + 5w)2 — w2]/2 = ml2w(t)Sw.
(' учетом этого получаем
Это же соотношение справедливо и для конечных (не малых) приращений частоты До; и индукции ДВ (поскольку коэффициенты перед Scj и SB в формуле (1) постоянны и не зависят от времени):
Ди = дДВ/(2т).
Если поле уменьшить втрое, то
ДВ = —2В/3, Да; = —(?B/(3m) = -и?о/3,
т. е. частота вращения уменьшится на одну треть.
361
Задача 328 (77-г)
Плоскость тонкого металлического кольца радиусом г, лежали' го на гладком столе, перпендикулярна силовым линиям постоянно! и однородного магнитного поля с индукцией В. По кольцу протекай постоянный ток, величина I которого достаточно мала. Найдите ш* личину сил натяжения, действующих в кольце.
Решение. На участок проводника с током, находящегося и магнитном поле, действует сила
AF = /[AlxB].
В нашем случае А1±В (рис. 130), поэтому
&F = IBM.	(I)
Сила AF направлена вдоль радиуса кольца наружу, если вектор магнитной индукции В входит в плоскость чертежа. Для опредо ленности рассмотрим именно этот случай.
Кроме силы AF на элемент кольца А/ действуют силы натяжг ния Ti и Тз, направленные по касательным к кольцу. Силой жг, действующей со стороны магнитного поля, создаваемого током п кольце, пренебрегаем, потому что индукция В этого поля пропор циональна току /, который по условию достаточно мал. Так кии элемент кольца находится в равновесии, то можно написать у pan нения
AF-(Ti4-T2)sin^ = 0,	(7)
(71-T2)cos/? = 0,	(3)
где — угол, под которым из центра кольца О видна половина »л<> мента Д/. Из системы уравнений (2), (3) следует равенство
ДГ = 271sin /3,	(-1)
362
где Т = Т\ — ?2—искомая сила натяжения в кольце. При А/ —> О выполняется $ —> О, поэтому
sin/?«/? = Д//(2г).	(5)
Подставляя выражения (1) и (5) в формулу (4), получаем величину искомой силы натяжения в кольце
Если же AF направлена по радиусу вовнутрь кольца, то изменятся направления всех сил. В том числе Т станет теперь силой сжатия, но модуль ее останется прежним.
Задача 329** (83-г)
В эксперименте, исследующем зависимость массы частицы от ее скорости, узкий пучок электронов с разными скоростями v пролетает вдоль оси у через область длиной /, в которой на него действуют постоянные электрическое и магнитное поля. Вектор напряженности электрического поля направлен против оси х, а вектор индукции магнитного поля — по оси х. Электроны регистрируют с помощью люминесцирующего экрана, нормального к оси пучка и расположенного на расстоянии L (L /) от области действия полей. Определите форму кривой, которую увидит экспериментатор на экране.
Решение. Обозначим массу электрона т, величину его заряда е и скорость, с которой он движется вдоль оси у до входа в область существования полей, vy. Так как силы, действующие на электрон со стороны обоих полей, направлены перпендикулярно к первоначальному направлению движения, то изменить величину составляющей скорости на ось у они не могут. Область существования нолей электрон проходит за время
Д/1 = 1/Vy}
а оставшееся расстояние L до экрана — за время
Д/2 = L/vy.
За время Д/i сила Fx — еЕу действующая на электрон со стороны электрического поля, сообщает ему в направлении оси х импульс
I
рх = Fx Д/i = еЕ—.	(1)
vv
363
Сила Fz = evyB} действующая на электрон со стороны магниты ин поля с индукцией В, сообщает ему за время Д/i импульс в напри влении z
pz = FzAti = elB.	(V|
Если предположить, что импульс и скорость частицы связаны обыч ным соотношением
р = mv,	(.’1|
то легко увидеть, что электрон попадает на экран в точку с коор динатой
рх	eElL
х = — Д/2 =---г-	'•)
т	mv~
Принимая во внимание равенства (2) и (3), для координаты z точки, в которую попадает на экране электрон, находим
pz А	eBlL
z=—^t2 = ------.	(Ь)
т	mvy
Формулы (4) и (5) показывают, что чем меньше скорость электро на Vy} тем дальше он попадает на экран от начала координат и, наоборот, чем больше эта скорость, тем точка попадания ближе н началу. Исключая из равенств (4) и (5) величину vyi получаем урпи пение кривой, на которой расположены точки попадания частиц пи экран:
2 тЕ
x = z2	(II)
eB2lL
Проведем анализ полученной кривой в области, соответствую щей большой скорости частицы (из формул (4) и (5) следует, ч I»» эта область на экране находится вблизи начала координат). Равен ство (6) показывает, что касательная к кривой в точке с z = 0 пи правлена по оси z. Однако результаты опытов противоречат такому выводу. В начале координат касательная к экспериментальной крн вой составляет некоторый отличный от нуля угол с осью г.
Чтобы получить с помощью формулы (6) не противоречащиП опыту результат, необходимо предположить, что при стремлении скорости электрона к бесконечности его импульс растет быстрее скорости. В релятивистской физике справедливость этого прсдио ложения полностью подтверждается.
Импульс частицы в теории относительности зависит от ее скор»» сти следующим образом:
_ mv
Р у/1 ~ V2/с2 ’
364
где с—скорость света в вакууме. При скоростях, много меньших скорости света, импульс, вычисленный по классической формуле (3), практически совпадает со значением, получаемым из равенства (7). При скоростях частицы, близких к скорости света, нельзя пользоваться приближенной формулой (3).
Используя точное выражение (7) для компонент импульса рх и учтем, что скорости частицы вдоль осей х и z много меньше скорости vy. Тогда
mvx
px —
(8)
Pz =
mv2
(9)
Легко видеть, что вместо выражений (4) и (5) теперь имеем
PxJi- «у/с2 eElLJl-v2/c2
__Y_______А / „ — __Y_______
(10)
рг ,/1 - v2/c2 eBlL Jl- vl/c?
z = —-----------Д/2 =--------i.	(11)
mvy
Координаты точки на экране, вычисленные по формулам (10) и (11), стремятся к нулю при стремлении скорости vy к скорости света. Тангенс угла о между касательной к кривой в начале координат и осью z определяется просто отношением координат х и z при стремлении
vv к с:
х
tga = lim - = vv->c Z
El
Be'
Вместо формулы (6) для уравнения кривой на экране получаем
выражение
о тЕ I (eBlL \2 eB2lL V \ mcz J
(12)
Видим, что выражение (12) отличается от выражения (6) вторым слагаемым под знаком корня, которым нельзя пренебречь в области кривой, удовлетворяющей условию
тс eBlL'
(13)
Используя табличные значения констант и разумные параметры экспериментальной установки (L « 1 м, I « 1 см, В « 0,1 Тл), можно
365
увидеть, что граница применимости классической механики, онрни ляемая неравенством (13), соответствует z < 1 см. Скорость, нео(н»» димая для попадания в эту область, составляет (см. формулу (11 Н всего 107м/с. Такую скорость электрон может набрать, пройдя |ш • ность потенциалов в 250 В.
Задача 330* (75-г)
Горизонтально расположенный металлический стержень длипмН I вращается с угловой скоростью си вокруг вертикальной оси, про ходящей через один из его концов. Вдоль оси вращения направлпь* однородное магнитное поле с магнитной индукцией В. При кпкмм значении В разность потенциалов на концах стержня равна пул и» Чему равна эта разность потенциалов, если нет магнитного поля Удлинением стержня пренебречь. Отношение е/т заряда электрпцн к его массе считать известным.
Решение. Величина силы F, действующая на электрон, дня жущийся со скоростью v в плоскости, перпендикулярной к вектор* магнитной индукции В, определяется формулой
F = evB,
где е — заряд электрона. Сила F направлена перпендикулярно векторам v и В. Так как, по условию задачи, вектор В направл' и вдоль оси вращения стержня, то сила F направлена вдоль стгр^ ня к оси вращения или от нее (векторы F, В и v образуют праву i" систему в случае отрицательного заряда частицы).
Рассмотрим электрон, вращающийся вместе со стержнем на при извольном расстоянии г от оси с угловой скоростью и). Для ЭИ Им электрона evB = mv2/г. Полагая здесь v = сиг, получаем ewli ш2. Отсюда
В = vnw/e.	(I)
Равенство (1) не содержит г. Таким образом, в поле с индукпп ей, определяемой формулой (1), электроны, находящиеся на любьи расстояниях г от оси, будут двигаться по окружностям и разноги потенциалов на концах вращающегося стержня будет равна пул in
В отсутствии магнитного поля электроны будут смещаться в пн рону от оси до тех пор, пока не возникнет электрическое поло । достаточной напряженностью Е. Теперь роль центростремительпиП силы будет выполнять сила еЕ = ?тгси2г, откуда
Е — тш2г/е.
366
Напряженность поля вдоль стержня растет линейно от г. В соответствии с этим разность потенциалов между осью и произвольной Iникой стержня, находящейся на расстоянии г от оси, будет равна
2 *>
" 2е
Задача 331 (75-р)
Плоский контур с сопротивлением R площадью S может без трения вращаться относительно оси, лежащей в плоскости контура. По-। гоянное однородное магнитное поле с индукцией В направлено по нормали к оси вращения. Под действием внешних сил контур вращается с постоянной угловой скоростью он Индуктивность контура пренебрежимо мала.
а)	Найдите среднюю за период мощность внешних сил.
б)	Найдите количество джоулева тепла, выделяемого в контуре и единицу времени.
Решение. Поток вектора магнитной индукции через поверхность S меняется со временем по закону
Ф = BS cos о»/.
При этом в контуре возникает ЭДС индукции
£ = wBS sin art
и течет ток
wBS .
—-—smart. R
В контуре выделяется мощность
R	2К
(’реднее значение мощности
— a?2B2S2
IV =
2R
Вся работа, совершаемая внешними силами, тратится на выделение тепла.
Задача 332* (87-р)
Атом в модели Томсона представляет собой тяжелый равномерно заряженный шар, внутри которого находится легкая частица
367
(“ядро”), имеющая заряд, равный и противоположный заряду шарп Такой “атом” влетает со скоростью v в электрическое перпендику лярное его скорости поле Е протяженностью L. Найдите амплитуд) колебаний “ядра” после вылета “атома” из поля, считая, что
а)	“атом” пролетает область поля быстро, т. е. “ядро” практичг ски не успевает сдвинуться относительно шара;
б)	“атом” пролетает область поля за большое время, и к момги ту вылета “атома” из поля колебания “ядра” относительно шири прекратятся.
Масса “ядра” тп, заряд q, радиус шара R. Считать, что “ядро” движется в пределах “атома” свободно и R2 <
Решение, а) “Ядро” практически не сдвинулось относительно шара, но приобрело некоторую скорость v вдоль поля. При этом mv = Fiy где F = qE} а / = L/v — время пролета “атомом” облает поля. Заряженный шар создает на расстоянии г (г < R) от свои.< центра поле
E'(r) = qr/(4ireoR3).
Потенциальная энергия “ядра” в этом поле
U(г) = дг2/(8тгбоЯ3).
Амплитуду колебаний А найдем из закона сохранения энергии:
пти2/2 = U (А),
что дает	_________________
А = s/4m0E2L2R3/(mv2).
При этом считаем А < R.
б)	К моменту вылета “атома” из области действия поля “ядро” находится в покое относительно “атома”. Следовательно, уравнопг шены силы, действующие на него со стороны полей Е и Е(. Дли расстояния го “ядра” от центра “атома” получаем
г0 = 47T€QER3/q.
Вылетев из поля, “ядро” будет совершать колебания с амплшу дой Го-
368
12. ГЕОМЕТРИЧЕСКАЯ ОПТИКА
При решении многих задач на геометрическую оптику требуется построение изображения в тонких линзах.
Линза представляет собой прозрачное (обычно стеклянное) тело, ограниченное двумя сферическими поверхностями. Тонкой называют линзу, толщина которой много меньше радиуса поверхностей и расстояния от предмета до линзы.
Введем некоторые вспомогательные определения.
Главная оптическая ось — прямая, проходящая через центры сферических поверхностей, образующих линзу. Точка пересечения главной оптической оси с линзой—оптический центр линзы. Прямые, проходящие через оптический центр линзы, — побочные оптические оси.
Лучи, идущие параллельно главной оптической оси, после преломления в линзе проходят так, что пересекают главную оптическую ось в фокусе линзы F (рис. 131 а, б). Если эти лучи сходятся к точке Г (рис. 131, а), линза называется собирающей. Если лучи расходятся так, что их продолжения пересекаются в точке F' (рис. 131, б), линза называется рассеивающей.
Расстояние / от оптического центра линзы О до ее фокуса F — фокусное расстояние. Фокусное расстояние для тонких линз (в воздухе) рассчитывается по формуле
7 = ("-1) (яГ+я^)‘
Здесь Ki, Яз — радиусы сферических поверхностей линзы, причем предполагается, что R > 0 для выпуклой поверхности линзы и R < О для вогнутой. Величина f может оказаться положительной (собирающая линза) или отрицательной (рассеивающая).
369
Величина, обратная фокусному расстоянию, носит начв/нш» оптической силы линзы.
Из выражения (1) следует, что фокусные расстояния тонкой л ни зы одинаковы по обе стороны от нее.
Плоскость, перпендикулярная к главной оптической оси и нрпш* дящая через фокус линзы, называется фокальной плоскостью, Jh чи, параллельные побочной оптической оси, проходят после прслн мления через точку пересечения этой оси с фокальной плоскостью
Основываясь на перечисленных правилах и определениях, мп и» но построить ход лучей в тонкой линзе.
Рис. 132
На рис. 132, а показан ход луча AAi после преломления в спЬн рающей линзе. Для построения проведена побочная оптическая п» i ВО, параллельная лучу AAi и проходящая через оптический цгн i р линзы О. Луч AAi после преломления должен пройти через точм пересечения оси ВО с фокальной плоскостью Д, т. е. через точку (
Аналогично строится луч AAi после преломления в рассеиваю щей линзе (рис. 132, б). Различие состоит в том, что не сам луч /I 1| после преломления в линзе пересекает побочную оптическую • и i ВО, а его продолжение CAi в точке С, лежащей в левой фокалвпий плоскости F.
На рис. 133 построены изображения источника А в собираюпюН линзе при разных расстояниях от источника до линзы (а — болит фокусного, б—равном фокусному, в, г — меньше фокусного) и в pin сеивающей линзе (5, е).
Различные построения можно обобщить формулой линзы:
1 1 1
7 — “ + г, fab
(У1
где а — расстояние от объекта до линзы, b — расстояние от линзы ,п»» изображения, т. е. до точки пересечения преломленных лучей. Hr
370
ц|*1ина b положительна для случая “действительного” изображения (преломленные лучи пересекаются в некоторой точке за линзой) и отрицательна для случая “мнимого” изображения (лучи за линзой расходятся, а их продолжения пересекаются в некоторой точке перед линзой). Отрицательной может быть и величина а: на линзу может падать не расходящийся от действительного источника пучок лучей, а сходящийся в некоторой точке за линзой. В этом случае источник “мнимый” и величина а отрицательна.
Рис. 133
* * *
Задача 333 (78-г)
Собирательная линза с фокусным расстоянием f имеет две поверхности— одну плоскую, вторую выпуклую. Плоскую поверхность посеребрили так, что она стала полностью отражать свет. Как в общем случае построить изображение в этой оптической системе?
Решение. Рассмотрим ход луча, идущего параллельно главной оптической оси (рис. 134). После преломления в линзе этот луч
371
Рис. 134. К задаче 333.
должен был пройти через правый фокус F. Но отразившись от -юр калыюй поверхности линзы, луч направлен к первому фокусу, (hi нако, проходя второй раз через линзу, луч опять преломляется н пересекает главную оптическую ось на некотором расстоянии /| •• • линзы, в точке Fi, являющейся фокусом системы. Найдите j\ л<ч из следующего соображения: оптические силы последовательно pin положенных тонких линз складываются. В рассматриваемом слупи лучи проходят одну и ту же линзу дважды, следовательно, oiiiii ческая сила линзы с плоской зеркальной поверхностью в два pn m больше, чем самой линзы, а фокусное расстояние в два раза мен i.пн
/1 = 7/2.
Итак, изображение надо строить, как в линзе с фокусным рн* стоянием //2, а затем зеркально отразить построенное изображспп» в плоскости линзы.
Задача 334* (74-г)
Имеется система из N параллельных плоских прозрачных пли стин. Если свет интенсивностью J падает по нормали на одну и • пластин, то часть интенсивностью a J отражается, а часть интепсин ностью (1 — проходит через пластину. Найдите интенсивное и света, прошедшего через N пластин.
Решение. Пусть для определенности свет падает на систему N пластин слева. Пусть — интенсивность света, падающего ин к-ю пластину слева, a Jk — интенсивность света, уходящего от к И пластины налево.
Тогда на fc-ю пластину падает свет интенсивностью 7л+1 спрппп от этой же fc-й пластины направо уходит свет интенсивности 11 По условию задачи
Jk =aJk+(1-a)Jk+l.	(I)
На первую пластину падает слева свет с интенсивностью J\, и уходит налево свет с интенсивностью	где а/у
372
коэффициент отражения от N пластин. Из-за отсутствия поглощения суммарный поток, проходящий через первую пластину, прохо-н,пт и через все остальные:
<71 J1 = (1 ow)«7i = Jk	Jk	— <Zfc+i	(2)
Подставляя Jki полученное из равенства (2), в формулу (1) имеем
Л+1 ~ Л- ~	J _	а ‘	(з)
Написав равенство (3) k раз, получим
4+1 = 4 - kaJi \	л	- kaJ^\_a^	(4)
Учтем, что на последнюю пластину	свет справа	не падает:
,/^j = 0, а направо уходит свет интенсивностью Jn = «71 (1 — ajv).
При этом условии из формулы (4) получаем искомую величину относительной интенсивности света, прошедшего через N пластин:
, _ 1
1 aN~ l + (N-l)a
Задача 335 (81-р)
Расстояние между экраном и источником света равно L. Линза дает четкое изображение источника при двух положениях, расстояние между которыми равно I. Найдите фокусное расстояние линзы.
Решение. Обозначим расстояния от линзы до источника при первом и втором положениях линзы как oi и 02; расстояния от линзы до экрана — bi и 62 соответственно.
По условию задачи
01 + ^1 — а2 4" ^2 —	(1)
а2 — а1 =	— ^2 — I-	(2)
Применим формулу линзы к каждому из двух ее положений:
1-2. 1
f ~ а1 + ’
1 1 1
*7 —----h т~-
J <*2
(3)
(4)
Приравнивая левые части формул (3) и (4), получаем
+ bi ~ 02 + ^2 01&1	02^2
373
Отсюда в силу (1)
Ctjbl = Д2&2 -
Исключая из последнего равенства с помощью (1) и (2) неизвестш.11 величины ai, аг и Ь?, имеем
откуда следует, что
bl =	(Ы
Теперь из (1)
L — 1
ai=L-6i = ——.	(II)
Подставляя левые части равенств (5) и (6) в формулу линзы (3), находим искомое фокусное расстояние
di 4- 61 4L
Задача 336 (86-г)
Как бы мы стали видеть, если бы скорость света возросла в III” раз?
Решение. При увеличении скорости света в 105 раз длина волны возрастет во столько же раз и превысит 1 см, что существен ни больше размеров зрачка. Поэтому мы ничего не увидим.
Задача 337 (85-р)
На глубине h = 10 см под гладкой поверхностью воды (к<гм|« фициент преломления п = 1,33) в большом сосуде горит маленький лампочка. Что увидит наблюдатель, находящийся над водой, сели непосредственно над поверхностью воды расположить лист белой бумаги?
Решение. Бумага рассеивает свет. Благодаря полному вну треннему отражению от поверхности воды на бумаге будет видни светлое пятно с радиусом г = fttga, где а = arcsin(l/n) — угол пол ного внутреннего отражения. Поэтому
h
г = .	« 11,3 см.
х/п2 - 1
Задача 338 (82-г)
Оцените оптическую силу линз в очках, необходимых для чтении дальнозоркому человеку, который резко видит предметы, распило женные не ближе одного метра. Какими должны быть эти линзы собирающими или рассеивающими?
374
Решение. Очки должны создать мнимое изображение текста, удаленного от них на расстояние а к 0,25 м (наилучшего зрения), на расстоянии b = — 1 м от них. Знак минус показывает, что изображение должно находиться по ту же сторону от очков, что и рассматриваемый текст. С помощью формулы тонкой линзы для ее оптической силы находим
1	1	1	о
D=-=T + -& Здптр.
f	о	а
Положительное значение D указывает, что линзы должны быть собирающими.
Задача 339 (77-г)
Матовая пластинка отражает свет во все стороны равномерно. Объясните, почему пластинка кажется более яркой, если ее рассматривать под малым углом к ее поверхности.
Решение. Очевидно, что пластина кажется тем более яркой, чем больше освещенность Е изображения картинки, создаваемого па сетчатке. Для Е запишем выражение
Е = Ф/S,	(1)
где Ф — световой поток, проходящий через зрачок, и S — площадь изображения пластинки на сетчатке. Обозначим через I силу света, отражаемого пластинкой в направлении, составляющем угол а с плоскостью пластинки. В зрачок попадает световой поток
Ф = /Q.	(2)
Здесь Q— телесный угол, под которым из малой площадки dS виден зрачок. Очевидно, что
n = й/г2,	(3)
где Si —площадь зрачка и г — расстояние от площадки до зрачка. Так как по условию задачи величина /, как и угол П, не зависят от угла а, то и световой поток Ф, попадающий в зрачок, не зависит от о.
Площадь S изображения на сетчатке прямо пропорциональна видимой площади пластинки S' = So sin а:
s = r2S' = r2Sosina,	(4)
S' — площадь проекции площадки Sq на плоскость, нормальную к направлению, с которого рассматривается пластинка; Г — линейное увеличение хрусталика (Г < 1).
375
Подставляя в формулу (1) Ф (2) и S (4) и принимая во внимашп формулу (3), для освещенности находим
^=r2f1-	(''>
r2r2sina Sq
Формула (5) показывает, что освещенность изображения тем больше, чем меньше угол а. Именно поэтому пластинка и кажете и нам более яркой при рассматривании ее с направления, составляю щего малый угол а с ее поверхностью.
Отметим, что подобным образом ведет себя не любая пластин к» Чаще всего величина / является функцией угла а и при каком-m избранном значении qq достигает своего максимального значения, п при стремлении а к нулю убывает быстрее, чем sin а. В этом слупи* пластинка кажется наиболее яркой именно с этого направления Говорят, что она “блестит” в направлении (Со-
существуют поверхности, для которых
1 = /0 sin а,	((I)
где /о — некоторая константа. Такие поверхности называют абсо лютно матовыми. В случае пластинки с абсолютно матовой поверх ностыо освещенность ее изображения на сетчатке дается формулой
Е = IS1 = IoSl	Cl\
SorMsina 5о(Гг)2
Очевидно, что в этом случае Е не зависит от а. Рассматривая абсо лютно матовую пластинку, мы видим ее одинаково яркой с любого направления.
Задача 340 (77-г)
Над столом висит зеркало, плоскость которого составляет угол а = 45° с горизонтом. Центральная часть зеркала расположена над столом на высоте h = 1 м. На той же высоте на расстоянии по горизонтали I = 1 м от центра зеркала висит лампа с силой светя I = 100 кд. Найдите освещенность стола точно под центром зеркали
Решение. Свет от источника А в точку стола В попадшч двумя путями: по прямой АВ и отражаясь от зеркала по ломаной АО В. Во втором случае свет после отражения распространяется так, как распространялся бы свет от'источника с той же силой евс та /, помещенного в точке А1, являющейся изображением точки А в зеркале. Точки Л', О и В расположены на одной вертикали. Иско мую освещенность Е стола в точке В надо рассматривать как сумму
376
освещенностей Е\ и Е{, создаваемых источниками А и А'. Источник /I создает в точке В освещенность
„	/ cos а
1 “ (ЛЯ)2'
Так как АВ = x/h2 + I2 и cos а = h/x/h2 +12, то
(Л2 4- Z2)<'
Источник А' создает в точке В освещенность
Г>_ 1 _ 1
1	(Л'Я)2 (Л + /)2'
Суммарная освещенность
E=E'+E'=\w^+^ywm-
Задача 341 (77-г)
Тонкая собирающая линза имеет фокусное расстояние F. Из того же материала, что и линза, изготовлена плоскопараллельная пластинка с полостью, форма которой повторяет форму линзы. Полость заполнена воздухом. Найдите оптическую силу пластинки с полостью.
Решение. Фокусное расстояние f линзы связано с радиусами R1 и /?2 поверхностей соотношением
Плоскопараллельную пластинку с полостью, форма которой повторяет форму линзы, можно рассматривать как составленную из двух вплотную сдвинутых плосковогнутых стеклянных рассеивающих линз. Для нахождения их фокусных расстояний воспользуемся формулой (1). Полагая в ней R2 = оо и беря первый член суммы со знаком минус, а затем, полагая Ri = 00 и беря со знаком минус второй член суммы, для фокусных расстояний находим выражения л = --^4 > л = —•	(2)
п — 1	п - 1
377
Получим формулу, связывающую фокусное расстояние опти'Н’ ской системы f из двух вплотную приставленных друг к другу тон КИХ ЛИНЗ С фокусными расстояниями /1 И f2- Фокусным расстои нием оптической системы из двух линз будем называть расстоянн» от оптического центра системы до точки, в которой сходится иогп» прохождения системы пучок лучей, параллельных главной oimi ческой оси. Для нахождения фокусного расстояния системы лип» воспользуемся общей формулой линзы
Применяя формулу (3) по отношению к первой линзе, все входи щие в нее расстояния будем снабжать индексом 1, по отношению ю» второй —индексом 2.
Пусть слева на первую линзу падает пучок лучей, параллельны» главной оптической оси. Тогда, полагая di = оо, найдем, «пн
= Л ♦
Ко второй линзе лучи будут подходить так, как если бы они бы ли испущены из фокуса первой линзы. Это означает, что а2 = —6| —Д. После прохождения второй линзы лучи будут распространи i ь ся так, как если бы они были испущены из фокуса системы, т, с i>2 = /'• Следовательно,
__1_ 1 _ ±
/1 + Г ~ Л 
Или, что то же самое,
Oi+O2 = Oz,	(Ь)
т. е. оптическая сила системы из двух вплотную сложенных лип। равна алгебраической сумме оптических сил этих линз. Подставляй в (4) выражения для Д и Д (2), получаем
я' = -(п-1)(4- + 7г\	(,1)
Сравнивая между собой формулы (1) и (6), находим
£>' = -!//< О,	(7)
378
т. е. пластинка с воздушной полостью представляет собой рассеивающую линзу с фокусным расстоянием /.
Отметим, что формула (5) может быть обобщена и на произвольное число п вплотную сдвинутых плоских линз с оптическими силами Di,  • • j Оптическая сила D такой системы определяется формулой
D = Di + Di + ... + Dn .	(8)
Величина D не зависит от порядка следования линз друг за другом.
Покажем, как более общая формула (8) может быть использована при решении данной задачи. Рассмотрим систему из трех сдвинутых вплотную линз — исходной тонкой собирающей и двух плоско-вогнутых, образующих плоскопараллельную пластинку с полостью. Для этой системы
Do — Di + D2 + D .	(9)
Здесь
D = 1//	(10)
— оптическая сила собирающей линзы, D± и D2 —оптические силы плосковогнутых линз, Do — оптическая сила плоскопараллельной пластинки. Найдем оптическую силу D' системы из двух плосковогнутых линз. Заменяя в (9) сумму Di 4- D2 на D', величину D — правой частью формулы (10) и принимая во внимание, что Do = 0, снова приходим к формуле (7).
Задача 342 (80-р)
Докажите, что формула тонкой собирающей линзы имеет вид г*1Х2 = /2, где Xi — расстояние от предмета, находящегося (для определенности) слева от линзы до левой фокальной плоскости; а?2 — расстояние от правой фокальной плоскости до изображения; f — фокусное расстояние линзы.
Решение. Обычную формулу тонкой линзы записывают в
где b — расстояние от предмета до линзы и а— расстояние от изображения до линзы. Рассматриваемые в условии задачи величины и Х2 связаны с величинами b и а очевидными формулами:
b = X! 4- f , а = х2 4- /.	(2)
379
Из (1) и (2) получаем
1 1 _ 1
*1 + / +	+ f ~ f
Отсюда с помощью простых алгебраических преобразований нахп дим требуемую формулу простой линзы:
= /2-
Задача 343 (80-г)
Тонкая плосковыпуклая линза с оптической силой D вставлена в стенку аквариума плоской стороной к воде. Найдите фокусные расстояния получившейся системы. Абсолютный показатель прело мления воды пр
Решение. Пусть линза вставлена в левую стенку аквариума (рис. 135). Найдем сначала положение правого фокуса. С этой ц<» лью рассмотрим луч, идущий в воздухе слева направо параллельно главной оптической оси 00' на малом расстоянии h от нее. Этот лун сначала преломляется на выпуклой поверхности линзы и падает на ее плоскую поверхность на расстоянии Л' < Л от главной оптической оси под некоторым углом а. В результате преломления на плоской поверхности линзы рассматриваемый луч входит в воду под некото рым углом /?. Углы а и (3 связаны друг с другом соотношением
где п — показатель преломления воды относительно стекла. В воде луч распространяется по прямой линии и пересекает главную онти ческую ось в правом фокусе на некотором расстоянии Д от линзы Это расстояние связано с расстоянием h' и углом /? формулой
7- = tg 0-	со
J2
380
Рассмотрим теперь ход этого же луча, когда в аквариуме воды нет и справа от линзы находится воздух. В этом случае луч падает па плоскую поверхность линзы под тем же углом а и на том же расстоянии h! от главной оптической оси, что и раньше. Выходит в воздух этот луч под некоторым углом 7, связанным с углом а соотношением
sina sin 7
где п* — показатель преломления воздуха относительно стекла. Луч пересекает главную оптическую ось на расстоянии f от линзы, которое связано с расстоянием h* и углом 7 формулой: h'/f = tg 7, которую, принимая во внимание, что f — D"1 * *, можно записать так:
h! D = tg 7.
Из (2) и (4) для /2 получаем
f - _L
(4)
(5)
Так как расстояние Л, а следовательно и h!, мало, то малы углы а, /3 и у. В связи с этим sina « tg а « a, sin/? « tg /? « /3 и sin 7 « tg 7 w у. С учетом этого формулы (1), (3) и (5) принимают
соответственно вид	?="	(6> °- = <	(П Л=5 i	‘8>
Из системы уравнений (6)—(8) получаем
1 п
D nf'
(9)
Показатели преломления воды относительно стекла и воздуха относительно стекла можно выразить через абсолютные показатели преломления воды щ, стекла П2 и воздуха П3: п = П1/П2; п' = пз/пг-С учетом этих равенств формула (9) принимает вид
1 П1
/2 i-х
D пз
381
Абсолютный показатель преломления воздуха практически не <н личается от единицы, т. е. пз = 1, поэтому окончательно:
А = ni/D.
Для нахождения левого фокусного расстояния надо рассмотри ! ь ход луча, идущего справа налево параллельно главной оптической оси. Поскольку этот луч падает на правую плоскую поверхносп. линзы под углом а = 0, то, независимо от того, какая среда запо.н няет аквариум, ход его один и тот же. Поэтому
fi=f= 1/D.
382
13.	ОЛИМПИАДНАЯ СМЕСЬ
* * *
Задача 344 (85-г)
Идет дождь. Скорость капель направлена под углом а к вертикали, количество воды в единице объема воздуха равно р. Вам надо пройти под дождем путь /, причем ветер попутный. С какой скоростью надо двигаться, чтобы промокнуть как можно меньше, если:
а)	вы сверху защищены, а можете промокнуть только спереди или сзади;
б)	вы хотите, чтобы на вас попало минимальное количество воды все равно откуда, причем ваша площадь при виде сверху равна Si, а при виде спереди — 5*2 •
Решение. Пусть скорость капель v, ваша скорость—и. Перейдем в систему отсчета, связанную с вами; вектор скорости капель в этой системе отсчета обозначим v'. Вертикальная составляющая скорости в этой системе та же, что и в исходной:
vfx = v cos а;
горизонтальная составляющая будет равной
vfy = vsina — и.
За время t через площадку Si пройдет количество воды
mi = SiVytp
— та вода, которая попадет на вас сверху. За то же время через S2 пройдет масса воды
m2 = S2|*4|М = S2|vsina - u\tp.
Путь I вы прошли за время i — 1/и. Полное количество упавшей на вас воды:
т = mi + m2 = (Sivcosa + S^lvsina — u\)pl/u. (1)
383
а)	. В этом случае вода сверху не попадает, Si = 0 и из формулы (1) получаем
mmin = 0 при U — V sin Of.
При такой скорости дождь падает на вас только сверху.
б)	. Перепишем равенство (1) в виде
т = plv (5i cosatz-1 + 52 sin a |u-1 — (vsina)-1|) .
Теперь нетрудно показать следующее: при
Si/S2 > tgа
надо идти как можно быстрее (так, чтобы и оо), при этом
^min — plS2 J
при
Si/S2 < tga
надо двигаться со скоростью и = и sin а, при этом
^min = p/5iCtgOf.
Задача 345 (85-р)
В произвольный момент времени t координата тела, движущем и ся вдоль оси ж, определяется равенством я(/) = (ct — 6)2. Найди и-ускорение и скорость тела в произвольный момент времени.
Решение. Зависимость координаты х от времени t можи»» представить в виде
х = b2 — 2bct + c2t2.	(I)
Сравнивая это выражение с уравнением равноускоренного движг ния:
х = zq + vqI + at2/2,
видим, что уравнение (1) описывает равноускоренное движение • начальной скоростью vo = —2bc и ускорением а = 2с2. При panim ускоренном движении можем написать для скорости в произвол!, ный момент времени выражение
v(t) = Vq + at,
которое в нашем случае принимает вид v(t) = —2c(b — ct).
384
Задача 346 (85-р)
Диск вращается с постоянной угловой скоростью о/ вокруг оси, проходящей через его центр и перпендикулярной к плоскости диска. Муха ползет из центра диска вдоль его радиуса с постоянной скоростью v. Найдите скорость мухи в системе отсчета, связанной г землей.
Решение. Введем обозначения: и — скорость мухи в системе отсчета, связанной с землей; v — скорость мухи относительно диска; vz — скорость относительно земли той точки диска, в которой в данный момент находится муха. Имеет место равенство
и = v + vz,
причем вектор v направлен вдоль радиуса, по которому движется муха, a vz -по нормали к этому радиусу. Так как и—диагональ прямоугольника, построенного на v и vz, то очевидно, что величина искомой скорости и определяется равенством
и = |u| = \/|v|2 + |v'|2.
Принимая во внимание, что |v| = v и |vz| = о>т, где г — расстояние мухи от центра диска в момент времени t (г = /и), окончательно получаем:
и = \/v2 + Ш2Г2 — v2y/l + w2t2.
Задача 347 (85-г)
Колесо, вся масса Af которого заключена в ободе радиусом /?, катят по горизонтальной поверхности с постоянной скоростью v, прикладывая некоторую силу F, направленную горизонтально. К внутренней поверхности обода прикреплен маленький шарик массой т. Найдите силу F(t) и силу давления колеса на поверхность p(t). Считать, что в начальный момент времени шарик находится в нижней точке.
Решение. Перейдем в систему отсчета, связанную с центром колеса. В этой системе отсчета колесо вращается с постоянной угловой скоростью Угловую координату шарика а будем отсчитывать от наинизшей точки: а(/) = wt. На шарик действуют сила тяжести mg, направленная вертикально вниз, и сила Q(Z), направленная так, чтобы сумма сил, действующих на шарик, была всегда направлена к центру колеса и равна Т = mu;2/?. Проекции этой силы на горизонтальное и вертикальное направления зависят от времени следующим образом:
Тх = mw2T?sin a(t) =	(1)
Ту = mw2Rcos а(/) = Q(t)y — mg.	(2)
385
По III закону Ньютона шарик действует на колесо с силой N(Z) —Q(t). Из этого факта и равенств (1), (2) следует, что
= —mw2T?sin о(/),	(3)
Ny = — mw2T?cos a(t) — mg.	(4)
Сумма сил, действующих на колесо, должна равняться нулю:
N(/)+K(/) + F(/) + Mg = 0.	(Ь)
(Сила Р(/) действует со стороны колеса на поверхность, а си ла К(/) = — Р(/)—со стороны поверхности на колесо.) Учиты вая, что сила F(/) всегда направлена горизонтально, а сила Р(/) вертикально, спроецируем равенство (5) на горизонтальное и вер г и кальное направления, подставив вместо проекций силы N правы»’ части равенств (3) и (4):
—mu>27?sin a(i) + F(t) = 0,	(II)
—n?w2licos а(/) — тд — Мд + K{t) = 0.	(7)
Из последних равенств получаем ответ:
F(t) = ?nw2Tlsin а(/);
P(t) = тд + Мд + ты2 R cos а(/).
Анализируя ответ, заметим, что приведенное в задаче движение н<и можно только при выполнении условия
cv2T? < д (1 + М/т) ,
в противном случае колесо будет подскакивать.
Задача 348 (85-г)
На абсолютно гладкой горизонтальной поверхности стола па рп« стоянии 21 друг от друга находятся два шарика массой т каждый Радиусы шариков пренебрежимо малы в сравнении с I. Шарики спи заны невесомой нерастяжимой нитью длиной 21. В начальный ми мент скорости шариков равны нулю. Среднюю точку нити А нами нают двигать с постоянной скоростью v в направлении, перпелдп кулярном нити и параллельном поверхности (рис. 136, а).
1. Какой путь пройдет точка А до момента столкновения шароп7 2. Какой путь пройдет точка А до того момента, когда скорое i и шариков относительно поверхности стола снова станут равны пули ' Удар шариков абсолютно упругий.
3. Какую силу надо прикладывать к точке А, чтобы она дшпа лась с постоянной по величине и направлению скоростью?
386
Рис. 136. К задаче 348.
Решение. Перейдем от неподвижной жестко связанной с поверхностью стола системы отсчета к другой инерциальной системе отсчета, в которой точка А покоится, а шарики в начальный момент имеют скорости v, направленные так, как показано на рис. 136, 61
В выбранной системе отсчета шарики движутся по дугам окружности с радиусом I. Так как действующие на шарики со стороны нитей силы натяжения все время перпендикулярны к направлению движения, то шарики движутся по окружности равномерно со скоростью v. До момента столкновения каждый шарик пройдет по дуге окружности путь <§, равный 1/4 длины окружности:
s = тг1/2 .
Время движения
i — s/v = 7r//(2v) .
За это же время точка А, двигаясь со скоростью г, пройдет в исходной системе отсчета такой же путь:
L = vt = 7г//2 .
Для ответа на второй вопрос нам нужно найти путь, пройденный точкой А в исходной системе отсчета к моменту, когда шарики будут иметь скорости v в системе отсчета, связанной с точкой А. В этой системе отсчета шарики двигаются из точек Р и Р1 по дугам окружности с радиусом I до упругого столкновения в точке Q. Обменявшись в процессе упругого соударения своими скоростями, шарики вновь двигаются по окружности в обратном направлении; они проходят точки Р и Р1 с одинаковыми скоростями —V и во второй раз сталкиваются в точке R. После обмена скоростями в точке Р шарики возвращаются в точки Р и Р', имея одинаковые скорости v. Таким образом, от начала движения до этого момента каждый шарик, двигаясь равномерно по дуге окружности со скоростью и,
387
проходит путь, равный длине окружности. За это же время, дним ясь прямолинейно с той же скоростью v, точка А в исходной сие н м» отсчета проходит такой же путь L = 2тг/.
Для ответа на третий вопрос заметим, что движение периоднчи' во времени и происходит с периодом Т = 2ttI/v. Рассмотрим сини действующие на шарики и точку А в момент времени, когда пи in составляют с осью х угол а (рис. 136, в). В силу того, что обе пи hi невесомы, они действуют на шарики и точку А с равными по ikmiii чине силами Fi, —Pi и F2, — F2. Под действием сил Fi и F2 шарИ1«н равномерно движутся по окружности с центростремительным угм» рением v2/l и, в соответствии со II законом Ньютона, для Fy и / можно написать: о _	„ mv
Fi = F2 = — •	(Н
На точку А действует внешняя сила F, которую надо найти, »» также силы —Fi и — F2 натяжения нитей. Так как точка А в и игр циальной системе отсчета не имеет ускорения, то
f = Fi4-f2.	ел
Отсюда ясно, что сила F направлена вдоль оси у. Проецируй равенство (2) на ось у и принимая во внимание равенства (1), дни величины силы F находим
F = (Fi + F2) sin а = sina.	(3)
Найдем зависимость угла а от времени; при этом условимся у I • • считать положительным, когда шарики находятся над осью ж, и m рицательным, когда они находятся под ней. После начала дни ж» ния, двигаясь равномерно, шарики за время i проходят путь s ~ Н за это время нити описывают угол а, удовлетворяющий равепс иц s = 1а. Отсюда ясно, что а = vt/ly и (3) можно записать так:
„ mi? . vt „	. vt
F = 2-у- sin у = Fmax sm у.
Здесь
__ mv2 ^max = 2 -
— амплитудное значение силы F.
Положительные значения F отвечают силе, направленной при тивоположно оси уу а отрицательные — силе, направленной по о» и У-
388
Задача 349 (85-г)
Горизонтальный стержень вращается вокруг вертикальной оси, проходящей через один из его концов, с постоянной угловой скоро-I'Tbiow. На стержень насажена шайба массой Л/, соединенная с осью вращения невесомой пружиной, имеющей жесткость к (рис. 137). Длина нерастянутой пружины /. Трение между шайбой и стержнем отсутствует. При каком соотношении между параметрами задачи шайба находится в устойчивом равновесии?
Рис. 137. К задаче 349.
Решение. Пусть шайба находится на расстоянии х от оси вращения. Тогда на нее со стороны пружины действует сила
F(x) = k(x-l).	(1)
Только при х > I эта сила направлена к оси вращения и может выполнять роль центростремительной силы. Величина центростремительной силы F4c, под действием которой масса т будет двигаться по окружности радиусом х с угловой скоростью ц>, дается формулой
Гцс(я) = то/2ж.	(2)
Очевидно, что шайба неподвижна относительно вращающегося стержня, т. е. находится в равновесии, если имеет место равенство
F(x) = Рцс(х).	(3)
Приравнивая правые части равенств (1) и (2), определяем координату д?о, в которой достигается равновесие:
_ I
Xq 1 — тш2/к'
Заметим, что это равенство имеет смысл только при
а?о > 0, т. е. к > то/2.	(4)
389
Если
F(x) >	(bl
то шайба смещается к оси вращения; если же наоборот, то шаПЬн смещается от оси вращения.
Исследуем равновесие на устойчивость. Равновесие устойчит», если при сдвиге шайбы от этого положения я о возникают силы, ни правленные к положению равновесия. Следовательно, для устойчн вости равновесия необходимо, чтобы неравенство (5) выполнялись при х > 2?о, а противоположное неравенство имело место при х < j'n Сравнивая правые части равенств (1) и (2), получаем, что нерп венство (5) имеет место при х > хо, а обратное неравенство— при х < xq, если выполнено условие (4).
Окончательный результат: устойчивое равновесие имеет меги» только при выполнении условия (4). В противном случае нет равно весия и, естественно, нет устойчивости.
Критическое значение частоты вращения, до которой существу г i состояние устойчивого равновесия:
wo = \/k/m
есть не что иное, как собственная частота колебаний тела массой т на пружине с жесткостью к.
Задача 350 (85-р)
Шарик массой т прикреплен к пружине с жесткостью к и ми жет свободно двигаться вдоль прямого горизонтального стержни, закрепленного на вращающейся опоре (рис. 138). Систему приводи! во вращение вокруг вертикальной оси с угловой скоростью w. ()< i вращения проходит через центр шарика при w = 0. Найдите полп жения равновесия системы. Являются ли они устойчивыми? Отп<ч обоснуйте.
Рис. 138. К задаче 350.
390
Решение. При w = 0 пружина не деформирована, и шарик при этом находится в положении устойчивого равновесия. Систему раскручивают до угловой скорости w. Предположим, что шарик покоится на расстоянии I от оси вращения и не касается стенок. При этом шарик движется с центростремительным ускорением и2/ и должно выполняться равенство
mw2l = kl.	(1)
а)	Если тиг = к, то это равенство (1) справедливо при любых значениях Z, т. е. шарик находится в положении “безразличного” равновесия при любом I (малые возмущения могут перемещать шарик вдоль стержня).
б)	Если ты2 < fc, то равенство тш21 = kl имеет место лишь при 1 = 0. Таким образом, положение равновесия шарика находится на оси вращения. Это устойчивое положение равновесия шарика, так как сила, действующая на него со стороны пружины (fc/), больше, чем произведение массы на центростремительное ускорение шарика на расстоянии I от центра.
в)	Наконец, если mw2 > к, то положение равновесия шарика при I = 0 неустойчиво.
Задача 351 (85-г)
Проволочная спираль расположена вертикально. Радиус спирали Я, расстояние между ее витками h, причем h R. На спираль надета маленькая бусинка, которая может скользить по ней; коэффициент трения между проволокой и бусинкой ц < Л/(2тгЯ). Определите скорость бусинки в установившемся режиме.
Решение. Рассмотрим силы, действующие в произвольный момент времени t на бусинку, скользящую вниз по спирали. Ее скорость в этот момент обозначим через v. Рассмотрение будем вести в инерциальной системе декартовых координат (неподвижной относительно спирали); ее начало совместим с точкой О, в которой находится центр бусинки в момент времени t. Ось х направим вдоль вектора v, ось у проведем перпендикулярно к оси х и направим ее в сторону оси спирали, ось z направим вверх.
Заметим, что ось х составляет с горизонталью такой угол а, что
h tg“=2rf
Так как по условию задачи h Л, то угол а мал, и для него можно написать:
afatgQ;=97R-	t1)
391
На бусинку действует сила тяжести mg, направленная вши вдоль вертикали, сила нормальной реакции спирали Q, направлен ная перпендикулярно к оси а?, и сила трения скольжения FT, напри вленная противоположно оси х. Для величины FT силы FT можно написать:
FT=pQ.	(2)
Согласно II закону Ньютона
ma=mg + Q + FT.	(3)
Здесь т — масса бусинки и а — ее ускорение. Спроецируем это рн венство, отнесенное к моменту времени /, на оси построенной нами системы декартовых координат:
тах = m^sina — FT, may = Qy} maz = Qz — mg cos a.
Отсюда, принимая во внимание, что угол а мал и, вследствие этого, sina « а и cos a « 1, с учетом равенств (1) и (2) получаем
may = Qyy	('I)
maz = Qz - mg.
Здесь ах,ау,аг и Qy, Qz —проекции векторов а и Q на соответстпу ющие оси.
Будем считать, что начало координат бусинка проходит в уже установившемся режиме, т. е. с постоянной линейной скоростью гц, которую требуется найти. Так как Л <£ 7?, то с хорошей степе! и do точности можно считать, что вблизи начала координат траекторией бусинки служит дуга окружности с радиусом Я, расположенной п плоскости ху. В связи с этим,
ах = 0, ау = Vq/Я, az = 0.	(b)
С учетом равенств (5) система уравнений движения (4) принимаем вид
meh
- С, = о, mg-Qz = 0.
392
Пополняя эту систему очевидным равенством
Q2 = Q2y + Q2
и исключая затем из нее неизвестные Qy, Qz и Q, находим
2	1 1/4
^о = (<?Я)1/2 (^р) -1 у Л7Г [Л J
Задача 352 (85-г)
Определите радиус выпуклого моста, имеющего форму дуги. Известно, что давление автомобиля, движущегося по мосту со скоростью v = 90 км/ч, в верхней точке моста в два раза меньше давления неподвижного автомобиля.
Решение. Автомобиль обладает центростремительным ускорением
а = v2/R,	(1)
где R— радиус моста. Когда автомобиль проходит через верхнюю точку моста, его ускорение а направлено вниз. В этот момент в вертикальном направлении на него действует сила тяжести mg (вниз) и сила нормальной реакции моста N (вверх). В соответствии со II законом Ньютона
та = тд — N.	(2)
Величина силы нормальной реакции No моста, приложенной к покоящемуся в его верхней точке автомобилю, равна величине силы тяжести тд, т. е.
Nq = тд.
По условию задачи
N = ^Na = ^тд.	(3)
Принимая во внимание (3), из уравнений (1) и (2) находим
R = 2v2 /д = 125 м.
Задача 353 (85-г)
Гимнаст падает с высоты h = 12 м на горизонтальную сильно растянутую абсолютно упругую сетку, в результате чего сетка прогибается на ДЛ. — 1 м. Оцените, во сколько раз сила, действующая в этот момент на гимнаста со стороны сетки, больше его веса. Линейные размеры сетки много больше ДЛ. Собственным весом сетки и сопротивлением воздуха пренебречь.
393
Решение. Так как по условию задачи сетка абсолютно упру hi н и при падении гимнаста прогибается на малую в сравнении г г» размерами величину, то с хорошим приближением можно считпн. что при всех деформациях сетки справедлив закон Гука
F = kx.	(II
Здесь к— коэффициент жесткости упруго растянутой сетки; х величина прогиба На гимнаста со стороны сетки действует макси мальная сила fin ах при х = ДА. Таким образом, можно написан.
fin ах ~ АДА.
По условию задачи требуется оценить величину отношения
fin ах   к Ah тд тд
Для этого рассмотрим переходы энергии из одного вида в друroll, совершающиеся при падении гимнаста на сетку. От начала падении до полной своей остановки гимнаст проходит расстояние h + ДА и его потенциальная энергия уменьшается на величину
W = Tng(h + &h),	(3|
а кинетическая — не меняется. Работой сил сопротивления вощу ха и изменением потенциальной энергии сетки в поле тяготения пн условию задачи можно пренебречь. Таким образом, начальная in» тенциальная энергия гимнаста (3) переходит в потенциальную энг|» гию упругой деформации сетки, растянутой на величину ДА, т. с
Ж = *(ДЛ)2/2.	(II
Приравнивая правые части (3) и (4), получаем следующее соотпп шение:
^А(ДА)2 = mg(h + ДА).
Отсюда
А ДА
— = 26 тд \
Из (2) и (5) следует, что
^• = 26 + тд \
Таким образом, максимальная сила, действующая на гимнаста при падении на сетку, превосходит его вес в 26 раз.
394
ДОБАВЛЕНИЯ
I.	Движение шарика в жидкости (комментарий к закону Архимеда)
Найдем ускорение, с которым начинается движение шарика с плотностью рт, полностью погруженного в жидкость с плотностью р.
Прежде всего рассмотрим небольшой объем жидкости, имеющий вид длинного цилиндра, направленного вдоль оси Ох, с высотой Дж и площадью основания S. Давление в жидкости р(ж,у, z). Площадь основания цилиндра настолько мала, что давление во всех точках основания можно считать одинаковым. Пусть координата одного основания цилиндра ж, а координата другого основания ж + Дж. Вдоль оси Ох на цилиндр действуют сила давления Fx = (р(ж,у, z) — р(ж + Дж, у, z))S, сила тяжести тдх = pAxSgx (здесь дх— проекция ускорения свободного падения на ось Ох). Считаем, что выделенный объем покоится, т. е. имеет нулевую скорость по отношению к окружающей жидкости, и поэтому не рассматриваем сил вязкого трения, которые могут возникать при относительных смещениях соседних слоев жидкости.
Под действием перечисленных выше сил выделенный объем жидкости может двигаться с некоторым ускорением ах :
pS Ахах = (р(ж,?/, z) - р(х + Дх,у, z))S + рДх Sgx.
Из этого уравнения движения получаем p(x,y,z) -р(ж + &xty,z)
или переходя к пределу Дж —> 0:
Аналогичные соотношения имеют место для осей Оу и Оz. В векторной записи искомое уравнение движения жидкости имеет вид
-Vp=p(a-g).
395
Для несжимаемой жидкости можно внести р под знак градиентп и обозначив р/р = Ф, получить зависимость
-УФ = а-^,	(1|
аналогичную связи потенциала с напряженностью электрически!•• поля. Взяв дивергенцию от обеих частей соотношения (1) и учи гы вая уравнение неразрывности div v = 0, получим для приведенном' давления Ф уравнение Лапласа:
ДФ = 0.	(7|
Это уравнение было впервые получено Эйлером, а впоследствии пи лучило название уравнения Лапласа.
Для вычисления выталкивающей силы, действующей на поперм ность тела, погруженного в жидкость, необходимо знать давление на поверхности тела, т. е. решить уравнение (2) с учетом граничных условий конкретной задачи.
Рассмотрим движение шарика радиусом Я, погруженного в ж ни кость. Будем считать, что размеры шарика много меньше размерив сосуда и расстояния до поверхности жидкости. Для решения in кой задачи надо сформулировать граничные условия на поверхни сти шарика и на бесконечности. Для этого выберем сферичсскуи» систему координат с началом в центре шарика. Ось Oz направим вертикально вверх. Точки на поверхности шарика будут описыва и. ся координатами (/?, 0, ^), причем зависимость давления от <р отеу i ствует в силу осевой симметрии задачи. Если плотность шарика /», отличается от плотности жидкости р, шарик начнет двигаться с и» которым ускорением а. Точки на поверхности шарика будут имгп. то же самое ускорение а. Составляющая этого ускорения по норма ли к поверхности будет равна an = acos0. Жидкость у поверхног i и шарика будет смещаться с ускорением, нормальная составляющий которого будет такой же.
Используя связь (1), можно записать первое граничное условие для уравнения (2):
= (a + £)cosfl,	(3)
r=R
Знак плюс перед д в правой части (3) вместо знака минус перед ц в правой части (1) появился из-за того, что вектор g направлен вер тикально вниз. На бесконечности влияние шарика на распределение давления в жидкости должно отсутствовать, и само давление буде i
5Ф дп
396
зависеть от вертикальной координаты z по обычному линейному закону. С учетом связи z = г cos 0 в сферической системе координат можно написать:
ф1г-юо = фо - дг соз0.	(4)
Здесь- введена величина Фо = Ф|Г_НЮ| ^=7Г/2 —значение искомой функции на уровне z — 0. На этом описание граничных условий закончено, и мы можем искать решение уравнения (2) с учетом (3) и (4).
Общий вид решения уравнения (2) запишем в виде ряда по полиномам Лежандра:
Ф(г, 0) = 52 tf(cos0)	+ В(г') ,	(5)
1=0
где Ai и Bi—неизвестные постоянные коэффициенты. Подставляя общее решение (5) в граничное условие (3), получаем, что все коэффициенты Bi = 0 кроме Во = Фо и В± = -д. Из условия (4) получаем, что равны нулю и все Ai кроме А± = aR3/2. Общее решение (5) принимает вид
/ а рЗ \
Ф(г, 0) = Фо + cos 0 ( —j- - gr 1.	(6)
Давление на поверхности шарика (г = R):
Ф(Я, 0) = Фо + cos 0	- gli).	(7)
Теперь можем вычислить силу гидростатического давления, действующую на шарик:
Знак минус в правой части равенства (8) отражает тот факт, что сила давления направлена в каждой точке поверхности по нормали “вовнутрь” тела, а вектор dS направлен по нормали “наружу” от замкнутой поверхности. В силу осевой симметрии задачи вектор FA направлен вертикально. Спроецируем векторное равенство (8) на вертикальную ось:
[ [p$(R,0)cos9dS
397
и подставим в интеграл выражение (7):
Проинтегрируем это выражение по поверхности шарика с учетом того, что в сферической системе координат dS = R2d cos 0 dip. Первое слагаемое при интегрировании обратится в ноль, а остальные дадут
(д - £) .
О	V аи /
Видим, что полученный результат отличается от силы Архимеда, вычисленной при ускорении а = 0: F& = ^TrR3pg. При а > 0 (шарик всплывает) < Fa, а при а < 0 (шарик тонет) ЕД > F&. Теперь мы можем написать уравнение движения шарика
4	о	4	о 4	о
-тгЯ3рта = --тгЯЗртд + -тгЯгр О	и	и
откуда и найдем ускорение а:
а = 2д
Р Рт
Р 4“ ^Рт
398
II.	Столкновения тел при наличии трения
Абсолютно упругое столкновение тела с шероховатой поверхностью подробно рассмотрено в задаче 73.
Рассмотрим другой предельный случай — абсолютно неупругий удар. В этом случае в процессе удара полностью гасится составляющая импульса тела, направленная по нормали к поверхности. Полный импульс силы, действовавшей в процессе удара по нормали к поверхности, будет равен (в обозначениях задачи 73)
=pi,
изменение импульса тела вдоль поверхности
<5рц = FT6t = min {pp_L,P||},
а импульс тела после удара будет направлен вдоль поверхности и равен
pfl = тах{рц - рр±,0}.	(1)
Таким образом, при рц > ppj. тело после удара будет скользить вдоль поверхности, замедляясь под действием сил трения и тяжести (величина силы трения будет зависеть от силы нормального давления, которая, в свою очередь, определяется ориентацией поверхности).
При рц < ppi тело после удара останавливается, а его дальнейшее состояние будет также зависеть от ориентации поверхности.
В приведенных рассуждениях предполагается, что тело не переворачивается в процессе удара. Условия, необходимые для выполнения этого предположения, зависят от формы тела. Так, например, для кубика легко увидеть, что при р < 1 и столкновении с поверхностью всей гранью куба переворота не будет.
В некоторых задачах рассматриваются процессы, аналогичные столкновению при наличии трения. Рассмотрим для примера тело, съезжающее по наклонной плоскости с углом а в основании и продолжающее движение по горизонтальной поверхности. Нас будет интересовать только момент перехода с наклонной плоскости на горизонтальную поверхность. Пусть скорость тела в нижней точке наклонной плоскости была равна vq. При абсолютно упругом ударе тела о горизонтальную поверхность, оно подскочит с некоторой скоростью (аналогично удару в задаче 73). При абсолютно неупругом
399
ударе из равенства (1) следует, что скорость тела будет направлкчн» вдоль горизонтальной поверхности и равна
v = vgmax {cos а — sin а, 0}.
Рис. 139
Можно рассмотреть и другую модель перехода тела с наклонной на горизонтальную поверхность — сделать этот переход плавным, сопрягая две плоскости с дугой а окружности некоторого радиус п R (рис. 139). Тело переходит с наклонной плоскости на дугу, со вершает поворот па угол а и выходит на горизонтальную поверх пость. Как меняется его скорость при движении по дуге? Прежде всего будем считать, что радиус окружности настолько мал, чи» изменением скорости под действием силы тяжести в процессе дни жения по окружности можно пренебречь (другими словами, измг нение потенциальной энергии тела много меньше его кинетической энергии). Можно считать, что тело двигается под действием силы реакции опоры, создающей центростремительное ускорение v2/R> и силы трения скольжения, уменьшающей скорость. Введем для удоб ства угловую координату тела на дуге: пусть она меняется от 0 до а. Если тело сдвинулось по дуге на небольшой угол 6а, пройдя при этом путь R6a, то его энергия уменьшилась на величину <5 (nw2/2) за счет работы силы трения, равной iimv2/R:
w г» Г с f ™v* 1
Суммируя эти равенства для элементарных перемещений по всей дуге, получаем интегральное равенство:
J0	J vo V
или
= - In (v2/vo) >
Следовательно, скорость тела после прохождения дуги равна v = voe“Ma.
400
Выше речь шла о столкновениях, не приводящие к переворотам тела. Теперь рассмотрим столкновение тела, которое может поворачиваться, с шероховатой плоскостью. Эффекты, связанные с поворотом, легче всего рассмотреть и а примере обруча.
Пусть плоскость обруча перпендикулярна поверхности и скорость центра масс обруча лежит в этой же плоскости. Абсолютно упругий удар обруча, налетающего на стенку по нормали, был рассмотрен в задаче 80. Здесь рассмотрим обруч радиусом Я, налетающий на стенку под углом а к нормали, вращаясь с угловой скоростью о>о (рис. 140, а).
Начальную скорость обруча vo разложим на две составляющие: Гц и . В процессе удара импульс силы реакции опоры равен удвоенному импульсу обруча по направлению нормали 2mvj_. Как и в задаче 80, этот импульс силы приводит к появлению импульса силы трения 2pmvj_. Будем, для определенности, считать, что иц > т. е. точка соприкосновения обруча и поверхности проскальзывает по поверхности в ту же сторону, куда направлена скорость гц. При этом импульс силы трения уменьшает поступательную скорость вдоль плоскости до значения глц — 2/iv± и увеличивает угловую скорость до о>о + 2pv^/Rj если гц > Rwq + 4/zvj., в противном случае конечная скорость вдоль плоскости будет равна (иц 4- Ясио)/2, а конечная угловая скорость (^о 4-^ц/Я)/2. Заметим, что при определенных соотношениях между параметрами задачи скорость вдоль плоскости в процессе удара может измениться не только по величине, но и по направлению; при этом обруч отскочит “назад” по отношению к первоначальному направлению движения. Обруч отскочит строго назад, если гц = и о?о = —3/wi/J?.
Аналогичный анализ при г?ц < wqR приводит к следующему результату: при Ruo > V|| 4- 4/wjl конечная скорость вдоль плоскости равна vy 4- 2^vi, конечная угловая скорость — 2^vj_/7?; при Rwq < V|| 4- 4/wj. конечная скорость вдоль плоскости (иц 4- Ядъ>о)/2, а конечная угловая скорость о»о/2 4- иц/(27?).
401
III.	Взаимодействия при наличии нецентральных сил
Общая картина движения тела в поле с потенциалом С7(г), m висящим только от расстояния г до некоторой точки*, подро(»п» > рассмотрена в задаче 100. Там показано, что в этом поле па I действует сила F, направленная против вектора г и равная
dU(r) dr
или в векторной форме
YdU(r} г dr
(И
По III закону Ньютона сила F', действующая со стороны толп н<» источник поля, равна силе F и направлена в противоположную <• i• • рону:
F, = _F = £d£(r) г dr
Рис. 141
Для нецентрального потенциала силы “действия” и “противо,иН1 ствия” направлены, вообще говоря, не по линии, соединяющей точи и приложения этих сил, а по параллельным линиям, как показано ни рис. 141. Величины этих сил, как и при центральном потенцию!» равны, а направления противоположны. При этом оказывается, ч i • • суммарный момент сил в системе двух взаимодействующих тел /I и В отличен от нуля!
* Такой потенциал называется центральным. Любой другой (нецелтри»»» ный) потенциал зависит не только от величины радиус-вектора (г = |г|), пи и от его направления.
402
Докажем, что отличный от нуля момент сил, действующих на некоторую систему из N тел, приводит к изменению ее момента количества движения.
Выпишем уравнения движения для каждого из N тел в следующем виде:
F, А/ = mi Av; = Ар,, г = 1,2,..., TV.	(2)
Введенные здесь обозначения очевидны: F,—суммарная сила, действующая на тело г; v, и р, —скорость и импульс этого тела в данный момент; А/— промежуток времени, малый настолько, чтобы можно было пренебречь изменением любых параметров системы.
Умножим равенство (2) слева векторно на радиус-вектор точки приложения силы F»:
[г,-xFi]Af = [г,-х Др,], г = 1,2,...,Л\	(3)
Правую часть равенства (3) можно переписать в виде A [r: х р,], так как имеет место тождество A [r< х pj = [г, х Ар,] 4- [Аг» х pj. I [оследнее слагаемое в правой части этого тождества обращается в ноль в виду того, что Аг,- || р* (векторное произведение параллельных векторов равно нулю).
Проведем в (3) суммирование по г:
(JV	\	дг
^2 [г< X F,]) Д/ = Д [г< X р,].
»=1 /	: = 1
В левой части этого равенства стоит импульс суммарного момента сил М, а в правой— изменение суммарного момента количества движения L всей системы:
МА/ = AL.	(4)
Из этого равенства следует, что отличный от нуля момент сил приводит к изменению вектора момента количества движения системы.
В системе двух тел, изображенной на рис. 141, суммарный момент сил отличен от нуля. Такая система должна, в соответствии с формулой (4), ускоренно вращаться относительно общего центра масс, что противоречит, в частности, закону сохранения энергии. Могут ли существовать системы, подобные изображенной на рис. 141?
Приведем конкретный пример такой системы (рис. 142). В точке А расположен заряд д, а на расстоянии I в точке В— диполь,
403
состоящий из зарядов +q и — </. Длина диполя d I. Покажем, чт»• потенциальное поле, создаваемое диполем, нецентральное.
F
Рис. 142
d
Рис. 143
Для этого обратимся к рис. 143 и вычислим потенциал в про извольной точке па расстоянии R от центра диполя под углом 0 к вектору d, соединяющему заряды диполя:
= |R-d/2| “ |R + d/2| ‘	(Г,)
Это выражение при расстояниях R^> d можно упростить, используя разложение по малой величине d/R'.
1 _______________1________~ 1Л +
|R Т d/2|	^008 0 +</2/4 ~ /Д 2Л C0S ) 
С учетом этого приближения формула (5) может быть переписали в виде
^(R) =	tf3dR-
Видим, что потенциальное поле диполя не обладает сферической симметрией —потенциал явно зависит не только от расстояния //, но и от угла 0.
Силы Fi и F2, действующие на заряд со стороны диполя, равны по величине. Их векторная сумма равна F. Силы, действующие нп диполь со стороны заряда, равны —Fi и —F2 соответственно, а их сумма —F. Момент сил F и — F относительно любой точки отличен от нуля, направлен по часовой стрелке и равен М = FI. Но заметим, что момент сил —Fi и —F2 тоже отличен от нуля, направлен про тив часовой стрелки и равен М' « Fid. Из сравнения треугольника сил Fi, F2, и F с треугольником, в вершинах которого находятся заряды, видим, что Fi/F « l/d. Отсюда получаем, что F\d « Fl и М = М*> а в векторной форме М = — М'. Следовательно, суммарный момент сил, действующих в этой системе, равен нулю. Заряд,
404
находящийся в точке 0, действуя на диполь, расположенный в точке Л, создает не только силу — F, но и момент сил —М\
Таким образом, взаимодействие в нецентральных полях описывается не только силами, но и моментами сил, т. е. имеет более сложный вид, чем в взаимодействие центральных полях.
405
IV. Движение центра масс изолированной механической системы
Известно, что в силу II и III законов Ньютона центр масс толп или системы тел не может прийти в движение за счет одних лишь внутренних сил (без воздействия извне). Так, например, трудно по верить рассказу барона Мюнхгаузена о том, как он, почти утонув в болоте, сумел выбраться, вытащив себя за волосы. Тем не менее, воспользовавшись предложенным ниже способом, барон смог бы вы браться из болота.
Представим себе человека массой М, который раскрутил в вер тикальной плоскости, гирю массой т до некоторой скорости в (рис. 144). Гиря движется по окружности радиусом г (длина ру ки человека) под действием силы тяжести ?ng и силы натяжения Т Центростремительное ускорение гири йцс — ^2/г. М атем ати чес кап запись II закона Ньютона применительно к гире имеет вид
о
Г mg + T =--------.
Г г
Напишем это равенство для верхней точки траектории гири в про екции на вертикальное направление:
т ж'2
----.	(I) г
Пока человек стоит на земле, на него действуют сила тяжести сила со стороны гири —Т и сила реакции опоры. Если человек оторвется от земли, сила реакции опоры обратится в ноль. По II закону Ньютона для человека
Mg - Т — Ма
406
в проекции на вертикальное направление:
Мд - Т = Ма.
(2)
Из равенства (1) видно, что, раскручивая гирю с достаточно большой скоростью можно добиться того, чтобы величина Т стала больше Мд, так что ускорение человека в равенстве (2) может оказаться отрицательным, т. е. направленным вверх, следовательно человек оторвется от земли.
Барон Мюнхгаузен, применив этот способ, действительно смог бы выбраться из болота без посторонней помощи!
Покажем, что рассмотренный пример не противоречит II закону Ньютона.
Из (1) и (2) следует, что человек отрывается от земли при скорости гири
IM + m v > \ ------гд,
V m
а его ускорение в этот момент равно
mv2 (М 4- тп)д
Mr М
(3)
Человек движется вверх с ускорением а, а гиря при этом движется вниз с ускорением v2/г. Ускорение центра масс системы ао определяется формулой
_ 1
а° m + М
Подставляя сюда вместо а выражение (3), получаем а0 = — д в соответствии со II законом Ньютона.
Рассмотрим другой пример механической системы, которая приходит в состояние движения без видимых внешних воздействий.
Рис. 145
Рис. 146
407
Представьте себе тележку, на которой установлен закры п.Н1 ящик (рис. 145), никак не связанный с колесами. Что находи пн в этом “черном ящике”—неизвестно. В какой-то момент телшим начинает движение и в дальнейшем движется с некоторой скор»» стью, несмотря па наличие сил трения. Что же заставляет телгжн двигаться?
Приведем пример устройства, работа которого вызовет движешь тележки. На рис. 146 показано содержимое “черного ящика”: лиги, который вращается с постоянной угловой скоростью о;, стержни., шарнирно соединенный с диском и с муфтой. Муфта может ст» бодно скользить по горизонтальному неподвижному штоку. Полип и масса тележки равна Л/, масса муфты т. Те, кому посчастливил< и i видеть паровоз, могут узнать в этом механизме один из элемс1пнн паровоза, работающий “в обратную сторону”. Действительно, у пи ровоза аналогичный механизм преобразует поступательное движг пие поршня паровой машины во вращательное движение колеси II нашем случае вращательное движение диска преобразуется в пос । у пательное движение муфты. А причем же здесь колеса тележки, и почему она будет двигаться?
Рассмотрим относительное движение частей системы. Пуги, муфта смещается вдоль штока от некоторого среднего положении по закону
я = A coscut,	(4)
скорость муфты относительно тележки
v = —Аы sina?/.	(li)
При этом тележка будет двигаться относительно.земли с некоторой скоростью V2, а муфта — со скоростью
Vi = v 4- v2.	(В)
Если на систему “тележка+муфта” никакие силы в горизонталь ном направлении не действуют, ее центр масс должен находиться и покое, т. е.
mvi + (Л/ — m)vz = 0.	(7)
Из равенств (5), (6) и (7) находим скорость тележки и скорость муфты относительно земли
т	т	.
V2 = ——V = —Awsmwt	8
М	М
408
При таком движении тележка периодически смещается от некоторого начального положения и, в среднем, остается на месте.
Теперь поставим на колеса стопор, который позволяет им вращаться только в одну сторону (рис. 146 — по часовой стрелке). Кроме того, сделаем трение скольжения между колесами и землей достаточно большим, чтобы колеса не проскальзывали при включенном стопоре. В результате таких действий тележка сможет двигаться только в одну сторону (на рис. 146 — направо). При этом к выписанным выше уравнениям (4)—(6) добавляется условие V2 > О, а закон сохранения импульса (7) перестает выполняться, так как на тележку в горизонтальном направлении при включении стопора действует сила трения.
Пусть движение механизма начинается так, как описано формулами (5), (8) и (9), справедливыми до момента Т/2 (здесь Т = 2тг/о? — период). В первую половину периода тележка движется направо и стопор не работает.
Вторую половину периода тележка должна была бы двигаться налево, но из-за стопора она некоторое время будет стоять на месте, а муфта будет двигаться направо. Ее скорость определяется выражением (5), так как при v? = 0 имеет место равенство v = t>i. Видно, что скорость их увеличивается и достигает максимального значения в момент ЗТ/4. Центр масс системы до этого момента движется ускоренно направо, следовательно, сила трения скольжения между колесами и землей действует на систему направо. При I = ЗТ/4 ускорение груза меняет направление (его скорость начинает уменьшаться) и, если бы тележка оставалась неподвижной, ускорение центра масс было бы направлено налево. Однако сила трения скольжения налево действовать не будут, так как колеса по часовой стрелке могут вращаться свободно (сила трения качения обычно очень мала по сравнению с силой трения скольжения). Следовательно, начиная с / = ЗТ/4 центр масс будет двигаться равномерно направо.
Количество движения системы при t = ЗТ/4 равно
ГПЩ 1зТ/4 =
и будет сохраняться, пока стопор не сработает снова. Из равенств (5), (6) и закона сохранения импульса в рассматриваемый период
nwi + (М — m)v2 = тАы
следует
777,
V2 = — АЛ1 -f- sin о;/), М
(10)
409
т . М ~ т
vi — — Аси------——Аси sincut.
М	М
(II)
Среднее значение скорости тележки в дальнейшем движении равни
т л ^2 = М
Графики функций (10) и (11) изображены на рис. 147 сплошной ли нией (штриховая линия соответствует функциям (8) и (9)). Из фор мулы (10) и графика видно, что скорость v? в процессе дальнейшего движения остается положительной, а стопор больше не работает.
Если теперь учесть возможное сопротивление движению тележ ки (трение качения и прочее), то скорость центра масс системы бу дет постепенно уменьшаться, и скорость тележки обратится в ноль не в момент времени 7Т/4, как это изображено на рис. 147, а раньше* (рис. 148). Стопор опять сработает, и тележка будет неподвижна до момента 7Т/4. Затем процесс будет периодически повторяться.
Рис. 148
Мы показали на двух примерах, что движение изолированной механической системы типа “черного ящика” может начаться без видимого воздействия со стороны окружающих тел. В действительности это воздействие существует и является причиной начала движения. В первом примере это — сила реакции опоры, которая в процессе вращения груза в некоторый момент превышает силу тяжести и двигает центр масс системы вверх. При дальнейшем движе
410
нии центам асе будет двигаться ускоренно вниз, следовательно сила реакции опоры становится меньше силы тяжести и может даже обратиться в ноль. Во втором примере воздействие на систему передается силой трения покоя, которая сообщает тележке импульс в горизонтальном направлении, когда препятствует ее движению в противоположную сторону.
411
V. Движение в потенциальных полях с трением
Рассмотрим необычную, с житейской точки зрения, ситуацию, которая возникает в некоторых потенциальных полях, когда нал и чие силы трения (сопротивления движению) приводит к увеличению скорости движущихся тел.
В качестве примера разберем задачу о движении тела в хороню известном читателю центральном потенциальном поле тяготения.
Пример. Спутник, двигаясь по орбите радиусом R = Лз в верх них слоях атмосферы, испытывает трение. За один оборот его эпср гия меняется на величину 8Е <0, малую настолько, что орбиту спутника можно считать круговой. Требуется найти изменение ско рости спутника 8v и радиуса орбиты 8R за один оборот.
Используя равенства (11) и (12) из раздела 4, получаем измене ние полной энергии тела как функцию расстояния и скорости соответственно:
SE = Е(П+1Я)-Е{Л) =	+	= 5™»^. (>)
SE=E(v + Sv) - Д(») = - t~	(2)
Из (1) и (2) находим
6R =	< 0, 5v = -— >0.
mgR£	mv
Итак, при трении в верхних слоях атмосферы радиус орбиты спутника уменьшается, а его скорость возрастает.
В этом примере наличие силы трения увеличивает скорость тела (уменьшая при этом полную механическую энергию).
Строго говоря, движение при наличии трения не является потенциальным. Но при малом трении подобное движение можно рассматривать как движение в потенциальном поле с медленным изменением параметров системы (в данном случае медленно меняются радиус, скорость, механическая энергия). Такие изменения параметров называют адиабатическими.
Выясним, в каких потенциальных полях наличие слабого трения приводит к увеличению скорости. Пусть тело движется по окружности радиусом R в поле с потенциалом U(R) = bRn, где b и п — некоторые константы. Сила, действующая на тело в таком поле,
412
равна (формула 11) в Добавлении III)
f RdU(r) L Rd„_i
F = - — —= -bn — R x.
R dr	R
Сила направлена к центру, т. е. имеет место притяжение, если
Ьп > 0.	(3)
Этот случай мы и будем рассматривать, так как в противном случае движение по окружности невозможно.
При движении по окружности радиусом R со скоростью v сила направлена к центру и равна mv2/R, откуда следует, что
Кинетическая энергия при таком движении
гр — г>п
-С'КИН — 2	’
а полная механическая энергия
„	„	п + 2,Л„ п + 2 mv2	. /J4
Е = Екин 4“ U(7?) = - bR =	- .	(4)
Z	п Z
При незначительном уменьшении Е скорость будет возрастать, если (п + 2)/п < 0. Это неравенство эквивалентно условию 0 > п > —2. В таких потенциальных полях скорость орбитального движения возрастает при наличии трения. Из (4) с учетом условия (3) следует, что радиус орбиты при движении с трением уменьшается.
А как будет влиять трение на орбитальное движение в других потенциальных полях? В соответствии с (4) при п > 0 уменьшение энергии приводит к уменьшению и скорости, и радиуса орбиты.
Аналогичным образом в случае п < —2 приходим к парадоксальному результату: при наличии трения скорость орбитального движения уменьшается, а радиус орбиты увеличивается! Чем больше энергии мы отберем у спутника, тем дальше он улетит! И вообще, из (4) следует, что в рассматриваемом случае полная энергия орбитального движения положительна, т. е. больше энергии тела в бесконечно удаленной точке. При стремлении же радиуса орбиты к нулю энергия, необходимая для кругового движения, стремится к бесконечности.
Чтобы разобраться в этом, вычислим две характерные величины— I и II космические скорости — в таком поле. Напомним, что I
413
космическая скорость—это скорость орбитального движения, а II космическая скорость — это минимальная скорость, которую надо сообщить телу на орбите радиусом R для того, чтобы оно улетело бесконечно далеко от притягивающего центра.
Скорость в формуле (4) и есть I космическая:
Найдем II космическую скорость из закона сохранения энергии
-^кин(Я) + U (Я) — ^кин(оо) + (7(оо).
Очевидно, что при минимальном значении начальной скорости, до статочном для достижения бесконечно далекой точки, правая часть этого равенства обращается в ноль, следовательно
(6)
Сравнивая выражения (5) и (6), видим, что при п < — 2 получается неравенство vj > иц! Это означает “нестабильность” орбитального движения в таких потенциальных полях. Движение по окружности при п < —2 формально имеет место, но любые возмущения приводят или к удалению тела на бесконечность, или к падению тела на притягивающий центр.
414
VI. Размерность в физике
В задачах с небольшим числом параметров, имеющих, к тому же, различную размерность, иногда удается определить структуру ответа, исходя только из анализа размерностей. Особенно полезен анализ размерностей в задачах, где нужно найти относительные изменения физических величин.
В качестве примера проведем анализ размерностей в задаче 161:
Пример 1. На горизонтальной равнине установлена пушка, стреляющая под углом а к горизонту. Во сколько раз надо увеличить начальную скорость снаряда, чтобы:
а)	дальность полета снаряда увеличилась вдвое;
б)	максимальная высота подъема снаряда увеличилась вдвое;
в)	время полета снаряда увеличилось вдвое?
Здесь мало исходных параметров, все они имеют различную размерность (начальная скорость и, ускорение свободного падения угол наклона ствола а), найти надо не абсолютные значения величин, а их относительные изменения.
При анализе размерностей употребляют следующие обозначения:
М —размерность массы,
L — размерность длины,
Т — размерность времени.
Размерность некоторой физической величины обычно обозначают прямыми скобками: [Л] — размерность величины А.
Возвращаясь к задаче, введем обозначения: t—время полета, h — высота подъема, I — дальность полета. Очевидно, что
И = Т, [Л] = [/] = £.
Как эти величины зависят от начальной скорости v ([u] = LT-1) и ускорения свободного падения д ([/?] = £Т“2)?
Построить из v и д величины с размерностью Т и L можно единственным (с точностью до произвольного числового множителя) образом:
[v/j] = Т,	(1)
[»2Zff] = L.	(2)
415
Следовательно, t ~ v/g, а I ~ Л ~ v2/д. Таким образом, для уп»1 личения высоты и дальности в два раза надо увеличить начальную • скорость в х/2 раз, для увеличения времени полета в два раза пали увеличить начальную скорость скорость в два раза.
В этом примере для получения ответа методом анализа размер ностей нам не пришлось использовать никаких формул кинематики Кинематическое же решение этой задачи хотя и не сложное, но до вольно длинное.
Теперь рассмотрим пример, в котором без анализа размерное гей ответ не может быть получен на элементарном уровне.
Пример 2. На поверхности Земли песочные часы отмеряют 5-ми путные интервалы. Оцените, какие интервалы будут отмерять тг же часы на высоте, равной двум радиусам Земли, считая, что силы трения песчинок не зависят от их скорости.
Интервал времени /, отмеряемый песочными часами, завис и i от ускорения силы тяжести д, размеров и формы часов, размером песчинок и т. д. Среди всех этих величин только д имеет размер ность, в которую входит время: [51] = LT~2. Отсюда ясно, что вы ражение для t должно содержать ускорение свободного падения д в степени —1/2:
t ~0_1/2.
Отсюда следует, что t(frl) = /g(M	,.п
*(Л2)
где t(h)—интервал времени, отмеряемый часами на высоте h над поверхностью Земли, и gfji) —ускорение свободного падения на этой высоте. Когда тело находится на поверхности Земли, h = hi = 0; во втором случае h =	= 27?з, где R3— радиус Земли. На тело
массой 771, находящееся на расстоянии h от поверхности Земли, с ее стороны действует сила
О _ Г, ,пМ3
которая сообщает телу ускорение
F Мз
Здесь G — постоянная всемирного тяготения, М3 —масса Земли. Полагая в равенстве (3) hi = 0, /го — 2Яз и принимая во внимание
416
формулу (4), находим
<(2Я3) =f/-A-i(0) = 3/(0) = 15 мин.
У 5(2Яз)
В общем случае мы можем воспользоваться методом анализа размерностей для получения выражений, состоящих из трех (или менее) физических величин независимых* размерностей.
Пусть /1, /2 и /3 — три величины различной размерности, из которых надо построить величину F. Пусть размерности всех величин известны:
[fi] = MaiLb‘Td‘, i =1,2,3;
[F] = MaLbTd.
Искомое выражение должно иметь вид
^~ЛЛ/2В/з°>	(5)
где А, В и D — некоторые числа.
Потребовав равенства размерностей в правой и левой частях формулы (5), получим
а =	Асц 4- Ва2 4- Раз,	(6)
Ь =	АЬг + Bb2 + Db3)	(7)
d =	Adi 4" Bd2 + Dd3.	(8)
Система уравнений (6) — (8) имеет решение, если размерности величин fi независимы. Решая эту систему относительно А, В и Р, находим показатели степеней в выражении (5), т. е. определяем искомую функциональную зависимость.
В классической физике все явления и процессы можно описать, используя или три вышеперечисленные основные размерности (масса, длина, время), или любые три независимые комбинации этих единиц. Соответствующие эталоны размерностей разрабатывались учеными на протяжении многих лет. На сегодняшний день общепринятыми являются эталоны: времени (секунда— 9192 631770 периодов колебаний, происходящих в атоме изотопа цезия-133), длины
* Независимость размерностей некоторого набора физических величин [/,] означает, что ни одна размерность не может быть выражена через комбинацию других.
417
(метр — путь, проходимый светом в вакууме за 1/299792458 секунды) и массы (килограмм — масса эталонной платино-иридиевой гири). Системы единиц, содержащие больше эталонных размерностей (например, в международной системе единиц СИ содержится четвертая "фундаментальная” размерность—сила тока), с необходимостью включают в свою структуру некоторые размерные константы, не имеющие никакого физического смысла. Так, диэлектрическая проницаемость вакуума в системе СИ равна
А2 -с4
е0 = 8,854187817-10"12----=•
кг • мл
Подобные константы возникают как дополнительные условия на избыточное число “фундаментальных” величин.
В так называемой “современной” физике — физике XX века— существуют интересные возможности формирования системы размерностей.
Экспериментально установлен и теоретически обоснован факт постоянства скорости света во всех инерциальных системах отсчета. Эта инвариантная скорость с = 2,99792458 • 108 м/с может быть принята за один из эталонов универсальной, не зависящей от нашего конкретного выбора, системы размерностей.
Положив с = 1, получаем систему единиц, в которой скорости безразмерны (измеряются в единицах скорости света), а длина и время измеряются в одних и тех же единицах.
Кроме скорости света в современной физике вводится константа h = 1,05457266 • 10“34кг • м2 • с-1 —постоянная Планка — квант момента количества движения. Эксперимент и теория утверждают, что любой орбитальный момент количества движения кратен Л, а собственный момент количества движения любого тела кратен Если положить Л = 1 и измерять моменты количества движения в единицах постоянных Планка, то в физике останется только одна размерность— метры, или секунды, или килограммы, или любая комбинация этих единиц. (В физике элементарных частиц принято все физические величины измерять в энергетических единицах — электрон-вольтах (эВ).)
В построенной на основе скорости света и постоянной Планка системе единиц многие физические константы оказываются безразмерными. Так, например, квадрат заряда электрона можно представить в виде
п 4тгЛс е * ТзТ*
418
Только постоянная всемирного тяготения остается размерной:
м3
G = 6,67259 • 10"11-г = he  2,2 • 1017 кг-2.
кг • с2
Если положить и G = 1, Л все физические величины станут безразмерными. Массы будут тгзмеряться в единицах
7710 = У?! 10~8 КГ’
расстояния — в единицах
/о = У^«10“35м,
а время — в единицах
*°=У5*1о-43с-
Эти единицы не удобны для использования, но они отражают фундаментальные свойства нашего пространства-времени.
419
VII. Редукция размерных констант
Продолжая тему размерности в физике (см. Добавление VI), ни кажем на конкретных примерах, что исключение размерных пари метров задачи (редукция) позволяет не только упростить вычисли ния, но и проясняет характер зависимости решения от значения ни раметров.
Идея метода редукции состоит в том, что некоторые размерные параметры конкретной задачи можно положить равными единице Для этого необходимо, чтобы размерности этих параметров были линейно независимыми. Такая процедура приводит к существен!к» му упрощению уравнений и позволяет или решить их в общем виде, или проанализировать характер решения.
Пример 1. Электрон с зарядом е и массой т с нулевой начальной скоростью с расстояния R падает на ядро с зарядом Ze. Как изме нится время падения электрона, если начальное расстояние увели чить в четыре раза?
Уравнение движения имеет вид
- Ze2 = та 4п€оГ2 ~ г2 ’
Произведем подстановку г = Rry t = ri. При этом ускорение а = Rr~2a. Уравнение движения примет вид
. _ kr2
mR3r2
Произвольный параметр г положим равным т = y/mR3/ky после чего уравнение движения приобретет простую форму
а = —г ,
с начальным условием г = 1 при 1=0. Решение этого уравнения (величина i) не зависит от параметра R. Следовательно, зависимость t от R определяется только видом параметра т:
i = ri = \/mR3/ki.
Увеличение R в четыре раза приведет к увеличению i в восемь раз.
420
Пример 2. Найти уравнения процессов с отрицательной теплоемкостью для идеального газа.
Рассмотрим семейство термодинамических процессов с идеальным газом, описываемых на плоскости р, V прямыми, проходящими через точку (ро, Vb) - Введем редуциЛванпые безразмерные переменные:	’
’Г = р/ро, Ч> = V/Vo, Т = Т/То.
Здесь То = PoVo/(vR)-
В этих обозначениях уравнение состояния принимает вид
7Г1р = Г,
(1)
определение теплоемкости
^Aip
С — (Jy + vH.—?—.
Дг
Точка (ро, Vo) на плоскости р, V переходит в точку с координатами % = 1,	= 1 на плоскости тг,
Уравнение прямой на плоскости тг,у>, проходящей через точку (1,1), имеет вид
7г-1 + 6(^-1) = 0,	(2)
где Ь — безразмерный параметр, определяющий угол /3 наклона прямой: b = — tg /3. Заметим, что угол наклона на плоскости тг, р совпадает с углом наклона на плоскости р, V.
Связь малых приращений переменных из уравнения состояния
(1)
<рДтг + 7гД^> = ДТ,
из уравнения процесса (2)
Дтг + ЬД<р = 0.
Исключая из этих равенств % и Дтг, имеем
Дг = Д<р ( — bip Ч— ) .
Подставляя полученный результат в определение теплоемкости и учитывая, что в исследуемой точке все переменные равны единице, находим
„ i т Al	„ i — ib + 2
С = vR I - + - 2 	= I'R ——7—
\2 — Dtp2 + Т) 1т=¥>=1	1 — 0
421
Из этого равенства видно, что при 1 < b < (/ + 2)/i теплоемкость си стемы отрицательна. Для тангенса угла наклона прямой получаем -1 > tg /? > —(г + 2)/г.
Легко видеть, что изотерма (Дт = 0) имеет в исследуемой точке наклон tg ,в = —1, а адиабата (С = 0) —наклон tg /3 = —+ 2)/г.
В использованном здесь методе исключены конкретные значения ро, Vo. Следовательно, любая кривая на плоскости р, V, проходящая через произвольную точку (ро, Vo) с наклоном, промежуточным между наклоном адиабаты и изотермы, описывает неравновесный процесс с отрицательной теплоемкостью.
Пример 3, Определить закон движения тела массой пг, подвеше-пого на пружине жесткостью к в поле тяжести с ускорением свободного падения д.
Уравнение движения этого тела
та = — кх + тд.	(3)
В этом уравнении исходно присутствуют три параметра: т} д. Покажем, что в действительности характер движения этого тела определяется уравнением, не содержащим никаких параметров.
Введем новые переменные х = qx, t = ri, где q и г— некоторые константы. При этом изменится и ускорение: а = qr~~a. В новых переменных уравнение движения (3) имеет вид
mqr~~a = — kqx + тд,	(4)
Теперь выберем значения констант: q = тд/ку т = у/т/к и перепишем уравнение движения (4):
а = -х + 1.	(5)
В уравнении (5) не осталось параметров. Более того, сдвигом х -> х + 1 приводим (5) к уравнению вида
а — — х.
Таким образом, характер движения тела в рассмотренном примере не зависит от параметров задачи и описывается уравнением гармонических колебаний.
Пример 4- Определить закон движения тела массой т, подвешенного на пружине жесткостью к в поле тяжести с ускорением свободного падения д и помещенного в вязкую среду с сопротивлением, пропорциональным скорости (коэффициент пропорциональности р).
422
Уравнение движения в этом случае имеет вид
та = —kx — fiv -f- тд.
Производя те же замены, что и в предыдущем примере, и полагая у, =	v = qr-iv, получаем уравнение
а = —х — fiv.
Видим, что в уравнении движения остался один безразмерный параметр Д, значения которого и определяют характер процесса — колебания с затуханием (при Д < 2) или монотонное движение к положению равновесия (при Д > 2).
423
VIII.	Процессы с постоянной теплоемкостью
Выведем уравнение процесса с постоянной теплоемкостью для идеального газа. Общее определение теплоемкости
формулы для идеального газа, приведенные в первом разделе,
Cv = \vR, Cp = l-^uR z	z
и закон сохранения энергии
AQ = l-vRAT + pAV	(2)
Z
позволяют написать соотношение
ДИ c = cv+p—.
Используя уравнение состояния идеального газа р — vRT/V или р = (Ср - Cv)T/V\ получаем
дт	ДИ
(C-Cv)— = (CP-Cv)^.
Это равенство справедливо лишь при условии
TC-CvVCv-Cp _const.	(3)
Учитывая, что из уравнения состояния Т ~ рИ, запишем (3) в виде
pc-CvyC-cp _ constj или ру с-Ср _ const.	(4)
Для адиабатических процессов С = 0, и уравнение адиабаты принимает вид
~р
рУ?у=рУ i = const.	(5)
Из равенств (4) следует, что в процессе, в котором давление и объем связаны соотношением
рИ7 = const
424
при значениях 7 в интервале Ср/Су > 7 > 1, система имеет отрицательную теплоемкость.
В процессах, отличных от (4), теплоемкость неапостоянна. Ее значения в каждой точке определяются углом наклоВа касательной к графику на плоскости р, V. Подробный анализ этой зависимости приведен в Добавлении VII (пример 2).
425
IX.	Исследование простейших термодинамических процессов
Для вычисления коэффициента полезного действия тепловой мп шины, работающей по замкнутому циклу, необходимо определять знак выражения
Д<? = ^vR&T + pAV	(I)
на разных этапах цикла. Действительно, как известно, КПД есть отношение совершенной работы (т. е. площади цикла на плоскости р, У) к количеству теплоты, переданной рабочему телу. Для вычи сления этого количества теплоты надо сложить все положительные значения AQ, вычисленные по формуле (1). На тех участках зп мкнутого цикла, где величина AQ отрицательна, тепло переходит от рабочего тела в окружающую среду.
Из уравнения состояния идеального газа следует связь малых изменений параметров р, V и Т:
рД7 + Vkp=vfiAT.	(2)
Рассмотрим некоторый участок графика процесса на плоскости р, V, описываемого прямой линией, проходящей через точку (ро, Vo):
p = p0 + a(V-V<0).	(3)
Здесь а— параметр, определяющий наклон прямой к оси V. Из равенства (3) следует еще одно соотношение между изменениями параметров:
Др = аД V.	(4)
Учитывая (3) и (4), получаем вместо равенства (1)
д<?= Q(p + aV)+p) ДК	(5)
Из (5) следует, что тепло к системе подводится на прямом участке процесса в окрестности точки (po,Vo) при условии, что параметр наклона при Д V > 0 удовлетворяет неравенству
426
а при ДУ < О— неравенству
г + 2 ро
2 Уо
График любого процесса на плоскости р, У в ок
(П
Четности любой
точки (р0) Vb) можно представить отрезком прямой с определенным параметром наклона а. В тех случаях, когда этот параметр удовлетворяет неравенствам (6), (7), к системе подводится тепло и его
надо учитывать при нахождении количества теплоты, полученной
системой от нагревателя в замкнутом процессе.
427
X.	Модель атома Бора
Объяснить закономерности спектров атомов и описать их строг ние можно в модели атома Бора на основе корпускулярно-волновым представлений. В этой модели постулируется, что электрон в атом»1 может находиться только на определенных (разрешенных) орбитах Их параметры определяются из требования, чтобы на орбите ради усом г укладывалось целое число волн, т. е.
Л х Л
2тгг = пл = п —,
Р
откуда следует
nh	/и
mvr=2ir'
Вместе с тем электрон (заряд е) в атоме движется под действием сил притяжения со стороны ядра (заряд Ze) по орбите радиусом г со скоростью v.
mv2 * _ 1 Ze2
Г 4?Г£о г2
Из равенств (1) и (2) можно получить радиус орбиты
2 ЕоЛ2
Г~П irmZe2
и скорость электрона
1 Ze2 
V —-------.
2бо пЛ
Низшая орбита (n = 1) в атоме водорода (Z = 1) называется боровской орбитой. Для этой орбиты радиус и скорость принимают значения
го « 0,5 • 10"1Ом, vo « —.
1о I
Величина го характеризует размеры атома, а так как vo <£ с, то электрон в атоме можно считать нерелятивистским по крайней мере для легких ядер (при Z < 100).
Зная скорость электрона, можно вычислить его энергию на разрешенной орбите:
_	1 mZ2e4 * * * * *
~ ”8^ п2Л2 ‘
428
Энергия электрона на боровской орбите Е « —13 эВ. Такая энергия требуется для ионизации атома водорода. На других орбитах электрон имеет большую энергию (возбужденные состояния). Переходы электрона с одной орбиты (из состояния «нергией Е') на другую (в состояние с энергией Е) происходят с Опусканием или поглощением фотонов определенной частоты:
hv = E' - Е.
429
XI.	Постоянство скорости света как следствие принципа относительности
В основе теории относительности лежит известный и в ньютонок ской механике принцип равноправия инерциальных систем отсчета Этот принцип был распространен на все физические явления, вклю чая электромагнитные.
Новым в релятивистской физике стал постулат постоянства ски рости света в вакууме (с) во всех инерциальных системах отсчета а ее независимость от скоростей источника и приемника излучен их Этот постулат противоречит классическому закону сложения ско ростей.
Принцип и постулат теории относительности позволили вывг сти законы преобразования физических величин при переходе in одной инерциальной системы отсчета в другую, отличные от клас сических.
Наблюдатель в одной инерциальной системе отсчета описывает событие, произошедшее в точке с координатами ж, уу z в момент вро мени /. В другой инерциальной системе, движущейся со скоростью и вдоль оси Ох относительно первой, это же событие наблюдает ся в точке с координатами а/, у', z* в момент времени tl. Связь этих координат и моментов времени определяется преобразованиями Лоренца:
t> = t - (и/с2)х
— и2/с2 ’ t х — ut
^/1 — и2/с2 ’
V = У,
(1)
(2)
(3)
(4)
Приведем вывод преобразований Лоренца, основанный только на принципе относительности (т. е. на равноправии всех инерциальных систем отсчета). В этом выводе постулат постоянства скорости света не используется, а оказывается следствием принципа относительности.
Рассмотрим две инерциальные системы отсчета Л' и А'', движущиеся друг относительно друга со скоростью и. Выберем декартовы координаты в этих системах отсчета так, чтобы в начальный момент времени начала координат совпадали, а оси были параллельны.
430
В одной системе отсчета моменты времени и координаты вдоль направления и будем обозначать t и х соответственно, в другой системе отсчета— f и
Оси Ох и Ох' направим так, чтобы система К' двигалась со скоростью и относительно системы К вдоль оси Охь а сисЖма К— со скоростью — и вдоль оси Ох' системы К'.
Равномерное движение свободной материальной точки вдоль оси О а? в системе К описывается уравнением
x = a?o + vt,	(5)
а в системе К'— уравнением
х* ** = a?Q 4- vftf.	(6)
Преобразование координат х, t системы К в координаты х*, tf системы К' должно быть таким, чтобы уравнение (5) переходило в уравнение (6). Это означает, что прямая на плоскости х9 t должна переходить в прямую на плоскости х* 9 t*. Таким свойством обладает линейное преобразование*:
tf = d[u)t + е(и)х)	(7)
я' = а(и)ж +	(8)
где а (и), Ь(и), сЦи), е(и) зависят только от скорости относительного движения систем отсчета и.’*
Повернем оси координат в двух системах отсчета вокруг некоторого направления, перпендикулярного скорости и, на 180°. Это приведет к замене х и х* на —х п —х' соответственно. Если теперь
* В книге В. А. Фока “Теория пространства, времени и тяготения” (1961) в Добавлении А на стр. 510-514 показано, что самым общим преобразованием, переводящим прямую в прямую, является дробно-линейное преобразование. Однако, если ввести дополнительное физическое требование, чтобы точки, наг ходящиеся на конечных расстояниях в системе К', находились на конечных расстояниях в системе преобразование сводится к линейному.
**Для упрощения рассматриваем соотношения (7), (8), включающие только преобразования координат х, t. В более общем виде надо было бы включить и преобразования координат у, z:
t' =	4-	ata:(u)a;4- aty(u)y 4- atzfujz,
x' = axt(u)t +	4- axy(u)y + аЛ2(ф|
у* =	4-	ayx(u)z? 4- ayy(u)y 4- ayz(u)zf
z = a2t(u)t 4-	4- a>zy(u)y + azz(u)z.
Предлагаем читателю самостоятельно проделать вывод в общем виде и получить равенства (3), (4).
431
заменить направление скорости и на противоположное, т. е. на —и, то преобразования (7), (8) примут вид
if = d(—u)i — е(—и)х,	(9)
xl = а(—и)х — b(—u)i.	(10)
Заметим, что произведенные преобразования привели к тому, что система К1 движется, как и прежде, вдоль оси Ох системы К со скоростью и. В силу изотропности нашего пространства (равноправия всех направлений) вид равенств (7), (8) не зависит от направления осей Ох (Ож'), следовательно формулы (9), (10) должны совпадать с (7), (8). Это возможно, если функции a(u), d(u) четные, а функции 6(u), е(и) нечетные.
Теперь учтем, что точка х = 0 движется со скоростью —ив системе /<': xf = —uif. Из уравнений (7), (8) следует 6(u) = — ud(u). Аналогично точка х* = 0 движется со скоростью и в системе К: х = ui. Из тех же уравнений следует Ь{и) = —ua(u) или а(и) = rf(u). Вводя для симметрии вместо нечетной функции е(и) четную функцию д(и) по формуле е(и) = — иад(и)} получаем
tf = а(и)(/ — ид(и)х),	(11)
xf = a(u)(z — ui).	(12)
Рассмотрим еще одну инерциальную систему отсчета К", движущуюся со скоростью и' вдоль оси Ох' системы К'. Закон преобразования из системы К* в систему К" должен иметь вид, аналогичный (И). (12):
i" — а^и')^ — и/^(и/)ж/),	(13)
х" = a(u,)(a?' — u7/').	(14)
Подставляя в (13), (14) выражения для х" > ift (7), (8), получаем
t" = a(u')a(u)(l + ии'д(и’)) (t -	ж) >	(i5)
*" = a(u')a(u)(1 + ии'д(и)) (® - t	•	(16)
С другой стороны, равенства (15), (16) описывают переход из системы Л’ в систему К", движущуюся вдоль оси Ох с некоторой скоростью и":
t" =	- и"д(и")х),	(17)
х" = а(и")(х - u"t).	(18)
432
Из сравнения соотношений (17), (18) с (15), (16) следуют равенства
д(и) = д(и') = д = const,
и и + и'
и = ----------j
1 4- uufg
а(и") = a(u)a(u')(l +
(19)
(20)
(21)
Равенство (19) вводит некоторую постоянную величину, размерность которой — обратный квадрат скорости. Эта величина одинакова во всех системах отсчета, и на современном уровне развития теоретической физики ее численное значение не может быть выведено из каких-либо общих принципов. Экспериментальное значение этой величины д = с“2, где с — скорость света в вакууме. В классической механике (нерелятивистской) д = 0.
Равенство (20) определяет закон сложения скоростей для движения вдоль оси Ох (Ох1}: если uf — скорость движения точки вдоль оси Ох1 в системе отсчета К*, то и"— скорость движения точки вдоль оси Ох в системе отсчета К. Важное свойство этого закона: если и1 < с, то и" < с; если и1 = с, то и” = с; если и' > с, то и" > с. Следовательно, если скорость частицы (или электромагнитной волны) равна с в одной системе отсчета, то она одинакова во всех инерциальных системах отсчета.
Равенство (21) —функциональное уравнение, из которого можно определить вид неизвестной функции а(и):
а(и) =
1
\/1 — iz2/c2
(22)
Итак, мы вывели соотношения (1), (2) из принципа относительности и получили следствием постоянство скорости с во всех инерциальных системах отсчета. Важно отметить принципиальное отличие данного подхода к выводу преобразований Лоренца от общепринятого. Постоянство скорости света во всех инерциальных системах отсчета—это экспериментальный факт, установленный с определенной степенью точности. Приведенный выше вывод не опирается на этот факт, из него следует только существование скорости, одинаковой во всех инерциальных системах отсчета.
Из преобразований Лоренца можно получить следующие интересные следствия.
1.	Если в одной системе отсчета некоторые события происходят в точках Я] и Х2 в один и тот же момент времени i, то в другой
433
системе отсчета эти события происходят в точках х\ и х2 в разные моменты времени t\ и t2:
<-^ = ^(^2-^)-
Понятие одновременности оказывается зависящим от выбора системы отсчета.
2.	Если в одной системе отсчета между двумя событиями, происходящими в одной и той же точке, проходит время Z, то в другой системе отсчета между этими же событиями проходит время
— и2/с2
Это соотношение выражает релятивистский эффект замедления времени в движущихся объектах.
3.	Если в одной системе отсчета покоящаяся линейка имеет длину /, то в системе отсчета, в которой линейка движется со скоростью и вдоль своей оси, ее длина
/' = /л/1 - U2/c2.
Этот эффект называется релятивистским сокращением продольных размеров тела. При этом поперечные размеры тела не изменяются.
4.	Если в одной системе отсчета скорость тела v(vr, vz), то в другой системе отсчета его скорость v'(?4, г^, v') равна
, _ vx - и , _ vy y/1 - и2/с2 t _ vz y/1 - u2/c2
“ 1 - (u/c2)vx ’	” 1 - (u/c2)vx ’ Vz “ 1 - (u/c2)^ ’
Так, например, если тело движется со скоростью v вдоль оси Ох в одной системе отсчета, то его скорость в другой системе отсчета:
V — и
1 — uv/c2
(23)
Из соотношения (23) следует постоянство скорости с в различных системах отсчета. Действительно, положив v = с в правой части равенства (23), получаем и' = с.
В релятивистской механике доказывается, что импульс и энергия свободного тела (или замкнутой системы тел) сохраняются, однако их связь с массой и скоростью определяется иначе:
mpv р _ трс2 у/1 — v2 jc2'	у/1 — v2/с2
(24)
434
Здесь mo —масса покоя, т. е. масса ^ла, измеренная при скоростях много меньше скорости света.
Из равенств (24) следует интересное соотношение
Р _ v
Е с2’
Особое значение в современной физике приобрело определение энергии тела по формуле (24). Действительно, при скоростях много меньше скорости света все приведенные равенства совпадают с соответствующими выражениями классической физики. Соотношение же для энергии при малых скоростях можно записать в виде
2
Е « тос2 +	•	(25)
Второе слагаемое в равенстве (25) — это обычное выражение для кинетической энергии. Первое же слагаемое в классической физике было неизвестно. Оно означает, что каждое тело, даже находящееся в состоянии покоя, имеет некоторую энергию, пропорциональную его массе покоя. Эта энергия называется энергией покоя
Eq = mQc2.
Если тело, оставаясь в состоянии покоя, отдает или получает энергию Д£/, то его масса покоя меняется на величину
А ДЯ
Amo =
с2
Получить экспериментальное подтверждение этой связи изменений массы и энергии удалось при исследовании ядерных реакций и реакций с элементарными частицами.
435
Содержание
Предисловие............................................... 3
Используемые обозначения.................................. 5
Единицы физических величин................................ 7
1.	Закон Архимеда......................................... 8
2.	Динамика прямолинейного движения...................... 32
3.	Динамика вращательного движения....................... 81
4.	Закон всемирного тяготения........................... 102
5.	Механические колебания............................... 115
6.	Законы сохранения.................................... 146
7.	Статика.............................................. 171
8.	Кинематика........................................... 183
9.	Термодинамика и молекулярная физика.................. 222
10.	Электростатика...................................... 268
11.	Электрический ток................................... 306
12.	Геометрическая оптика............................... 369
13.	Олимпиадная смесь................................... 383
Добавления.............................................. 395
I.	Движение шарика в жидкости (комментарий к закону Архимеда)......................................... —
II.	Столкновения тел	при	наличии	трения........... 399
III.	Взаимодействия при наличии нецентральных сил.	402
IV.	Движение центра масс изолированной механической системы......................................... 406
V.	Движение в потенциальных полях с трением...... 412
VI.	Размерность в физике.......................... 415
VII.	Редукция размерных констант.................. 420
VIII.	Процессы с постоянной теплоемкостью......... 424
IX.	Исследование простейших термодинамических процессов ......................................... 426
X.	Модель атома Бора.............................. 428
XI.	Постоянство скорости света как следствие принципа относительности................................. 430
Учебное издание
Сергей Николаевич Манида
ФИЗИКА.
РЕШЕНИЕ ЗАДАЧ ПОВЫШЕННОЙ СЛОЖНОСТИ По материалам городских олимпиад школьников 2-е издание
Учебное пособие
Директор Издательства СПбУ проф. Р. В. Светлов Главный редактор Т. Н. Пескова
Редактор И. Н. Рязанова
Обложка Е. И. Егоровой Корректор Г. В. Маркичева Оригинал-макет подготовили С. Л. Кузьмина, А. М. Вейшторт
Издание подготовлено в ЖЩК Лицензия ИД № 056790 от 24.08.01 Подписано в печать 26.07.04. Формат 60x90 1/16. Бумага офсетная. Печать офсетная. Усл. печ. л. 27,5. Заказ Xе 153.
Издательство СПбГУ. 199034, Санкт-Петербург, Университетская наб., 7/9.
Тел. (812)328-96-17; факс (812)328-44-22 E-mail: editor@unipress.ru www.unipress.ru
По вопросам реализации обращаться по адресу: С.-Петербург, 6-я линия В. О., д. 11/21, к. 21 Телефоны: 328-77-63, 325-31-76 E-mail: post@unipress.ru Типография Издательства СПбГУ.
199061, С.-Петербург, Средний пр., 41.